Test Bank For Health Promotion Throughout the Life Span (Health Promotion Throughout the Lifespan (E

Page 1


Chapter 01: Health Defined: Objectives for Promotion and Prevention Edelman: Health Promotion Throughout the Life Span, 8th Edition MULTIPLE CHOICE 1. Which model of health is most likely used by a person who does not believe in preventive health care? a. Clinical model b. Role performance model c. Adaptive model d. Eudaimonistic model ANS: A The clinical model of health views the absence of signs and symptoms of disease as indicative of health. People who use this model wait until they are very sick to seek care. DIF:

Cognitive Level: Remember (Knowledge)

REF:

p. 3

2. A person with chronic back pain is cared for by her primary care provider as well as receives acupuncture. Which model of health does this person likely favor? a. Clinical model b. Role performance model c. Adaptive model d. Eudaimonistic model ANS: D The eudaimonistic model embodies the interaction and interrelationships among physical, social, psychological, and spiritual aspects of life and the environment in goal attainment and creating meaning in life. Practitioners who practice the clinical model may not be enough for someone who believes in the eudaimonistic model. Those who believe in the eudaimonistic model often look for alternative providers of care. DIF:

Cognitive Level: Apply (Application)

REF:

p. 3

3. A state of physical, mental, spiritual, and social functioning that realizes a person’s potential and is experienced within a developmental context is known as: a. growth and development. b. health. c. functioning. d. high-level wellness. ANS: B Health is defined as a state of physical, mental, spiritual, and social functioning that realizes a person’s potential and is experienced within a developmental context. DIF:

Cognitive Level: Remember (Knowledge)

4. Which of the following best describes a client who has an illness? a. Someone who has well-controlled diabetes b. Someone with hypercholesterolemia c. Someone with a headache d. Someone with coronary artery disease without angina ANS: C

REF:

p. 5


Someone with a headache represents a person with an illness. An illness is made up of the subjective experience of the individual and the physical manifestation of disease. It can be described as a response characterized by a mismatch between a person’s needs and the resources available to meet those needs. A person can have a disease without feeling ill. The other choices represent disease. DIF:

Cognitive Level: Analyze (Analysis)

REF:

p. 6

5. Which US report is considered a landmark document in creating a global approach to health? a. The 1990 Health Objectives for the Nation: A Midcourse Review b. Healthy People 2020 c. Healthy People 2000 d. The U.S. Surgeon General Report ANS: C Healthy People 2000 and its Midcourse Review and 1995 Revisions were landmark documents in which a consortium of people representing national organizations worked with US Public Health Service officials to create a more global approach to health. DIF:

Cognitive Level: Remember (Knowledge)

REF:

p. 6

6. Which of the following represents a method of primary prevention? a. Informational session about healthy lifestyles b. Blood pressure screening c. Interventional cardiac catheterization d. Diagnostic cardiac catheterization ANS: A Primary prevention precedes disease or dysfunction. It includes health promotion and specific protection and encourages increased awareness; thus, education about healthy lifestyles fits this definition. Blood pressure screening does not prevent disease, but instead identifies it. DIF:

Cognitive Level: Apply (Application)

REF:

p. 11

7. Which of the following represents a method of secondary prevention? a. Self–breast examination education b. Yearly mammograms c. Chemotherapy for advanced breast cancer d. Complete mastectomy for breast cancer ANS: B Screening is secondary prevention because the principal goal of screenings is to identify individuals in an early, detectable stage of the disease process. A mammogram is a screening tool for breast cancer and thus is considered a method of secondary prevention. DIF:

Cognitive Level: Apply (Application)

REF:

p. 15

8. Which of the following represents a method of tertiary prevention? a. Drunk driving campaign b. Road blocks for drunk driving c. Emergency surgery for head trauma after a motor vehicle accident d. Physical and occupational therapy after a motor vehicle accident with head trauma ANS: D


Physical therapy and occupational therapy are considered tertiary prevention. Tertiary prevention occurs when a defect or disability is permanent and irreversible. It involves minimizing the effect of disease and disability. The objective of tertiary prevention is to maximize remaining capacities. DIF:

Cognitive Level: Apply (Application)

REF:

p. 15

9. In reviewing a person’s medical claims, a nurse realizes that the individual with moderate persistent asthma has had several emergency department visits and is not on inhaled steroids as recommended by the NHLBI asthma management guidelines. The nurse discusses this with the person’s primary care provider. In this scenario, the nurse is acting as a(n): a. advocate. b. care manager. c. consultant. d. educator. ANS: B Care managers act to prevent duplication of service and reduce cost. Care managers base recommendation on reliable data sources such as evidence-based practices and protocols. DIF:

Cognitive Level: Apply (Application)

REF:

p. 15

10. During a home visit, a nurse assists an individual to complete an application for disability services. The nurse is acting as a(n): a. advocate. b. care manager. c. consultant. d. educator. ANS: A The advocacy role of the nurse helps individuals obtain what they are entitled to receive from the health care system, tries to make the system more responsive to individuals’ community needs, and assists individuals in developing skills to advocate for themselves. DIF:

Cognitive Level: Apply (Application)

REF:

p. 15

11. During a home visit, a nurse discusses the dangers of smoking with an individual. In this scenario the nurse is acting as a(n): a. advocate. b. care manager. c. consultant. d. educator. ANS: D Health education is a primary prevention technique available to avoid major causes of disease. Teaching can range from a chance remark to a planned lesson. DIF:

Cognitive Level: Apply (Application)

REF:

p. 16

12. A nurse is asked to provide an expert opinion about the development of an education program for newly diagnosed diabetics. In this scenario, the nurse is acting as a(n): a. advocate. b. care manager. c. consultant. d. educator.


ANS: C Nurses with a specialized area of expertise provide education about health promotion and disease prevention to individuals and groups as consultants. DIF:

Cognitive Level: Apply (Application)

REF:

p. 16

13. A nurse is planning to deliver an educational program to individuals with diabetes. Which of the following should be the initial action taken by the nurse to ensure the success of the program? a. Assess the motivation level of the individuals b. Assess the knowledge level of the individuals c. Establish teacher-learner goals with the individuals d. Establish multiple teaching sessions with the individuals ANS: B Selection of the methods most likely to succeed involves the establishment of teacher-learner goals. Thus, the first step by the nurse should be establishment of goals. DIF:

Cognitive Level: Analyze (Analysis)

REF:

p. 16

14. The conscientious, explicit, and judicious use of current best evidence in making decisions about the care of individuals is known as: a. health-related quality of life. b. evidence-based practice. c. a Healthy People 2010 goal. d. the ecological model of health. ANS: B Evidence-based practice is defined as the conscientious, explicit, and judicious use of current best evidence in making decisions about the care of individuals. DIF:

Cognitive Level: Remember (Knowledge)

REF:

p. 16

15. Which research methodology should be used to address the question, “What is the difference in the infection rates between individuals who receive twice-a-day dressing changes versus once-a-day dressing changes?” a. Evidence-based practice research b. Qualitative research c. Quantitative research d. Clinical judgment research ANS: C Quantitative research studies describe situations, correlate different variables related to care, or test causal relationships among variables related to care. Evidence-based practice research and clinical judgment research are not research methodologies; they are used to answer clinical questions. DIF:

Cognitive Level: Apply (Application)

REF:

p. 16

16. The question, “What is the experience of teenagers who lose a sibling to cancer?” can best be answered by using which research methodology? a. Evidence-based practice research b. Qualitative research c. Quantitative research d. Clinical judgment research ANS: B


Qualitative research studies describe phenomena or define the historical nature, cultural relevance, or philosophical basis of aspects of nursing care. Evidence-based practice research and clinical judgment research are not research methodologies; they are used to answer clinical questions. DIF:

Cognitive Level: Apply (Application)

REF:

p. 16

17. A nurse who uses findings from a randomized, controlled trial on the care of Foley catheters to change practice at an institution is practicing: a. evidence-based medicine. b. qualitative research. c. quantitative research. d. clinical judgment. ANS: A The practice of evidence-based medicine means integrating individual clinical expertise with the best available external clinical evidence from systematic research. DIF:

Cognitive Level: Apply (Application)

REF:

p. 16

18. Which of the following is most influenced by the social and economic environment of a community? a. Social health policies b. Quality of care c. Evidence-based practice d. Practice guidelines ANS: A Social policies concerning health are influenced by the social and economic environment of a population. Analysis of population trends and projections is necessary to help health professionals determine changing needs. DIF:

Cognitive Level: Remember (Knowledge)

REF:

p. 17

19. A major cause of death in the early twentieth century was: a. cancer. b. cerebrovascular disease. c. heart disease. d. infections. ANS: D Infections and acute disease were the major causes of death in the early part of the twentieth century. DIF:

Cognitive Level: Remember (Knowledge)

REF:

p. 17

20. A mainstay of therapy for the management of chronic diseases is: a. lifestyle changes. b. occupational and physical therapy. c. medications. d. surgery. ANS: A The application of complex technology is not only costly but contributes minimally to improvement of health. One needs to focus on the cause of disease. The needs of a person with chronic disease are related to and affected by the individual’s biochemical functioning, genetics, environment, and personal choices. DIF:

Cognitive Level: Apply (Application)

REF:

p. 17


21. Which of the following demonstrates a nurse taking action to promote health and prevent disease? a. Making a home visit to a person who is recovering from a heart attack b. Administering medications to a cardiac client in the hospital c. Providing cardiopulmonary resuscitation during a heart attack d. Educating a person about the advantages of a heart-healthy diet during a home visit ANS: D Solutions for health promotion are focused on individual and government involvement. To promote health and wellness, an emphasis must be placed on primary prevention. This is often related to actions such as education that influence lifestyle choices. In the preceding example, educating a person about the advantages of a hearthealthy diet during a home visit serves to influence lifestyle choices. DIF:

Cognitive Level: Apply (Application)

REF:

p. 17

22. Which factor may have the most influence in changing the health behavior of a single, adult woman who smokes and is the care provider for her mother, her own children, and granddaughter? a. Education regarding effects of smoking on her health b. The satisfaction that she will not contribute to secondhand smoke c. The availability of a weekly support group d. A gift card for $10 to a local grocery store for every week she is smoke free ANS: D Motivational factors play a role in influencing attitudinal changes. A financial incentive is an example of a motivating factor. For this woman, who is financially responsible for the health and well-being of other individuals, finances will likely play a significant role in motivating her actions. Remember that education regarding the benefits of not smoking is not enough. Thus, the $10 gift card may have the most influence in changing her health behavior. DIF:

Cognitive Level: Analyze (Analysis)

REF:

p. 14 (Box 1-6) | p. 18

23. An example of a Hispanic American nurse promoting cultural competency is: a. providing translation services for all Hispanic Americans. b. focusing on episodic care of illnesses with Hispanic Americans. c. taking a class about the Hispanic American culture. d. providing care for all Hispanic American persons in her clinic. ANS: C Nurses must be aware of their beliefs, values, and customs. They should not assume that their perspective is correct and shared by others. Additionally, they should avoid stereotyping. Broadening one’s educational base regarding cultural beliefs is a way to develop and thus promote cultural competency. DIF:

Cognitive Level: Apply (Application)

REF:

p. 17 (Box 1-2)

24. The devaluing of beliefs, values, and customs of others is known as: a. ethnocentrism. b. racism. c. cultural competency. d. empathy. ANS: B Racism is defined as the devaluing of beliefs, values, and customs of others. DIF:

Cognitive Level: Remember (Knowledge)

REF:

p. 7 (Box 1-2)

25. Building a partnership with an obese individual who is trying to lose weight requires the nurse to:


a. b. c. d.

schedule a day to exercise with the individual. go grocery shopping for the individual. learn about the methods of weight loss. give the individual a weight loss goal.

ANS: C Nurses must work with individuals to determine what they are willing to do to lose weight. To do this successfully, nurses must know what options are available to the individuals for weight loss. Once nurses learn about the methods of weight loss, they can then develop goals for weight loss with the individual. DIF:

Cognitive Level: Apply (Application)

REF:

p. 8 (Box 1-3)

26. Assuming that an individual’s own perspective is correct and shared by others is known as: a. ethnocentrism. b. racism. c. cultural competency. d. empathy. ANS: A Assuming that an individual’s own perspective is correct and shared by others is known as ethnocentrism. DIF:

Cognitive Level: Remember (Knowledge)

REF:

p. 8 (Box 1-2)

MULTIPLE RESPONSE 1. Which of the following “investment” themes has been identified by the National Institute of Nursing Research? (select all that apply) a. Interprofessional collaboration b. Improving quality of life c. End-of-life care d. Increasing physical activity among Americans ANS: B, C The National Institute of Nursing Research has identified five themes that promote health and prevent disease, improve quality of life through symptom management, and support palliative and end-of-life care, innovation, and nurse scientists. DIF:

Cognitive Level: Remember (Knowledge)

REF: p. 16

2. Which interventions address both the National Institute of Nursing Research themes and the overall goals of the Healthy People 2020 leading health indicators? (select all that apply) a. Establishing a new park with a well-lit track b. Establishing a smoking cessation campaign c. Providing reduced-cost transportation passes to senior citizens d. Providing free condoms at all federally funded health clinics ANS: A, B, D


Establishing a new park with a well-lit track, establishing a smoking cessation campaign, and providing free condoms at all federally funded health clinics all address the National Institute of Nursing Research themes and the Healthy People 2020 leading health indicators. A park, smoking cessation campaign, and free condoms address the themes of promoting health and preventing disease and the goal of increasing quality and years of healthy life. Providing reduced-cost transportation passes to senior citizens may indirectly affect the health of this population; however, it is not directly related to the themes of the National Institute of Research (promote health and prevent disease, improve quality of life through symptom management, and support palliative and end-of-life care, innovation, and nurse scientists) or the overall goals of Healthy People 2020 (increase quality and years of healthy life and eliminate health disparities). DIF:

Cognitive Level: Analyze (Analysis)

REF:

p. 7 | p. 16

Chapter 02: Emerging Populations and Health Edelman: Health Promotion Throughout the Life Span, 8th Edition MULTIPLE CHOICE 1. Which of the following statements about ethnic minorities in the United States is accurate? a. It is estimated that the percentage of ethnic minorities will decrease during the next 30 years. b. It is estimated that ethnic minorities will increase to one in two by 2050. c. The increasing population of refugees has been a significant contributor to the increase in ethnic minorities. d. The increasing population of ethnic minorities has helped decrease the health disparities faced by this population. ANS: B It is estimated that the number of ethnic minorities will increase to one in two by 2050. In 2010, it was estimated that 33% of the population was from an ethnic minority. The increasing population of immigrants has been a significant contributor to the increasing populations of major ethnic groups. The increasing populations of ethnic groups is one factor that is producing disparities in health status and access of the health care system. DIF:

Cognitive Level: Understand (Comprehension) REF:

p. 24

2. A person states, “My grandmother is the decision maker in our family.” Which of the following is being described by the person? a. Culture b. Race c. Ethnicity d. Values ANS: A Culture, as an element of ethnicity, refers to integrated patterns of human behavior that include the language, thoughts, communications, actions, customs, beliefs, values, and institutions of racial, ethnic, religious, or social groups. The term “ethnicity” encompasses more than a cultural practice, which is what is being described by the person; it focuses on differences in meanings, values, and ways of living. Race is associated with power and indexes the history or ongoing imposition of one’s group’s authority above another. Values are beliefs about the worth of something and serve as standards that influence behavior and thinking. DIF:

Cognitive Level: Analyze (Analysis)

REF:

p. 24

3. The nurse recommended to a 50-year-old woman that she schedule a routine mammogram. Which of the following would be the most important factor in this woman’s decision to schedule this exam? a. Race b. Ethnicity c. Cultural values


d.

Value orientation

ANS: C Cultural values guide actions and decision-making that facilitates self-worth and self-esteem. They shape human behaviors and determine what individuals will do to maintain their health status, how they will care for themselves, and others who become ill, and where and from whom they will seek health care. Race is associated with power and indexes the history or ongoing imposition of one’s group’s authority above another. Ethnicity focuses on differences in meanings, values, and ways of living. Value orientations reflect the personality type of a particular society. DIF:

Cognitive Level: Apply (Application)

REF:

p. 24

4. Which of the following actions demonstrates a health care professional providing culturally competent care? a. Encouraging the person to take medications as prescribed b. Asking the person to describe his folk healing methods c. Demonstrating the proper way to administer an insulin injection d. Assisting the person with discussing his health problems with the family ANS: B It is very important for health care providers to be aware of how people interpret their health issues or illnesses to be capable to provide culturally competent care. A culturally competent health care professional should be able to consistently and thoroughly recognize and understand the differences in his or her culture and that of the patient or client, to respect the person’s values and beliefs, and adjust the approach of delivering care to meet each person’s needs and expectations. Asking the person to describe his folk healing methods is the only action that demonstrates the health care professional seeking input from the person into the care that is received. DIF:

Cognitive Level: Analyze (Analysis)

REF:

p. 26

5. A person reports that she has been seeking care from an acupuncturist to help relieve the chronic pain that she has been experiencing. Which of the following statements would be the most appropriate response from the nurse? a. “You should have told me that the current treatments were helping your pain.” b. “Tell me more about your treatments from the acupuncturist.” c. “Tell me why you decided to not to continue with your treatment plan.” d. “You should not be seeing an acupuncturist while receiving professional care.” ANS: B Through a culturally sensitive assessment process, nurses can determine what specific remedies individuals are using and whether their continued use would interfere with the prescribed method. The nurse asking the person to describe the treatments from the acupuncturist allows the nurse to learn this information. The other responses demonstrate an ethnocentric perspective by the nurse, viewing the treatments from the acupuncturist as inferior to professional care. DIF:

Cognitive Level: Analyze (Analysis)

REF:

p. 26

6. When providing an educational session about the Arab American population, which of the following information would be included? a. The largest group of Arab Americans was refugees in the 1960s. b. The largest groups of Arab Americans are from Palestine and Iraq. c. Members of the Arab American population are most likely to live in rural communities. d. Members of the Arab American population are more likely to have college degrees than Americans at large. ANS: D


Members of the Arab American population are more likely to have college degrees (+45%) than Americans at large (28%). About 94% of Arab Americans live in metropolitan areas. The largest groups of Arab Americans are the Lebanese, Syrians, and Egyptians. Arab Americans came to the United States in three immigration waves; the last occurred in the 1960s and consisted of many professionals, entrepreneurs, and skilled and semiskilled laborers. DIF:

Cognitive Level: Understand (Comprehension) REF:

p. 27

7. A health care professional is caring for an Arab American individual. Which of the following cultural practices of this ethnic minority should be considered when planning care? a. This ethnic culture tends to be future oriented. b. Religion plays an important role in this culture. c. Traditional cultural practices are infrequently used during a health crisis. d. Members of this culture tend to have smaller families. ANS: B Religion plays an important part in Arab culture, and there are dietary rules and prescribed rituals for praying and washing. Arab Americans are present oriented and view the future as uncertain. During a health crisis, many Arab Americans seek out their family, community, and traditional values and cultural practices. Arab American families are, on average, larger than non–Arab American families. DIF:

Cognitive Level: Apply (Application)

REF:

p. 27

8. A health care professional is providing education to the parents of an Asian American child who has recently been diagnosed with Type I diabetes. Which of the following actions should be taken by the health care professional? a. Provide instructions to the child’s father. b. Encourage the parents to bring other siblings into the clinic for screening. c. Schedule multiple educational sessions for the child and family. d. Watch for nonverbal gestures by the adults to indicate understanding. ANS: A In Asian American culture, the oldest male family member often is the decision maker and spokesperson. Maintaining harmony is an important value in Asian cultures, and it is strongly emphasized to avoid conflict and direct confrontation. As a result of this, Asian Americans may not show their disagreement with the recommendations of health care professionals. Type I diabetes is not a common health problem experienced by this minority, so it is probably not necessary to encourage screening for siblings. Additional educational sessions may be necessary; however, Asian Americans tend to be more highly educated than any other population in the United States. DIF:

Cognitive Level: Analyze (Analysis)

REF:

p. 29

9. An Asian American family has recently immigrated to the United States. Which of the following would provide an appropriate rationale when encouraging the family to send their child to school? a. The child will get a good education. b. According to law, all children must go to school. c. The child can get health care at school. d. Exposure to different cultures in school will enhance socialization. ANS: D Exposure to different cultures in school facilitates the adoption of other cultural beliefs and aids in the socialization of the child into a new environment. DIF:

Cognitive Level: Apply (Application)

REF:

p. 29

10. Which of the following was the fastest-growing minority group in the United States between 2000 and 2010?


a. b. c. d.

Asian American Arab American Hispanic American Native American

ANS: C A 43% increase in the Hispanic population between 2000 and 2010 makes Hispanics the fastest-growing minority group in the United States. DIF:

Cognitive Level: Remember (Knowledge)

REF:

p. 31

11. The interrelationship of poverty and health care dollars spent by Blacks and other minorities is affected greatly by: a. lack of access to preventive health care services b. low numbers of minority health care providers c. use of emergency rooms for care d. increased infant mortality rates in African American populations ANS: A A decrease in resources for preventive care leads to the use of emergency rooms and other more expensive health care services that are often used as resources when severe illness occurs. DIF:

Cognitive Level: Understand (Comprehension) REF:

p. 32

12. A health care provider is working with an African American woman who has recently suffered a stroke and is homebound. She insists that she must get out of the house and attend Sunday worship services. What is the most likely explanation for her insisting that she participate in this cultural practice? a. The church is the only place where prayer can be performed. b. The church serves as a social support for its members. c. The church is the place where the family meets on a weekly basis. d. The church serves as a site for folk healing practices. ANS: B The church is significant support system many African Americans. It serves many purposes beyond worship and formation, including serving as a place to meet where members could pass news, take care of business, and find strength of purpose; providing direct social welfare services; acting as a stabilizing force in the community; facilitating citizenship training and community social action; serving as a transmitter of cultural history; and providing the means for coping and surviving in a hostile world. African Americans often find comfort in the support their religious leader can give them, but it does not have to happen within the church. African Americans believe in the healing power of prayer, but that can happen outside of the church as well. Family is the strongest source of support for African Americans, and most meet more often than weekly at church. DIF:

Cognitive Level: Apply (Application)

REF:

p. 33

13. Which of the following ethnic groups has a disproportionately high death rate from unintentional injuries and suicide? a. American Indian/Alaska Native Americans b. Asian Americans c. Latino/Hispanic Americans d. Black/African Americans ANS: A American Indian/Alaska Native Americans have disproportionately high death rates from unintentional injuries and suicide. Difficult life situations and stresses of daily life contribute to an array of problems, including feelings of hopelessness, desperation, family dissolution, and substance abuse.


DIF:

Cognitive Level: Apply (Application)

REF:

p. 34

14. A health care professional is offering an educational session about providing culturally congruent care. Which of the following information would be included the presentation? a. Hispanic Americans value keeping balance and harmony with the earth. b. The oldest male is the decision maker in African American families. c. Native Americans are present oriented, taking one day at a time. d. The hot and cold concept of disease is part of the Asian American culture. ANS: C Native Americans are generally present oriented, emphasizing events that are occurring now rather than events that will happen later. They take one day at a time and in times of illness they cope by hoping for improvements the next day. Native Americans value keeping balance and harmony with the earth. The oldest male is the decision maker and spokesperson in Asian American families. The hot and cold concept of disease is part of the Hispanic culture. DIF:

Cognitive Level: Apply (Application)

REF:

p. 34

15. A family has recently become homeless. Which of the following factors most likely contributed to this situation? a. Being from an ethnic minority background b. Declining rates of poverty c. Having multiple chronic illnesses d. Being unable to find affordable housing ANS: D The inability to find affordable housing, decline in public assistance, poverty, and eroding work opportunities all contribute to homelessness. The increasing prevalence of poverty has caused an increase in the homeless population. Other factors that may affect this situation are lack of affordable health care, domestic violence, mental illness, and addiction disorders. DIF:

Cognitive Level: Apply (Application)

REF:

p. 35

16. A health care professional is caring for an individual who is homeless. Which of the following considerations should be made? a. The prevalence of substance abuse is much lower among the homeless population than the general population. b. The prevalence of HIV/AIDS is much lower among the homeless population than the general population. c. The percentage of the population who has health insurance is much lower among the homeless than the general population. d. The percentage of the population who has limited access to medical care is much lower among the homeless than the general population. ANS: C Most homeless people do not have health insurance or the ability to pay for needed health care, and many providers refuse to deliver treatments to these people. The prevalence of substance abuse, HIV/AIDS, and mental health disorders is higher among the homeless population than the general population. DIF:

Cognitive Level: Apply (Application)

REF:

p. 36

17. A health care professional is caring for an individual who is homeless and has recently been diagnosed with Type II diabetes. Which of the following factors is the most important to consider when planning care? a. Considering the cost of the purchasing medications b. Determining the pharmacy where medications will be obtained c. Obtaining insurance that will pay for the follow-up care


d.

Finding supportive housing for the individual

ANS: D Research and practice have shown that permanent supportive housing works because housing is an essential part of treatment; thus, this is the most important factor that should be considered. If supportive housing is found, the stability will help the homeless individual to follow the prescribed medical regimen. DIF:

Cognitive Level: Analyze (Analysis)

REF: p. 36

18. A health care professional is leading a community action coalition to address the problem of homelessness within the neighborhood. Which of the following statements would most likely be made by health care professional? a. Homelessness should be addressed by improving financial assistance programs. b. Homelessness should be of concern to everyone in the neighborhood. c. Homelessness should be addressed by encouraging job growth in the area. d. Homelessness should be of concern to the mental health providers in the area. ANS: B Homelessness is everyone’s problem, and people can ultimately affect the establishment of priorities to facilitate an improved quality of life. As more people understand homelessness, this will serve as an excellent guide in providing input, taking necessary action, and making the final decision as to what will make a healthy nation. DIF:

Cognitive Level: Analyze (Analysis)

REF:

p. 37

19. What is the main focus of the National Institutes of Health (NIH)? a. Addressing and reducing health disparities b. Outlining nationwide health promotion and disease prevention c. Protecting minority populations through development of health policies d. Supporting communities in addressing health disparities ANS: A The main concern of the National Institutes of Health is addressing and reducing health disparities involving cancer, diabetes, infant mortality, AIDS, cardiovascular illnesses, and many other diseases. Healthy People 2020 outlines a comprehensive, nationwide health promotion and disease prevention agenda. The Office of Minority Health improves and protects the health of racial and ethnic minority populations through the development of health policies and programs that concentrate on eliminating health disparities. The Centers for Disease Control and Prevention’s Racial and Ethnic Health Disparities Action Institute supports communities to take action in addressing health disparities. DIF:

Cognitive Level: Remember (Knowledge)

REF:

p. 37

20. A health care professional is searching for a funding source to develop a colorectal cancer screening program for ethnic and racial minorities in the community. Which of the following federal agencies would most likely be able to assist with this initiative? a. The National Institute on Minority Health and Health Disparities b. The Centers for Disease Control and Prevention c. The Office of Minority Health d. The National Institutes of Health ANS: C The Office of Minority Health improves and protects the health of racial and ethnic minority populations through the development of health policies and programs that concentrate on eliminating health disparities. The National Institute on Minority Health and Health Disparities, The Centers for Disease Control and Prevention, and National Institutes of Health all address health disparities among racial and ethnic minorities, but their priority is not in funding these initiatives.


DIF:

Cognitive Level: Analyze (Analysis)

REF:

p. 37

21. Which of the following best demonstrates the practice of transcultural nursing? a. Using previous knowledge about ethnic minority cultures to plan care b. Adapting nursing care to meet the need of a person from an ethnic minority c. Requesting an interpreter when caring for a person from an ethnic minority d. Attending a presentation about cultural diversity ANS: Blm Transcultural nursing is defined as an area of nursing study and practice that focuses on discovering and explaining cultural factors that influence the health, well-being, illness, or death of individuals or groups and seeks to provide culturally based appropriate care to people of diverse cultures. Adapting nursing care to meet the needs of a person from an ethnic minority best meets this definition of transcultural nursing. A nurse may initially use previous knowledge about minority cultures to plan care, but then must individualize the care based on individual differences within the culture. Having an interpreter present will not be necessary when working with all persons from ethnic minorities. Attending a presentation about cultural diversity would assist the nurse in becoming more culturally competent, but it is not the best example of practicing transcultural nursing because no nursing care is being provided when attending a presentation. DIF:

Cognitive Level: Analyze (Analysis)

REF:

p. 38

22. A nurse would like to improve his cultural competence. Which of the following is the best strategy to use? a. Explore complementary and alternative medicine practices used by other cultures. b. Discuss cultural differences with coworkers. c. Provide care to diverse populations. d. Participate in continuing education programs about cultural diversity. ANS: D Salman et al. (2007) assessed cultural awareness and cultural competence levels among staff nurses who participated in a continuing education program aimed at increasing knowledge of culturally competent care of geriatric populations. Staff nurses who participated in this training program had higher cultural competence levels as compared with those who did not join this training program. Exploring complementary and alternative medicine practices used by other cultures will assist if a nurse is using these practices; however, depending on whom the nurse is caring for, this information may have limited usefulness. Discussing cultural differences with coworkers may help the nurse learn about other cultures, but it may not provide the best source of information. Providing care to diverse populations may not help the nurse to become more culturally competent without having some baseline knowledge about cultural diversity. DIF:

Cognitive Level: Apply (Application)

REF:

p. 26 (Box 2-3)

23. A nurse is told by a colleague that an Asian American individual on home care is using complementary and alternative medicine (CAM). Which of the following conclusions can accurately be drawn from this statement? a. The client will have adverse complications from using this form of medicine. b. The client may be using meditation, acupuncture, or another therapy. c. The client will soon be returning to his or her native country for further treatment. d. The client does not agree with the current professional care medication regimen. ANS: B Examples of complementary and alternative medicine (CAM) include acupuncture, feedback, relaxation, music therapy, massage, art, music, and dance therapy. Persons who do not experience relief from chronic conditions often resort to complementary alternative medicine. It is important the nurse learn more about the CAM that the person is using so that any potential interactions between the use of CAM and professional care can be addressed. Complementary and alternative medicine can be used in conjunction with the use of professional care. DIF:

Cognitive Level: Apply (Application)

REF:

p. 30 (Box 2-4)


24. A woman reports that she has strong spiritual practices. Which of the following is she most likely to experience? a. Improved coping skills and social support b. Increased understanding of religious differences c. Decreased pain and improved healing d. Decreased use of Western medicine ANS: A Spiritual practices are likely to improve coping skills and social support, promote feelings of optimism and home, encourage healthy behavior, decrease feelings of depression and anxiety, and support a sense of relaxation. DIF:

Cognitive Level: Apply (Application)

REF:

p. 39 (Box 2-5)

MULTIPLE RESPONSE 1. Which of the following individuals will most likely experience a disparity in health and health care? (select all that apply) a. African American man b. Unemployed woman c. White middle-aged man d. Single white woman ANS: A, B, D Health disparities adversely affect groups of people who have systematically experienced greater obstacles to health based on their racial or ethnic group; religion; socioeconomic status; gender; age; mental health; cognitive, sensory, or physical disability; sexual orientation or gender identity; geographic location; or other characteristics historically linked to discrimination or exclusion. A white middle-aged man is the only individual who does not meet that definition. DIF:

Cognitive Level: Apply (Application)

REF:

p. 23

2. A health care professional is providing culturally competent care. Which of the following actions is being performed by the professional? (select all that apply) a. Recognizing and accepting cultural diversity b. Respecting the patient’s values, beliefs, and expectations c. Understanding the pathophysiology of disease processes d. Providing health care services that are respectful of the individual’s cultural beliefs ANS: B, D A culturally competent health care professional should be able to consistently and thoroughly recognize and understand the differences in his or her culture and that of the patient or client; respect the individual’s values and beliefs; and adjust the approach of delivering care to meet each individual’s needs and expectations. Simply recognizing and accepting cultural diversity and understanding the pathophysiology of disease processes are insufficient measures to reach cultural competency in health care. DIF:

Cognitive Level: Apply (Application)

REF:

p. 26

3. A health care provider is discussing the importance of receiving routine preventive care with a Hispanic family who has recently immigrated to the United States. Which of the following would best describe why they may be disinterested in receiving professional care? (select all that apply) a. Lack of folk remedies b. Lack of interpreter services c. Lack of health insurance d. Lack of family support


ANS: B, C Barriers experienced by Hispanic Americans in receiving appropriate health care services include lack of racial and ethnic diversity in the leadership and workforce of the health care system, lack of interpreter services for Spanish-speaking people, lack of health insurance, and lack of or inadequate culturally appropriate health care resources. They may not readily seek care because of their continued reliance on their folk system of healing. The family is the most important source of support for Hispanic Americans. DIF:

Cognitive Level: Apply (Application)

REF:

p. 31

4. A nurse is developing a continuing education program about strategies to work with culturally diverse populations for health care professionals in the community. Which of the following organizations would provide information that could be included in this presentation? (select all that apply) a. The American Nurses Association b. Sigma Theta Tau International c. The National League for Nursing d. The American Association of Colleges of Nursing ANS: A, C, D Major organizations, such as the American Nurses Association (ANA), the National League for Nursing (NLN), and the American Association of Colleges of Nursing (AACN), publish culturally relevant materials to guide students, clinicians, and educators. DIF:

Cognitive Level: Apply (Application)

REF:

p. 38

Chapter 03: Health Policy and the Delivery System Edelman: Health Promotion Throughout the Life Span, 8th Edition MULTIPLE CHOICE 1. Which addition to a community best demonstrates the concept of the Healthy People 2020 report? a. New cardiothoracic intensive care unit at a major hospital b. New rehabilitation center c. New recreational health center d. New children’s hospital ANS: C The Healthy People 2020 report promotes health care, not illness care. A hospital, ICU, and rehabilitation center emphasize episodic care after an illness. The recreation health center serves to preserve health in the community and helps meet the goals of the Healthy People 2020 report. DIF:

Cognitive Level: Apply (Application)

REF:

p. 47 | p. 47 (Box 3-1)

2. Which nurse is at risk of making a medical error? a. Working overtime b. Works in a hospital that advocates multidisciplinary care c. Caring for four clients during the shift d. Attends regular continuing education programs ANS: A Health care systems are the basic cause of medical errors. Organizational and workforce management, work design, and organizational culture are problem areas that contribute to medical errors. Poor management leads to increased nurse turnover, the need for increased client-to-nurse ratios, increased need for overtime, and decreased number of nurses, all of which can lead to medical errors. A nurse working overtime is at risk of making a medical error.


DIF:

Cognitive Level: Apply (Application)

REF:

p. 50

3. What is the most effective policy a staff nurse can implement to help decrease medical errors on the unit? a. Forgo opportunities for continuing education so the unit is never understaffed. b. Foster a collaborative working environment on the unit. c. Check all medications a minimum of three times before administering them. d. Develop a policy that mandates the firing of any nurse who commits an error. ANS: B Health care systems are the basic cause of medical errors. Organizational and workforce management, work design, and organizational culture are problem areas that contribute to medical errors. Creating a collaborative working environment helps improve organizational culture, thereby reducing the chance of medical errors. DIF:

Cognitive Level: Apply (Application)

REF:

p. 50

4. The person known as the father of British and American public health is: a. Lillian Wald. b. Edwin Chadwick. c. Lemuel Shattuck. d. Paul Ehrlich. ANS: B Edwin Chadwick is known as the father of British and American public health. DIF:

Cognitive Level: Remember (Knowledge)

REF:

p. 51

5. A community health nurse most effectively preserves the health of a person with tuberculosis (TB) and the community by: a. administering and reading the purified protein derivative (PPD) of all close contacts b. providing direct observed therapy (DOT) for the individual with TB c. isolating the person because it is likely drug resistant d. telling the person to wear a mask when leaving the home ANS: B Community health nursing practice promotes, preserves, and maintains the health of populations and the effect of their health status on that of the community as a whole through care provided to individuals, families, and groups. Thus, the most effective strategy of treating the individual and preventing the spread of TB is to provide DOT. DIF:

Cognitive Level: Analyze (Analysis)

REF:

p. 52

6. A community planning committee is working on the development of a community nursing center. Which of the following essential components should be included in this center? a. Physician as medical director b. Interdisciplinary staff c. Nurse as chief manager d. Partnership with an academic institution ANS: C The essential components of a community nursing center include a nurse as chief manager, a nursing staff that is accountable and responsible for care and professional practice, and nurses as the primary providers of care. DIF:

Cognitive Level: Apply (Application)

REF:

p. 54


7. Considering the provider and client perspective, which of the following is the most autonomous form of insurance? a. Fee-for-service plan b. Health maintenance organization (HMO) c. Preferred provider organization (PPO) d. Independent practice association (IPA) ANS: A In the fee-for-service plan, a provider provides a service and bills the individual’s insurance company. The individual is also allowed to choose his or her provider. DIF:

Cognitive Level: Apply (Application)

REF:

p. 55

8. Which could result in a change in a nurse practitioner’s practice? a. Holding a master’s degree or higher b. Moving to another state when she gets married c. Caring for Medicare clients d. Changing to another office within the same practice ANS: B Nurse practitioners generally hold master’s degrees or higher. The nurse practice act within their states may be more comprehensive than the institutions for which they work. Legislation exists for reimbursement of nurse practitioners by Medicare. However, practice acts and prescriptive authority vary from state to state and thus influence practice patterns. DIF:

Cognitive Level: Apply (Application)

REF:

p. 55 | p. 70

9. Which health care provider represents a primary care provider? a. Psychiatric advanced practice nurse b. Clinical nurse specialist c. Pediatric nurse practitioner d. Acute care nurse practitioner ANS: C A primary care provider serves as a gatekeeper, coordinating care of individuals by determining the need for referrals and procedures. A primary care provider can be a physician, physician’s assistant, or advanced practice nurse in a primary care setting. The primary care provider provides basic and routine care usually in an office or a clinic. A pediatric nurse practitioner is an example of a primary care provider. DIF:

Cognitive Level: Apply (Application)

REF:

p. 55 | p. 56 (Box 3-5)

10. Which form of managed care restricts providers to caring for individuals who are members of their organization? a. Fee-for-service plan b. Health maintenance organization (HMO) c. Preferred provider organization (PPO) d. Independent practice association (IPA) ANS: B The traditional HMO was a group or staff model in which a group of physicians and some specialty services provided care to its members. Providers generally spent all their time serving members of the HMO. Fee-forservice, IPAs, and PPOs are not restricted to serving clients for any one organization. DIF:

Cognitive Level: Remember (Knowledge)

REF:

p. 55


11. A Medicare client reports to the home care nurse that he is receiving care through an accountable care organization (ACO). Which of the following considerations should be made when delivering care to this individual? a. The individual must see a primary care provider before being referred to a specialist. b. The focus of care is prevention and management of individuals with chronic disease. c. The individual has paid a membership fee to be part of this organization. d. The focus of care is to conserve money in a health savings account. ANS: B The focus of care of accountable care organizations (ACOs) is to focus on prevention and management of individuals with chronic disease out of the hospital. In an ACO, physicians accept the responsibility for the quality of care provided and overall costs of delivering care to a defined population of patients. Accountable care organizations are composed of physicians, specialists, and hospitals, so a specialist will be able to be seen within the ACO. Individuals who are part of concierge care pay a membership fee in return for enhanced health care services or amenities. Health savings accounts are used in conjunction with high deductible health insurance plans and are not related to the use of ACOs. DIF:

Cognitive Level: Apply (Application)

REF:

pp. 56-57

12. An individual states that his family has insurance, but that paying for routine medical costs is very expensive because most routine services are paid for out of pocket. Which type of insurance plan does this individual most likely have? a. High deductible health insurance plan (HDHP) b. Point-of-service plan (POS) c. Health maintenance organization (HMO) d. Preferred provider organization (PPO) ANS: A High deductible health insurance plans (HDHPs) are structured in a way similar to traditional managed care plans and fee-for-service plans but have a very high annual out-of-pocket deductible. Thus, a family with this type of insurance will pay out-of-pocket until they reach the deductible, which may make receiving health services expensive for them. Point-of-service plans allow members, for an additional fee and higher copayment, to use providers outside of the HMO network. Health maintenance organizations (HMOs) deliver comprehensive health maintenance and treatment services for a group of enrolled individuals who prepay a fixed fee. Preferred provider organizations (PPOs) have a preselected list of providers who have agreed to provide health services for those enrolled in the plan. DIF:

Cognitive Level: Apply (Application)

REF:

p. 58

13. Which of the following is a health plan consisting of hospitals and physician providers providing health care services to plan members (usually at discounted rates) in return for expedited claims payment? a. Health maintenance organization (HMO) b. Health savings account (HSA) c. Preferred provider organization (PPO) d. Independent practice association (IPA) ANS: C A health plan consisting of hospitals and physician providers providing health care services to plan members (usually at discounted rates) in return for expedited claims payment is known as a preferred provider organization (PPO). DIF:

Cognitive Level: Remember (Knowledge)

REF:

p. 58 | p. 56 (Box 3-5)

14. Two working adults do not have access to health insurance for their family from their employers and do not meet the financial criteria for Medicaid. For which of the following federal programs may the children in this family be eligible?


a. b. c. d.

Medicare Civilian Health Medical Program for Uniformed Services (CHAMPUS) State Children’s Health Insurance Program (SCHIP) Veteran’s Administration Program

ANS: C The State Children’s Health Insurance Program is a public state insurance program established to provide insurance to uninsured children whose family is typically described as the working poor and do not meet Medicaid requirements. In this case, both parents work but do not have access to insurance through their employers. Additionally, they do not meet the financial Medicaid requirement. DIF:

Cognitive Level: Apply (Application)

REF:

p. 60

15. A nurse complies with the Patient Self-Determination Act when asking: a. a person upon admission to the hospital if he or she has an advanced directive b. the family in the recovery room if the client has an advanced directive c. a person before discharge from the hospital if he or she has an advance directive d. the family about an advanced directive after the person has been intubated ANS: A The Patient Self-Determination Act is designed to increase individual involvement in decisions about lifesustaining treatments. The nurse must ensure that advanced directives are available to physicians at the time the medical decision is being made. Therefore, the nurse complies with the act when she asks a person upon admission to the hospital if he or she has an advanced directive. DIF:

Cognitive Level: Apply (Application)

REF:

p. 64

16. A nurse discussing the care of a person on the surgical unit following gastric bypass surgery with a friend is in violation of the: a. Patient Self-Determination Act b. Health Insurance Portability and Accountability Act of 1996 c. Americans with Disabilities Act d. Civil Rights Act ANS: B The Health Insurance Portability and Accountability Act of 1996 (HIPAA) is a federal privacy standard that requires safeguards to protect the security and confidentiality of health information. Disclosures without individual authorization are allowed only to public health authorities authorized by law to collect and receive information for the purpose of preventing or controlling disease, injury, or disability. DIF:

Cognitive Level: Apply (Application)

REF:

p. 64

17. Which is a violation of the Health Insurance Portability and Accountability Act of 1996? a. Nurse reporting a case of TB to the health department b. Nurse reporting a case of child abuse to the Department of Human Services c. Nurse discussing the person’s case with his or her physician d. Nurse discussing the person’s case with his or her school nurse ANS: D The Health Insurance Portability and Accountability Act of 1996 (HIPAA) is a federal privacy standard that requires safeguards to protect the security and confidentiality of health information. Disclosures without individual authorization are allowed only to public health authorities authorized by law to collect and receive information for the purpose of preventing or controlling disease, injury, or disability. Before speaking to the school nurse, the nurse must obtain authorization from the client’s parents.


DIF:

Cognitive Level: Apply (Application)

REF:

p. 64

18. Which nurse is functioning as a care manager? a. Nurse working with a family to coordinate care after their child experiences a second hospitalization for an asthma exacerbation b. Nurse providing asthma education in the office setting to a child with moderate, persistent asthma c. Nurse administering the appropriate antiinflammatory medication to a child admitted to the hospital with an asthma exacerbation d. Nurse making a home visit for a respiratory assessment to a child following an admission for an asthma exacerbation ANS: A Care managers help determine what medical care is necessary, monitor care, and arrange for individuals to receive the most cost-effective care in the most appropriate settings. They must collaborate with providers and with the client/family. Care managers are especially helpful following a client after discharge and clients with complex needs. A nurse working with a family to coordinate service after a hospitalization is a good example of services provided by a care manager. DIF:

Cognitive Level: Apply (Application)

REF:

p. 71

19. A pregnant woman with two toddlers living at less than 135% of the federal poverty level would be eligible for: a. Medicare Part A b. Medicare Part B c. Medicaid d. State Children’s Health Insurance Program (SCHIP) ANS: C State Medicaid programs must cover all pregnant women and children up to 6 years of age with a family income of less than 133% of the federal poverty level. States Children’s Health Insurance Program provides insurance coverage to children whose family income is below 200% of the federal poverty level or whose income is 50% higher than the state’s Medicaid eligibility threshold. In this case, the woman is pregnant, her children are under the age of 6, and they meet the financial criteria for Medicaid but not SCHIP. DIF:

Cognitive Level: Apply (Application)

REF:

pp. 74-75

20. Which of the following is a major factor limiting health care services in the United States? a. Lack of nurses b. Lack of health care facilities c. Lack of funding d. Lack of client interest in health promotion ANS: C The lack of nurses may prevent health care providers from offering health promotional education activities. The cost of health care and prescription drugs is a major limitation to health care services. The United States has the highest proportion of population with no health insurance, thereby limiting health care services available to Americans. DIF:

Cognitive Level: Remember (Knowledge)

REF:

pp. 75-76

21. Which person is at highest risk for being uninsured? a. 68-year-old retired mailman b. 48-year-old on renal dialysis c. 2-year-old whose parents work but do not have benefits through their employers d. 27-year-old who attends college and works part time in a small pizza parlor


ANS: D Young adults (27 years of age or older) are not eligible to be covered under their parents’ insurance. Medicare is available for those over the age of 65 and for people who require dialysis. Medicaid and SCHIP are available for children whose families meet the financial requirements. This child would likely meet criteria for either Medicaid or SCHIP because the parents are considered working poor. Because health care insurance is so expensive, it would be difficult for a 27-year-old to afford his or her own plan. Young adults often go without insurance because of the high costs. DIF:

Cognitive Level: Apply (Application)

REF:

pp. 75-76

22. Adoption of a Canadian-style health care system in the United States has the potential to: a. decrease waiting times for tests and procedures. b. create further health disparities. c. increase the number of HMOs. d. increase the amount of funding available for health care. ANS: B Canadians with private health insurance and higher incomes have access to greater health care services and more expedient health care. This translates to a two-tier system of health care, which can contribute to health disparities in health care access and quality. DIF:

Cognitive Level: Analyze (Analysis)

REF:

p. 77

23. A system used to evaluate the necessity, appropriateness, and efficiency of the use of the health care system, the purpose of which is to lower costs by discouraging unnecessary treatments, is known as: a. managed care b. gate keeping c. utilization review d. capitation ANS: C A system used to evaluate the necessity, appropriateness, and efficiency of the use of the health care system, the purpose of which is to lower costs by discouraging unnecessary treatments, is known as utilization review. DIF:

Cognitive Level: Remember (Knowledge)

REF:

p. 56 (Box 3-5)

24. The country with the lowest infant mortality rate in 2011 was: a. Canada b. Mexico c. Sweden d. the United States ANS: C Sweden had the lowest infant mortality in 2011 with a mortality rate of 2.74 per 1000 live births. DIF:

Cognitive Level: Remember (Knowledge)

REF:

p. 48 (Table 3-1)

25. The country with the highest life expectancy for women in 2011 was: a. France b. Germany c. Sweden d. the United States ANS: A France was the country with the highest life expectancy for women (85 years of age) in 2011.


DIF:

Cognitive Level: Remember (Knowledge)

REF: p. 48 (Table 3-1)

MULTIPLE RESPONSE 1. A community health nurse is presenting information about the current health of the nation to a local political action group in 2011. Which of the following information would be included in this presentation? (select all that apply) a. Funding of public health programs has improved the health of the nation. b. Provision of health care has improved the health of the nation. c. Advances in technology have improved the health of the nation. d. Consumer education has improved the health of the nation. ANS: A, B, D According to the 2010 Report of Health, the health of the nation has improved in many areas as a result of substantial funding of public health programs, research, provision of health care, and initiatives to support consumer education. DIF:

Cognitive Level: Apply (Application)

REF:

p. 42

2. Which of the following statements about the financing of health care in the United States is accurate? (select all that apply) a. Employee-sponsored health insurance premiums experienced minimal increases from 2001 to 2011. b. State tax dollars must fund at least 50% of a state’s Medicaid cost. c. The largest percentage of health care dollars is spent on hospital care. d. Most workers who have an employer-sponsored health insurance plan are enrolled in preferred provider organizations (PPOs). ANS: C, D In 2010, almost one third (31%) of health care dollars was spent on hospital care. The majority of employeecovered workers were enrolled in PPOs (55%). Employee-sponsored health insurance premiums rose dramatically from 2001 to 2011 by 113%. Federal dollars must fund a minimum of 50% of a state’s Medicaid cost; the state is responsible for contributing whatever percentage is not funded by the federal government. DIF:

Cognitive Level: Understand (Comprehension) REF:

p. 67

3. Which of the following would be considered part of a vulnerable population? (select all that apply) a. Deaf man b. Cocaine-addicted teenage girl c. Handicapped child d. College graduate ANS: A, B, C Those who are physically disabled or handicapped, substance abusers, and those with communication difficulties are considered part of the vulnerable population in the United States. DIF:

Cognitive Level: Apply (Application)

REF:

Chapter 04: The Therapeutic Relationship Edelman: Health Promotion Throughout the Life Span, 8th Edition MULTIPLE CHOICE 1. Which of the following statements about values is true? a. It is impossible for someone to understand his or her values.

p. 47 (Box 3-2


b. c. d.

Values evolve over time; they are not static. Values assimilated in childhood are those held for a lifetime. Values clarification tells a person how to act.

ANS: B Values take time to develop, and they may change with education, experience, exposure, or a combination of these. DIF:

Cognitive Level: Understand (Comprehension) REF:

p. 83

2. A nurse is providing education about improving self-awareness to a small group of women who have been victims of domestic violence. Which of the following should the nurse recommend the clients do first to improve their self-awareness? a. Use self-disclosure to share aspects of self. b. Listen to and learn from others. c. Listen to oneself and pay attention to emotions, thoughts, and reactions. d. Use others to bounce back your own thoughts and recollections. ANS: C To achieve a high self-awareness, three steps must be taken. The first step is listening to oneself and paying attention to emotions, thoughts, memories, reactions, and impulses. The second step is listening to and learning from others. The third step is using self-disclosure to share aspects of self. DIF:

Cognitive Level: Apply (Application)

REF:

p. 84

3. The nurse shares with her client the news that she, the nurse, is going to be married soon and tells the client about her wedding plans. Which of the following best describes the nurse’s actions? a. The nurse is sharing inappropriate personal information with her client. b. The nurse is exhibiting a communication technique called self-disclosure. c. The nurse is attempting to show empathy with her client. d. The nurse is violating client confidentiality rights of the Health Insurance Portability and Accountability Act (HIPAA). ANS: B Sharing aspects of the self enriches interpersonal life and is a use of the technique labeled self-disclosure. DIF:

Cognitive Level: Apply (Application)

REF:

pp. 84-85

4. A nurse has started admitting a new person on home care and is beginning to establish a relationship. Which of the following would be the most important thing for the nurse to do? a. Complete the paperwork in a timely fashion b. Establish open communication c. Conduct a complete physical health assessment d. Provide feedback to the person’s questions ANS: B Communication is the foundation for any professional relationship. It is the cornerstone of a positive nurseperson relationship. It refers to a set of strategies and actions to enhance reciprocity, mutual understanding, and decision making. DIF:

Cognitive Level: Apply (Application)

REF:

p. 85 | p. 87

5. Which of the following best describes person-centered communication? a. The nurse says to the person seeking care, “Why do you continue to drink when you know it is not good for you?”


b. c. d.

The nurse responds to person seeking care’s request for pain medicine by saying, “It is not time for your pain medicine. You had it hours ago.” The client says to the nurse, “I think my husband is cheating on me.” The nurse responds by saying, “You think your husband is cheating on you?” The client says to the nurse, “I can’t believe I have cancer.” The nurse responds by saying, “One of my closest friends had cancer and had to have chemotherapy.”

ANS: C The nurse recognizes that the person seeking care has some concerns that she would like to discuss and paraphrases her statement to give the person an opportunity to elaborate on thoughts that are important to him or her. DIF:

Cognitive Level: Analyze (Analysis)

REF:

p. 87

6. A nurse is counseling someone who has recently been diagnosed with diabetes. The nurse says, “Tell me more about how this diagnosis has affected your daily activities.” Which of the following best describes the type of communication the nurse is using? a. Verbal communication b. Nonverbal communication c. Metacommunication d. Functional communication ANS: A Verbal communication is the transmission of messages using words, spoken or written. Nonverbal communication encompasses all messages that are not spoken or written. Metacommunication refers to a message about the message, the relationship aspect of communication. DIF:

Cognitive Level: Apply (Application)

REF:

p. 88

7. Which of the following statements about nonverbal communication is true? a. Nonverbal behavior is not usually contextual. b. Nonverbal behavior is culturally and situationally bound. c. Nonverbal communication is the most important type of communication. d. Nonverbal communication is easy to interpret. ANS: B Cultural exposure and situational impact are essential components of interpreting nonverbal communication and must be taken into consideration in attempts to interpret nonverbal communication. DIF:

Cognitive Level: Understand (Comprehension) REF:

p. 89

8. Which of the following scenarios best describes a nurse using metacommunication? a. Using both touch and silence when counseling an individual b. Practicing reflection when interaction with an individual c. Discussing with an individual how to solve a problem d. Understanding that an individual needs a break before proceeding ANS: D Metacommunication refers to a message about the message. It is the relationship aspect of communication. In a sense, it involves reading between the lines or going past the surface content of the message to glean nuances of meaning. DIF:

Cognitive Level: Analyze (Analysis)

REF: p. 89


9. Which of the following statements can be identified as a method for clarifying a message? a. “I get very upset when you talk to me in that tone.” b. “You make me very angry when you drink alcohol.” c. “I can make you happy. I know I can.” d. “What I want from you is to be left alone!” ANS: A Use of “I” statements is one technique that can be used for clarifying and qualifying messages. DIF:

Cognitive Level:

Analyze (Analysis)

REF:

p. 90

10. A nurse and a man are involved in a conversation. When the person folds his arms across his chest, the nurse almost simultaneously folds her arms across her chest. This behavior is recognized as: a. empathy. b. reciprocity. c. flexibility. d. spacing. ANS: B The patterning of similar activities within the same interval by two people can facilitate communication. This patterning is known as reciprocity. DIF:

Cognitive Level: Apply (Application)

REF:

pp. 90-91

11. A nurse and her client are engaged in meaningful conversation when suddenly there is silence between the two. To facilitate effective client-centered communication, the nurse should: a. ask the person what he is thinking, so they can understand each other. b. restate what the person said before silence ensued, to get clarification. c. change the subject to one that is more pleasant for the person to discuss. d. wait quietly to give the person time to reflect where he wants to lead the conversation. ANS: D Silence allows the person to reflect on what is being discussed or experienced and lets him or her know that the nurse is willing to wait until he or she is ready to say more. DIF:

Cognitive Level: Apply (Application)

REF:

p. 91

12. A person has just been admitted to the hospital. In talking with the person, the nurse is able to elicit from her that the reason for her hospitalization is that her husband beat her up. Which characteristic of the therapeutic relationship is being demonstrated? a. Empathy b. Trust c. Rapport d. Purposeful communication ANS: D The nurse focuses communication for a particular aim: to obtain information to be used in care of the person. DIF:

Cognitive Level: Apply (Application)

REF:

p. 93 13. An individual reveals to the nurse that he is having an affair with his wife’s best friend. Which of the following statements will enhance effective therapeutic communication between him and the nurse? a. “You know that what you are doing is wrong.” b. “Why would you want do something like that?” c. “How could you do that to your wife and children?”


d.

“This affair seems to be troubling you.”

ANS: D The nurse is seeking to determine the client’s own feelings about his affair. By seeking feedback, the nurse helps explain the meaning further. The nurse suspends judgment and responds in such a way as to encourage him to keep talking rather than make judgments about his behavior, which would shut down communication. DIF:

Cognitive Level: Apply (Application)

REF:

p. 95

14. A nurse is working with a person to increase his daily exercise and activity. Which of the following statements by the nurse best demonstrates the use of reflection? a. “I have also struggled with finding time to exercise on a regular basis.” b. “Regular exercise plays an important role in preventing many health problems.” c. “You said that you have difficulty making time in your day for exercise.” d. “It may be helpful if we developed a goal related to daily exercise. “ ANS: C Reflection is the restatement of what the individual has said in the same or different words. This technique can involve paraphrasing or summarizing the person’s main point to indicate interest and to focus the discussion. It is the selective paraphrasing or literal repetition of the person’s words to underscore the importance of what has been said, summarize a main concern or theme, or elicit elaborated information. DIF:

Cognitive Level: Analyze (Analysis)

REF:

p. 95

15. A woman tells the nurse that she is very nervous about their meeting today. Which of the following relationship stages are the nurse and woman most likely experiencing? a. Orientation phase b. Working phase c. Therapeutic phase d. Termination phase ANS: A The orientation phase begins when the nurse and individual meet. This meeting typically involves some feeling of anxiety because neither party knows what to expect. DIF:

Cognitive Level: Apply (Application)

REF:

p. 98

16. Which of the following is the first step in the valuing process? a. Acting out the choice b. Selecting from alternatives c. Being happy with personal beliefs and actions d. Choosing freely ANS: D The seven-step valuing process involves (1) choosing freely, (2) choosing from alternatives, (3) choosing after careful consideration of potential outcomes of each alternative, (4) cherishing and being happy with personal beliefs and actions, (5) affirming the choice in public, when appropriate, (6) acting out the choice, and (7) repeatedly acting in some type of pattern. DIF:

Cognitive Level: Remember (Knowledge)

REF:

p. 83 (Box 4-2)

17. The nurse makes sure that the distance between himself and the client is at least 6 feet before he begins to ask questions related to the client’s health history. Which of the following statements is true? a. This is the ideal space for intimate communication. b. This distance is too far for the nurse to build a therapeutic relationship while obtaining the


c. d.

information. This is the recommended distance between client and nurse for effective therapeutic communication. The nurse should position himself an additional foot away to facilitate the conversation.

ANS: B Personal space of 18 inches to 4 feet is appropriate for close relationships in which touching may be involved and good visualization is desired. DIF:

Cognitive Level: Apply (Application)

REF:

p. 91 (Box 4-8)

MULTIPLE RESPONSE 1. A nurse is working on values clarification with a person who has chosen to smoke during her pregnancy. Which of the following statements would be the most appropriate for the nurse to assist in this process? (select all that apply) a. “Why do you smoke cigarettes? b. “What do you value most in life?” c. “Who has influenced your decision to smoke?” d. “Is your decision to smoke consistent with your values?” ANS: B, D Techniques to clarify values include assisting the person to identify her own values (“What is important to you?”), identify values conflicts or conflicts between values and actions (“Are your actions consistent with your values?”), and using reflection to restate the value and make it explicit. DIF:

Cognitive Level: Analyze (Analysis)

REF:

p. 83 (Box 4-3)

2. A nurse is in the process of establishing a therapeutic relationship with a person. Which of the following techniques should be used by the nurse? (select all that apply) a. Learn about the person’s interests. b. Sit close to the person. c. Show interest in the person’s concerns. d. Define the parameters of the relationship with the person. ANS: C, D Purposeful communication should be an aim of the therapeutic relationship. Social chitchat, communication without a goal, should not make up the bulk of the therapeutic interaction. The amount of space between communicators varies from culture to culture, so sitting close to the person may not be culturally appropriate. Building rapport by showing the person that his or her concerns interest the nurse is important. Also, trust can be built with the person by clearly defining the relationship parameters and expectations. DIF:

Cognitive Level: Analyze (Analysis)

REF:

p. 93

3. Which of the following is an essential question that nurses should encourage individuals to ask at every health visit? (select all that apply) a. “What is my main problem?” b. “What health information do I not understand?” c. “What do I need to do?” d. “What is the plan for the next visit?” ANS: A, C


Health literacy is the capacity to read, comprehend, and follow through on health information; it is a critical component of health promotion. To combat low health literacy, nurses can encourage individuals to ask three essential questions at every health visit: “What is my main problem?” “What do I need to do?” “Why is it important for me to do this?” DIF:

Cognitive Level: Remember (Knowledge)

REF:

p. 100

Chapter 05: Ethical Issues Related to Health Promotion Edelman: Health Promotion Throughout the Life Span, 8th Edition MULTIPLE CHOICE 1. Which of the following statements is true with regard to health promotion? a. Health promotion efforts are not concerned with addressing environmental obstacles to health. b. Advocates of health promotion are not involved in political campaigns against harmful products. c. The business of eliminating health disparities is a unique function of Healthy People 2010 goals. d. Health promotion involves collaboration of many professional groups. ANS: D Health promotion is not the province of a single discipline but involves individuals, health care providers, and institutions working together to create a positive environment for health and to achieve health goals. DIF:

Cognitive Level: Understand (Comprehension) REF:

p. 104

2. Which of the following types of ethical theories tells us how people act toward each other and their environments and what they believe are good or moral actions? a. Descriptive value theories b. Normative theories c. Consequentialism theory d. Duty-based theories ANS: A Descriptive theories do not tell us what actions we ought to take. They are not directive; they tell us how people act toward each other and their environments and what they seem to believe are good or moral actions. DIF:

Cognitive Level: Remember (Knowledge)

REF:

p. 105

3. A nurse includes the statement, “Treat others the way you would like to be treated.” when teaching a group of adolescents about bullying. Which type of ethical theory is being used in this example? a. Descriptive value theory b. Normative theory c. Consequentialism theory d. Duty-based theory ANS: B Normative theories are concerned with ensuring good actions. They are reasoned explanations of the moral purpose of human interactions, or they are divinely “revealed” truths about good action (religious ethics). DIF:

Cognitive Level: Apply (Application)

REF:

p. 105

4. A nurse believes that it is his responsibility to ensure the safety of the public by administering flu shots to everyone in the community. Which ethical theory is being used by the nurse? a. Descriptive value theory b. Normative theory


c. d.

Consequentialism theory Duty-based theory

ANS: C Consequentialism theory proposes that actions are good insofar as they are aimed at yielding the greatest amount of happiness or pleasure or cause the least amount of harm or pain to individuals and overall in society. Administration of flu vaccines proposes the best result for individuals and decreased risk for others in the community. DIF:

Cognitive Level: Apply (Application)

REF:

pp. 105-106

5. Which of the following statements best describes the application of moral or ethical theory to the practices involved in health promotion? a. One can be assured of morally correct action in a given situation if one adheres to a theory of moral principles. b. There are no identifiable criteria that can be used effectively to determine correct actions in a given situation. c. The good of the larger population always takes precedence over the good of one individual. d. Making ethical decisions regarding human health involves the consideration of multiple factors. ANS: D Making moral or ethical decisions about human health is not an easy task and requires assessing various criteria or theories. DIF:

Cognitive Level: Apply (Application)

REF:

p. 106

6. An emergency department provides care for all individuals regardless of circumstances and ability to pay. This is an example of: a. ethic of justice. b. ethic of care. c. applied ethics. d. metaethics. ANS: A The ethic of justice is impartial and nondiscriminatory. An ethic of care, however, requires an understanding of situational particularities, ensuring that we try to understand a given individual’s needs in the context of his or her life. DIF:

Cognitive Level: Apply (Application)

REF:

p. 108

7. A nurse uses ethical reasoning to solve an ethical dilemma. Why would the nurse use this process? a. Assists in predicting all of the possible consequences of future actions b. Facilitates gathering of the most important information to solve the problem c. Considers the importance of caring when solving the problem d. Highlights salient aspects of future actions ANS: B The purpose of ethical inquiry is to gain clarity on actual or potential moral issues arising in the context of health-promotion endeavors and to understand what is expected of the health-promotion agent viewed as a moral agent. Ethical reasoning can facilitate appropriate and in-depth data gathering, permit the uncovering of hidden agendas and interests, and focus on the most salient aspects of a particular problem. DIF:

Cognitive Level: Apply (Application)

REF:

p. 109


8. A nurse whose religious beliefs prohibit abortion has been asked to participate in a termination of the pregnancy of a 16-year-old victim of date rape. According to the revised American Nurses Association’s (ANA’s) Code of Ethics for Nurses, which of the following actions should the nurse take next? a. Suspend her religious beliefs and provide comfort and support for the client. b. Quit her job and find another source of employment. c. Notify her supervisor about her beliefs and request a change in assignment. d. Refuse to participate in the abortion. ANS: C In the immediacy of the situation, the ANA Code of Ethics provides for the right of nurses to refuse to participate in procedures that violate their own values, but the nurse’s ethical behavior must ensure that arrangements for the care of the client are provided. DIF:

Cognitive Level: Analyze (Analysis)

REF: pp. 109-110

9. A liver for which two people are tissue-typed has become available after the death of a donor in a car accident. Client A is a 45-year-old substance abuser whose liver is damaged as a result of his use of alcohol. Client B is a 16-year-old adolescent in need of a liver transplant because of a birth abnormality. Which of the following would be useful for a nurse in giving input as to which of the two should receive the liver? a. Advocate for Client B because he is the younger of the two and will live longer. b. Use feminist moral theory to advocate for Client A in spite of his alcohol use. c. Use a guided set of moral principles in decision-making before advocating for either. d. Advocate for Client A because he has 15 to 20 years of productive life left. ANS: C Systematically using a set of moral principles in making ethical decisions assists the nurse in resolving ethical dilemmas such as that described. DIF:

Cognitive Level: Analyze (Analysis)

REF:

pp. 110-111

10. Which concept is aimed at interrupting potential ethical problems before they develop? a. Feminist ethics b. Preventive ethics c. Metaethics d. Normative ethics ANS: B Preventive ethics is a requirement of health promotion in which practitioners envision potential problems and institute actions that stop their development. DIF:

Cognitive Level: Remember (Knowledge)

REF:

p. 111

11. When providing preoperative teaching for a client who will be undergoing total knee arthroplasty, the nurse asks the client questions to assess his understanding of the surgery. Which type of ethics is the nurse implementing? a. Duty-based ethics b. Normative ethics c. Metaethics d. Preventive ethics ANS: D Preventive ethics aims to forestall ethical problems before they develop. Preventive ethics is an important requirement of health-promotion endeavors that includes individual action by the nurse, as well as social and political activism with other nurses or professional nursing organizations. Assessing the client’s understanding of what is going to happen during a surgical procedure before the client signs the consent for the procedure makes sure the client understands the information and should help prevent ethical problems.


DIF:

Cognitive Level: Apply (Application)

REF:

p. 111

12. A 24-year-old man with severe cognitive impairment, and no relatives participating in his care, needs to make a decision about removal of a brain tumor that will render him totally dependent for completion of all activities of daily living. The nurse participates in ethical decision making on his behalf, knowing that the client is unable to benefit from which of the following ethical concepts? a. Autonomy b. Justice c. Beneficence d. Advocacy ANS: A Because the client is cognitively delayed and is unable to understand explanations of treatment that would be given to him, the client is unable to be autonomous in making a decision regarding neurosurgery. DIF:

Cognitive Level: Apply (Application)

REF:

pp. 113-115

13. A client is scheduled to have open-heart surgery. His physical condition is such that he is at high risk to experience complications during the procedure, but he has not been advised of this possibility. Which of the following ethical concepts has been ignored in rendering care for this client? a. Beneficence b. Autonomy c. Justice d. Informed consent ANS: D The client is able to “substantially understand” his care. Ethical care ensures that a person has all of the appropriate information necessary to make an autonomous decision about his or her care. This client has not been informed of his chances of dying during the surgery. DIF:

Cognitive Level: Apply (Application)

REF:

p. 114

14. An 18-year-old woman in whom a sexually transmitted disease (STD) was recently diagnosed asks the nurse not to tell her mother that she has an STD. Her mother asks the nurse what is causing her daughter’s vaginal discharge. Which of the following actions should the nurse take? a. Follow the principle of veracity and tell the mother the diagnosis. b. Respect the principle of confidentiality and support the client’s request not to tell her mother the diagnosis. c. Tell the client’s mother that she has a urinary tract infection, to protect the client’s privacy and honor the mother’s request. d. Ignore the mother’s request for information. ANS: B The Health Insurance Portability and Accountability Act (HIPAA) guarantees client confidentiality. The client is the only person who can inform her mother of the diagnosis and the only person who can give the nurse permission to inform her mother of the diagnosis. The nurse could appropriately respond to the mother by saying, “I cannot share that information with you because of client confidentiality legislation. You might ask your daughter to share that information with you.” DIF:

Cognitive Level: Analyze (Analysis)

REF:

p. 117

15. A nurse is providing care to a 15-year old female who has recently been diagnosed with a sexually transmitted infection (STI). Which of the following actions should the nurse take next? a. Seek advice from an ethics expert. b. Facilitate interaction between the adolescent and her parents/guardians.


c. d.

Report the STI to the appropriate authorities. Contact the adolescent’s parents/guardians.

ANS: B Federal and state laws generally serve to protect the privacy and autonomy of adolescents. Responsibilities of the nurse include helping an adolescent to grasp his or her authentic options and rights, facilitating interaction between the adolescent and parents or guardians, maintaining trust, and preserving confidentiality. DIF:

Cognitive Level: Apply (Application)

REF:

p. 117

16. An elderly client has recently been diagnosed with cancer. The client’s family has asked the nurse to withhold this information from the client because they feel that this information would cause the client to give up on life and become very depressed. The nurse believes the client should be told this information. Which ethical principle is being upheld by the nurse? a. Justice b. Beneficence c. Veracity d. Nonmaleficence ANS: C Veracity is also known as devotion to the truth. Veracity in giving people information about their health care needs facilitates autonomous choice and enhances personal decision-making. Withholding information, or providing information that is misleading or incomprehensible, in an attempt to influence someone to agree to a treatment or intervention, conflicts with veracity. DIF:

Cognitive Level: Apply (Application)

REF:

p. 118

17. Which of the following concepts of ethics is described as the “duty to maximize the benefits of actions while minimizing harm”? a. Advocacy b. Justice c. Beneficence d. Autonomy ANS: C Beneficence governs actions taken to further the overall health or well-being of an individual or society. “Beneficence” means doing good. DIF:

Cognitive Level: Remember (Knowledge)

REF:

p. 118

18. A nurse reflects on previous experiences with minority populations and considers how these interactions have influenced her present care to these populations. Which part of values clarification and reflection has the nurse used? a. Formulate a possible course of action. b. Examine the influence of beliefs. c. Reflect on practice. d. Determine the prevalent values. ANS: B It is important for the nurse to think about the influence that beliefs and values have on his or her practice. An understanding of how personal beliefs and values are either congruent, or are liable to interfere, with the task at hand is crucial to ethical problem solving. DIF:

Cognitive Level: Apply (Application)

REF:

p. 121


MULTIPLE RESPONSE 1. A nurse is working with a community to increase its awareness about the dangers of lead poisoning. Which of the following provides the best explanation as to why the nurse is performing this action? (select all that apply) a. It is the right thing to do. b. It is a moral responsibility. c. It is an ethical responsibility. d. It is the role of the nurse. ANS: B, C When nurses provide service to society through health promotion interventions, their care for clients can be seen as a moral endeavor. Moral issues are confronted in the process of attempting to enhance the well-being of a society overall, as well as promoting and protecting health for individual members of a society. The ANA Code of Ethics for Nurses Interpretive Statements promises that “nursing encompasses the prevention of illness, the alleviation of suffering, and the protection, promotion, and restoration of health in the care of individuals, families, groups and communities.” The terms ethical and moral are used interchangeably throughout the chapter. DIF:

Cognitive Level: Apply (Application)

REF:

p. 104

2. A nurse is using feminist ethics when planning a program for women to promote screening and early detection of breast cancer. Which of the following considerations would be made by the nurse? (select all that apply) a. Importance of relationships b. Understanding of the oppression of women c. Importance of responsibility when caring for others d. Understanding of the imbalance of power ANS: B, C Characteristics of feminist ethics include: understanding that human beings are inseparable from their relationships with others; a focus on care and responsibility aspects of relationships, rather than application of abstract principles; a concern with the development of character and attitudes that result in caring actions reflective of a person who is related to rather than detached from context; and a concern for the rights and equality of all individuals that is not limited to the oppression of women. DIF:

Cognitive Level: Apply (Application)

REF:

p. 108

Chapter 05: Ethical Issues Related to Health Promotion Edelman: Health Promotion Throughout the Life Span, 8th Edition MULTIPLE CHOICE 1. Which of the following statements is true with regard to health promotion? a. Health promotion efforts are not concerned with addressing environmental obstacles to health. b. Advocates of health promotion are not involved in political campaigns against harmful products. c. The business of eliminating health disparities is a unique function of Healthy People 2010 goals. d. Health promotion involves collaboration of many professional groups. ANS: D Health promotion is not the province of a single discipline but involves individuals, health care providers, and institutions working together to create a positive environment for health and to achieve health goals. DIF:

Cognitive Level: Understand (Comprehension) REF:

p. 104


2. Which of the following types of ethical theories tells us how people act toward each other and their environments and what they believe are good or moral actions? a. Descriptive value theories b. Normative theories c. Consequentialism theory d. Duty-based theories ANS: A Descriptive theories do not tell us what actions we ought to take. They are not directive; they tell us how people act toward each other and their environments and what they seem to believe are good or moral actions. DIF:

Cognitive Level: Remember (Knowledge)

REF:

p. 105

3. A nurse includes the statement, “Treat others the way you would like to be treated.” when teaching a group of adolescents about bullying. Which type of ethical theory is being used in this example? a. Descriptive value theory b. Normative theory c. Consequentialism theory d. Duty-based theory ANS: B Normative theories are concerned with ensuring good actions. They are reasoned explanations of the moral purpose of human interactions, or they are divinely “revealed” truths about good action (religious ethics). DIF:

Cognitive Level: Apply (Application)

REF:

p. 105

4. A nurse believes that it is his responsibility to ensure the safety of the public by administering flu shots to everyone in the community. Which ethical theory is being used by the nurse? a. Descriptive value theory b. Normative theory c. Consequentialism theory d. Duty-based theory ANS: C Consequentialism theory proposes that actions are good insofar as they are aimed at yielding the greatest amount of happiness or pleasure or cause the least amount of harm or pain to individuals and overall in society. Administration of flu vaccines proposes the best result for individuals and decreased risk for others in the community. DIF:

Cognitive Level: Apply (Application)

REF:

pp. 105-106

5. Which of the following statements best describes the application of moral or ethical theory to the practices involved in health promotion? a. One can be assured of morally correct action in a given situation if one adheres to a theory of moral principles. b. There are no identifiable criteria that can be used effectively to determine correct actions in a given situation. c. The good of the larger population always takes precedence over the good of one individual. d. Making ethical decisions regarding human health involves the consideration of multiple factors. ANS: D Making moral or ethical decisions about human health is not an easy task and requires assessing various criteria or theories. DIF:

Cognitive Level: Apply (Application)

REF:

p. 106


6. An emergency department provides care for all individuals regardless of circumstances and ability to pay. This is an example of: a. ethic of justice. b. ethic of care. c. applied ethics. d. metaethics. ANS: A The ethic of justice is impartial and nondiscriminatory. An ethic of care, however, requires an understanding of situational particularities, ensuring that we try to understand a given individual’s needs in the context of his or her life. DIF:

Cognitive Level: Apply (Application)

REF:

p. 108

7. A nurse uses ethical reasoning to solve an ethical dilemma. Why would the nurse use this process? a. Assists in predicting all of the possible consequences of future actions b. Facilitates gathering of the most important information to solve the problem c. Considers the importance of caring when solving the problem d. Highlights salient aspects of future actions ANS: B The purpose of ethical inquiry is to gain clarity on actual or potential moral issues arising in the context of health-promotion endeavors and to understand what is expected of the health-promotion agent viewed as a moral agent. Ethical reasoning can facilitate appropriate and in-depth data gathering, permit the uncovering of hidden agendas and interests, and focus on the most salient aspects of a particular problem. DIF:

Cognitive Level: Apply (Application)

REF:

p. 109

8. A nurse whose religious beliefs prohibit abortion has been asked to participate in a termination of the pregnancy of a 16-year-old victim of date rape. According to the revised American Nurses Association’s (ANA’s) Code of Ethics for Nurses, which of the following actions should the nurse take next? a. Suspend her religious beliefs and provide comfort and support for the client. b. Quit her job and find another source of employment. c. Notify her supervisor about her beliefs and request a change in assignment. d. Refuse to participate in the abortion. ANS: C In the immediacy of the situation, the ANA Code of Ethics provides for the right of nurses to refuse to participate in procedures that violate their own values, but the nurse’s ethical behavior must ensure that arrangements for the care of the client are provided. DIF:

Cognitive Level: Analyze (Analysis)

REF: pp. 109-110

9. A liver for which two people are tissue-typed has become available after the death of a donor in a car accident. Client A is a 45-year-old substance abuser whose liver is damaged as a result of his use of alcohol. Client B is a 16-year-old adolescent in need of a liver transplant because of a birth abnormality. Which of the following would be useful for a nurse in giving input as to which of the two should receive the liver? a. Advocate for Client B because he is the younger of the two and will live longer. b. Use feminist moral theory to advocate for Client A in spite of his alcohol use. c. Use a guided set of moral principles in decision-making before advocating for either. d. Advocate for Client A because he has 15 to 20 years of productive life left. ANS: C


Systematically using a set of moral principles in making ethical decisions assists the nurse in resolving ethical dilemmas such as that described. DIF:

Cognitive Level: Analyze (Analysis)

REF:

pp. 110-111

10. Which concept is aimed at interrupting potential ethical problems before they develop? a. Feminist ethics b. Preventive ethics c. Metaethics d. Normative ethics ANS: B Preventive ethics is a requirement of health promotion in which practitioners envision potential problems and institute actions that stop their development. DIF:

Cognitive Level: Remember (Knowledge)

REF:

p. 111

11. When providing preoperative teaching for a client who will be undergoing total knee arthroplasty, the nurse asks the client questions to assess his understanding of the surgery. Which type of ethics is the nurse implementing? a. Duty-based ethics b. Normative ethics c. Metaethics d. Preventive ethics ANS: D Preventive ethics aims to forestall ethical problems before they develop. Preventive ethics is an important requirement of health-promotion endeavors that includes individual action by the nurse, as well as social and political activism with other nurses or professional nursing organizations. Assessing the client’s understanding of what is going to happen during a surgical procedure before the client signs the consent for the procedure makes sure the client understands the information and should help prevent ethical problems. DIF:

Cognitive Level: Apply (Application)

REF:

p. 111

12. A 24-year-old man with severe cognitive impairment, and no relatives participating in his care, needs to make a decision about removal of a brain tumor that will render him totally dependent for completion of all activities of daily living. The nurse participates in ethical decision making on his behalf, knowing that the client is unable to benefit from which of the following ethical concepts? a. Autonomy b. Justice c. Beneficence d. Advocacy ANS: A Because the client is cognitively delayed and is unable to understand explanations of treatment that would be given to him, the client is unable to be autonomous in making a decision regarding neurosurgery. DIF:

Cognitive Level: Apply (Application)

REF:

pp. 113-115

13. A client is scheduled to have open-heart surgery. His physical condition is such that he is at high risk to experience complications during the procedure, but he has not been advised of this possibility. Which of the following ethical concepts has been ignored in rendering care for this client? a. Beneficence b. Autonomy c. Justice d. Informed consent


ANS: D The client is able to “substantially understand” his care. Ethical care ensures that a person has all of the appropriate information necessary to make an autonomous decision about his or her care. This client has not been informed of his chances of dying during the surgery. DIF:

Cognitive Level: Apply (Application)

REF:

p. 114

14. An 18-year-old woman in whom a sexually transmitted disease (STD) was recently diagnosed asks the nurse not to tell her mother that she has an STD. Her mother asks the nurse what is causing her daughter’s vaginal discharge. Which of the following actions should the nurse take? a. Follow the principle of veracity and tell the mother the diagnosis. b. Respect the principle of confidentiality and support the client’s request not to tell her mother the diagnosis. c. Tell the client’s mother that she has a urinary tract infection, to protect the client’s privacy and honor the mother’s request. d. Ignore the mother’s request for information. ANS: B The Health Insurance Portability and Accountability Act (HIPAA) guarantees client confidentiality. The client is the only person who can inform her mother of the diagnosis and the only person who can give the nurse permission to inform her mother of the diagnosis. The nurse could appropriately respond to the mother by saying, “I cannot share that information with you because of client confidentiality legislation. You might ask your daughter to share that information with you.” DIF:

Cognitive Level: Analyze (Analysis)

REF:

p. 117

15. A nurse is providing care to a 15-year old female who has recently been diagnosed with a sexually transmitted infection (STI). Which of the following actions should the nurse take next? a. Seek advice from an ethics expert. b. Facilitate interaction between the adolescent and her parents/guardians. c. Report the STI to the appropriate authorities. d. Contact the adolescent’s parents/guardians. ANS: B Federal and state laws generally serve to protect the privacy and autonomy of adolescents. Responsibilities of the nurse include helping an adolescent to grasp his or her authentic options and rights, facilitating interaction between the adolescent and parents or guardians, maintaining trust, and preserving confidentiality. DIF:

Cognitive Level: Apply (Application)

REF:

p. 117

16. An elderly client has recently been diagnosed with cancer. The client’s family has asked the nurse to withhold this information from the client because they feel that this information would cause the client to give up on life and become very depressed. The nurse believes the client should be told this information. Which ethical principle is being upheld by the nurse? a. Justice b. Beneficence c. Veracity d. Nonmaleficence ANS: C Veracity is also known as devotion to the truth. Veracity in giving people information about their health care needs facilitates autonomous choice and enhances personal decision-making. Withholding information, or providing information that is misleading or incomprehensible, in an attempt to influence someone to agree to a treatment or intervention, conflicts with veracity.


DIF:

Cognitive Level: Apply (Application)

REF:

p. 118

17. Which of the following concepts of ethics is described as the “duty to maximize the benefits of actions while minimizing harm”? a. Advocacy b. Justice c. Beneficence d. Autonomy ANS: C Beneficence governs actions taken to further the overall health or well-being of an individual or society. “Beneficence” means doing good. DIF:

Cognitive Level: Remember (Knowledge)

REF:

p. 118

18. A nurse reflects on previous experiences with minority populations and considers how these interactions have influenced her present care to these populations. Which part of values clarification and reflection has the nurse used? a. Formulate a possible course of action. b. Examine the influence of beliefs. c. Reflect on practice. d. Determine the prevalent values. ANS: B It is important for the nurse to think about the influence that beliefs and values have on his or her practice. An understanding of how personal beliefs and values are either congruent, or are liable to interfere, with the task at hand is crucial to ethical problem solving. DIF:

Cognitive Level: Apply (Application)

REF:

p. 121

MULTIPLE RESPONSE 1. A nurse is working with a community to increase its awareness about the dangers of lead poisoning. Which of the following provides the best explanation as to why the nurse is performing this action? (select all that apply) a. It is the right thing to do. b. It is a moral responsibility. c. It is an ethical responsibility. d. It is the role of the nurse. ANS: B, C When nurses provide service to society through health promotion interventions, their care for clients can be seen as a moral endeavor. Moral issues are confronted in the process of attempting to enhance the well-being of a society overall, as well as promoting and protecting health for individual members of a society. The ANA Code of Ethics for Nurses Interpretive Statements promises that “nursing encompasses the prevention of illness, the alleviation of suffering, and the protection, promotion, and restoration of health in the care of individuals, families, groups and communities.” The terms ethical and moral are used interchangeably throughout the chapter. DIF:

Cognitive Level: Apply (Application)

REF:

p. 104

2. A nurse is using feminist ethics when planning a program for women to promote screening and early detection of breast cancer. Which of the following considerations would be made by the nurse? (select all that apply) a. Importance of relationships b. Understanding of the oppression of women c. Importance of responsibility when caring for others d. Understanding of the imbalance of power


ANS: B, C Characteristics of feminist ethics include: understanding that human beings are inseparable from their relationships with others; a focus on care and responsibility aspects of relationships, rather than application of abstract principles; a concern with the development of character and attitudes that result in caring actions reflective of a person who is related to rather than detached from context; and a concern for the rights and equality of all individuals that is not limited to the oppression of women. DIF:

Cognitive Level: Apply (Application)

REF:

p. 108

Chapter 06: Health Promotion and the Individual Edelman: Health Promotion Throughout the Life Span, 8th Edition MULTIPLE CHOICE 1. Healthy People 2020 objectives provide a framework for: a. assessment. b. diagnosis. c. prevention. d. treatment. ANS: C The health promotion initiative named Healthy People 2010 provides a framework for prevention. DIF:

Cognitive Level:

Remember (Knowledge)

REF:

p. 128

2. Which of the following best describes a primary prevention method for colon cancer? a. Hemoccult testing b. High fiber diet c. Colonoscopy d. Laparoscopy ANS: B Primary prevention includes generalized health promotion and specific protection from disease. Hemoccult and colonoscopy are forms of screening, not prevention. Eating a healthy diet high in fiber is a preventive measure. DIF:

Cognitive Level: Apply (Application)

REF:

pp. 128-129

3. Who authored the framework which provides the foundation for nursing assessment and diagnosis using the functional health patterns? a. Erikson b. Gordon c. Newman d. Nightingale ANS: B Gordon’s framework provides the foundation for most NANDA nursing diagnoses using the functional health pattern. Nurses use the framework to combine assessment skills with subjective and objective data to construct patterns. DIF:

Cognitive Level: Remember (Knowledge)

REF:

p. 129


4. Over the last week, a person has had finger stick glucose levels of 127, 132, 140, 138, 143, 145, and 140. This information allows the nurse to characterize the person’s function pattern by utilizing which area of focus? a. Age-developmental b. Functional c. Individual-environmental d. Pattern ANS: D Pattern focus implies that the nurse explores patterns or sequences of behavior over time. Pattern recognition occurs during information collection. Functional health patterns then provide structure to analyze factors. DIF:

Cognitive Level: Apply (Application)

REF:

p. 130

5. A nurse is using a functional focus to assess a person. Which of the following the nurse be evaluating? a. Visual acuity b. Pupil reactivity c. Ability to drive d. The red reflex ANS: C Functional focus refers to the individual’s performance level. Nurses assess how particular visual patterns affect lifestyle. The ability to drive would affect a person’s lifestyle and might require a change in how the person functions. DIF:

Cognitive Level: Analyze (Analysis)

REF:

p. 131

6. A nurse working with a Hispanic family is explaining the plan for managing a child’s asthma to the child’s mother, father, and grandmother. To whom should the nurse direct the education? a. Mother b. Father c. Grandmother d. Parents and grandmother ANS: D Culturally competent care is delivered with understanding of and sensitivity to cultural factors influencing health behaviors. Nurses provide culturally competent care when they identify and use cultural norms and values. In the Hispanic population, the male figure is usually the decision maker, and the family elders are highly respected. However, assumptions about cultural norms should not be made. As a result, the nurse should direct education to all three adults because they may all have an impact on the child’s health care needs. DIF:

Cognitive Level: Analyze (Analysis)

7. A 27-year-old woman has not received a Pap test in which functional pattern? a. Health-perception–health-management pattern b. Elimination pattern c. Activity-rest pattern d. Self-perception–self-concept pattern

REF: p. 131 years. This assessment finding identifies an alteration in

ANS: A Assessment objectives for the health-perception–health-management pattern consist of obtaining data about perceptions, management, and preventive health practices. Exploring these values identifies potential health hazards. A 27-year-old woman should receive a Pap test every 2 years. Failing to do so could place her at risk for health problems; thus, this finding identifies an alteration in the health-perception–health-management pattern.


DIF:

Cognitive Level: Apply (Application)

REF:

p. 131

8. A client is experiencing an alteration in the health-perception–health-management pattern and an alteration in the values-beliefs pattern. Which of the following best describes the behavior of this person? a. Never sees a physician b. Only sees a physician if not feeling well c. Sees a physician for screenings only d. Sees a physician for follow-up care of a chronic disease ANS: B Health beliefs and perceptions directly affect participation in care. Dimensions of assessment in the valuesbeliefs pattern include the individual’s values, beliefs, or goals that guide choices or decisions that are related to health. People who do not believe in health promotion activities will likely see a physician only when sick. Thus, someone with an alteration in the health-perception–health-management and values-beliefs patterns will likely only see a physician if not feeling well. DIF:

Cognitive Level: Apply (Application)

REF:

p. 131 | p. 142

9. When assessing a person’s nutritional-metabolic pattern, which objective finding would have implications for nursing intervention? a. The person’s 24-hour diet diary b. The person’s dentition c. The person’s food preferences d. The person’s financial status ANS: B Although all of the assessment parameters listed have implications for nursing diagnosis and planning for this client, the only objective measure is the client’s dentition. It is the only one that can be validated with a physical exam. DIF:

Cognitive Level: Apply (Application)

REF:

p. 134

10. When assessing a client’s activity-exercise pattern, which subjective finding has implications for nursing practice? a. A person’s decreased muscle tone b. A person’s amount of leisure time c. A person’s decreased range of motion d. A person’s use of a cane ANS: B Although all findings are important in assessing the activity-exercise pattern, the only subjective finding is the amount of leisure time that the person reports having. All others are objective findings and can be validated with a physical exam. DIF:

Cognitive Level: Apply (Application)

REF:

pp. 135-136

11. During a health history, a person reports getting 5 hours of sleep a night. What does this information indicate to the nurse? a. The person is not receiving enough sleep. b. The person is receiving adequate sleep. c. The nurse must determine where the person sleeps. d. The nurse must ask additional questions. ANS: D


The single most important factor assessed in the sleep-rest pattern is probably the perception of adequacy of sleep and relaxation. The objective when assessing the sleep-rest pattern is to describe the effectiveness of the pattern from the person’s perspective. Wide variation in sleep time does not necessarily affect functional performance. Different individuals require different amounts of sleep. Thus, without further subjective data, the nurse is not able to make a diagnosis in this functional pattern. DIF:

Cognitive Level: Apply (Application)

REF:

p. 136

12. A nurse assesses the cognitive-perceptual pattern of a Type 1 diabetic client. Which finding has implications for the individual’s nursing plan of care? a. Decreased sense of hearing b. Decreased sense of smell c. Decreased sense of taste d. Decreased visual acuity ANS: D Assessment parameters in the cognitive-perceptual pattern include hearing, vision, smell, and taste. A person with Type 1 diabetes mellitus requires insulin injections. A decrease in visual acuity will make it difficult for the individual to draw up his or her medication and therefore will influence the nurse’s plan of care. DIF:

Cognitive Level: Analyze (Analysis)

REF:

p. 137

13. Which scenario indicates a potentially dysfunctional pattern? a. Adult with frequent urination b. Woman who lost her job c. Elderly person with blurred vision d. Overweight adult with a sweet tooth ANS: B A pattern is potentially dysfunctional when sufficient evidence exists or enough risk factors are present to indicate that a pattern of dysfunction will likely occur if interventions are not instituted. A dysfunctional pattern is a problem when it represents a deviation from established norms or from the individual’s previous condition or goal. The woman who lost her job indicates a potential dysfunction pattern because the stress of losing her job places her at risk for ineffective coping. The other scenarios are not potentially dysfunctional; by definition, they are dysfunctional. DIF:

Cognitive Level: Apply (Application)

REF:

p. 144

14. Which scenario represents a dysfunctional pattern? a. Sexually active teenager who does not use condoms b. Salesman who sleeps only 5 hours a night c. Single mother of three children d. Woman with a small extended family ANS: A A pattern is potentially dysfunctional when sufficient evidence exists or enough risk factors are present to indicate that a pattern of dysfunction will likely occur if interventions are not instituted. A dysfunctional pattern is a problem when it represents a deviation from established norms or the individual’s previous condition or goal. Dysfunctional patterns may be present in the absence of disease, and nursing care may be necessary for health promotion and maintenance. The teenager, although free of disease, is in need of health promotion and disease prevention strategies because her sexual behavior indicates a dysfunction in her sexuality-reproductive pattern that places her at risk for a sexually transmitted disease and pregnancy. DIF:

Cognitive Level: Apply (Application)

REF:

p. 144


15. A nurse is counseling a person with a dysfunctional sleep pattern. Which of the following recommendations would the nurse most likely give the person? a. Read in bed until he falls asleep. b. Avoid fluids after 7 PM. c. Exercise immediately before bedtime. d. Watch television in the recliner in the evening. ANS: B Etiological factors of most dysfunctional patterns often lie within another pattern or patterns. Outcomes and plans are based on probable cause. Exercising before bed, watching television, and reading in bed are not considered appropriate sleep hygiene. Frequent urination may be the cause of his dysfunctional sleep pattern and, if so, avoiding fluids before bed would be an appropriate plan. DIF:

Cognitive Level: Analyze (Analysis)

REF:

p. 144

16. A nurse is caring for a person with a potential dysfunction in the health-perception–health-management pattern. Which of the following nursing interventions would most likely be performed? a. Arranging for home delivery of medication from the pharmacy b. Providing education regarding the dangers of smoking c. Instituting visiting nurse services for blood pressure checks d. Providing direct observed therapy for tuberculosis medications ANS: B Potential problems are risk states. Nursing interventions are directed toward risk reduction through education. Health promotion requires the individual to participate in his own care, and he cannot do this if he does not recognize his susceptibility to an impending health problem. Providing education addresses the risk and provides the person with information needed to change beliefs. The other options make the person a passive participant rather than an active one. DIF:

Cognitive Level: Apply (Application)

REF:

p. 144

17. The nurse has determined that a person has a dysfunction in the nutritional-metabolic pattern. Which action would be the next step for the nurse to take? a. Weigh the person. b. Set a goal weight with the person. c. Ask the person what her favorite foods are.0 d. Develop a plan for weight loss. ANS: B The individual’s goals and the determined diagnosis provide the basis for planning. Before developing a plan, a goal must be set. Clarity of the goals and diagnosis is critical to the development of an effective plan. In this case, the diagnosis has already been established and thus assessment of this pattern has occurred (weight, favorite foods). The next step before developing a plan is to set a goal weight with the client. DIF:

Cognitive Level: Apply (Application)

REF:

p. 145

18. A nurse weighs a person who has been diagnosed with a dysfunction in the nutritional-metabolic pattern. Which aspect of the nursing process is being performed? a. Assessment b. Implementation c. Planning d. Evaluation ANS: D


The nursing process consists of assessment, diagnosis, planning, implementation, and evaluation. A person who has been diagnosed with a dysfunction has already been assessed. The process of analyzing changes experienced by a person after a plan has been implemented occurs in the evaluation phase. In this question, a weight will determine whether or not the person is moving toward her goals of weight loss. DIF:

Cognitive Level: Analyze (Analysis)

REF:

p. 145

19. A nurse administers the T-ACE test to a pregnant woman. The woman’s responses result in a score of 3. This score indicates that the woman: a. requires interventions for problem drinking. b. lacks evidence of problem drinking. c. requires interventions for sexually transmitted disease risks. d. lacks evidence of sexually transmitted disease risks. ANS: A The T-ACE provides a sensitive measure of alcohol-intake pattern in pregnant women. A score of 2 or more indicates evidence of problem drinking. This client had a score of 3, which would require an intervention for problem drinking. DIF:

Cognitive Level: Apply (Application)

REF:

p. 128 (Think About It Box)

20. A Hispanic mother tells the nurse that she has been using home remedies for her child’s asthma. Which home remedy might this mother be using? a. Acupuncture b. Cupping c. Hot tea d. Massage ANS: C In the Hispanic population, asthma is viewed as a cold disease (hot-cold imbalance) and thus is treated with warm therapies. Diet is often used to maintain equilibrium. Thus, warm tea added to the child’s diet might be used to restore equilibrium between hot and cold in this child who has asthma. DIF:

Cognitive Level: Apply (Application)

REF:

p. 133 (Box 6-2)

21. Which classification system fulfills needs that are exclusive to nursing? a. The International Classification of Nursing Practice (ICNP) b. The International Classification of Functioning, Disability, and Health (ICF) c. The International Nursing Diagnoses Classification (NANDA-I) d. The Nursing Diagnostic System (NDS) ANS: C The NANDA-I system includes diagnostic criteria, and related etiologies in addition to the description. The NANDA-I fulfills needs that are exclusive to nursing. DIF:

Cognitive Level: Remember (Knowledge)

REF: p. 144 (Box 6-5)

22. Erikson’s task of autonomy vs. shame and doubt occurs during which stage of development? a. Infancy b. Early childhood c. Late childhood d. Early adolescence ANS: B Erikson’s task of autonomy vs. shame and doubt occurs during early childhood.


DIF:

Cognitive Level: Remember (Knowledge)

REF:

p. 139 (Table 6-3)

23. A young couple is deciding if they should get married and start a family. Which of Erikson’s life stages are they experiencing? a. Identity vs. role confusion b. Intimacy vs. isolation c. Generativity vs. stagnation d. Ego integrity vs. despair ANS: B During early adulthood individuals experience Erikson’s life stage of intimacy vs. isolation. Examples of life events in this stage include committing to a mate and family responsibilities and selecting a career. Identity vs. role confusion occurs during adolescence. Intimacy vs. isolation occurs during middle adulthood. Ego integrity vs. despair occurs during maturity. DIF:

Cognitive Level: Remember (Knowledge)

REF:

p. 139 (Table 6-3)

24. Which cultural group defines illness as a price that is being paid for the past or the future? a. African b. Native American c. Arabian d. Asian ANS: B American Indians define illness as a price that is being paid for the past or the future. DIF:

Cognitive Level: Remember (Knowledge)

REF:

p. 133 (Box 6-2)

25. A man is telling a nurse that he feels that his health is a gift from God. This statement most closely coincides with beliefs of which cultural group? a. African b. Alaska Native c. Asian d. Hispanic ANS: D Hispanics define health as a gift from God. DIF:

Cognitive Level: Apply (Application)

REF:

p. 133 (Box 6-2)

26. Which of the following is the leading cause of death among women? a. Accidents b. Cancer c. Coronary artery disease d. Stroke ANS: C The leading cause of death in women is coronary artery disease. DIF:

Cognitive Level: Remember (Knowledge)

MULTIPLE RESPONSE

REF:

p. 144 (Hot Topics Box)


1. A client who fails to take his insulin on a regular basis may have a conflict in which of the following functional health patterns? (select all that apply) a. Health-perception–health-management b. Cognitive-perceptual c. Elimination d. Values-beliefs ANS: A, B, D A problem in one area serves as a clue to dysfunction in other areas. Cognitive patterns include the ability of the individual to understand and follow directions, retain information, make decisions, solve problems, and use language appropriately. As a result, this person may not understand how to give himself the insulin properly. The values-beliefs pattern describes values including the individual’s spiritual values, beliefs, and goals. This person may not believe in the use of medications unless he is symptomatic. The health-perception–health-management pattern involves the individual’s health status and health practices used to reach the current level of health or wellness, with a focus on perceived health status and meaning of health to the individual. This person may not believe in health promotion and prevention. Thus, a person who fails to take his insulin on a regular basis may have a conflict in the health-perception–health-management, cognitive-perceptual, and values-beliefs patterns. DIF:

Cognitive Level: Apply (Application)

REF:

p. 131 | p. 137 | p. 142

2. Which individual is at risk for a dysfunction in elimination pattern? (select all that apply) a. 46-year-old mother of two b. 32-year-old African American man c. 15-year-old girl d. 72-year-old white woman ANS: A, B, C, D When evaluating elimination patterns, nurses must consider age, developmental level, and cultural considerations. A 46-year-old mother of two is at risk for urinary stress incontinence because of the two vaginal births; an older adult is at risk for urinary control problems; African Americans often have a diet low in fiber, which can lead to constipation; and teenagers, especially girls, may have problems with body image, leading to abuse of laxatives. Thus, all persons listed are at risk for a dysfunction in elimination patterns. DIF:

Cognitive Level: Analyze (Analysis)

REF:

p. 134

Chapter 06: Health Promotion and the Individual Edelman: Health Promotion Throughout the Life Span, 8th Edition MULTIPLE CHOICE 1. Healthy People 2020 objectives provide a framework for: a. assessment. b. diagnosis. c. prevention. d. treatment. ANS: C The health promotion initiative named Healthy People 2010 provides a framework for prevention. DIF:

Cognitive Level:

Remember (Knowledge)

REF:

p. 128

2. Which of the following best describes a primary prevention method for colon cancer? a. Hemoccult testing b. High fiber diet


c. d.

Colonoscopy Laparoscopy

ANS: B Primary prevention includes generalized health promotion and specific protection from disease. Hemoccult and colonoscopy are forms of screening, not prevention. Eating a healthy diet high in fiber is a preventive measure. DIF:

Cognitive Level: Apply (Application)

REF:

pp. 128-129

3. Who authored the framework which provides the foundation for nursing assessment and diagnosis using the functional health patterns? a. Erikson b. Gordon c. Newman d. Nightingale ANS: B Gordon’s framework provides the foundation for most NANDA nursing diagnoses using the functional health pattern. Nurses use the framework to combine assessment skills with subjective and objective data to construct patterns. DIF:

Cognitive Level: Remember (Knowledge)

REF:

p. 129

4. Over the last week, a person has had finger stick glucose levels of 127, 132, 140, 138, 143, 145, and 140. This information allows the nurse to characterize the person’s function pattern by utilizing which area of focus? a. Age-developmental b. Functional c. Individual-environmental d. Pattern ANS: D Pattern focus implies that the nurse explores patterns or sequences of behavior over time. Pattern recognition occurs during information collection. Functional health patterns then provide structure to analyze factors. DIF:

Cognitive Level: Apply (Application)

REF:

p. 130

5. A nurse is using a functional focus to assess a person. Which of the following the nurse be evaluating? a. Visual acuity b. Pupil reactivity c. Ability to drive d. The red reflex ANS: C Functional focus refers to the individual’s performance level. Nurses assess how particular visual patterns affect lifestyle. The ability to drive would affect a person’s lifestyle and might require a change in how the person functions. DIF:

Cognitive Level: Analyze (Analysis)

REF:

p. 131

6. A nurse working with a Hispanic family is explaining the plan for managing a child’s asthma to the child’s mother, father, and grandmother. To whom should the nurse direct the education? a. Mother b. Father c. Grandmother d. Parents and grandmother


ANS: D Culturally competent care is delivered with understanding of and sensitivity to cultural factors influencing health behaviors. Nurses provide culturally competent care when they identify and use cultural norms and values. In the Hispanic population, the male figure is usually the decision maker, and the family elders are highly respected. However, assumptions about cultural norms should not be made. As a result, the nurse should direct education to all three adults because they may all have an impact on the child’s health care needs. DIF:

Cognitive Level: Analyze (Analysis)

7. A 27-year-old woman has not received a Pap test in which functional pattern? a. Health-perception–health-management pattern b. Elimination pattern c. Activity-rest pattern d. Self-perception–self-concept pattern

REF: p. 131 years. This assessment finding identifies an alteration in

ANS: A Assessment objectives for the health-perception–health-management pattern consist of obtaining data about perceptions, management, and preventive health practices. Exploring these values identifies potential health hazards. A 27-year-old woman should receive a Pap test every 2 years. Failing to do so could place her at risk for health problems; thus, this finding identifies an alteration in the health-perception–health-management pattern. DIF:

Cognitive Level: Apply (Application)

REF:

p. 131

8. A client is experiencing an alteration in the health-perception–health-management pattern and an alteration in the values-beliefs pattern. Which of the following best describes the behavior of this person? a. Never sees a physician b. Only sees a physician if not feeling well c. Sees a physician for screenings only d. Sees a physician for follow-up care of a chronic disease ANS: B Health beliefs and perceptions directly affect participation in care. Dimensions of assessment in the valuesbeliefs pattern include the individual’s values, beliefs, or goals that guide choices or decisions that are related to health. People who do not believe in health promotion activities will likely see a physician only when sick. Thus, someone with an alteration in the health-perception–health-management and values-beliefs patterns will likely only see a physician if not feeling well. DIF:

Cognitive Level: Apply (Application)

REF:

p. 131 | p. 142

9. When assessing a person’s nutritional-metabolic pattern, which objective finding would have implications for nursing intervention? a. The person’s 24-hour diet diary b. The person’s dentition c. The person’s food preferences d. The person’s financial status ANS: B Although all of the assessment parameters listed have implications for nursing diagnosis and planning for this client, the only objective measure is the client’s dentition. It is the only one that can be validated with a physical exam. DIF:

Cognitive Level: Apply (Application)

REF:

p. 134


10. When assessing a client’s activity-exercise pattern, which subjective finding has implications for nursing practice? a. A person’s decreased muscle tone b. A person’s amount of leisure time c. A person’s decreased range of motion d. A person’s use of a cane ANS: B Although all findings are important in assessing the activity-exercise pattern, the only subjective finding is the amount of leisure time that the person reports having. All others are objective findings and can be validated with a physical exam. DIF:

Cognitive Level: Apply (Application)

REF:

pp. 135-136

11. During a health history, a person reports getting 5 hours of sleep a night. What does this information indicate to the nurse? a. The person is not receiving enough sleep. b. The person is receiving adequate sleep. c. The nurse must determine where the person sleeps. d. The nurse must ask additional questions. ANS: D The single most important factor assessed in the sleep-rest pattern is probably the perception of adequacy of sleep and relaxation. The objective when assessing the sleep-rest pattern is to describe the effectiveness of the pattern from the person’s perspective. Wide variation in sleep time does not necessarily affect functional performance. Different individuals require different amounts of sleep. Thus, without further subjective data, the nurse is not able to make a diagnosis in this functional pattern. DIF:

Cognitive Level: Apply (Application)

REF:

p. 136

12. A nurse assesses the cognitive-perceptual pattern of a Type 1 diabetic client. Which finding has implications for the individual’s nursing plan of care? a. Decreased sense of hearing b. Decreased sense of smell c. Decreased sense of taste d. Decreased visual acuity ANS: D Assessment parameters in the cognitive-perceptual pattern include hearing, vision, smell, and taste. A person with Type 1 diabetes mellitus requires insulin injections. A decrease in visual acuity will make it difficult for the individual to draw up his or her medication and therefore will influence the nurse’s plan of care. DIF:

Cognitive Level: Analyze (Analysis)

13. Which scenario indicates a potentially dysfunctional pattern? a. Adult with frequent urination b. Woman who lost her job c. Elderly person with blurred vision d. Overweight adult with a sweet tooth ANS: B

REF:

p. 137


A pattern is potentially dysfunctional when sufficient evidence exists or enough risk factors are present to indicate that a pattern of dysfunction will likely occur if interventions are not instituted. A dysfunctional pattern is a problem when it represents a deviation from established norms or from the individual’s previous condition or goal. The woman who lost her job indicates a potential dysfunction pattern because the stress of losing her job places her at risk for ineffective coping. The other scenarios are not potentially dysfunctional; by definition, they are dysfunctional. DIF:

Cognitive Level: Apply (Application)

REF:

p. 144

14. Which scenario represents a dysfunctional pattern? a. Sexually active teenager who does not use condoms b. Salesman who sleeps only 5 hours a night c. Single mother of three children d. Woman with a small extended family ANS: A A pattern is potentially dysfunctional when sufficient evidence exists or enough risk factors are present to indicate that a pattern of dysfunction will likely occur if interventions are not instituted. A dysfunctional pattern is a problem when it represents a deviation from established norms or the individual’s previous condition or goal. Dysfunctional patterns may be present in the absence of disease, and nursing care may be necessary for health promotion and maintenance. The teenager, although free of disease, is in need of health promotion and disease prevention strategies because her sexual behavior indicates a dysfunction in her sexuality-reproductive pattern that places her at risk for a sexually transmitted disease and pregnancy. DIF:

Cognitive Level: Apply (Application)

REF:

p. 144

15. A nurse is counseling a person with a dysfunctional sleep pattern. Which of the following recommendations would the nurse most likely give the person? a. Read in bed until he falls asleep. b. Avoid fluids after 7 PM. c. Exercise immediately before bedtime. d. Watch television in the recliner in the evening. ANS: B Etiological factors of most dysfunctional patterns often lie within another pattern or patterns. Outcomes and plans are based on probable cause. Exercising before bed, watching television, and reading in bed are not considered appropriate sleep hygiene. Frequent urination may be the cause of his dysfunctional sleep pattern and, if so, avoiding fluids before bed would be an appropriate plan. DIF:

Cognitive Level: Analyze (Analysis)

REF:

p. 144

16. A nurse is caring for a person with a potential dysfunction in the health-perception–health-management pattern. Which of the following nursing interventions would most likely be performed? a. Arranging for home delivery of medication from the pharmacy b. Providing education regarding the dangers of smoking c. Instituting visiting nurse services for blood pressure checks d. Providing direct observed therapy for tuberculosis medications ANS: B Potential problems are risk states. Nursing interventions are directed toward risk reduction through education. Health promotion requires the individual to participate in his own care, and he cannot do this if he does not recognize his susceptibility to an impending health problem. Providing education addresses the risk and provides the person with information needed to change beliefs. The other options make the person a passive participant rather than an active one. DIF:

Cognitive Level: Apply (Application)

REF:

p. 144


17. The nurse has determined that a person has a dysfunction in the nutritional-metabolic pattern. Which action would be the next step for the nurse to take? a. Weigh the person. b. Set a goal weight with the person. c. Ask the person what her favorite foods are. d. Develop a plan for weight loss. ANS: B The individual’s goals and the determined diagnosis provide the basis for planning. Before developing a plan, a goal must be set. Clarity of the goals and diagnosis is critical to the development of an effective plan. In this case, the diagnosis has already been established and thus assessment of this pattern has occurred (weight, favorite foods). The next step before developing a plan is to set a goal weight with the client. DIF:

Cognitive Level: Apply (Application)

REF:

p. 145

18. A nurse weighs a person who has been diagnosed with a dysfunction in the nutritional-metabolic pattern. Which aspect of the nursing process is being performed? a. Assessment b. Implementation c. Planning d. Evaluation ANS: D The nursing process consists of assessment, diagnosis, planning, implementation, and evaluation. A person who has been diagnosed with a dysfunction has already been assessed. The process of analyzing changes experienced by a person after a plan has been implemented occurs in the evaluation phase. In this question, a weight will determine whether or not the person is moving toward her goals of weight loss. DIF:

Cognitive Level: Analyze (Analysis)

REF:

p. 145

19. A nurse administers the T-ACE test to a pregnant woman. The woman’s responses result in a score of 3. This score indicates that the woman: a. requires interventions for problem drinking. b. lacks evidence of problem drinking. c. requires interventions for sexually transmitted disease risks. d. lacks evidence of sexually transmitted disease risks. ANS: A The T-ACE provides a sensitive measure of alcohol-intake pattern in pregnant women. A score of 2 or more indicates evidence of problem drinking. This client had a score of 3, which would require an intervention for problem drinking. DIF:

Cognitive Level: Apply (Application)

REF:

p. 128 (Think About It Box)

20. A Hispanic mother tells the nurse that she has been using home remedies for her child’s asthma. Which home remedy might this mother be using? a. Acupuncture b. Cupping c. Hot tea d. Massage ANS: C


In the Hispanic population, asthma is viewed as a cold disease (hot-cold imbalance) and thus is treated with warm therapies. Diet is often used to maintain equilibrium. Thus, warm tea added to the child’s diet might be used to restore equilibrium between hot and cold in this child who has asthma. DIF:

Cognitive Level: Apply (Application)

REF:

p. 133 (Box 6-2)

21. Which classification system fulfills needs that are exclusive to nursing? a. The International Classification of Nursing Practice (ICNP) b. The International Classification of Functioning, Disability, and Health (ICF) c. The International Nursing Diagnoses Classification (NANDA-I) d. The Nursing Diagnostic System (NDS) ANS: C The NANDA-I system includes diagnostic criteria, and related etiologies in addition to the description. The NANDA-I fulfills needs that are exclusive to nursing. DIF:

Cognitive Level: Remember (Knowledge)

REF:

p. 144 (Box 6-5)

22. Erikson’s task of autonomy vs. shame and doubt occurs during which stage of development? a. Infancy b. Early childhood c. Late childhood d. Early adolescence ANS: B Erikson’s task of autonomy vs. shame and doubt occurs during early childhood. DIF:

Cognitive Level: Remember (Knowledge)

REF:

p. 139 (Table 6-3)

23. A young couple is deciding if they should get married and start a family. Which of Erikson’s life stages are they experiencing? a. Identity vs. role confusion b. Intimacy vs. isolation c. Generativity vs. stagnation d. Ego integrity vs. despair ANS: B During early adulthood individuals experience Erikson’s life stage of intimacy vs. isolation. Examples of life events in this stage include committing to a mate and family responsibilities and selecting a career. Identity vs. role confusion occurs during adolescence. Intimacy vs. isolation occurs during middle adulthood. Ego integrity vs. despair occurs during maturity. DIF:

Cognitive Level: Remember (Knowledge)

REF:

p. 139 (Table 6-3)

24. Which cultural group defines illness as a price that is being paid for the past or the future? a. African b. Native American c. Arabian d. Asian ANS: B American Indians define illness as a price that is being paid for the past or the future. DIF:

Cognitive Level: Remember (Knowledge)

REF:

p. 133 (Box 6-2)


25. A man is telling a nurse that he feels that his health is a gift from God. This statement most closely coincides with beliefs of which cultural group? a. African b. Alaska Native c. Asian d. Hispanic ANS: D Hispanics define health as a gift from God. DIF:

Cognitive Level: Apply (Application)

REF:

p. 133 (Box 6-2)

26. Which of the following is the leading cause of death among women? a. Accidents b. Cancer c. Coronary artery disease d. Stroke ANS: C The leading cause of death in women is coronary artery disease. DIF:

Cognitive Level: Remember (Knowledge)

REF:

p. 144 (Hot Topics Box)

MULTIPLE RESPONSE 1. A client who fails to take his insulin on a regular basis may have a conflict in which of the following functional health patterns? (select all that apply) a. Health-perception–health-management b. Cognitive-perceptual c. Elimination d. Values-beliefs ANS: A, B, D A problem in one area serves as a clue to dysfunction in other areas. Cognitive patterns include the ability of the individual to understand and follow directions, retain information, make decisions, solve problems, and use language appropriately. As a result, this person may not understand how to give himself the insulin properly. The values-beliefs pattern describes values including the individual’s spiritual values, beliefs, and goals. This person may not believe in the use of medications unless he is symptomatic. The health-perception–health-management pattern involves the individual’s health status and health practices used to reach the current level of health or wellness, with a focus on perceived health status and meaning of health to the individual. This person may not believe in health promotion and prevention. Thus, a person who fails to take his insulin on a regular basis may have a conflict in the health-perception–health-management, cognitive-perceptual, and values-beliefs patterns. DIF:

Cognitive Level: Apply (Application)

REF:

p. 131 | p. 137 | p. 142

2. Which individual is at risk for a dysfunction in elimination pattern? (select all that apply) a. 46-year-old mother of two b. 32-year-old African American man c. 15-year-old girl d. 72-year-old white woman ANS: A, B, C, D


When evaluating elimination patterns, nurses must consider age, developmental level, and cultural considerations. A 46-year-old mother of two is at risk for urinary stress incontinence because of the two vaginal births; an older adult is at risk for urinary control problems; African Americans often have a diet low in fiber, which can lead to constipation; and teenagers, especially girls, may have problems with body image, leading to abuse of laxatives. Thus, all persons listed are at risk for a dysfunction in elimination patterns. DIF:

Cognitive Level: Analyze (Analysis)

REF:

p. 134

Chapter 07: Health Promotion and the Family Edelman: Health Promotion Throughout the Life Span, 8th Edition MULTIPLE CHOICE 1. A nurse is determining which family assessment tool would be best to implement with a family when planning home visits for health promotion. Which of the following criteria should the nurse use to evaluate these tools? a. The number of questions asked on the assessment tool b. The linkages of the assessment to Healthy People 2020 c. The amount of involvement that the family has in completing the tool d. The capability of the tool to assess goals and outcomes ANS: C Useful health-promotion family assessments involve listening to families, engaging in participatory dialogue, recognizing patterns, and assessing family potential for active, positive change. DIF:

Cognitive Level: Apply (Application)

REF:

pp. 150-151

2. Which of the following best describes the nurse’s role in health promotion and disease prevention? a. Educating about home safety measures b. Identifying areas for family improvement c. Implementing the nursing process using a systems perspective d. Acting as a role model for the family ANS: D The nurse’s role in health promotion and disease prevention is best described as acting as a role model for the family. Implementing the nursing process, identifying areas for family improvement, and educating about home safety measures are all part of the nurse’s role but do not describe the comprehensive role of the nurse. DIF:

Cognitive Level: Remember (Knowledge)

REF:

pp. 151-152

3. Which of the following theories is an attempt to explain families as a set of interacting individuals with patterns of living that influence health decisions? a. Feminist theory b. Systems theory c. Developmental theory d. Resiliency theory ANS: B Systems theory is an attempt to explain patterns of living among the individuals who make up the family system. DIF:

Cognitive Level: Remember

(Knowledge)

REF:

p. 151

4. A nurse is assessing how a family will transition and adapt after their youngest child leaves for college. By using this framework, which of the following perspectives is the nurse implementing? a. Risk-factor


b. c. d.

Structural-functional Open systems Developmental

ANS: Dj Duvall and Miller identified stages of the family life cycle and critical family developmental tasks, through a developmental perspective. This conceptual model helps to anticipate family events and discusses how families complete basic family tasks as they transition through these events. DIF:

Cognitive Level: Apply (Application)

REF:

p. 152

5. Which of the following would be described as a family structural component? a. Income earner of the house b. Socialization for the family c. Immunization of infants d. Launching of children ANS: A Structural components of the family refer to family roles and relationships. DIF:

Cognitive Level: Apply (Application)

REF:

p. 152

6. A nurse is collecting data for a family assessment using Gordon’s functional health patterns. The nurse learns that the family has no books in the home to read to the preschool-age children. To which of the following functional health patterns does this information pertain? a. Roles-relationship b. Cognitive-perceptual c. Health-perception–health-management d. Self-perception–self-concept ANS: B The cognitive-perceptual pattern identified characteristics of language, cognitive skills, and perception that influence desired or required family activities. The availability of books in the home for preschool age children impacts this functional health pattern. DIF:

Cognitive Level: Analyze (Analysis)

REF:

p. 157

7. A nurse who is using Gordon’s functional health patterns is planning to assess its roles-relationships pattern. Which of the following questions would be most appropriate for the nurse to ask? a. What is the family’s philosophy of health? b. What does the family do to have fun? c. How are problems in the family resolved? d. Who decides when and how children go to sleep? ANS: C How problems in the family are resolved relates to assessment of the roles-relationships pattern. The family’s philosophy of health relates to the health perception-health management pattern. What the family does to have fun relates to the activity-exercise pattern. The decision concerning when and how children go to sleep relates to the sleep-rest pattern. DIF:

Cognitive Level: Analyze (Analysis)

REF:

pp. 158-159

8. A nurse is using a genogram to represent a family. Which of the following statements is accurate? a. A genogram identifies the genetic disorders of the family. b. A genogram includes information about the past two generations.


c. d.

A genogram can be used to make connections about family health patterns. A genogram begins with a circle in the center of the page.

ANS: C A genogram shows a variety of family structures and highlights family health patterns, which can be used for anticipatory health guidance. Significant diseases and disorders of the family members are highlighted on the genogram. Data on at least three generations are reported on a genogram. The genogram uses a variety of symbols to demonstrate connections but does not begin with a circle in the center of the page. DIF:

Cognitive Level: Apply (Application)

REF:

p. 160

9. The ecomap of a client’s family has slashed lines drawn from the son to the family church. Based on this information, what conclusion can the nurse make? a. The son is deceased. b. The son is actively involved with the family church. c. The son has a stressful relationship with the church. d. The son has no relationship with the church. ANS: C Slashed lines on an ecomap signify stressful relationships. DIF:

Cognitive Level: Apply (Application)

REF:

p. 160

10. A client reports that her family will be moving because her husband is taking a new job in another state. She is very unhappy about the decision and doesn’t want to move. What action should the nurse take next? a. Assess the client’s and family’s coping mechanisms in handling stress. b. Encourage the client to act excited about the move. c. Talk to the husband to get his perspective on the move. d. Tell her that all families must cope with new situations from time to time. ANS: A The family’s ability to cope with demands of everyday living determines its level of success. The nurse needs to assess how the family usually copes with stressful situations to find ways that might be available to the family now to cope with the current situation. DIF:

Cognitive Level: Analyze (Analysis)

REF:

p. 164

11. In a family in which the mother and the father differ on how to spend and save money, the parents are constantly arguing with each other. Which of the following interventions should the nurse include in the plan of care for this couple? a. Assist them to develop strategies that are congruent with their values. b. Allow each of them to defend his or her own values. c. Focus on outcomes that each wants to accomplish. d. Divert their attention to areas in which they are successful. ANS: A When strategies are used that are congruent with each individual’s values, the couple adjusts. DIF:

Cognitive Level: Apply (Application)

REF:

p. 166

12. A blended family has six children, ages 2, 4, 4, 5, 7, and 10. During a visit to the home, the nurse notices that the 7-year-old seems quiet and withdrawn, whereas the other children are playing loudly in the garage. Which of the following conclusions can the nurse make from this observation? a. This child has most likely been abused. b. This child is one of multiple children closely spaced in age.


c. d.

This family suffers from low self-esteem. This family provides harsh punishment for their children.

ANS: B Risks associated with role relationships in blended families include multiple closely spaced children, which limits the parents’ time for interaction to meet individual children’s needs. DIF:

Cognitive Level: Apply (Application)

REF:

p. 167

13. The nurse is caring for a family who has 2-year-old twins. Which of the following health promotion advice would be included in the nurse’s plan of care for this couple? a. Wear bicycle helmets for safety. b. Use caution around the family swimming pool. c. Cross the street at using the crosswalks. d. Advocate for the day care to provide adequate socialization. ANS: B Two-year-olds are prone to wandering to where water is and could fall into a swimming pool without being noticed. DIF:

Cognitive Level: Apply (Application)

REF:

p. 167

14. A nurse is conducting a health promotion assessment for a family with a 9-month-old. Which of the following should be of most concern to the nurse? a. The age of the house in which the family lives b. Genetic diseases in the family c. Driving practices in the family d. Toilet training for the child ANS: A A typical 9-month-old is beginning to crawl. Houses built before 1974 may contain lead-based paint, to which a crawling baby might have access. Lead causes neurological damage and anemia. DIF:

Cognitive Level: Analyze (Analysis)

REF:

p. 167

15. A nurse is caring for a family with an adolescent child. Which of the following problems would the nurse anticipate that the family would report? a. Concerns about career decisions b. Concerns about exposure to environmental hazards c. Difficulty with parents finding fulfillment with raising the child d. Difficulty with open communication with the child ANS: D Open communication with parents is often difficult during the adolescent stage, partly because of the differing developmental tasks of adolescents and adults. Concerns about exposure to environmental hazards and parents finding fulfillment with raising the child typically are concerns with families with younger children. Concerns about career decisions are typically problematic for families with older children/young adults. DIF:

Cognitive Level: Apply (Application)

REF:

p. 168

16. The nurse is working with a middle-age married couple whose son has just graduated from college. Which developmental tasks would the nurse expect to find in this family? a. Attending activities for their son b. Strengthening their marital relationship for future family stages c. Acting as a launching center for their son


d.

Responding to the prospect of changing careers

ANS: C Families with young adults act as launching centers for children ready to leave home. DIF:

Cognitive Level: Apply (Application)

REF:

pp. 168-169

17. A nurse has developed a family nursing diagnosis. Which of the following best describes the purpose of this action? a. Describes the strengths of the family b. Allows for creation of goals for the family c. Promotes behavioral change among family members d. Validates health problems with the family ANS: D Writing a family nursing diagnosis helps families promote health throughout the life cycle and prevents disease through decreasing risk-taking behaviors. Nurses derive diagnoses from assessed validated data. The nursing diagnosis describes and validates potential or actual health problems with families. The diagnosis provides direction for outcomes and interventions first identifying what the problem is. DIF:

Cognitive Level: Apply (Application)

REF:

p. 170

18. A home care nurse is planning an intervention with a family focusing on decreasing susceptibility. Which of the following nursing interventions would be most appropriate to implement? a. Education about building on current strengths of the family b. Education about hand hygiene c. Education about health care resources in the community d. Education about child safety seats ANS: B Four types of nursing interventions appear in health-promotion and disease-prevention planning: increasing knowledge and skills; increasing strengths; decreasing exposure; and decreasing susceptibility. Decreasing susceptibility means educating families about prevention principles. Examples include education about hand hygiene and how diseases are spread from person to person and by other factors in the environment. DIF:

Cognitive Level: Analyze (Analysis)

REF:

p. 171

19. A nurse is making a final home visit with a family to evaluate the nursing care plan. Which of the following actions would the nurse most likely complete during this visit? a. Obtain vital signs from all members of the family. b. Ask the family members to state the goals that were previously developed. c. Collect data similar to that which was collected at the initial visit for comparison. d. Educate about the importance of using role relationships to create a healthy family. ANS: C The purpose of evaluation is to determine how the family has responded to the planned interventions and whether these interventions were successful. The family’s baseline data are used as comparative criteria in evaluation; thus, it would be worthwhile to collect data similar to that collected at the initial visit to see if any changes have occurred. The nurse reassesses the situation and compares the new information with that on the original assessment to determine whether change has occurred. DIF:

Cognitive Level: Analyze (Analysis)

REF:

p. 172

20. A nurse is planning a home visit for a family. Which of the following actions would be most appropriate for the nurse to take?


a. b. c. d.

Study information regarding the family from agency records and other sources. Make a contract with the family that states specific goals and objectives. Identify how the home visit will be financed. Understand the situation from the family’s perspective.

ANS: A Part of planning the home visit is studying information regarding the family from agency records, referral forms, and other sources. Making a contract, identifying how the visit will be financed, and understanding the situation from the family’s perspective are all part of the process of making the visit. DIF:

Cognitive Level: Apply (Application)

REF:

p. 151 (Box 7-1)

21. A nurse is providing follow-up care for a family who has recently had a baby. Which of the following topics should the nurse anticipate discussing with the family? a. Type 1 diabetes b. Fetal alcohol syndrome c. Communicable diseases d. SIDS ANS: D Families in the beginning childbearing stage need education about the risk of SIDS for their infant. After the birth of the child, it is most likely that they do not need education about fetal alcohol syndrome, unless there were issues with alcohol use during the prenatal period. Discussion about communicable diseases and potentially Type 1 diabetes would be more common with families with school-aged children. DIF:

Cognitive Level: Apply (Application)

REF:

p. 154 (Table 7-1)

MULTIPLE RESPONSE 1. A home health nurse is admitting a 54-year-old man for services following a coronary artery bypass graft (CABG). As part of the initial visit the nurse completes a family assessment. What is the purpose of this nursing action? (select all that apply) a. Allows for health promotion and disease prevention appraisal b. Allows for inclusion of family members in decision-making c. Allows for data collection necessary for comparison to Healthy People 2020 d. Allows for development of patient-centered care ANS: A, B Families provide the structure for many health promotion practices; therefore, family assessment informs health promotion and disease prevention appraisal. Additionally, health providers are encouraged to include families in decision-making and encouraging their presence and participation in all aspects of care from acute care to health promotion. Thus, it is important to have assessed the family so that the family can be included in the care process. DIF:

Cognitive Level: Apply (Application)

REF:

p. 150

2. A nurse is providing care for a family in the community. Which of the following characteristics would the nurse assess to determine the health of the family? (select all that apply) a. Developmental stage of each family member b. Coping mechanisms of each family member c. Potential risk factors within the family d. Maintenance of trust within the family ANS: B, C, D


Coping mechanisms of each family member, potential risk factors within the family, and maintenance of trust within the family are all necessary characteristics for the nurse to assess when determining the health of a family. When assessing a family, the nurse will consider the developmental stage of the family before considering the developmental stage of each of the individual family members. DIF:

Cognitive Level: Apply (Application)

REF:

p. 152 (Box 7-2)

3. A nurse is conducting an environmental assessment as part of a family assessment. Which of the following would the nurse assess? (select all that apply) a. Garbage collection in the neighborhood b. Convenience stores in the neighborhood c. Safety of the home d. Climate of the home ANS: A, B, C, D The family environment is made up of the home, neighborhood, and community. Thus, aspects of the community as well as aspects of the home environment need to be analyzed during this assessment. DIF:

Cognitive Level: Apply (Application)

REF:

p. 165

Chapter 08: Health Promotion and the Community Edelman: Health Promotion Throughout the Life Span, 8th Edition MULTIPLE CHOICE 1. A nurse is employed by a local health department. Which of the following would be one of her primary responsibilities? a. Providing appropriate treatment for the flu to an elderly man b. Providing education to the community about prevention of the flu c. Ensuring that the family members of the man with the flu get tested for the flu d. Ensuring that the family members of the man with the flu receive the flu vaccine ANS: B The role of the community health nurse is to promote the health of the population. It is not limited to any particular individual or group of individuals. It is important that the nurse view the community structure as a population and consider existing health services. Therefore, the primary responsibility of this nurse is to the community she serves by providing education about flu prevention. DIF:

Cognitive Level: Analyze (Analysis)

REF:

p. 179

2. A parish nurse is concerned about the number of parishioners who smoke. How would the parish nurse best address this issue? a. Ban smoking on parish property. b. Invite local experts to participate in a health fair at the parish. c. Find literature about smoking cessation. d. Hand out free nicotine-replacement systems. ANS: B Nurses interact with communities to promote health. They supply educational information to at-risk groups to develop health-oriented skills as well as encourage behavior change. As such, nurses act as advocates collaborating with other disciplines and agencies. The success of health promotion programs depends on support from prominent community members. Therefore, arranging for a health fair and inviting local experts to participate is the best way for the nurse to address the smoking issue at her parish.


DIF:

Cognitive Level: Analyze (Analysis)

REF:

p. 180 | p. 183

3. Partnering with school-based clinics and local pediatric dentists so that children can have access to preventive dental care is an example of a(n): a. codependent function. b. independent function. c. interdependent function. d. dependent function. ANS: C Collaboration with community members and interdisciplinary teamwork functions crucial to effective community health are considered interdependent functions. Partnering with school-based clinics and local dentists is a collaborative effort to improve the community’s health. DIF:

Cognitive Level: Apply (Application)

REF:

p. 181

4. Which of the following health concerns should the nurse have as a priority when planning care for a community? a. The limited recreational areas identified through a windshield survey b. The high crime rate reported in the town records c. The absence of health clinics on the local bus route d. The lack of grocery stores within walking distance identified by the community ANS: D As a community liaison, the nurse establishes priorities for programming and matches resources with needs determined by a community-needs assessment. The goal is to maintain the community’s vision. Nurses’ concerns should be based on the community’s concerns. Therefore, the lack of grocery stores identified by the community should take first priority. DIF:

Cognitive Level: Analyze (Analysis)

REF:

pp. 181-182 | p. 184

5. A nurse who is new to the area wants to get a “feel for the community she will be working with.” Which of the following would be the best way for the nurse to start learning about the community? a. Driving around the neighborhood b. Going door to door asking people about the community c. Searching online to obtain population statistics d. Talking to the police captain at the local police department ANS: A Community nurses assess communities by observation, interview, and measurement. Driving around and observing the community uses one’s senses to obtain information about the community, its health problems, and its strengths. This type of assessment (driving around and observing) gives the nurse an idea of how the community functions. DIF:

Cognitive Level: Analyze (Analysis)

REF:

pp. 181-182

6. A community believes many people in that community lead sedentary lives because they lack safe areas in which to exercise. What action should the nurse take in order to confirm or reject these beliefs? a. Walk around the neighborhood. b. Interview members of the community. c. Obtain town morbidity and mortality rates. d. Go to the town hall to obtain the number of parks in the community. ANS: B


The nurse has generated a hypothesis (people lead sedentary lives because of a lack of safe areas). To explore this hypothesis, the nurse should follow it up with interviews and measurement data. In this situation, town statistics would not be sufficient to address the hypothesis. The nurse needs to interview members of the community to determine how they perceive it. DIF:

Cognitive Level: Apply (Application)

REF:

pp. 181-182

7. A nurse is interviewing members of a substance abuse unit at a local Veterans Administration (VA) hospital. Which of the following parts of a community is the nurse assessing? a. Structure of a community b. Subsystem of a community c. Supra system of a community d. Interaction of a community ANS: B Structural parts of a community form subsystems within a larger supra system. A substance abuse unit of a local VA hospital is a subsystem of a larger system (the VA hospital and family of hospitals). The structure of a community system or subsystem forms a formal or informal arrangement of parts. As a result, because interviewing is a form of assessment, the nurse in this example is assessing the subsystem (substance abuse unit of a local VA hospital) of a community (the VA hospital and family of hospitals). DIF:

Cognitive Level: Apply (Application)

REF:

p. 182

8. The study of a population is referred to as (a): a. community pattern. b. windshield survey. c. demography. d. community diagnosis. ANS: C Demography is defined as the study of a population. DIF:

Cognitive Level: Remember (Knowledge)

REF:

pp. 182-183

9. The process of dynamic change with adaptation in the system’s parts, and how community systems and subsystems interact is known as: a. structure of a community. b. community health promotion. c. community diagnosis. d. function of a community. ANS: D The function of a community is defined as the process of dynamic change with adaptation in the system’s parts and how community systems and subsystems interact. DIF:

Cognitive Level: Remember (Knowledge)

REF:

p. 183

10. A nurse is assessing a community from both a developmental and risk perspective. Which of the following characteristics would be of most interest to the nurse? a. Gender b. Age c. Race d. Socioeconomic level ANS: B


Community nurses use a developmental-age-correlated approach to identify health promotion and health protection activities. Age is an indication of development. Additionally, community nurses assess those risk factors in a community that can cause adverse health outcomes. Risk factors can vary from person to person but include age, gender, race, geographical location, and lack of health services. Age can thus be used to assess a community from both a developmental and a risk perspective. DIF:

Cognitive Level: Apply (Application)

REF:

p. 183

11. Using developmental theory, a community nurse assesses the community and determines that the majority of residents who are 45 years of age or older lead sedentary lives. Which of the following would be the priority for this nurse to promote? a. Health promotion activities b. Accident prevention activities c. Drug prevention activities d. Pregnancy prevention activities ANS: A Developmental theory can be used to identify existing or potential health problems for a particular age group in the community. Age-related risk factors associated with individuals can be extended to the community. It is well known that physical activity decreases with age and chronic illnesses increase with age. Therefore, health promotion and disease prevention activities become a priority for the nurse serving this community. DIF:

Cognitive Level: Analyze (Analysis)

REF:

p. 183

12. Which of the following demonstrates the nurse using subjective data to indicate a health concern in the healthperception–health-management pattern? a. Examining high sexually transmitted disease rates recorded by the health department b. Listening to community members discuss the high smoking rate among teenagers c. Analyzing the high infant mortality rate reported by state hospitals d. Discussing the high rate of teenage motor vehicle crashes recorded by the local police department ANS: B Subjective data is based on the perceptions of individuals in the community. Interviewing provides a way to learn how members perceive their communities and their perceptions of health. Therefore, the high smoking rates among teenagers reported by community members indicates subjective data. The others are forms of objective data. DIF:

Cognitive Level: Analyze (Analysis)

REF:

p. 181 | p. 184

13. After assessing the community, the nurse concludes that the community is having difficulty meeting its nutritional-metabolic pattern. Which of the following findings would the nurse most likely have discovered during the assessment? a. Decreased availability of grocery stores b. Poor nutritional habits c. Lack of subsidized food programs d. Inadequate knowledge about proper nutrition ANS: B The nutritional-metabolic pattern identifies data relevant to community eating habits. These habits are the result of decreased availability of grocery stores, lack of subsidized food programs, and inadequate knowledge. The habits are also influenced by a community’’s culture. Therefore, the end results of health concerns in the nutritional-metabolic pattern of a community are poor nutritional habits. DIF:

Cognitive Level: Apply (Application)

REF:

p. 184


14. A nurse is assessing a community’s exposure to pollutants by examining contaminated soil, air, water, and food. Which functional health pattern is being assessed by the nurse? a. Nutritional-metabolic pattern b. Elimination pattern c. Health-perception–health management pattern d. Values-beliefs pattern ANS: B The elimination pattern identifies factors including exposure to pollutants in the community through contaminated soil, water, air, and food. DIF:

Cognitive Level: Apply (Application)

REF:

pp. 184-185

15. Which functional health pattern provides information about problem-solving and decision-making within communities? a. Cognitive-perceptual pattern b. Self-perception–self-concept pattern c. Coping-stress tolerance pattern d. Health-perception–health-management pattern ANS: A The cognitive-perceptual pattern provides information about problem-solving and decision making within communities. DIF:

Cognitive Level: Remember (Knowledge)

REF:

p. 185

16. Within a community there is an elevated high-school dropout rate and its high school students are experiencing a decreased sense of self-worth. Based on this data, the community is at highest risk to experience an alteration in which of the following functional health patterns? a. Self-perception–self-concept pattern b. Role-relationships pattern c. Cognitive-perceptual pattern d. Sexuality-reproduction pattern ANS: A The elevated high-school dropout rate is a risk factor in the values-beliefs pattern, and the decreased sense of self-worth is a risk factor in the self-perception–self-concept pattern. Risk factors from several pattern areas may form clusters of risks for certain groups. Elevated high-school dropout rates and a decreased sense of selfworth may lead to crimes that the community cannot adequately control or cope with. DIF:

Cognitive Level: Analyze (Analysis)

REF:

pp. 185-186

17. A nurse is working with a community that is experiencing an alteration in its values-beliefs pattern. Which of the following is most likely to result because of a disturbance in this functional health pattern? a. Respiratory disorders b. Hearing loss c. Coronary artery disease d. Stress ANS: C The values-beliefs pattern identifies the community values and beliefs. Values underlie decisions regarding prevention programs. A lack of primary prevention methods (e.g., exercise and a healthy diet) can lead to coronary artery disease. DIF:

Cognitive Level: Analyze (Analysis)

REF:

p. 186


18. A nurse is using the technique of mapping while implementing the nursing process in the community setting. The nurse is engaging in which aspect of the nursing process? a. Collection b. Planning c. Analysis d. Implementation ANS: C Analysis refers to data categorization and pattern determination. The organization of data is used to determine patterns. Mapping is an organizational technique used in data analysis. DIF:

Cognitive Level: Apply (Application)

REF:

pp. 186-187

19. A nurse is collecting data regarding the number of crosswalks within a community. Which perspective is being used to gather this data? a. Developmental perspective b. Functional perspective c. Risk-factor perspective d. Systems perspective ANS: C One way to assess a community is from a risk-factor perspective. Risk factors associated with community diseases, illness, and death rates play a role in predicting the likelihood of adverse health conditions. Risk factors include a combination of demographic, psychological, physiological, or environmental characteristics. Select groups may be at risk based on shared risks. Knowing which risk factors are present assists community nurses in developing action plans for health promotion and disease prevention. DIF:

Cognitive Level: Apply (Application)

REF:

p. 183 | p. 188

20. A community nurse has collected rates of alcohol-related motor vehicle accidents involving teenagers. Which action should the nurse take next? a. Develop a plan to address the high rates of alcohol-related motor vehicle accidents. b. Partner with high school nurses to educate students about the risks of drinking and driving. c. Diagnose the community with ineffective health management patterns related to the high levels of drunk driving. d. Establish support groups for teenagers with alcohol abuse. ANS: C The role of the community nurse includes the interaction of independent, interdependent, and dependent functions. Independent functions include assessing, analyzing, diagnosing, planning, implementing, and evaluating. In this case, once the nurse analyzes the alcohol-related motor vehicle accident rates, the next step is to diagnose the community. Therefore, the nurse would diagnose this community with ineffective health management patterns related to the high levels of drunk driving. DIF:

Cognitive Level: Analyze (Analysis)

REF:

21. Which of the following best describes the primary goal of community nurses? a. Create charts/graphs of data collected. b. Create plans for community health promotion programs. c. Create evaluation tools for health promotion programs. d. Create a culture of health promotion/health protection with the community. ANS: D

p. 181 | pp. 188-189


Over the last 2 decades, several social trends in the United States have increased the public interest in health promotion and disease prevention. Community health nursing combines nursing practice and public health concepts to promote the health of populations. Community participation in health planning facilitates effective assignment of priorities. Additionally, change results from efforts by individuals or groups and involves fundamental shifts of behavior. It is important that the community nurse create a culture of health promotion and health protection within a community in order to facilitate positive change in health behaviors. DIF:

Cognitive Level: Analyze (Analysis)

REF:

p. 177 | p. 190

22. A community nurse develops a plan to address the problem of teenage pregnancies. Which of the following actions should the nurse take next? a. Determine the rate of teenage pregnancies. b. Evaluate the success of her plan. c. Provide educational programs at local schools regarding pregnancy prevention. d. Determine which factors related to teenage pregnancy require intervention. ANS: C Plans guide nursing actions. Implementation of the nursing process begins based on the health promotion/health protection plan. Providing education is an action item in a plan and thus should be the next step. Determining the rate occurs before diagnosis. Determining which factors require intervention occurs during the planning phase. DIF:

Cognitive Level: Apply (Application)

REF:

p. 190

23. A nurse has implemented a smoking cessation program for teenagers and is now evaluating the rate of teenage smoking in the community. Which of the following functions is being demonstrated by the nurse? a. Codependent function b. Independent function c. Interdependent function d. Dependent function ANS: B Evaluating nursing activities such as health promotion and health education is an independent function. In this case, the nurse is evaluating the nursing activity (smoking cessation program), and therefore it is an independent function. DIF:

Cognitive Level: Apply (Application)

REF:

p. 181 | p. 192

24. Which factor has the greatest implication for a community nurse when developing a health promotion program for church members? a. The majority of members live within a 3-mile radius. b. The majority of members are African American. c. The majority of members are high school graduates. d. The majority of members are employed. ANS: B Demographic data provides the basis for analysis and a means to identify groups that may be at high risk for health concerns. Age, gender, race, geographical location, consumption pattern, or lack of health services may be considered risk factors because one or more may contribute to disease or death and place the population sharing them at risk. Such information also provides clues for the direction of health strategies. It is known that poverty rates among African Americans are rising, placing them at risk for health disparities. Additionally, African Americans are at high risk of developing Type 2 diabetes mellitus and hypertension. As a result, knowing that a majority of church members are African American has great implications for a community nurse when developing health promotion programs. DIF:

Cognitive Level: Analyze (Analysis)

REF:

p. 183 | p. 188 (Box 8-4)


25. A community is in the maintenance stage of change. Which of the following would be an appropriate intervention for the community health nurse? a. Highlight past successes b. Discuss the benefits of changing c. Develop strategies to prevent relapse d. Provide information ANS: C In the maintenance stage of change, a community nurse should highlight past successes and future benefits. During the contemplation phase, the benefits of changing should be discussed. Developing strategies to prevent relapse should happen during the action phase. Providing information should occur during the precontemplation stage. DIF:

Cognitive Level: Analyze (Analysis)

REF:

p. 187 (Table 8-2)

26. A community is considering banning smoking in all schools. Based on the community’s current stage of change, which action should the nurse take next? a. Provide the community with statistics regarding childhood smoking rates. b. Point out positive outcomes associated with banning smoking such as maintenance of lung function. c. Help the community develop a plan to implement the ban. d. Praise the members of the community for their actions. ANS: B A community that is considering implementing a smoking ban is in the contemplation stage of change. In this stage, interventions should be focused on discussing the risks of not changing and the benefits of changing. Maintaining the lung function of children is a benefit of the ban the community members are considering and thus an appropriate intervention for the nurse to utilize. DIF:

Cognitive Level: Analyze (Analysis)

REF:

p. 187 (Table 8-2)

MULTIPLE RESPONSE 1. In response to the growing elderly population, community health nurses should do which of the following? (select all that apply) a. Work with the board of education to emphasize the importance of adequate physical activity for children and the importance of healthy meals. b. Work with local nursing schools to emphasize the importance of geriatric training in the curriculum. c. Work with the state and local departments of health to devise a plan to increase nursing home beds. d. Work with local health departments to increase recreational activities for elderly members of the community. ANS: A, B, C, D Community leadership provides direction for health promotion and health protection. The change in age distribution indicates that people will be living longer and thus have more chronic illness issues. Services need to be made available for this population. Health promotion services need to be made available to the younger population as well to decrease their risk factors for future illnesses such as obesity and heart disease. Therefore, all responses are correct. DIF:

Cognitive Level: Analyze (Analysis)

2. Which of the following is a stage of change? (select all that apply) a. Reflection b. Action c. Maintenance

REF:

p. 177 | p. 182


d.

Acceptance

ANS: B, C According to Pender, the five stages of change include precontemplation, contemplation, planning, action, and maintenance. DIF:

Cognitive Level: Remember (Knowledge)

REF:

p. 187 (Table 8-2)

Chapter 09: Screening Edelman: Health Promotion Throughout the Life Span, 8th Edition MULTIPLE CHOICE 1. Which is an example of asymptomatic pathogenesis? a. Blood pressure of 170/98 experiencing headaches b. Positive finding on colonoscopy and blood in his or her stool c. Elevated prostate-specific antigen (PSA) d. Elevated TSH who is always tired ANS: C The primary objective of screening is the detection of a disease in its early stages, to treat it and deter its progression. The screening process is based on the principle that disease is preceded by a period of asymptomatic pathogenesis when risk factors predisposing a person to the pathological condition are building momentum toward manifestation of the disease. Therefore, someone with an elevated PSA without any symptoms is an example of asymptomatic pathogenesis. The other three examples demonstrate manifestation of disease (headaches, blood in stool, and tiredness). DIF:

Cognitive Level: Apply (Application)

REF:

p. 196

2. An occupational health nurse is planning a cholesterol screening with the employees at the factory. Which of the following would be an advantage of conducting this screening? a. Allows for the beginning of a multiple test screening process b. Provides an opportunity for health education c. Allows for preliminary diagnosis of coronary artery disease d. Provides the opportunity for a referral to a physician ANS: B Screenings create an opportunity for providing health education to a group of individuals who may not otherwise receive it. This allows the nurse to take advantage of a teachable moment with the employees in the factory. DIF:

Cognitive Level: Apply (Application)

REF:

p. 197

3. Influenza occurs among Americans at a rate of 36 per 100 people annually. Which type of rate is this statement describing? a. Prevalence rate b. Incidence rate c. Morbidity rate d. Mortality rate ANS: A


Incidence rate is the rate of a new population problem and estimates the risk of an individual developing the specific disease or condition during a specific period or over a lifetime. Prevalence is the proportion of a given population with the disease or condition at any one point in time. Usually acute conditions are assessed by their incidence (rate of occurrence), whereas chronic conditions are measured by their prevalence (generally existing). DIF:

Cognitive Level: Apply (Application)

REF:

p. 198

4. A nurse is examining the incidence, prevalence, and mortality rates of colon cancer in the community. Which of the following measures of life is being investigated? a. Quality adjusted life year (QALY) b. Quantity of life c. Disability adjusted life year (DALY) d. Satisfaction of life ANS: B Measures of quantity of life affected by a disease are more readily attainable than quality of life measures. Quantity of life can be measured by using incidence and prevalence rates as well as disease-specific mortality rates. DIF:

Cognitive Level: Apply (Application)

REF:

p. 199

5. Which represents a disadvantage of screening? a. Utilization of group screening methods b. Utilization of multiple test screening c. Utilization of a test with high specificity d. Utilization of a test with low sensitivity ANS: D Group screening and multiple test screening are advantages of screening programs. A disadvantage of screening occurs when the test is unable to distinguish those who probably have the disease from those who do not. Tests with low sensitivity produce a large number of false-negative tests and leave those screened with a false sense of a healthful state, resulting in them losing the opportunity to receive early treatments that could prevent irreversible damage. DIF:

Cognitive Level: Understand (Comprehension) REF:

p. 197 | p. 200

6. The nurse is examining the ability of a phenylketonuria (PKU) screening test to distinguish correctly between newborns who have and who do not have the disease. Which of the following measures of accuracy of the instrument is being evaluated? a. Sensitivity b. Specificity c. Validity d. Efficacy ANS: C Validity is defined as a test’s ability to distinguish correctly between diseased and nondiseased individuals. DIF:

Cognitive Level: Apply (Application)

REF:

p. 199

7. The proportion of people with a condition who correctly test positive when screened is known as: a. sensitivity. b. specificity. c. validity. d. efficacy.


ANS: A Sensitivity refers to the proportion of people with a condition who correctly test positive when screened. DIF:

Cognitive Level: Remember

(Knowledge)

REF:

p. 200

8. The Denver Developmental Screening Test (DDST) has been said to have excellent test specificity. What does this statement mean? a. Rarely identifies children who have developmental delays b. Rarely identifies children who do not actually have developmental delays c. Has a large number of false positive results d. Has a large number of false negative results ANS: B Specificity measures a test’s ability to recognize negative reactions or nondiseased individuals. A test with excellent specificity will rarely be positive if the disease is not present. DIF:

Cognitive Level: Remember (Knowledge)

REF:

p. 200

9. A nurse determined the interobserver reliability of a blood pressure reading. Which of the following describes the method that was used by the nurse? a. Blood pressure readings of 124/82 were obtained two days in a row by two different nurses. b. Three consecutive blood pressure readings of 124/82 were obtained by the same nurse. c. Blood pressure readings of 124/82 in the right arm and 124/82 in the left arm were obtained. d. A blood pressure reading of 124/82 was obtained immediately followed by another blood pressure reading of 147/92. ANS: A Reliability is an assessment of the reproducibility of the test’s results when different individuals with the same level of skill perform the test during different periods and under different conditions. If the same result emerges when two individuals perform the test, interobserver reliability is shown. Therefore, a client with a blood pressure reading of 124/82 two days in a row by two different nurses is an example of interobserver reliability. DIF:

Cognitive Level: Apply (Application)

REF:

p. 199

10. A nurse is using a sweat test to screen people for cystic fibrosis. Which of the following results demonstrates poor sensitivity? a. When 6 out of every 10 sweat tests performed are negative, but the six individuals actually have cystic fibrosis b. When 6 out of every 10 sweat tests performed are positive, but the six individuals do not actually have cystic fibrosis c. When 6 out of every 10 sweat tests performed are negative, and the six individuals really do not have cystic fibrosis d. When 6 out of every 10 sweat tests performed are positive, and the six individuals really do have cystic fibrosis ANS: A Sensitivity refers to the proportion of people with a condition who correctly test positive when screened. A test with poor sensitivity will miss individuals with the condition, and there will be a large number of false-negative results; individuals actually have the condition but were told they were disease free. Thus when 6 out of every 10 sweat tests performed are negative but the six individuals actually have cystic fibrosis, it is an example of poor sensitivity. DIF:

Cognitive Level: Apply (Application)

REF:

p. 200

11. A nurse is creating a hypertension screening program. Which of the following methods would be the best way to design a successful program?


a. b. c. d.

Work with stakeholders to conduct a community assessment. Purchase state-of-the-art sphygmomanometers to measure blood pressures. Use the program developed at a previous place of employment. Contact a local church to see if the program can be implemented there.

ANS: A Partnerships are essential to developing health programs and screening programs. The primary rule is to never assume that what is appropriate and effective for one community will be appropriate and effective for another community. A community assessment conducted as a partnership with key stakeholders provides information about the high-risk population, available health care resources, and the high-risk population’s health needs. By conducting the assessment, the nurse can identify the necessary community resources and mobilize them to achieve maximal benefits and positive outcomes. DIF:

Cognitive Level: Analyze (Analysis)

REF:

p. 201

12. A nurse is planning to offer a depression screening at a local community center. Which of the following should be considered prior to implementation of the program? a. Limited referral sources in the community b. Limited support groups in the community c. Insufficient evidence that depression screening tools are cost effective d. Insufficient evidence mental health screening is appropriate ANS: A Constraints affecting the operation of a screening program include financial concerns, political issues, cultural constraints, follow-up and referral services, and accessible treatment facilities. An efficient referral system should link the follow-up resources to the screening program, providing continuity of care. A method must be devised to encourage the participant to take positive action on the referral. Depression screening for adults is a covered preventive service for adults. DIF:

Cognitive Level: Apply (Application)

REF:

p. 201

13. A nurse is implementing a test that screens for hypercholesterolemia. Which of the following parameters should this test have? a. No cutoff point b. Low cutoff point c. Intermediate cutoff point d. High cutoff point ANS: B The goal of a screening program, identifying an individual as high risk or not, depends on the numerical value of the screening instrument. When the parameter for this distinction is not clear, a cutoff point is set. Above this point, the person is considered disease positive; below this point, the individual is considered disease negative. Thus, if the disease were potentially life-threatening or if a disease is relatively benign in terms of stigmatization, anxiety, and problems with treatment, the lower cutoff would be preferred. High cholesterol, if left untreated, could contribute to life-threatening cardiac disease. Additionally, it is benign in terms of stigmatization. Therefore, a lower cutoff point should be set. DIF:

Cognitive Level: Analyze (Analysis)

REF:

p. 203

14. During a screening, a test with a high specificity and low sensitivity is utilized. Which of the following issues could arise by using this test? a. Ethical issues b. Race issues c. Gender issues d. Cultural issues


ANS: A Misinterpretation caused by screening instruments is of great ethical concern. A difficult ethical issue in screening is determining whether the benefits received by those who receive correct results are worth the problems experienced by those who receive incorrect results. In this case, the high specificity of the test would result in low false-positive rates and would correctly identify nondiseased individuals. However, the low sensitivity would result in a high false-negative rate and therefore would miss a large number of people who are actually positive, resulting in ethical issues. DIF:

Cognitive Level: Apply (Application)

REF:

p. 200 | p. 202 | p. 203

15. A nurse is educating a group of community members about how hypertension screening is effective in reducing the rate of cardiovascular disease, thus reducing the expenses that are spent on management of this disease. Which of the following ratios is being described? a. Cost-disease analysis b. Cost-efficiency analysis c. Cost-benefit ratio analysis d. Cost-effectiveness analysis ANS: C Cost-benefit ratio analysis allows the comparison of various outcomes in monetary terms. The cost of the screening versus the cost of chronic care management is considered. DIF:

Cognitive Level: Apply (Application)

REF:

pp. 203-204

16. Which of the following analyses is used to determine the optimal use of resources to reach a predetermined constant end point or the desired health outcome? a. Cost-benefit ratio analysis b. Cost-effectiveness analysis c. Cost-efficiency analysis d. Cost-disease analysis ANS: B An analysis that determines the optimal use of resources to reach a predetermined, constant end-point or the desired health outcome is known as cost-effectiveness analysis. DIF:

Cognitive Level: Remember (Knowledge)

REF:

p. 204

17. Which of the following is an example of a screenable population for hypertension? a. High school students b. Professional hockey players c. High-level business executives attending an annual conference d. Cardiac rehabilitation clients ANS: C The objective of identifying a screenable population is to identify a high-risk group that, when tested, will yield a significant number of diseased individuals. The main criterion used to define an appropriate population is the definitive presence of risk factors related to the disorder. Most high-level business executives are middle-aged men with stressful jobs, placing them at high risk for heart disease. Thus, this would be the best example of a screenable population for hypertension. DIF:

Cognitive Level: Analyze (Analysis)

REF:

pp. 204-205

18. A nurse is assessing a low-income population in a community. Which of the following would be most appropriate for this population? a. HIV screening


b. c. d.

Blood pressure screening Colorectal cancer screening Breast cancer mammography screening

ANS: B According to a 2010 Gallup poll, ‘Low-income Americans are more likely than their high-income counterparts to say they have been diagnosed with each of the chronic conditions........the differences are largest for depression, high blood pressure, and diabetes......the high level of obesity among low-income Americans is likely a contributing factor in these differences.’ DIF:

Cognitive Level: Analyze (Analysis)

REF:

p. 205

19. A nurse is working at a women’s health clinic and is asked by a client when she should return for her next Pap smear. Which of the following resources would the nurse use to find the most current recommendations? a. National Health Information Center b. Healthy People 2020 c. U.S. Preventive Services Task Force (USPSTF) website d. Centers for Disease Control and Prevention (CDC) website ANS: C The most current information about recommendations for screening tests can be found on the Agency for Healthcare Research and Quality (AHRQ) U.S. Preventive Services Task Force (USPSTF) website. These recommendations evolve as new scientific evidence becomes available. DIF:

Cognitive Level: Apply (Application)

REF:

p. 206

20. A nurse is educating a 26-year-old, sexually active, female client about the screening tests that are now covered without a copayment or co-insurance because of the Affordable Care Act. Which of the following preventive services would the nurse include in this discussion? a. Tobacco use screening b. HIV screening c. Cervical cancer screening d. Breast cancer mammography screening ANS: B One result of the Affordable Care Act that was passed in 2010 was the movement toward prevention and health promotion. One result is that preventive services are required to be covered by new health insurance plans or policies. When covered by a network provider, HIV screening must be provided by insurance plans without copayment or co-insurance. DIF: REF:

Cognitive Level: Apply (Application) p. 206 | p. 207 (Box 9-5) | p. 208 (Box 9-6)

21. A nurse is planning a comprehensive health promotion activity to provide community members with a better opportunity to manage their own risk. Which of the following activities would be most appropriate for the nurse to perform? a. Obtaining blood pressures and cholesterol levels during a screening b. Obtaining blood pressures and family histories during a screening c. Obtaining blood pressures and discussing the importance of exercise during a screening d. Obtaining blood pressures, cholesterol levels, and glucose levels during a screening ANS: C


Providing health education during a screening falls under the Scope and Standards of Nursing Practice. Many chronic diseases are the result of health behaviors. The nurse’s role as educator is essential in the screening process because nurses provide individuals with the information necessary for choices that are made regarding healthy behavioral changes. Awareness is the first step in prevention. If awareness is combined with health education and health-promotion tools, people will have a better opportunity to manage their own risks. Thus, obtaining a person’s blood pressure while discussing the importance of exercise during a blood pressure screening is an example of a comprehensive health promotion activity. DIF:

Cognitive Level: Apply (Application)

REF:

p. 209

22. To screen for colorectal cancer, colonoscopy should be conducted every 10 years beginning at age: a. 30. b. 40. c. 50. d. 60. ANS: C It is recommended that men and women age 50 and older have a colonoscopy performed every 10 years to screen for colorectal cancer. DIF:

Cognitive Level: Remember (Knowledge)

REF:

p. 207 (Box 9-5)

23. For which of the following is Type 2 diabetes mellitus screening recommended? a. Overweight woman b. Teenager c. Man with hypertension d. Woman with a family history of Type 2 diabetes mellitus ANS: C The risk of developing Type 2 diabetes mellitus increases with age, obesity, and lack of exercise. Although the woman with a family history of Type 2 diabetes mellitus is at risk of developing the disease, screening is only recommended for those with hypertension. Therefore, the man with hypertension should be screened for Type 2 diabetes mellitus. DIF:

Cognitive Level: Apply (Application)

REF:

p. 207 (Box 9-5)

24. Which of the following women should be screened for breast cancer? a. A 35-year old woman with three children and no family history of breast cancer b. A 59-year-old woman with no children who still gets her period every month c. A 25-year-old woman with one child whom she gave birth to when she was a teenager d. A 17-year-old woman with one child who started menstruating at the age of 13 ANS: B In the United States, the incidence of breast cancer increases with age. Breast cancer mammography screenings are recommended every 1 to 2 years for women over 40. DIF:

Cognitive Level: Apply (Application)

REF:

p. 208 (Box 9-6)

25. A nurse is speaking to a women’s group in the community about the importance of completing mammography to screen for breast cancer. At which age should the nurse recommend that mammography begin? a. 18 b. 21 c. 35 d. 40


ANS: D It is recommended that all women age 40 and older have a mammography performed every 1 to 2 years to screen for breast cancer. DIF:

Cognitive Level: Understand (Comprehension) REF:

p. 208 (Box 9-6)

26. A nurse is reviewing the chart of a 15-year-old girl who has been sexually active since the age of 12. Which of the following findings would be of most concern? a. She does not perform self-breast exams. b. She has never had a Pap test. c. She had one HIV test performed at the age of 13. d. She does not use birth control pills. ANS: B Although self-breast exams are encouraged and considered an important aspect of breast health education, data regarding its efficacy is weak. Although she does not use birth control pills, there is no indication that she does not use other forms of birth control such as condoms that would also prevent against STDs. There is also no indication that she has had more than one sex partner and is at high risk for HIV infection. A cervical dysplasia screening is recommended for sexual active females. Therefore, the fact that she never had a Pap test to screen for cervical cancer and has been sexually active for the last 3 years is concerning. DIF:

Cognitive Level: Analyze (Analysis)

REF:

p. 208 (Box 9-7)

MULTIPLE RESPONSE 1. Which of the following is an example of screening? (select all that apply) a. Asking if someone performs self-breast exam b. Performing a self-breast exam c. Obtaining a mammogram d. Undergoing a needle biopsy ANS: B, C Screening is not considered a diagnostic measure. The ultimate goal could be curative, but more often, it is to prevent further development of the condition or disease. Screenings are done by oneself, or can be clinical, procedural, or lab based. Performing a self-breast exam and obtaining a mammogram are examples of screening. Asking if someone performs a self-breast exam may increase awareness but does not screen for disease. A needle biopsy would be diagnostic. DIF:

Cognitive Level: Apply (Application)

REF:

p. 196

2. A nurse is assessing a community and is trying to determine the appropriateness of conducting a Type 2 diabetes screening in the community. Which of the following questions would the nurse need to answer when making this decision? (select all that apply) a. Is Type 2 diabetes considered a community problem? b. What are the health benefits of screening for Type 2 diabetes? c. Can Type 2 diabetes be detected by screening? d. What is the cost-benefit ratio of implementing this screening? ANS: A, B, C The answers to the following three questions provide a basis for designating a disease as screenable or not screenable: Does the significance of the disorder warrant its consideration as a community problem? Can the disease by detected by screening? Should screening for the disease be done? DIF:

Cognitive Level: Apply (Application)

REF:

p. 198


Chapter 10: Health Education Edelman: Health Promotion Throughout the Life Span, 8th Edition MULTIPLE CHOICE 1. A nurse would like to assist Americans in improving their health. Which strategy would be most beneficial to improve the health of the American public? a. Encourage Americans to stop smoking. b. Lobby for state-of-the-art magnetic resonance imaging machines in all hospitals. c. Provide free medications for Americans. d. Offer free condoms to teenagers. ANS: A The greatest opportunity to improve the health of the American people lies in addressing unhealthy, personal, behavioral risk factors. Intensive lifestyle changes can be effective not only in preventing chronic diseases, but also in reversing their progression and significantly reducing health care costs. Improvement of the public’s health is more likely to come from behavior change than technology. Therefore, people deciding they are going to change their behavior and stop smoking are likely to lead to improvements in the health of the public. DIF:

Cognitive Level: Apply (Application)

REF:

p. 215

2. A nurse is using a health education component when teaching about smoking cessation. Which of the following actions is the nurse implementing? a. Setting a quit date for people in a smoking cessation class b. Providing education regarding the benefits of smoking cessation c. Allowing smokers to participate in a smoking cessation program only if they use the patch nicotine replacement system d. Encouraging attendees of a smoking cessation program to participate in a research study ANS: B Essential components of health education involve using teaching strategies, having learners maintain voluntary control over the decision to make changes in their actions, and focusing on behavior changes that have been found to improve heath. Providing smokers with the information regarding the benefits of smoking cessation allows them to make their own decision and provides them with information that is known to improve health. DIF:

Cognitive Level: Apply (Application)

REF:

pp. 215-216

3. Any combination of planned experiences based on sound theories that provide individuals, groups, and communities the opportunity to acquire the information and skills needed to make quality health decisions is known as health: a. promotion. b. counseling. c. education. d. knowledge. ANS: C Any combination of planned experiences based on sound theories that provide individuals, groups, and communities the opportunity to acquire the information and skills needed to make quality health decisions is known as health education. DIF:

Cognitive Level: Remember (Knowledge)

REF:

p. 216


4. An overweight woman joins a support group to help her lose weight. During her first session, the nurse explains the components of a healthy diet and discusses with the woman how she can eat out and still maintain a healthy diet. She asks the woman what her goal is and emphasizes that she herself is the key to success. What is the nurse promoting through the use of this strategy? a. Communication b. Values c. Advanced planning d. Empowerment ANS: D A goal of health education is empowerment. People who believe they can make a difference in their own health and who are included in decision making are more likely to make changes. By giving the woman the tools (education about a healthy diet) and involving her in the decision-making (set her own goal), the nurse empowers the woman to make a change in her eating habits. DIF:

Cognitive Level: Apply (Application)

REF:

p. 218

5. Which of the following illustrates that the objectives of health education and counseling are being met? a. Diabetic who attends a diabetes education program b. Diabetic who watches a video about self-administration of insulin c. Diabetic who starts taking his medications regularly d. Diabetic who is admitted in diabetic ketoacidosis ANS: C Health education encourages positive, informed changes in lifestyle behaviors that prevent acute and chronic disease, decrease disability, and enhance wellness. Two main objectives of health education and counseling are to change health behaviors and to improve health status. A diabetic who starts taking his medications regularly is an example of a behavior change intended to decrease disability. DIF:

Cognitive Level: Analyze (Analysis)

REF:

p. 218

6. Which of the following illustrates that the objectives of health education and counseling have been met? a. Asthmatic who has a decrease in emergency department visits b. Asthmatic who has been prescribed an albuterol inhaler c. Asthmatic who attends an asthma education program d. Asthmatic who visits the emergency department with an exacerbation ANS: A Health education encourages positive, informed changes in lifestyle behaviors that prevent acute and chronic disease, decrease disability, and enhance wellness. Two main objectives of health education and counseling are to change health behaviors and to improve health status. An asthmatic with a decrease in emergency department visits is an improvement in health status, which is a reflection of behavior changes used to prevent acute exacerbations of a chronic disease. DIF:

Cognitive Level: Analyze (Analysis)

REF:

p. 218

7. A teenager, who lives at home with her parents and school-aged brother, has been diagnosed with anorexia nervosa. Which of the following would be the most appropriate teaching goal for this family? a. Increased energy level b. Improved coping c. Enhanced self-esteem d. Facilitated family conversation ANS: B


The family plays an important role in health and illness. Understanding and intervening with the family is essential to promoting health and reducing risk. Health teaching includes all family members although the general teaching goal is the same for all, approaches and specific goals for each member will be different. An illness in one member of the family can alter the entire family’s ability to cope. As a result, improved coping is the most appropriate goal. DIF:

Cognitive Level: Analyze (Analysis)

REF:

pp. 218-219

8. A teenager, who lives at home with her parents and school-aged brother, has been diagnosed with anorexia nervosa. Which of the following would be the most appropriate teaching goal for this teenager? a. Increased energy level b. Improved coping c. Enhanced self-esteem d. Accomplished activities of daily living ANS: C Health teaching includes all family members; although the general teaching goal is the same for all, approaches and specific goals for each member will be different. Teenagers with anorexia nervosa often have decreased selfesteem. As a result, an appropriate goal for this teenager would be enhanced self-esteem. DIF:

Cognitive Level: Analyze (Analysis)

REF:

pp. 218-219

9. A nurse is using the health belief model as a framework when developing a community action campaign to increase the percentage of the population who receives the influenza vaccine. Which of the following considerations will need to be made? a. How empowerment can be used to motivate community members b. How modeling can be used in the community to increase public awareness c. Perceived susceptibility in the community about getting influenza d. Perceived stage of behavior change that exists in the community ANS: C The components of the health belief model that need to be considered are: individual perceptions or readiness for change; the value of health to the individual compared with other aspects of living; perceived susceptibility to a health problem, disease, or complications; perceived seriousness of the disease level threatening the achievement of certain goals or aims; risk factors to a disease attributed to heredity, race or culture, medical history, or other causes; perceived benefits of health action; perceived barriers to promotion action. DIF:

Cognitive Level: Analyze (Analysis)

REF:

p. 219

10. Which of the following emphasizes that an individual’s belief in being personally capable of performing the behavior is required to influence one’s own health? a. Social cognitive theory b. Self-efficacy theory c. Health belief model d. Transtheoretical model ANS: A The model that emphasizes an individual’s belief in being personally capable of performing the behavior required to influence one’s own health is known as the social cognitive theory. DIF:

Cognitive Level: Remember (Knowledge)

REF:

p. 219

11. A nurse is counseling a teenager who smokes one pack of cigarettes a day. The teenager states he likes to smoke with his friends and does not recognize the connection between his smoking and his asthma. When planning an intervention for this person, the nurse must first recognize that the teenager is in the: a. precontemplation stage of change.


b. c. d.

preparation stage of change. action stage of change. maintenance stage of change.

ANS: A A person in the precontemplation stage of change is not thinking about or considering quitting or adopting a behavior change within the next 6 months. Health education must be matched to the stage of change. This teenager does not see a connection between his smoking and his asthma. He likes to smoke and is therefore unlikely to make any changes at this time. DIF:

Cognitive Level: Apply (Application)

REF:

p. 220

12. An overweight woman is in the preparation stage of change. Which of the following interventions would be the most appropriate for the nurse to implement? a. Inform her of the health risks associated with being overweight. b. Prepare her for the setbacks. c. Develop a low-calorie/low-fat diet with her so she can follow it at home. d. Praise her for her recent successes. ANS: C A person in the preparation stage of change has made a change or is seriously thinking about making a change in the next month. At this stage, people make small or sporadic changes. Health education must be matched to the stage of change. During this stage, it is the educator’s role to provide the woman with the tools to move forward with her plan. Working on a diet together will decrease barriers to change by making it easier for the woman to follow a diet. Enlisting her cooperation achieves better results. DIF:

Cognitive Level: Analyze (Analysis)

REF:

p. 220

13. A nurse is providing education about safe driving with a group of newly licensed teenagers. Which factor would most likely influence a teenager’s decision to drive within the speed limit? a. A speed limit sign b. Having his parents telling him not to speed c. A previous traffic violation for speeding d. Seeing someone get pulled over for speeding ANS: C Effective health education requires an understanding of the influential factors affecting the individual’s decision making. These factors include values, beliefs, attitudes, life stresses, religion, previous experiences, and life goals. Having had a previous experience with receiving a speeding ticket may positively influence the teenager’s decision to drive within the speed limit because (1) he knows he is vulnerable to being caught and ticketed again, and (2) further tickets may have an effect on his ability to drive at all (e.g., his parents may punish him by not allowing him to drive, his driving privileges with the department of motor vehicles may be affected, or his insurance premium may be affected). Although seeing someone get pulled over may influence a person’s decision through modeling and observing others, teenagers believe they are invincible and therefore would probably not be effective in this age group. DIF:

Cognitive Level: Analyze (Analysis)

REF:

p. 220

14. A nurse is counseling a person who has recently started to receive treatment for alcohol abuse. Which factor would most negatively influence the person’s decision to stop abusing alcohol? a. He is a Catholic. b. His wife recently died from breast cancer. c. Buying alcohol is expensive. d. He was educated on the dangers of excessive drinking.


ANS: B Effective health education requires an understanding of the influential factors affecting the individual’s decisionmaking. These factors include values, beliefs, attitudes, life stresses, religion, previous experiences, and life goals. Although religion and education can influence a person’s decision, the stressor of his wife’s recent death is most likely to negatively influence his decision to stop abusing alcohol. DIF:

Cognitive Level: Analyze (Analysis)

REF:

p. 220

15. A nurse is educating a group of adults who are attempting smoking cessation. Which intervention would be most effective for the nurse to implement? a. Give the group members a handout on the dangers of smoking. b. Set a quit date with the group and determine which pharmacological method is most suitable for group members. c. Provide a month’s supply of nicotine replacement to use before the next visit. d. Have a guest speaker who has been diagnosed with lung cancer speak to the group. ANS: B Educators can increase a person’s motivation and capabilities to change by involving the person in the planning and goal setting. By picking a quit date and developing an action plan with the person, the educator is positively influencing the person’s motivation level. Education alone is not enough to influence change. DIF:

Cognitive Level: Analyze (Analysis)

REF: p. 220

16. The primary objective of social marketing is to: a. plan behavior. b. analyze behavior. c. limit behavior. d. change behavior. ANS: D The primary objective of social marketing is to change behavior. DIF:

Cognitive Level: Remember (Knowledge)

REF: pp. 221-222

17. A nurse distributes a demographic questionnaire to attendees of a weight management program. Which step in the teaching-learning process is being implemented? a. Assessment b. Development c. Implementation d. Evaluation ANS: A The assessment step in the teaching-learning process is comprised of assessing the students’ characteristics and needs. Obtaining demographic data from the attendees will help the nurse determine the characteristics of the students. DIF: Cognitive Level: Apply (Application) REF: pp. 222-223 | p. 222 (Box 10-6) 18. The goal for the educational session is for the individual to correctly demonstrate proper technique for checking blood sugar levels. Which of the following teaching strategies would be most effective? a. Provide the individual with one-on-one education about blood sugar monitoring. b. Provide the individual with a scenario about blood sugar monitoring. c. Provide the individual with the opportunity to practice blood sugar monitoring. d. Provide the individual with a blood sugar monitor.


ANS: C Teaching is directed toward one or more of the three learning domains. The goal of having the individual demonstrate the proper technique for checking blood sugar levels relates to the psychomotor domain and is best achieved through demonstration and practice. These strategies allow the nurse to provide the individual with feedback and encouragement. DIF:

Cognitive Level: Analyze (Analysis)

REF:

p. 223

19. The goal for the educational session is to have the individual explain when he should check his blood sugar level. Which of the following teaching strategies would be most effective for the nurse to implement? a. Provide the individual with one-on-one education about blood sugar monitoring. b. Provide the individual with a scenario about blood sugar monitoring. c. Provide the individual with the opportunity to practice blood sugar monitoring. d. Provide the individual with a blood sugar monitor. ANS: A Teaching is directed toward one or more of the three learning domains. The goal of having the individual explain when to monitor blood sugar levels relates to the cognitive domain, and it is best achieved by using strategies such as lecture, one-on-one education, and discussion. This allows the educator to provide the individual with a large amount of information at one time. DIF:

Cognitive Level: Analyze (Analysis)

REF:

p. 223

20. The goal is for the individual to verbalize the importance of checking blood sugar levels. Which of the following strategies would be best for the nurse to implement to achieve this goal? a. Provide one-on-one education about blood sugar monitoring. b. Provide the individual with a scenario about blood sugar monitoring. c. Provide the individual with the opportunity to practice blood sugar monitoring. d. Provide the individual with a blood sugar monitor. ANS: B Teaching is directed toward one or more of the three learning domains. The goals of having the individual verbalized the importance of checking blood sugar levels relates to the affective domain. Teaching strategies for this domain include role modeling and role playing. Such strategies allow the individual to formulate appropriate responses, attitudes, and feelings. DIF:

Cognitive Level: Analyze (Analysis)

REF:

p. 223

21. A nurse is developing learning objectives for an individual who has been placed on a low-fat, low-cholesterol diet. Which is an example of an appropriately written cognitive learning objective? a. The individual will understand the importance of a low-fat, low-cholesterol diet. b. The individual will correctly identify and purchase low-fat, low-cholesterol foods. c. The individual will believe that his heart will benefit from a low-fat, low-cholesterol diet. d. The individual will feel healthier if he follows a low-fat, low-cholesterol diet. ANS: B Cognitive learning refers to the development of new facts or concepts and builds on or applies knowledge to new situations. Objectives of this domain must incorporate action verbs that indicate observable learning. Identifying and purchasing low-fat, low-cholesterol foods demonstrates that the individual has learned which foods are low in fat and cholesterol. DIF:

Cognitive Level: Apply (Application)

REF:

p. 223 | p. 224 (Table 10-1)

22. A nurse is developing learning objectives for an individual who is becoming more physically active. Which of the following is an example of an appropriately written affective learning objective?


a. b. c. d.

The individual will understand the importance of daily exercise. The individual will demonstrate the importance of daily exercise. The individual will verbalize the importance of daily exercise. The individual will believe in the importance of daily exercise.

ANS: C Affective learning alludes to the recognition of values, religious and spiritual beliefs, family interaction patterns and relationships, and personal attitudes that affect decisions and the problem-solving process. Objectives of this domain must incorporate action verbs that indicate observable learning. Verbalizing the importance of daily exercise alludes to the recognition of the value of daily exercise to the individual. DIF:

Cognitive Level: Apply (Application)

REF:

p. 223 | p. 224 (Table 10-1)

23. A nurse is developing learning objectives for an individual who has been recently diagnosed with asthma. Which is an example of an appropriately written psychomotor learning objective? a. The individual will understand how to correctly use an inhaler. b. The individual will realize the importance of using an inhaler correctly. c. The individual will feel better when using the inhaler correctly. d. The individual will demonstrate proper use of the inhaler. ANS: D Psychomotor learning involves developing physical skills from simple to complex. Objectives of this domain must incorporate action verbs that indicate observable learning. Demonstrating proper use indicates that the individual has developed the necessary skills to use an inhaler correctly. DIF:

Cognitive Level: Apply (Application)

REF:

p. 223 | p. 224 (Table 10-1)

24. A nurse plans to use lectures, handouts, and a video to teach a group of teenagers the importance of safe sex practices. Which step in the teaching-learning process is the nurse demonstrating? a. Assessment b. Development c. Implementation d. Evaluation ANS: B The development of a teaching plan component of the teaching-learning process is comprised of developing the course content, teaching strategies, and learning action. Lectures, handouts, and videos are all examples of teaching strategies. DIF:

Cognitive Level: Apply (Application)

REF:

p. 224 | p. 222 (Box 10-6)

25. A nurse is providing health education about the importance of dental health to Latino residents in a community. Which of the following considerations should be made to promote health literacy within this program? a. Speak slowly and distinctly. b. Refer the client to an interpreter. c. Use photographs whenever possible. d. Consider methods to facilitate empowerment. ANS: A Strategies to promote health literacy include creating a shame-free environment (speaking slowly and distinctly), using clear and purposeful communication (being specific and concrete), communicating in a patient-centered manner (assess what is already known), and reinforcing the spoken word (using a variety of methods: photographs, audio, and drawings). DIF:

Cognitive Level: Apply (Application)

REF:

p. 217 (Box 10-2)


MULTIPLE RESPONSE 1. A nurse is developing health education content for an educational session with adults about the prevention of cardiovascular disease. Which of the following domains should be considered when developing the teaching plan? (select all that apply) a. Cognitive b. Psychomotor c. Gross motor d. Affective ANS: A, B, D The three domains of health education are cognitive, affective, and psychomotor. DIF:

Cognitive Level: Apply (Application)

REF:

p. 223

2. A nurse is providing a group presentation about the importance of healthy eating. Which of the following activities must be addressed to ensure an adequate learning climate? (select all that apply) a. Creating a sense of preparedness and organization b. Planning for alternative learning activities c. Assessing group needs d. Observing group interactions ANS: A, C, D Several activities need to be addressed in group presentations. They include creating a sense of preparedness and organization; anticipating group needs; assessing group needs; maintaining a high level of motivation and a sense of individualized attention and progression; and observing group interactions. DIF:

Cognitive Level: Apply (Application)

REF:

p. 225

3. A nurse is developing an educational session for adolescents related to avoidance of risky behaviors. Which of the following steps of the teaching-learning process will need to be considered? (select all that apply) a. Assessment b. Diagnosis c. Development d. Implementation ANS: A, C, D Steps in the teaching-learning process include assessment, development of learning outcomes and teaching plan, implementation of the teaching plan, and evaluation of expected outcomes. DIF:

Cognitive Level: Apply (Application)

REF:

p. 222 (Box 10-6)

Chapter 11: Nutrition Counseling for Health Promotion Edelman: Health Promotion Throughout the Life Span, 8th Edition MULTIPLE CHOICE 1. Which of the following statements about the current prevalence of obesity is accurate? a. The current generation of children may be less healthy and have a shorter life expectancy than their parents. b. The current incidence of obesity among American adults has stabilized. c. The current statistics show that greater than 75% of American adults are either overweight or obese. d. The current statistics report that about 50% of American children are either overweight or obese.


ANS: A Because of the increasing rates of obesity, unhealthy eating habits, and physical inactivity, we may see the first generation that will be less healthy and have a shorter life expectancy than their parents. The current statistics show that two in three American adults and one in three children are either overweight or obese. DIF:

Cognitive Level: Understand (Comprehension) REF:

p. 231

2. A nurse is working with a family that is experiencing food insecurity. Which of the following statements best describes this family? a. Members of this family are susceptible to food-borne illness. b. This family’s access to adequate food is limited by lack of money. c. This family is unable to purchase fresh fruits and vegetables. d. Members of this family are obese. ANS: B When a family experiences food insecurity it means that their access to adequate food is limited by a lack of money and other resources. Individuals and families may be more likely to be overweight or obese, potentially because the relatively lower cost of junk foods can promote overconsumption of calories. DIF: 3.

Cognitive Level: Apply (Application)

REF:

p. 234

Which of the following statements about the Dietary Reference Intakes (DRI) is correct? a. The DRI is used to monitor deficiencies in nutritional intakes of obese populations. b. The DRI is used to establish minimal amounts of nutrients needed to protect against nutrient deficiency. c. The DRI is used to plan and assess diets of healthy people. d. The DRI is used to set minimal guidelines to reduce risk of adverse outcomes from inadequate consumption of nutrients. ANS: C The DRI are a set of values for the dietary nutrient intakes of healthy people in the United States and Canada. They are used for planning and assessing diets. The four sets of values included in the DRI are Recommended Daily Allowance (RDA), Adequate Intake, Estimated Average Requirement, and Tolerable Upper Intake Levels. The previous RDA, which existed before 1997, established minimal amounts of nutrients needed to protect against nutrient deficiency. DIF:

Cognitive Level: Understand (Comprehension) REF:

p. 235

4. A community health nurse is using the Dietary Reference Intakes (DRIs) as a guide while developing an education presentation about nutrition for a community group. How will the use of the DRIs be helpful in developing this presentation? a. It considers the unique differences among children, pregnant women, and elderly people. b. It contains well-researched data that are reviewed on an annual basis. c. It is prescriptive in the amount of nutrients that are needed on a daily basis. d. It serves as an overall guideline for the population. ANS: D Dietary Reference Intakes may be used for individuals; however, ideally they are guidelines for population groups and apply over time. It is the trend that matters not the amount on a certain day. The DRIs are created even when limited data are available as this is deemed better than no guidance at all. The DRIs are only updated when deemed necessary. There are limited data relating to genetic diversity in the population or specific groups such as children, pregnant women, and elderly people. DIF:

Cognitive Level: Apply (Application)

REF:

p. 235


5. After the 2010 Dietary Guidelines were released, a community health nurse updated a nutrition presentation that was being used to educate various community groups about healthy eating habits. Which of the following information would have been added to the presentation? a. Consumption of increased amounts of carbohydrates b. Impact of the environment on the public’s food, drink, and activity choices c. Revised guidelines for the use of MyPyramid d. Inclusion of physical activity standards ANS: B For the first time, the 2010 Dietary Guidelines have an entire chapter devoted to addressing the impact of the broader food and physical environment on the public’s food, drink, and activity choices using the SocialEcological model. Physical activity standards were included in the 2005 guidelines. MyPyramid was replaced with MyPlate. There was a shift in food intake patterns emphasizing more vegetables, cooked dried beans, fruits, whole grains, nuts and seeds. DIF:

Cognitive Level: Apply (Application)

REF:

p. 236

6. A community health nurse is educating a community group about the 2010 Dietary Guidelines for Americans. Which of the following information would be included in this presentation? a. Limit consumption of dairy products. b. Purchase organic fruits and vegetables. c. Make at least half of your grains whole grains. d. Consume fish on a daily basis. ANS: C One of the principal messages for consumers is “make at least half of your grains whole grains.” Other messages include “switch to fat-free or low-fat (1%) milk” and “make half your plate fruits and vegetables.” There is not an essential message to consume fish on a daily basis. DIF:

Cognitive Level: Apply (Application)

REF:

p. 236

7. A person states to the nurse, “The guidelines for what we should eat as Americans is constantly changing, just when I got used to using MyPyramid they changed it to MyPlate. I wonder what will be next and if they really know what they are doing with all of this change.” Which of the following would be the best response from the nurse? a. “The USDA is constantly making changes in order to decrease the alarming rates of obesity in our country.” b. “MyPyramid was developed in 2005, so it was time for this graphic to be revised.” c. “The creation of MyPlate has decreased the need for additional teaching resources for nutrition.” d. “MyPlate serves as a better visual aid, reminding Americans what a healthy ‘plate’ looks like.” ANS: D MyPate was released in June 2011 providing Americans easy to understand and useful advice. MyPlate visually “reminds” consumers what a healthy “plate” looks like using a familiar place setting. It is not meant to stand alone as a teaching tool. DIF:

Cognitive Level: Analyze (Analysis)

REF:

pp. 238-239

8. A person asks the nurse if it is safe to use dietary supplements. Which of the following would be the best reply from the nurse? a. “It is important to be cautious when using any supplements.” b. “All dietary supplements must be approved by the FDA before they can be sold and are safe.” c. “No, dietary supplements have very limited regulations and should not be used.” d. “Be sure to read the product label before using the supplement to see what health claims have been made by the product.”


ANS: A The desirable way for the general public to obtain recommended levels of nutrients is by eating a variety of foods. Low-dose supplements that contain the recommended intakes for micronutrients appear to be generally safe. Caution is advised when using supplements, as natural is not always better. Dietary supplements are unregulated by the FDA in the sense that they do not require premarket review or approval by the FDA. DIF:

Cognitive Level: Apply (Application)

REF:

p. 240

9. A woman has asked the nurse about the benefits and risks of using ginseng to control her hot flashes associated with menopause. Which of the following governmental offices would most likely serve as a resource to answer the woman’s question? a. American Dietetic Association (ADA) b. National Center for Complementary and Alternative Medicine (NCCAM) c. Food and Drug Administration (FDA) d. United States Department of Agriculture (USDA) ANS: B The National Center for Complementary and Alternative Medicine (NCCAM) promotes and conducts the scientific study of the benefits and risks of dietary supplements, including medicinal herbs, in health maintenance and disease prevention. DIF:

Cognitive Level: Apply (Application)

REF:

p. 240

10. A 37-year-old woman asks the nurse if she should take a daily nutritional supplement. Which of the following would be the best reply by the nurse? a. “Postmenopausal women should take a calcium supplement to prevent osteoporosis.” b. “Many people use supplements as drugs instead of as supplements.” c. “Supplements can enhance the medications that you take.” d. “The best way to obtain nutrients is by eating a variety of foods.” ANS: D The desirable way for the general public to obtain nutrients is by eating a variety of foods. Dietary supplements needs to be used cautiously so that excess amounts are not consumed, which could lead to possible adverse effects. DIF:

Cognitive Level: Apply (Application)

REF:

pp. 240-241

11. A client who is taking Coumadin tells the nurse that she has started taking a vitamin E supplement as she has heard that this will help decrease the likelihood of her developing cancer. Which of the following would be the most appropriate response by the nurse? a. “Vitamin E supplements are unnecessary, so I don’t recommend you use this supplement.” b. “Vitamin E can interfere with the actions of vitamin K and enhance the effects of Coumadin .” c. “Vitamin E supplements can interfere with the absorption of iron and create additional health problems.” d. “Vitamin E must be used cautiously, as it is easy to experience a toxicity of this nutrient.” ANS: B High doses of vitamin E can interfere with vitamin K action and enhance the effect of Coumadin as one of the anticoagulant drugs. Because of this interaction, she should probably not be taking a supplement of vitamin E while taking the Coumadin. If she continues on the supplement, it may be necessary to adjust the dosage of Coumadin the woman is prescribed. DIF:

Cognitive Level: Apply (Application)

REF:

p. 241


12. A nurse is investigating an outbreak of a food-borne illness occurring at a local school. Which of the following would contribute to a chemical contamination resulting in food-borne illness? a. Unintended allergen added to food b. Bacteria on the countertops c. Pesticide on the fresh fruit d. Metal shavings in the food from a dull can opener ANS: C Food-borne illness is classified according to the source of its contamination. Food contaminants may be categorized as biological (bacteria), chemical (pesticides), or physical (metal shavings and unintended allergens). DIF:

Cognitive Level: Apply (Application)

REF:

p. 242

13. A school nurse is preparing to discuss food safety practices with high school students enrolled in a family and consumer science “foods” course. Which of the following information would be included in the presentation? a. Wash hands thoroughly with running warm water with soap for 30 seconds. b. Scrub firm services of all fruits and vegetables with a produce brush before eating. c. Pay close attention to assure that meat is cooked all of the way through. d. Refrigerate leftovers within 30 minutes of serving. ANS: A There are four principles at the cornerstones of Fight BAC!, a national food safety education campaign (Clean, Separate, Cook, and Chill). Clean—wash all parts of hands thoroughly with running warm water with soap and friction for about 20-30 seconds. Scrub firm surfaces like cantaloupe and cucumbers with a produce brush, other fresh fruits and vegetables can be rinsed under running water. A food thermometer should be used to make sure that food is cooked all the way through. Leftovers should be refrigerated within 2 hours or 1 hour in temperatures greater than 90° F. DIF:

Cognitive Level: Apply (Application)

REF:

p. 243

14. A community health nurse is working with a low-income family who is experiencing food insecurity. To assist the family in getting nutritious foods in their home, the family should be referred to which of the following programs? a. Nutrition Service Incentive Program (NSIP) b. Supplemental Nutrition for Women Infants and Children (WIC) c. National School Lunch Program (NSLP) d. Supplemental Nutrition Assistance Program (SNAP) ANS: D Supplemental Nutrition Assistance Program (SNAP) supplements the food-buying power of eligible low-income households and is the foundation of the Nations’ nutrition safety net. The program is designed to help lowincome families and individuals purchase nutritionally adequate foods. DIF:

Cognitive Level: Apply (Application)

REF:

pp. 244-245

15. A school nurse is discussing the income eligibility requirements for free and reduced-price school meals with a family who is new to the school district. Which of the following statements would the nurse most likely make? a. “If the employed member of your family earns less than $50,000 per year, your child will qualify for reduced-price meals.” b. “If your family’s income is less than 185% above the federal poverty level, your child will qualify for free or reduced-price meals.” c. “If you have more than five members living in your household, your child will qualify for reducedprice meals.” d. “If your family has an income above the federal poverty level, your child will not qualify for free or reduced-price meals.”


ANS: B Children from families with incomes greater than 130% to 185% of the federal poverty level are eligible for reduced-price school meals. Children from families with incomes at or less than 130% of the federal poverty level are eligible for free school meals. Consideration is given to both income level and number of members living in the household when interpreting the federal poverty guidelines. DIF:

Cognitive Level: Apply (Application)

REF:

p. 245

16. A nurse is counseling a person with heart disease. Which of the following statements made by the individual would require the nurse to provide additional education? a. “I drink skim milk on a regular basis.” b. “I try not to eat foods that have added salt on the label.” c. “I eat American cheese about once every 2 weeks.” d. “I add salt to many of the food that I eat to enhance the flavor.” ANS: D The 2006 AHA diet and lifestyle recommendations include the suggestion to reduce salt intake by comparing the sodium content of similar products and choosing products with less salt or sodium; choosing versions of processed foods, including cereals and baked goods, that are reduced in salt or sodium; and limited condiments. Sodium ingestion leads to retention of fluids and may be harmful to the cardiovascular system of someone who has heart disease. DIF:

Cognitive Level: Analyze (Analysis)

REF:

p. 248

17. A 45-year-old man has been diagnosed with metabolic syndrome. Which of the following characteristics would this man exhibit? a. Blood pressure: 146/88 mm Hg b. Waist measurement: 36 inches c. HDL: 48 mg/dL d. Triglycerides: 140 mg/dL ANS: A Metabolic syndrome is diagnosed when a person has three or more of these factors: excessive abdominal fat, as indicated by too large a waist measurement (greater than 40 inches in men); elevated blood pressure (higher than 130/85 mm Hg), low HDL level (lower than 40 mg/dL), and elevated triglyceride level (higher than 150 mg/dL). DIF:

Cognitive Level: Analyze (Analysis)

REF:

p. 249

18. Which of the following individuals has a modifiable, nutrition-related risk factor for stroke? a. African American man, age 65, who has a low salt intake b. White woman, age 32, who follows the MyPlate recommendations c. Russian man, age 40, who drinks 3 oz of vodka daily d. Hispanic man, age 42, who exercises regularly ANS: C Modifiable risk factors are those that the person can control. Race, age, and gender are nonmodifiable risk factors. Habitual alcohol intake is a modifiable risk factor for stroke. DIF:

Cognitive Level: Analyze (Analysis)

REF:

p. 250

19. A community health nurse is providing education about the “Dietary Approaches to Stop Hypertension” (DASH) eating plan to people attending cardiac rehabilitation. Which of the following recommendations would be included in this presentation? a. The use of this dietary plan has the same health effects as a daily exercise program. b. The use of this dietary plan will eliminate the need for medication to treat hypertension.


c. d.

This dietary plan is excludes the consumption of processed foods. This dietary plan includes eating greater amounts of fruits and vegetables.

ANS: D Clinical studies show that blood pressure can be lowered by following the DASH eating plan. The DASH eating plan includes abundant fruits and vegetables, more fat-free or low-fat milk and milk products, whole grains, fish, poultry, beans, seeds, and nuts. Depending on the severity of hypertension, the plan may allow for a reduction or elimination of medication. DIF:

Cognitive Level: Apply (Application)

REF:

pp. 250-251

20. A man states that he would like to change his dietary habits to decrease his risk of developing cancer in the future. Which of the following statements would be the best response for the nurse? a. b. c. d.

“Consume at least cups of fruits and vegetables each day.” “Dietary habits have minimal effect in reducing your risk of developing cancer.” “A glass of red wine should be consumed daily.” “Dietary supplements of specific nutrients will maximize the effects of dietary changes.”

ANS: A Eating at least cups of a variety of fruits and vegetables can help protect against cancers at many sites, particularly for cancers of the gastrointestinal and respiratory tracks. Following the dietary recommendations included in the 2010 Dietary guidelines for Americans and MyPlate are consistent with those recommended by the ACS. Food appears to have a protective effect by itself that dietary supplements do not provide. DIF:

Cognitive Level: Apply (Application)

REF:

pp. 251-252

21. Which of the following people is at greatest risk for a hip fracture secondary to osteoporosis? a. A 66-year-old White woman b. A 66-year-old African American woman c. A 66-year-old White man d. A 66-year-old African American man ANS: A White postmenopausal women have the greatest prevalence of osteoporosis and incidence of hip fracture by gender and race. DIF:

Cognitive Level: Apply (Application)

REF:

p. 252

22. A school nurse is using height and weight measurements to calculate the body mass index (BMI) of the students. How will the nurse use these data? a. BMI can be correlated to a specific percentage of body fat. b. BMI can be used to determine if a child is obese or underweight. c. BMI can be used to determine if a child will develop Type 2 diabetes. d. BMI can be compared with others of the same ethnic background. ANS: B Body mass index (BMI) is a helpful indicator of obesity and being underweight. Height and weight measurements are needed in the calculation to determine BMI. BMI is a good indicator of body fat, but it cannot be interpreted as a specific percentage of body fat. BMI is used to screen and monitor a population to detect the risk of health or nutritional disorders. In an individual, other data must be used to determine if a high BMI is associated with increased risk of disease; BMI alone is not diagnostic. The growth charts released in 2000 by the CDC more accurately reflect the nation’s cultural and racial diversity, but there is not a different chart for each ethnic background.


DIF:

Cognitive Level: Apply (Application)

REF:

p. 255

23. A community health nurse is educating health care providers in a local clinic about steps to take to help their clients improve their health-related behaviors. Which of the following would be included as the nurse teaches about the acronym LEARN? a. Explain how small changes can affect health. b. Acknowledge that action needs to be taken. c. Negotiate an agreement. d. Refer to community resources. ANS: C LEARN refers to the steps nurses can take to help the person who needs to improve health-related behavior. L = Listen with sympathy and understanding to the person’s perception of the problem, E = Explain personal perceptions of the problem, A = Acknowledge and discuss differences and similarities, R = Recommend treatment, N = Negotiate an agreement. DIF:

Cognitive Level: Apply (Application)

REF:

p. 256

24. Which of the following individuals would be at greatest risk to develop Type 2 diabetes? a. White, 50-year-old woman, obese, second child weighed 10 pounds at birth b. African American, 55-year-old man, hypertension, exercises regularly c. Hispanic, 22-year-old woman, healthy weight, family history of diabetes d. White, 75-year-old man, history of prostate cancer and high triglycerides ANS: A Risk factors for Type 2 diabetes include: having prediabetes (impaired glucose tolerance [IGT] and/or impaired fasting glucose [IFG]), age greater than 45, being overweight or obese, not exercising regularly, having a family history of diabetes, a personal history of gestational diabetes or a baby of at least 9 pounds at birth, other risk factors such as high blood pressure, a low HDL cholesterol or high triglycerides, and one’s race and ethnicity. The White, 50-year-old woman who is obese and gave birth to a child weighing 10 pounds has the most risk factors of the clients described. DIF:

Cognitive Level: Analyze (Analysis)

REF:

p. 256

25. A nurse is counseling a person who was recently diagnosed with diabetes about how to prevent the complications of this disease. Which of the following interventions would be stressed the most by the nurse? a. Adjusting sliding scale insulin therapy as needed b. Taking oral hypoglycemic medication as prescribed c. Monitoring blood glucose daily d. Adhering to medical nutrition therapy ANS: D Medicine may be used to control blood glucose level, and measuring the blood glucose daily helps in controlling its level. But the key to maintaining diabetic control is individualized medical nutrition therapy, the purpose of which is to delay or prevent diabetic complications. DIF:

Cognitive Level: Apply (Application)

REF:

pp. 257-258

26. A nurse is conducting an initial nutritional assessment on a person who is HIV positive. Which of the following questions would be most important for the nurse to ask? a. What fresh fruits and vegetables do you like to eat? b. Are you taking a daily multivitamin? c. How often do you eat at restaurants? d. Do you have any questions about your current medications?


ANS: B Evidence suggests that micronutrient deficiencies are common among those with HIV and that those not taking vitamin supplementation have increased morbidity and mortality. Thus, asking about use of a daily multivitamin is the most important information for the nurse to collect. A nutrient-dense, protein-rich, well-balanced diet should be stressed as well as vitamin and mineral supplementation. Because of an altered immune function, they get sicker when exposed to food-borne organisms, so discussion about how to avoid food-borne illness is also important. DIF:

Cognitive Level: Analyze (Analysis)

REF:

p. 259

MULTIPLE RESPONSE 1. Which of the following causes of death is/are directly associated with diet? (select all that apply) a. Diabetes mellitus b. Cirrhosis of the liver c. Stroke d. Coronary heart disease (CHD) ANS: A, C, D Four leading causes of death directly associated with diet are coronary heart disease (CHD), some types of cancer, stroke, and diabetes mellitus. Four more major causes of death—accidents, cirrhosis of the liver, suicide, and homicide—are associated with excessive alcohol intake. DIF:

Cognitive Level: Remember (Knowledge)

REF:

p. 231

2. A community health nurse is preparing an educational brochure about improving the nutritional behaviors of Americans. Which of the following topics would most likely be included in the brochure? (select all that apply) a. Increasing daily intake of milk and dairy products b. Decreasing daily sodium intake c. Increasing daily intake of fruits and vegetables d. Decreasing daily intake of sugar-sweetened drinks ANS: B, C, D Fewer than 15% of adults and 10% of adolescents eat the recommended amounts of fruits and vegetables every day. Sixty-three percent of adults and 84% of adolescents consume at least one sugar-sweetened drink each day. Most American adults consume more than twice the recommended average daily sodium intake level. DIF:

Cognitive Level: Apply (Application)

REF:

pp. 233-234

3. A community health nurse is developing educational materials related to nutrition. Which of the following resources would be most appropriate for the nurse to use? (select all that apply) a. Recommended dietary allowances (RDAs) b. MyPyramid c. Dietary reference intakes (DRIs) d. MyPlate ANS: C, D The most current food and nutrition recommendations are found using the dietary reference intakes (DRIs) and MyPlate. These two resources replaced the previously issued recommended dietary allowances (RDAs) and MyPyramid. DIF:

Cognitive Level: Analyze (Analysis)

REF:

p. 234

4. A person has been diagnosed with salmonellosis. Which of the following symptoms would the person most likely exhibit? (select all that apply) a. Bloody diarrhea


b. c. d.

Vomiting Hypoactive bowel sounds Abdominal cramping

ANS: B, D Symptoms of salmonellosis include abdominal cramping, mild to severe diarrhea, nausea, vomiting, and fever within 8 to 72 hours after infection. Bloody diarrhea is more likely seen with E. coli infection. DIF:

Cognitive Level: Apply (Application)

REF:

p. 242

Chapter 11: Nutrition Counseling for Health Promotion Edelman: Health Promotion Throughout the Life Span, 8th Edition MULTIPLE CHOICE 1. Which of the following statements about the current prevalence of obesity is accurate? a. The current generation of children may be less healthy and have a shorter life expectancy than their parents. b. The current incidence of obesity among American adults has stabilized. c. The current statistics show that greater than 75% of American adults are either overweight or obese. d. The current statistics report that about 50% of American children are either overweight or obese. ANS: A Because of the increasing rates of obesity, unhealthy eating habits, and physical inactivity, we may see the first generation that will be less healthy and have a shorter life expectancy than their parents. The current statistics show that two in three American adults and one in three children are either overweight or obese. DIF:

Cognitive Level: Understand (Comprehension) REF:

p. 231

2. A nurse is working with a family that is experiencing food insecurity. Which of the following statements best describes this family? a. Members of this family are susceptible to food-borne illness. b. This family’s access to adequate food is limited by lack of money. c. This family is unable to purchase fresh fruits and vegetables. d. Members of this family are obese. ANS: B When a family experiences food insecurity it means that their access to adequate food is limited by a lack of money and other resources. Individuals and families may be more likely to be overweight or obese, potentially because the relatively lower cost of junk foods can promote overconsumption of calories. DIF: 3.

Cognitive Level: Apply (Application)

REF:

p. 234

Which of the following statements about the Dietary Reference Intakes (DRI) is correct? a. The DRI is used to monitor deficiencies in nutritional intakes of obese populations. b. The DRI is used to establish minimal amounts of nutrients needed to protect against nutrient deficiency. c. The DRI is used to plan and assess diets of healthy people. d. The DRI is used to set minimal guidelines to reduce risk of adverse outcomes from inadequate consumption of nutrients. ANS: C


The DRI are a set of values for the dietary nutrient intakes of healthy people in the United States and Canada. They are used for planning and assessing diets. The four sets of values included in the DRI are Recommended Daily Allowance (RDA), Adequate Intake, Estimated Average Requirement, and Tolerable Upper Intake Levels. The previous RDA, which existed before 1997, established minimal amounts of nutrients needed to protect against nutrient deficiency. DIF:

Cognitive Level: Understand (Comprehension) REF:

p. 235

4. A community health nurse is using the Dietary Reference Intakes (DRIs) as a guide while developing an education presentation about nutrition for a community group. How will the use of the DRIs be helpful in developing this presentation? a. It considers the unique differences among children, pregnant women, and elderly people. b. It contains well-researched data that are reviewed on an annual basis. c. It is prescriptive in the amount of nutrients that are needed on a daily basis. d. It serves as an overall guideline for the population. ANS: D Dietary Reference Intakes may be used for individuals; however, ideally they are guidelines for population groups and apply over time. It is the trend that matters not the amount on a certain day. The DRIs are created even when limited data are available as this is deemed better than no guidance at all. The DRIs are only updated when deemed necessary. There are limited data relating to genetic diversity in the population or specific groups such as children, pregnant women, and elderly people. DIF:

Cognitive Level: Apply (Application)

REF:

p. 235

5. After the 2010 Dietary Guidelines were released, a community health nurse updated a nutrition presentation that was being used to educate various community groups about healthy eating habits. Which of the following information would have been added to the presentation? a. Consumption of increased amounts of carbohydrates b. Impact of the environment on the public’s food, drink, and activity choices c. Revised guidelines for the use of MyPyramid d. Inclusion of physical activity standards ANS: B For the first time, the 2010 Dietary Guidelines have an entire chapter devoted to addressing the impact of the broader food and physical environment on the public’s food, drink, and activity choices using the SocialEcological model. Physical activity standards were included in the 2005 guidelines. MyPyramid was replaced with MyPlate. There was a shift in food intake patterns emphasizing more vegetables, cooked dried beans, fruits, whole grains, nuts and seeds. DIF:

Cognitive Level: Apply (Application)

REF:

p. 236

6. A community health nurse is educating a community group about the 2010 Dietary Guidelines for Americans. Which of the following information would be included in this presentation? a. Limit consumption of dairy products. b. Purchase organic fruits and vegetables. c. Make at least half of your grains whole grains. d. Consume fish on a daily basis. ANS: C One of the principal messages for consumers is “make at least half of your grains whole grains.” Other messages include “switch to fat-free or low-fat (1%) milk” and “make half your plate fruits and vegetables.” There is not an essential message to consume fish on a daily basis. DIF:

Cognitive Level: Apply (Application)

REF:

p. 236


7. A person states to the nurse, “The guidelines for what we should eat as Americans is constantly changing, just when I got used to using MyPyramid they changed it to MyPlate. I wonder what will be next and if they really know what they are doing with all of this change.” Which of the following would be the best response from the nurse? a. “The USDA is constantly making changes in order to decrease the alarming rates of obesity in our country.” b. “MyPyramid was developed in 2005, so it was time for this graphic to be revised.” c. “The creation of MyPlate has decreased the need for additional teaching resources for nutrition.” d. “MyPlate serves as a better visual aid, reminding Americans what a healthy ‘plate’ looks like.” ANS: D MyPate was released in June 2011 providing Americans easy to understand and useful advice. MyPlate visually “reminds” consumers what a healthy “plate” looks like using a familiar place setting. It is not meant to stand alone as a teaching tool. DIF:

Cognitive Level: Analyze (Analysis)

REF:

pp. 238-239

8. A person asks the nurse if it is safe to use dietary supplements. Which of the following would be the best reply from the nurse? a. “It is important to be cautious when using any supplements.” b. “All dietary supplements must be approved by the FDA before they can be sold and are safe.” c. “No, dietary supplements have very limited regulations and should not be used.” d. “Be sure to read the product label before using the supplement to see what health claims have been made by the product.” ANS: A The desirable way for the general public to obtain recommended levels of nutrients is by eating a variety of foods. Low-dose supplements that contain the recommended intakes for micronutrients appear to be generally safe. Caution is advised when using supplements, as natural is not always better. Dietary supplements are unregulated by the FDA in the sense that they do not require premarket review or approval by the FDA. DIF:

Cognitive Level: Apply (Application)

REF:

p. 240

9. A woman has asked the nurse about the benefits and risks of using ginseng to control her hot flashes associated with menopause. Which of the following governmental offices would most likely serve as a resource to answer the woman’s question? a. American Dietetic Association (ADA) b. National Center for Complementary and Alternative Medicine (NCCAM) c. Food and Drug Administration (FDA) d. United States Department of Agriculture (USDA) ANS: B The National Center for Complementary and Alternative Medicine (NCCAM) promotes and conducts the scientific study of the benefits and risks of dietary supplements, including medicinal herbs, in health maintenance and disease prevention. DIF:

Cognitive Level: Apply (Application)

REF:

p. 240

10. A 37-year-old woman asks the nurse if she should take a daily nutritional supplement. Which of the following would be the best reply by the nurse? a. “Postmenopausal women should take a calcium supplement to prevent osteoporosis.” b. “Many people use supplements as drugs instead of as supplements.” c. “Supplements can enhance the medications that you take.” d. “The best way to obtain nutrients is by eating a variety of foods.”


ANS: D The desirable way for the general public to obtain nutrients is by eating a variety of foods. Dietary supplements needs to be used cautiously so that excess amounts are not consumed, which could lead to possible adverse effects. DIF:

Cognitive Level: Apply (Application)

REF:

pp. 240-241

11. A client who is taking Coumadin tells the nurse that she has started taking a vitamin E supplement as she has heard that this will help decrease the likelihood of her developing cancer. Which of the following would be the most appropriate response by the nurse? a. “Vitamin E supplements are unnecessary, so I don’t recommend you use this supplement.” b. “Vitamin E can interfere with the actions of vitamin K and enhance the effects of Coumadin .” c. “Vitamin E supplements can interfere with the absorption of iron and create additional health problems.” d. “Vitamin E must be used cautiously, as it is easy to experience a toxicity of this nutrient.” ANS: B High doses of vitamin E can interfere with vitamin K action and enhance the effect of Coumadin as one of the anticoagulant drugs. Because of this interaction, she should probably not be taking a supplement of vitamin E while taking the Coumadin. If she continues on the supplement, it may be necessary to adjust the dosage of Coumadin the woman is prescribed. DIF:

Cognitive Level: Apply (Application)

REF:

p. 241

12. A nurse is investigating an outbreak of a food-borne illness occurring at a local school. Which of the following would contribute to a chemical contamination resulting in food-borne illness? a. Unintended allergen added to food b. Bacteria on the countertops c. Pesticide on the fresh fruit d. Metal shavings in the food from a dull can opener ANS: C Food-borne illness is classified according to the source of its contamination. Food contaminants may be categorized as biological (bacteria), chemical (pesticides), or physical (metal shavings and unintended allergens). DIF:

Cognitive Level: Apply (Application)

REF:

p. 242

13. A school nurse is preparing to discuss food safety practices with high school students enrolled in a family and consumer science “foods” course. Which of the following information would be included in the presentation? a. Wash hands thoroughly with running warm water with soap for 30 seconds. b. Scrub firm services of all fruits and vegetables with a produce brush before eating. c. Pay close attention to assure that meat is cooked all of the way through. d. Refrigerate leftovers within 30 minutes of serving. ANS: A There are four principles at the cornerstones of Fight BAC!, a national food safety education campaign (Clean, Separate, Cook, and Chill). Clean—wash all parts of hands thoroughly with running warm water with soap and friction for about 20-30 seconds. Scrub firm surfaces like cantaloupe and cucumbers with a produce brush, other fresh fruits and vegetables can be rinsed under running water. A food thermometer should be used to make sure that food is cooked all the way through. Leftovers should be refrigerated within 2 hours or 1 hour in temperatures greater than 90° F. DIF:

Cognitive Level: Apply (Application)

REF:

p. 243


14. A community health nurse is working with a low-income family who is experiencing food insecurity. To assist the family in getting nutritious foods in their home, the family should be referred to which of the following programs? a. Nutrition Service Incentive Program (NSIP) b. Supplemental Nutrition for Women Infants and Children (WIC) c. National School Lunch Program (NSLP) d. Supplemental Nutrition Assistance Program (SNAP) ANS: D Supplemental Nutrition Assistance Program (SNAP) supplements the food-buying power of eligible low-income households and is the foundation of the Nations’ nutrition safety net. The program is designed to help lowincome families and individuals purchase nutritionally adequate foods. DIF:

Cognitive Level: Apply (Application)

REF:

pp. 244-245

15. A school nurse is discussing the income eligibility requirements for free and reduced-price school meals with a family who is new to the school district. Which of the following statements would the nurse most likely make? a. “If the employed member of your family earns less than $50,000 per year, your child will qualify for reduced-price meals.” b. “If your family’s income is less than 185% above the federal poverty level, your child will qualify for free or reduced-price meals.” c. “If you have more than five members living in your household, your child will qualify for reducedprice meals.” d. “If your family has an income above the federal poverty level, your child will not qualify for free or reduced-price meals.” ANS: B Children from families with incomes greater than 130% to 185% of the federal poverty level are eligible for reduced-price school meals. Children from families with incomes at or less than 130% of the federal poverty level are eligible for free school meals. Consideration is given to both income level and number of members living in the household when interpreting the federal poverty guidelines. DIF:

Cognitive Level: Apply (Application)

REF:

p. 245

16. A nurse is counseling a person with heart disease. Which of the following statements made by the individual would require the nurse to provide additional education? a. “I drink skim milk on a regular basis.” b. “I try not to eat foods that have added salt on the label.” c. “I eat American cheese about once every 2 weeks.” d. “I add salt to many of the food that I eat to enhance the flavor.” ANS: D The 2006 AHA diet and lifestyle recommendations include the suggestion to reduce salt intake by comparing the sodium content of similar products and choosing products with less salt or sodium; choosing versions of processed foods, including cereals and baked goods, that are reduced in salt or sodium; and limited condiments. Sodium ingestion leads to retention of fluids and may be harmful to the cardiovascular system of someone who has heart disease. DIF:

Cognitive Level: Analyze (Analysis)

REF:

p. 248

17. A 45-year-old man has been diagnosed with metabolic syndrome. Which of the following characteristics would this man exhibit? a. Blood pressure: 146/88 mm Hg b. Waist measurement: 36 inches c. HDL: 48 mg/dL d. Triglycerides: 140 mg/dL


ANS: A Metabolic syndrome is diagnosed when a person has three or more of these factors: excessive abdominal fat, as indicated by too large a waist measurement (greater than 40 inches in men); elevated blood pressure (higher than 130/85 mm Hg), low HDL level (lower than 40 mg/dL), and elevated triglyceride level (higher than 150 mg/dL). DIF:

Cognitive Level: Analyze (Analysis)

REF:

p. 249

18. Which of the following individuals has a modifiable, nutrition-related risk factor for stroke? a. African American man, age 65, who has a low salt intake b. White woman, age 32, who follows the MyPlate recommendations c. Russian man, age 40, who drinks 3 oz of vodka daily d. Hispanic man, age 42, who exercises regularly ANS: C Modifiable risk factors are those that the person can control. Race, age, and gender are nonmodifiable risk factors. Habitual alcohol intake is a modifiable risk factor for stroke. DIF:

Cognitive Level: Analyze (Analysis)

REF:

p. 250

19. A community health nurse is providing education about the “Dietary Approaches to Stop Hypertension” (DASH) eating plan to people attending cardiac rehabilitation. Which of the following recommendations would be included in this presentation? a. The use of this dietary plan has the same health effects as a daily exercise program. b. The use of this dietary plan will eliminate the need for medication to treat hypertension. c. This dietary plan is excludes the consumption of processed foods. d. This dietary plan includes eating greater amounts of fruits and vegetables. ANS: D Clinical studies show that blood pressure can be lowered by following the DASH eating plan. The DASH eating plan includes abundant fruits and vegetables, more fat-free or low-fat milk and milk products, whole grains, fish, poultry, beans, seeds, and nuts. Depending on the severity of hypertension, the plan may allow for a reduction or elimination of medication. DIF:

Cognitive Level: Apply (Application)

REF:

pp. 250-251

20. A man states that he would like to change his dietary habits to decrease his risk of developing cancer in the future. Which of the following statements would be the best response for the nurse? a. b. c. d.

“Consume at least cups of fruits and vegetables each day.” “Dietary habits have minimal effect in reducing your risk of developing cancer.” “A glass of red wine should be consumed daily.” “Dietary supplements of specific nutrients will maximize the effects of dietary changes.”

ANS: A Eating at least cups of a variety of fruits and vegetables can help protect against cancers at many sites, particularly for cancers of the gastrointestinal and respiratory tracks. Following the dietary recommendations included in the 2010 Dietary guidelines for Americans and MyPlate are consistent with those recommended by the ACS. Food appears to have a protective effect by itself that dietary supplements do not provide. DIF:

Cognitive Level: Apply (Application)

REF:

pp. 251-252

21. Which of the following people is at greatest risk for a hip fracture secondary to osteoporosis? a. A 66-year-old White woman b. A 66-year-old African American woman


c. d.

A 66-year-old White man A 66-year-old African American man

ANS: A White postmenopausal women have the greatest prevalence of osteoporosis and incidence of hip fracture by gender and race. DIF:

Cognitive Level: Apply (Application)

REF:

p. 252

22. A school nurse is using height and weight measurements to calculate the body mass index (BMI) of the students. How will the nurse use these data? a. BMI can be correlated to a specific percentage of body fat. b. BMI can be used to determine if a child is obese or underweight. c. BMI can be used to determine if a child will develop Type 2 diabetes. d. BMI can be compared with others of the same ethnic background. ANS: B Body mass index (BMI) is a helpful indicator of obesity and being underweight. Height and weight measurements are needed in the calculation to determine BMI. BMI is a good indicator of body fat, but it cannot be interpreted as a specific percentage of body fat. BMI is used to screen and monitor a population to detect the risk of health or nutritional disorders. In an individual, other data must be used to determine if a high BMI is associated with increased risk of disease; BMI alone is not diagnostic. The growth charts released in 2000 by the CDC more accurately reflect the nation’s cultural and racial diversity, but there is not a different chart for each ethnic background. DIF:

Cognitive Level: Apply (Application)

REF:

p. 255

23. A community health nurse is educating health care providers in a local clinic about steps to take to help their clients improve their health-related behaviors. Which of the following would be included as the nurse teaches about the acronym LEARN? a. Explain how small changes can affect health. b. Acknowledge that action needs to be taken. c. Negotiate an agreement. d. Refer to community resources. ANS: C LEARN refers to the steps nurses can take to help the person who needs to improve health-related behavior. L = Listen with sympathy and understanding to the person’s perception of the problem, E = Explain personal perceptions of the problem, A = Acknowledge and discuss differences and similarities, R = Recommend treatment, N = Negotiate an agreement. DIF:

Cognitive Level: Apply (Application)

REF:

p. 256

24. Which of the following individuals would be at greatest risk to develop Type 2 diabetes? a. White, 50-year-old woman, obese, second child weighed 10 pounds at birth b. African American, 55-year-old man, hypertension, exercises regularly c. Hispanic, 22-year-old woman, healthy weight, family history of diabetes d. White, 75-year-old man, history of prostate cancer and high triglycerides ANS: A


Risk factors for Type 2 diabetes include: having prediabetes (impaired glucose tolerance [IGT] and/or impaired fasting glucose [IFG]), age greater than 45, being overweight or obese, not exercising regularly, having a family history of diabetes, a personal history of gestational diabetes or a baby of at least 9 pounds at birth, other risk factors such as high blood pressure, a low HDL cholesterol or high triglycerides, and one’s race and ethnicity. The White, 50-year-old woman who is obese and gave birth to a child weighing 10 pounds has the most risk factors of the clients described. DIF:

Cognitive Level: Analyze (Analysis)

REF:

p. 256

25. A nurse is counseling a person who was recently diagnosed with diabetes about how to prevent the complications of this disease. Which of the following interventions would be stressed the most by the nurse? a. Adjusting sliding scale insulin therapy as needed b. Taking oral hypoglycemic medication as prescribed c. Monitoring blood glucose daily d. Adhering to medical nutrition therapy ANS: D Medicine may be used to control blood glucose level, and measuring the blood glucose daily helps in controlling its level. But the key to maintaining diabetic control is individualized medical nutrition therapy, the purpose of which is to delay or prevent diabetic complications. DIF:

Cognitive Level: Apply (Application)

REF:

pp. 257-258

26. A nurse is conducting an initial nutritional assessment on a person who is HIV positive. Which of the following questions would be most important for the nurse to ask? a. What fresh fruits and vegetables do you like to eat? b. Are you taking a daily multivitamin? c. How often do you eat at restaurants? d. Do you have any questions about your current medications? ANS: B Evidence suggests that micronutrient deficiencies are common among those with HIV and that those not taking vitamin supplementation have increased morbidity and mortality. Thus, asking about use of a daily multivitamin is the most important information for the nurse to collect. A nutrient-dense, protein-rich, well-balanced diet should be stressed as well as vitamin and mineral supplementation. Because of an altered immune function, they get sicker when exposed to food-borne organisms, so discussion about how to avoid food-borne illness is also important. DIF:

Cognitive Level: Analyze (Analysis)

REF:

p. 259

MULTIPLE RESPONSE 1. Which of the following causes of death is/are directly associated with diet? (select all that apply) a. Diabetes mellitus b. Cirrhosis of the liver c. Stroke d. Coronary heart disease (CHD) ANS: A, C, D Four leading causes of death directly associated with diet are coronary heart disease (CHD), some types of cancer, stroke, and diabetes mellitus. Four more major causes of death—accidents, cirrhosis of the liver, suicide, and homicide—are associated with excessive alcohol intake. DIF:

Cognitive Level: Remember (Knowledge)

REF:

p. 231


2. A community health nurse is preparing an educational brochure about improving the nutritional behaviors of Americans. Which of the following topics would most likely be included in the brochure? (select all that apply) a. Increasing daily intake of milk and dairy products b. Decreasing daily sodium intake c. Increasing daily intake of fruits and vegetables d. Decreasing daily intake of sugar-sweetened drinks ANS: B, C, D Fewer than 15% of adults and 10% of adolescents eat the recommended amounts of fruits and vegetables every day. Sixty-three percent of adults and 84% of adolescents consume at least one sugar-sweetened drink each day. Most American adults consume more than twice the recommended average daily sodium intake level. DIF:

Cognitive Level: Apply (Application)

REF:

pp. 233-234

3. A community health nurse is developing educational materials related to nutrition. Which of the following resources would be most appropriate for the nurse to use? (select all that apply) a. Recommended dietary allowances (RDAs) b. MyPyramid c. Dietary reference intakes (DRIs) d. MyPlate ANS: C, D The most current food and nutrition recommendations are found using the dietary reference intakes (DRIs) and MyPlate. These two resources replaced the previously issued recommended dietary allowances (RDAs) and MyPyramid. DIF:

Cognitive Level: Analyze (Analysis)

REF:

p. 234

4. A person has been diagnosed with salmonellosis. Which of the following symptoms would the person most likely exhibit? (select all that apply) a. Bloody diarrhea b. Vomiting c. Hypoactive bowel sounds d. Abdominal cramping ANS: B, D Symptoms of salmonellosis include abdominal cramping, mild to severe diarrhea, nausea, vomiting, and fever within 8 to 72 hours after infection. Bloody diarrhea is more likely seen with E. coli infection. DIF:

Cognitive Level: Apply (Application)

REF:

p. 242

Chapter 12: Exercise Edelman: Health Promotion Throughout the Life Span, 8th Edition MULTIPLE CHOICE 1. Activity that uses large muscle groups in a repetitive, rhythmic fashion over an extended period to improve the efficiency of the oxidative energy producing system and improve cardiorespiratory endurance is known as: a. aerobic activity. b. anaerobic activity. c. flexibility. d. muscular fitness.


ANS: A Aerobic activity is defined as activity that uses large muscle groups in a repetitive, rhythmic fashion over an extended period to improve the efficiency of the oxidative energy producing system and improve cardiorespiratory endurance. DIF:

Cognitive Level: Remember (Knowledge)

REF:

p. 265

2. A nurse is working with a group of individuals to start an exercise program. Which of the following would require the most attention and assistance? a. White man who works 50 to 60 hours a week as a business executive b. Unemployed, obese Hispanic woman c. Employed White woman with hypertension d. White man with a high school diploma ANS: B The number of adults with no leisure-time physical activity varies by race, ethnicity, gender, education level, geographic location, disability status, and age. Women are less active than men. Lower income and less educated people are not as physically active as those with higher socioeconomic and educational levels. In addition, African Americans and Hispanics are less active. The unemployed, obese Hispanic woman has three risk factors for inactivity: (1) she is unemployed and therefore likely has no or a low income level; (2) she is a woman, and (3) she is Hispanic. All of these factors place her at risk for inactivity. DIF:

Cognitive Level: Analyze (Analysis)

REF:

p. 265 | p. 286

3. According to the physical activity and fitness section of the Healthy People 2020 report, what percentage of adults 18 years of age or older report no leisure-time physical activity? a. 16% b. 26% c. 36% d. 46% ANS: D According to the physical activity and fitness section of the Healthy People 2020 report, 36% of adults 18 years of age or older report no leisure-time physical activity. DIF:

Cognitive Level: Remember (Knowledge)

REF:

p. 265

4. A nurse is developing a primary prevention strategy to address the problem of obesity. Which of the following interventions is being created? a. Encouraging an active lifestyle during childhood years b. Prescribing orlistat (Alli) c. Recommending the Jenny Craig program d. Recommending gastric banding surgery ANS: A Exercise is an important component of primary prevention. Less active children increase their risk of sedentary lifestyles as adults. Sedentary lifestyles lead to obesity. Therefore, a primary prevention method for obesity is to encourage an active lifestyle during the childhood years. DIF:

Cognitive Level: Apply (Application)

REF:

p. 266

5. A nurse is providing care for a 10-year-old child with a BMI at the 80th percentile. Which of the following best describes this child? a. Normal weight b. Overweight c. Obese


d.

Morbidly obese

ANS: A A child or adolescent is categorized as being overweight if his or her BMI is greater than or equal to the 95th percentile of age- and gender-specific BMI growth charts. Thus, the 10-year-old child with a BMI at the 80th percentile is considered to be normal weight. DIF:

Cognitive Level: Apply (Application)

REF:

p. 266

6. According to the 2009-2010 National Health and Nutrition Examination Survey, what percentage of 6- to 11year-old children are overweight? a. 14% b. 18% c. 24% d. 28% ANS: B According to the 2009-2010 National Health and Nutrition Examination Survey conducted by the CDC’s National Center for Health Statistics, 18% of 6- to 11-year-old children are overweight. DIF:

Cognitive Level: Remember (Knowledge)

REF:

p. 266

7. A nurse is providing care for a 40-year-old adult with a BMI of 32. Which of the following best describes this person? a. Normal weight b. Overweight c. Obese d. Morbidly obese ANS: C An adult 20 years of age or older is considered overweight if his or her BMI is 25 to 29.9, obese if his or her BMI is 30 or higher, and morbidly obese if his or her BMI is 40 or higher. Thus, the 40-year-old adult with a BMI of 32 is considered obese. DIF:

Cognitive Level: Apply (Application)

REF:

p. 266

8. During an office visit, a nurse meets with a woman who states she does not have time to exercise. Which of the following actions should the nurse take next? a. Do nothing because she has already made the decision to not exercise. b. Provide her with information on the benefits of exercising. c. Develop an exercise prescription with her. d. Schedule a follow-up in 2 months to see if she still feels the same way. ANS: B The woman is in the precontemplation state and thus should not be forced into a plan at this time. However, exercise has many benefits and it implementation should be encouraged. The nurse should continue to ask at each visit because when individuals are counseled by clinicians they are more likely to increase physical activity. As a result, providing the woman with information regarding the benefits of exercising and bringing up the topic of exercise is the best approach at this stage. DIF:

Cognitive Level: Analyze (Analysis)

REF:

p. 267 | p. 288

9. During an office visit, a nurse provides counseling to a 70-year-old woman with osteoporosis who is otherwise very healthy. Which of the following information should the nurse provide to the woman? a. Avoid all forms of exercise because of the risk of fractures.


b. c. d.

Walk with a partner at least 30 minutes a day 5 days a week. Use a wheelchair to get around so that she can prevent falls. Take a step aerobics class 5 days a week.

ANS: B Regular physical activity helps maintain functional independence and improve quality of life throughout the aging process. The benefits of exercise have been documented for both healthy and chronically ill older adults. Exercise improves bone mineral density, especially weight-bearing activities such as walking, aerobic, and dance. Women 30 to 40 years past menopause also need to consider decreasing the risk of fractures by preventing falls. Advanced age and osteoporosis should not exclude anyone from exercise. Therefore, walking with a partner provides the exercise that is needed as well as a companion to help prevent falls or notify someone if a fall occurs. DIF:

Cognitive Level: Apply (Application)

REF:

p. 267 | pp. 273-274

10. A nurse is providing health education for a group of older adults about the risks of developing coronary heart disease. Which of the following group members is at highest risk? a. Takes a beta-blocker for hypertension b. Has a total cholesterol level of 201 c. Doesn’t participate in any form of regular physical activity d. Has a family history of Type 2 diabetes mellitus ANS: C Physical inactivity affects more people in the development of coronary heart disease than any other risk factor. At any level or combination of risk factors, sedentary individuals are at greater risk of developing coronary heart diseases. Exercise provides both primary and secondary prevention. Thus, the person who does not participate in any form of regular physical activity is at most risk of developing coronary heart disease. DIF:

Cognitive Level: Apply (Application)

REF:

p. 268

11. During a home visit, a nurse notes that a person with hypertension lives on the third floor of a subsidized housing project on a busy road with no sidewalks and few street lights. Which of the following would be the best recommendation for exercise for the nurse to make to the person? a. Do not exercise because you have high blood pressure. b. Use the stairs whenever you are entering or leaving the building. c. Go for short walks around the block during the daytime only. d. Purchase an exercise bike. ANS: B Exercise decreases blood pressure and thus should be encouraged in those with hypertension, especially aerobic activity. Participation in physical activity depends on the availability of facilities and a safe environment. Walking in the road is not safe, and it is also unlikely that this person, who lives in a subsidized housing project, has the funds to purchase an exercise bike. Therefore, using the stairs as much as possible would be a safe, inexpensive form of exercise for this person. DIF:

Cognitive Level: Analyze (Analysis)

REF:

p. 269 | p. 290

12. A 51-year-old woman has begun menopause. The nurse emphasizes the importance of a diet high in calcium and vitamin D as well as exercise for this woman. This recommendation is important because by the time the client is 56, she will have experienced another bone loss. a. 3% b. 7% c. 10% d. 15%


ANS: D The rate of bone loss accelerates rapidly during the first 5 postmenopausal years, with annual losses of 3% to 5% being common. Thus, if at least 3 % is lost over 5 years, 3%  5 years (56 – 51) = 15%. DIF:

Cognitive Level: Apply (Application)

REF:

p. 273

13. A nurse is educating a 35-year-old woman about the best exercises to engage in to prevent bone loss. Which of the following exercises would the nurse recommend? a. Walking b. Swimming c. Weight training d. Bike riding ANS: A Weight-bearing activities have a positive effect on bone density, and walking is a weight-bearing activity. In young women, the goal is to increase bone mineral density; thus, walking is the best exercise to prevent osteoporosis in this woman. DIF:

Cognitive Level: Apply (Application)

REF:

pp. 273-274

14. A nurse is developing an exercise program for a person with moderate osteoarthritis of the knees. Which of the following forms of exercise would the nurse include in this program? a. Step aerobics b. Running c. Water aerobics d. Bike riding ANS: C Water activities are good exercise alternatives for people with musculoskeletal limitations who need some weight relief with exercise. The water provides buoyancy while also providing resistance to the limbs as they move. Studies have also demonstrated that people who participate in aquatic exercise experienced greater pain reduction than those who participated in land activities. Thus, water aerobics would be the most appropriate form of exercise for this individual. DIF:

Cognitive Level: Analyze (Analysis)

REF:

p. 275 | p. 282

15. A nurse is providing care to a person who has lower back pain. Which of the following exercises would the nurse recommend to assist with pain reduction? a. Walking b. Pilates c. Swimming d. Bike riding ANS: B Maintaining proper curves of lordosis and kyphosis in the vertebrae is important for sustaining the spring and shock absorption quality of the spine. Poor posture can increase stress on the back and lead to pain. Exercises that train the paraspinal muscles to improve posture (like Pilates) are effective for people with lower back pain. Aerobic activity is recommended to maintain endurance and prevent debilitation from inactivity. DIF:

Cognitive Level: Analyze (Analysis)

REF:

p. 276

16. Which of the following statements about exercise is true? a. People who engage in high-intensity exercise are at lowest risk for illness. b. Aerobic exercise should be part of daily exercise routines. c. Regular exercise may cause people to experience fatigue and increased susceptibility to mental


d.

health disorders. Low-level physical activity may have a protective effect on gastric and pancreatic cancer.

ANS: D Low-level physical activity may have a protective effect on gastric and pancreatic cancer. People who engage in moderate-intensity activity are at lowest risk for illness, whereas those who engage in high-intensity activity are at higher risk for illness. Aerobic exercise is recommended at a frequency of three to five times per week. Every other day is a good frame of reference as it allows for recovery between training sessions. Cross-training is recommended as part of daily exercise routines. Regular exercise promotes positive well-being and decreased susceptibility to mental health disorders. DIF:

Cognitive Level: Understand (Comprehension) REF:

p. 278

17. A nurse is monitoring the exercise of a 42-year-old woman. Which of the following would be the estimated maximum heart rate (MHR) for this woman? a. 126 b. 142 c. 178 d. 184 ANS: C A generic formula for determining MHR is 220 minus age: 220 – 42 = 178. DIF:

Cognitive Level: Apply (Application)

REF:

p. 279

18. A nurse is counseling a 40-year-old person who is going to start an exercise program. Which of the following heart rate ranges would the nurse encourage the person to maintain while exercising? a. 70 to 140 beats/min b. 80 to 143 beats/min c. 90 to 153 beats/min d. 100 to 180 beats/min ANS: C The maximum heart rate for a 40-year-old is 180. Exercise should produce a heart rate that is 50% to 85% of maximum heart rate. This percentage corresponds to a rate of perceived exertion of 11 to 14: 50% of 180 = 90, 85% of 180 = 153. Thus, the range should be 90 to 153. DIF:

Cognitive Level: Analyze (Analysis)

REF:

p. 279 | p. 281

19. A nurse is discussing exercise intensity with a client. Which of the following medications that the client is taking may limit the usefulness of heart rate as an indicator of exercise intensity? a. Ciprofloxacin (Cipro) b. Metoprolol (Toprol) c. Atorvastatin (Lipitor) d. Lansoprazole (Prevacid) ANS: B Beta-blockers may alter the relationship between heart rate and the rate of perceived exertion. Metoprolol is a beta-blocker and thus may limit the usefulness of heart rate as an indicator of exercise intensity. DIF:

Cognitive Level: Analyze (Analysis)

REF:

p. 281

20. The nurse is discussing an exercise prescription with a person. To help him determine the appropriate intensity, the nurse tells him that he should monitor his rate of perceived exertion. Which of the following should the nurse tell the man he should experience while exercising?


a. b. c. d.

Talking and singing without difficulty Feeling extremely fatigued Having mild musculoskeletal discomfort Being out of breath

ANS: C The rate of perceived exertion helps people determine a comfortable, beneficial level of exercise intensity. Being slightly short of breath, able to talk with difficulty, being pleasantly fatigued, and having mild musculoskeletal pain reflect a rate of perceived exertion that corresponds well with 50% to 85% of maximum heart rate, an appropriate target range for most individuals. DIF:

Cognitive Level: Analyze (Analysis)

REF:

p. 281

21. A nurse is discussing resistance training with a 24-year-old woman. The woman is resistant to weight lifting because she does not want to get too muscular. She prefers a program that is strictly aerobic. Which of the following would be the most appropriate response from the nurse? a. “Ok, just make sure you perform one hour of aerobics every day.” b. “Lifting weights is a necessary part of exercise.” c. “Weight lifting has many purposes and benefits; can we talk about them?” d. “Looking muscular at your age is attractive.” ANS: C Weight training helps develop healthy muscles that provide strength to perform activities of daily living (ADLs) without injury and it stimulates bone health. Additionally, the risk of injury outweighs the benefits of aerobic exercise more than 5 days a week. Not all weight lifting leads to a bodybuilder physique. Offering the patient the opportunity to discuss weight training will allow the patient the opportunity to open up and discuss fears and concerns. It also provides the nurse with the opportunity to discuss the risks and benefits. DIF:

Cognitive Level: Analyze (Analysis)

REF:

p. 279 | p. 282

22. An inborn set of physiological changes that offset those of the fight or flight response is known as: a. Self-efficacy b. Catecholamine surge c. Relaxation response d. Metabolism ANS: C Relaxation response is an inborn set of physiological changes that offset those of the fight or flight response. DIF:

Cognitive Level: Remember

(Knowledge)

REF:

p. 283

23. A nurse is caring for a man who states he is ready to start exercising. Which of the following actions should the nurse take next? a. Do nothing because he has already made the decision to implement a change. b. Provide him with information on the benefits of exercising. c. Develop an exercise prescription with him. d. Schedule a follow-up in 2 months to see if he still feels the same way. ANS: C Working with a person to choose an activity that he likes will improve adherence. The person is displaying that he is in the action phase of change, and he should be given a prescription that promotes exercise and places him in the maintenance phase. DIF:

Cognitive Level: Analyze (Analysis)

REF: p. 288


24. A nurse is counseling a 19-year-old person who has recently been diagnosed with Type 1 diabetes mellitus. The person asks if there is anything specific that should be done before going to play tennis with a friend. Which of the following instructions should the nurse give the person to avoid hypoglycemia? a. Inject the insulin into the abdomen. b. Inject the insulin into the arm. c. Inject the insulin into the thigh. d. Skip taking any insulin before exercising. ANS: A When insulin is injected in a muscle that will be active, it will speed up the absorption of insulin and quickly decrease blood glucose. Because tennis involves a lot of leg and arm movement, insulin should be given in the abdomen. Additionally, not all insulins have the same onset or duration of action. Changes in insulin dosage schedules should not be made without knowing the onset or duration of action. DIF:

Cognitive Level: Analyze (Analysis)

REF:

p. 287 (Box 12-10)

25. A woman is going for a 2-hour hike in the Grand Canyon and she asks the nurse how much fluid she should take along with her. Which of the following recommendations should be given by the nurse? a. Bring at least 24 ounces of water. b. Bring at least 72 ounces of water. c. Bring at least 24 ounces of sports drink. d. Bring at least 72 ounces of sports drink. ANS: D If the weather is hot, 5 to 12 oz of fluid should be taken every 15 to 20 minutes during exercise. Additionally, if exercise lasts more than 1 hour, a drink that provides carbohydrates and electrolytes helps delay fatigue and improve performance. DIF:

Cognitive Level: Analyze (Analysis)

REF:

p. 284

26. An occupational health nurse is trying to promote regular activity at her institution. Which of the following interventions would most likely be the most successful? a. Educate all employees regarding the benefits of exercise. b. Initiate a lunchtime walking group that employees can join. c. Provide all employees with an exercise prescription. d. Recommend that employees who do not exercise be terminated from their positions. ANS: B Individuals are more likely to initiate and maintain physical activity if they feel competent doing the activity, feel safe doing it, perceive no negative financial or social cost, can access the activity easily on a regular basis, and can fit the activity into their regular schedule. Knowledge is not enough to sustain activity. Role models are helpful. Initiating a lunchtime walking group demonstrates enthusiasm on the part of the nurse, provides an activity that is easy and mastered by most and provides a social activity that can promote reinforcement by others who participate. DIF:

Cognitive Level: Analyze (Analysis)

REF:

p. 281 | p. 288

MULTIPLE RESPONSE 1. A nurse is providing an educational program addressing the importance of being physically active. The nurse states that individuals should engage in exercise for a minimum of 30 minutes of activity on most days of the week. For which of the following individuals is that recommendation appropriate? (select all that apply) a. A 54-year-old with hypercholesterolemia b. A 66-year-old with hypertension c. A 48-year-old with obesity d. A 58-year-old with Type 2 diabetes mellitus


ANS: A, B, D The risk of CHD decreases with an increase in physical activity. Exercise positively affects lipoprotein metabolism and is a common part of the treatment for hypertriglyceridemia. Regular exercise also decreases blood pressure in adults. Exercise improves glycemic control by increasing insulin sensitivity in people with Type 2 diabetes mellitus. The Physical Activity Guidelines for Americans advise adults to perform at least 150 minutes of moderate intensity or 75 minutes of vigorous activity per week. Exercise also has a positive effect on obesity. However, the American College of Sports Medicine recommends a minimum of 40 to 60 minutes of exercise 5 to 7 days a week for those with obesity. DIF:

Cognitive Level: Analyze (Analysis)

REF:

p. 266 | pp. 268-269 | p. 272

2. A nurse is reviewing an exercise prescription for a person. Which of the following aspects should be included? (select all that apply) a. Frequency of exercise b. Intensity level of exercise c. Time and duration of exercise d. Type of exercise ANS: A, B, C, D An exercise prescription should include frequency of exercise, intensity level of exercise, time and duration of exercise, and type of exercise. DIF:

Cognitive Level: Apply (Application)

REF:

p. 279

Chapter 13: Stress Management Edelman: Health Promotion Throughout the Life Span, 8th Edition MULTIPLE CHOICE 1. A nurse is defining stress to a person. Which of the following statements should the nurse should make? a. “Happy events, such as the birth of a child, are rarely as stressful as unhappy events.” b. “An event that requires any change in behavior in stressful.” c. “All stressors create a challenge to health.” d. “Once stressors are identified, they can be controlled.” ANS: B Any event that requires a change in behavior is stressful. Some stressful events are beneficial to health and not inherently harmful. Many stressors cannot be controlled. DIF:

Cognitive Level: Apply (Application)

REF:

pp. 298-299

2. The nurse is conducting community education about stress. Which of the following statements should be made by the nurse? a. “Stress always has a negative effect on the body.” b. “A moderate amount of short-term stress can make a person more efficient and goal-focused.” c. “Long-term stress is useful in helping one to meet goals.” d. “Stress always has a positive effect on the body.” ANS: B Stress may be either harmful or helpful. Long-term stress is harmful. A moderate amount of stress is a motivator. DIF:

Cognitive Level: Apply (Application)

REF:

pp. 298-299


3. A nurse is discussing the various types of stress to a group of adults. Which of the following would be used as an example of eustress? a. Increasing alcohol intake to deal with loss of a job b. Avoidance of social activities because of anxiety c. Severely limiting food intake as a way to control stress d. Joining a cardiac rehabilitation exercise program after a myocardial infarction ANS: D Eustress is useful, not harmful. Although a myocardial infarction (MI) is stressful physically and psychologically, joining a cardiac rehab program in response to the MI will improve cardiac and overall health and well-being. The other examples indicate unhealthy coping strategies. DIF:

Cognitive Level: Apply (Application)

REF:

p. 299

4. A nurse is caring for a person who has experienced a minor injury following a fall. Which of the following physiological responses to stress would the nurse anticipate? a. Lowered blood pressure b. Elevated heart rate c. Lowered respiratory rate d. Elevated blood supply to the gut ANS: B Activation of the neuroendocrine system in response to stress results in elevated blood pressure, heart rate, respiratory rate, and an increased blood supply to large muscles. DIF:

Cognitive Level: Apply (Application)

REF:

p. 300

5. Which of the following statements concerning spirituality is correct? a. Spirituality is a search for the meaning of existence. b. To fully experience spirituality, one needs a religious affiliation. c. Stress always strengthens the quest for spirituality. d. Spirituality always prevents depression, which is common following a stressful event. ANS: A Religion and spirituality are complementary but not the same. Spirituality is possible without a religious affiliation. Although spirituality can help one cope with stress, depression may still result. DIF:

Cognitive Level: Understand (Comprehension) REF:

pp. 301-302

6. A nurse has stated that the person she is working with demonstrates stress hardiness. Which of the following individuals is most likely being described? a. Refuses to see family members after the nurse practitioner tells him or her about test results b. Conducts Internet research to learn about the latest treatment options for a health deviation c. Calls in sick when faced with a personal relationship problem d. Views stressors as negative life events ANS: B Stress hardiness is characterized by feeling in control of situations, viewing stressors as challenging rather than negative events, and using effective coping techniques. DIF:

Cognitive Level: Apply (Application)

REF:

p. 302

7. A nurse is describing adults who are in the “sandwich generation.” Which of the following provides the best description of this term?


a. b. c. d.

Middle-aged adults who are starting a family at an older age Young adults who are beginning to find jobs, marry, and start a family Middle-aged adults who are caring for children and aging parents Young adults who have several small children

ANS: C Because of current trends of delayed parenting and increased life span, more middle-aged adults are in the “sandwich generation,” in which they carry the responsibility to care for their children and aging parents. DIF:

Cognitive Level: Apply (Application)

REF:

p. 302

8. A nurse is collecting health assessment data about a person’s coping mechanisms in relation to stressors. Who would provide the best source of this information? a. Person’s primary care physician b. Person’s spouse/significant other c. The person d. Expert objective analysis skills of the nurse ANS: C People are the best source of information related to their coping skills. DIF:

Cognitive Level: Apply (Application)

REF:

p. 303

9. A nurse is assisting a person to cope with stress. Which statement by the nurse would be the most helpful? a. “Most people in this situation feel the way you feel.” b. “Tell me what this event means to you.” c. “There are many support groups available for you to use.” d. “Your family wants to help you through this stressful event. May I call them?” ANS: B Self-awareness is necessary to evaluate the meaning of an event and can only be assessed by asking the person to think about the meaning of the event. DIF:

Cognitive Level: Apply (Application)

REF:

p. 304

10. A nurse is counseling a person on how to effectively deal with stress. Which of the following actions would the nurse recommend the person take first? a. Recognize signs of stress. b. Take action to reduce stress. c. Connect warning signs with stressors. d. Develop effective coping skills. ANS: A To prevent negative consequences of stress, the person must first recognize signs of stress. He or she must then pay attention to these stressors, develop coping skills, and finally use the coping skills. DIF:

Cognitive Level: Apply (Application)

REF:

p. 304

11. A nurse is providing education to a community support group about ways to decrease stress. Which of the following would be the initial nursing intervention discussed by the nurse? a. Explain how support groups function b. Demonstrate deep breathing techniques c. Practice guided imagery techniques d. Help people to identify their response to stress


ANS: D The priority nursing intervention is to help the person become aware of the effects of stress on physical, emotional, and spiritual well-being. The other interventions are used after the person becomes aware of the individual reaction to stress. DIF:

Cognitive Level: Analyze (Analysis)

REF:

p. 304

12. A nurse is explaining the relaxation response to a person. Which of the following statements would be most appropriate for the nurse to make? a. “When the relaxation response is used, your sympathetic nervous system is activated and your pulse rate slows down.” b. “The relaxation response helps you to become more aware of your surroundings.” c. “Deep breathing is very effective in relaxing muscle groups and relieving stress.” d. “Holding your breath for 10 to 15 seconds will reduce your blood pressure and pulse rate.” ANS: C The relaxation response causes blood pressure and pulse rate to lower and muscles to relax. Activation of the sympathetic nervous system has the opposite result. DIF:

Cognitive Level: Apply (Application)

REF:

pp. 305-306

13. The nurse conducted a community health education program about the benefits and limitations of alternative therapies. The nurse recognizes that learning has occurred when a participant makes which of the following statements? a. “Acupuncture is a technique that can be easily learned and practiced.” b. “Hypnosis can be safely used by a licensed hypnotist to treat all cases of anxiety.” c. “Reiki is a type of massage that can be used to relieve stress.” d. “Herbal products must be used with caution because some can interact with prescription medications.” ANS: D Acupuncture is a technique that requires administration by a trained acupuncturist. Hypnosis should not be used for people with severe mental illness. Reiki is not a massage technique. DIF:

Cognitive Level: Analyze (Analysis)

REF:

pp. 306-307

14. A nurse responded to a natural disaster and used expressive writing as a means of coping with the stress related to this experience. Which of the following benefits of expressive writing should the nurse expect? a. Immediate relief of stress through storytelling b. Insight into the personal meaning of the experience c. Absence of the symptoms of stress d. Feeling a great sense of relief immediately after making a journal entry ANS: B Expressive writing helps one reflect on the personal meaning of an experience. Feelings of sadness or anxiety may accompany journal writing as events are analyzed. DIF:

Cognitive Level: Apply (Application)

REF:

p. 307

15. A nurse is helping a person reduce stress-disinhibition related to poor dietary choices. Which of the following nursing interventions would be most appropriate? a. Work with the person to develop a plan to slowly replace high-fat, high-calorie foods with more healthy choices. b. Encourage the person to eliminate all high-calorie, high-fat foods from the diet as quickly as possible.


c. d.

Give the person a standard list of low-fat, low-calorie foods. Provide the person a copy of the Food Guide Pyramid.

ANS: A Working with people to develop a realistic, culturally sensitive, healthy eating plan is more likely to produce lasting results than simply dictating what he or she should eat. DIF:

Cognitive Level: Analyze (Analysis)

REF:

pp. 307-308

16. The nurse is assisting a person to develop a positive attitude toward exercise. Which of the following nursing interventions would be most successful when working with this person? a. Educate the person that 60 minutes of moderate to strenuous exercise a day is required to lower the risk of heart disease. b. Suggest that the person participate in a formal exercise program that has been demonstrated to produce the best results. c. Educate the person that exercise patterns established in young adulthood are more effective than those established later in life. d. Suggest that the person walk at a moderate pace for 30 minutes a day, 5 days a week for improvement in physical and psychological health. ANS: D Formal exercise programs are no more effective than walking 30 to 60 minutes a day for 3 to 5 days a week. Exercise is beneficial at any stage of life. DIF:

Cognitive Level: Analyze (Analysis)

REF:

p. 308

17. A nurse is helping a man restructure his eating, sleeping, exercise, and thinking patterns to reduce stress. Which of the following nursing interventions would be most effective for this person? a. Encourage writing in a journal on a daily basis. b. Suggest joining a support group. c. Offer referral to individual or group therapy sessions. d. Promote purchasing a gym membership. ANS: A Before any meaningful change in behavior can take place, one must have an awareness of, and reflection about, personal behavior. DIF:

Cognitive Level: Analyze (Analysis)

REF:

pp. 304-307

18. A nurse is demonstrating how to use effective communication when dealing with conflict. Which of the following statements would be most appropriate for the nurse to make? a. “I am so angry that you are late again that you are grounded for a week.” b. “When you miss an appointment, I am hurt.” c. “Because you cannot be on time, you cannot have the car for a week.” d. “I feel anxious when you do not come home on time because I worry about your safety.” ANS: D Effective communication relies on assertive communication, which requires an assertive statement (emotion), expressed feelings (behavior), and perceived rights (explanation). DIF:

Cognitive Level: Apply (Application)

REF:

p. 311

19. A nurse is educating a client about the four-step approach to active listening. Which of the following actions should the nurse suggest the client take first? a. Formulate a nonaggressive response.


b. c. d.

Mirror the person’s feelings. Hear what is being said. Listen to the response.

ANS: C Active listening requires the listener to hear what the person is saying, mirror his or her feelings, listening to the person’s response, and communicate in a nonaggressive manner. DIF:

Cognitive Level: Apply (Application)

REF:

pp. 311-312

20. Which of the following statements concerning enjoying healthy activities is true? a. People enjoy pleasant activities even when they are stressed. b. People cherish happy activities and feel that such activities are important even during times of stress. c. People generally feel that they deserve to be happy. d. People who have hobbies are better able to handle stress than those without hobbies. ANS: B Hobbies, and other pleasurable activities, help people balance a stressful life. DIF:

Cognitive Level: Understand (Comprehension) REF:

p. 312

21. A nurse is assisting a client in values clarification. Which of the following actions should the nurse assist the client to complete first? a. Develop a religious affiliation to assist with the process. b. Identify what is meaningful in one’s life. c. Act in a way that is consistent with one’s beliefs. d. Use beliefs held by others as a guideline for life. ANS: B Values clarification begins with examining one’s beliefs and then acting in a way that supports those beliefs. Formal religious affiliation is not required for values clarification. DIF:

Cognitive Level: Apply (Application)

REF:

pp. 313-314

22. The nurse is caring for a person who is obese, sedentary, and has recently been diagnosed with Type 2 diabetes mellitus. Which of the following goals for the person is correctly stated? a. The person will exercise more often. b. The person will consume 900 calories a day. c. The person will walk one-quarter mile a day, 5 days a week. d. The person will eliminate all refined sugar and processed foods from her diet. ANS: C Goals must be realistic, specific, and measurable. Exercising “more often” is too broad a statement. Consuming “900 calories a day” or “eliminating all refined sugar, processed foods, and bread products from his or her diet” is not realistic. DIF:

Cognitive Level: Analyze (Analysis)

REF: p. 314

23. A nurse is caring for a person who has a diagnosis of “Sleep disturbance related to stress as evidenced by excessive daytime fatigue.” Which of the following nursing interventions would most likely to assist this person? a. Encourage daytime naps. b. Encourage exercise one hour before bedtime. c. Encourage a cool shower before bedtime. d. Encourage a consistent sleep-wake schedule.


ANS: D Daytime napping, exercising within 1 hour of bedtime, and taking a cool shower prior to bedtime will not promote nighttime sleep. Following a consistent sleep-wake cycle seven days a week will help establish a sleep pattern. DIF:

Cognitive Level: Apply (Application)

REF:

p. 309 (Box 13-6)

24. A nurse is educating a person about the process of cognitive restructuring. Which of the following actions should be taken at the end of this process? a. Take a deep breath. b. Select a realistic alternative thought. c. Reflect on the situation. d. Stop generating negative thoughts. ANS: B The most effective approach to restructuring thought patterns is to first stop generating negative thoughts, then take a deep breath to relax, then think about how the thought process is affecting well-being, and last, select a new realistic thought pattern to replace the negative thought pattern. DIF:

Cognitive Level: Apply (Application)

REF:

p. 310 (Box 13-7)

25. A nurse is explaining the concept of “presence.” The nurse should include that presence means that the person: a. is in the same room as another. b. multitasks in order to examines all aspects of a situation. c. sets goals in response to what another is saying. d. practices active listening and focuses on what is being said. ANS: D The concept of “presence” means that a person’s full attention is focused on the situation at hand. Multitasking produces the opposite effect because focus is lost. DIF:

Cognitive Level: Apply (Application)

REF:

p. 306 (Box 13-4)

MULTIPLE RESPONSE 1. A nurse is educating a group of adults about stress management strategies. Which of the following examples would the nurse include in the presentation? (select all that apply) a. Participation in a swimming class b. Keeping a food diary as part of a weight reduction program c. Joining a smoking cessation support group d. Setting a goal of walking one mile a day ANS: A, B, C, D Goal setting, self-monitoring, exercise, and problem solving are all effective stress reduction strategies. DIF:

Cognitive Level: Apply (Application)

REF:

pp. 304-308

2. The nurse in the pediatric oncology unit is working with a family whose toddler has been diagnosed with leukemia. The nurse can best assist the family to deal with this extrinsic stressor by doing which of the following? (select all that apply) a. Setting treatment goals for them until they are able to do so themselves b. Encouraging them to talk about their concerns c. Discussing relaxation techniques such as guided imagery with them


d.

Asking them if they would like to meet with a member of a pediatric oncology support group

ANS: B, C, D The nurse can assist this family by active listening, encouraging the family to discuss concerns, teaching them relaxation techniques, and providing access to a support group. It is not appropriate for the nurse to set goals for the family. DIF:

Cognitive Level: Apply (Application)

REF:

pp. 314-315

Chapter 14: Complementary and Alternative Strategies Edelman: Health Promotion Throughout the Life Span, 8th Edition MULTIPLE CHOICE 1. Which of the following statements best defines the concept of holistic nursing care? a. Holistic nursing care is a family-centered approach to care. b. Holistic nursing care relies primarily on alternative medicine. c. Holistic nursing care takes into consideration all aspects of a person’s life. d. Holistic nursing care focuses on wellness rather than illness. ANS: C Holistic nursing care takes into account all aspects of a person’s life experiences including environmental, physical, social, spiritual, and psychological constructs. DIF:

Cognitive Level: Understand (Comprehension) REF:

p. 320

2. In contrast to holistic health care, Western or allopathic medicine focuses on the: a. mind-body connection. b. impact of family relationships on health-seeking behaviors. c. physical aspects of health deviations. d. environmental impact on health. ANS: C Holistic health care focuses on the whole person, mind, body, spirituality, and so forth. Western or allopathic medicine focuses mainly on physical aspects of disease. DIF:

Cognitive Level: Remember (Knowledge)

REF:

p. 320

3. A nurse is discussing holistic health care with an individual. Which of the following explanations should the nurse include in this discussion? a. Empirical research has consistently demonstrated the value of alternative/complementary therapy. b. Self-reports of stress relief related to alternative/complementary therapy are numerous. c. The majority of research related to alternative/complementary therapy has utilized the scientific method. d. Research has, to date, been unable to verify the effectiveness of alternative/complementary therapy on health and well-being. ANS: B Empirical research related to the effectiveness of alternative/complementary therapy has been mixed. Some research studies have verified positive physiological reaction to this type of therapy. DIF:

Cognitive Level: Analyze (Analysis)

REF:

pp. 321-322 | p. 332


4. The nurse is conducting a community health education program concerning alternative/complementary therapies. Which of the following should be included in the presentation? a. Alternative/complementary therapy is primarily used when conventional medicine has failed to achieve desired results. b. Alternative/complementary therapy is used in both hospital and outpatient settings. c. Alternative/complementary therapy was popular in the late 1980s but is rarely used today. d. Alternative/complementary therapy primarily focuses on herbal remedies for common health deviations. ANS: B Alternative/complementary therapy is gaining popularity in both hospital and outpatient settings. It is used as an adjunct to traditional medicine and focuses on all aspects of being (e.g., environmental, spiritual), not just herbal products. DIF:

Cognitive Level: Apply (Application)

REF:

pp. 321-322

5. Which of the following complementary/alternative therapies involves emphasizing healthy lifestyle, strengthening and cleansing the body, and noninvasive treatments? a. Homeopathy b. Naturopathy c. Traditional Chinese medicine (TCM) d. Ayurvedic medicine ANS: B Naturopathy is guided by the healing power of nature. Practitioners do not use prescription drugs, injections, xrays, or surgery, but instead use a variety of complementary and alternative medicine (CAM) modalities, emphasizing healthy lifestyle, strengthening and cleansing the body, and noninvasive treatments. DIF:

Cognitive Level: Remember (Knowledge)

REF:

pp. 324-325

6. A nurse discussing aromatherapy at a community health fair. Which of the following aspects of this technique should be stressed? a. Aromatherapy can be used by individuals without special training. b. Aromatherapy is a recently developed therapy based on the work of contemporary natural healers. c. Aromatherapy uses herbal scents in candles or incense as the primary method of delivery. d. Aromatherapy can cause an allergic reaction in some people. ANS: D Aromatherapy uses aromatic plant materials and the essential oils of plants to treat physical imbalances, as well as to achieve psychological and spiritual well-being through inhalation, external application or ingestion. The use of aromatherapy without professional clinical training is strongly discouraged. Individuals must have this training to know the specific warnings and contraindications for each oil, as oils may cause allergic reactions or other undesirable side effects. DIF:

Cognitive Level: Apply (Application)

REF:

p. 325

7. A nurse is instructing a person on the use of breath meditation. Which of the following instructions would the nurse give the person? a. “You may use this form of meditation when walking down the street.” b. “The best time of day to engage in this form of meditation is while you are eating.” c. “Watching the sunset and reflecting on its beauty is a great way to use breath meditation.” d. “It is important to observe your breathing when using this technique.” ANS: D


Breath meditation is the simplest form of meditation. It can be done anywhere and can evoke the relaxation response. This type of meditation should not be done while completing other activities. One should stand, sit, or lie quietly when beginning to focus on breathing. Breathing should be observed using this technique, but not controlled. DIF:

Cognitive Level: Apply (Application)

REF:

p. 326 | p. 326 (Box 14-5)

8. A nurse is teaching a person about the technique of breath meditation. Which of the following aspects of this technique should the nurse stress to the person? a. Hold your breath for short amounts of time in order to regulate your breathing pattern. b. Focus on calming thoughts that come to mind during the meditation. c. Practice this technique when sitting or walking. d. Concentrate on the sensations associated with breathing. ANS: D This technique should be practiced when standing, sitting, or laying quietly. Thoughts that emerge during this technique should be let go, so the focus is on the sensation of breathing. Holding one’s breath is not part of the technique. DIF:

Cognitive Level: Apply (Application)

REF:

p. 326 | p. 326 (Box 14-5)

9. The nurse is teaching imagery as a method to reduce anxiety in preoperative individuals. Which of the following aspects should the nurse stress when teaching this technique? a. The benefits of guided imagery are primarily psychological. b. This technique promotes a sense of well-being. c. A “prayer circle” is needed for imagery to be most effective. d. Imagery produces only beneficial, calming results. ANS: B Imagery is a gentle but powerful technique that focuses and directs the imagination in order to promote a sense of well-being and help people relax. Imagery is an individual exercise and does not include a prayer circle. DIF:

Cognitive Level: Apply (Application)

REF:

p. 326

10. An individual asks the nurse to explain yoga. Which of the following statements would be made by the nurse? a. Yoga requires the ability to assume and maintain certain poses. b. There are many different types of yoga. c. Yoga is the most strenuous of the movement therapies. d. The major benefit of yoga is an increase in physical exercise and flexibility. ANS: B Not all forms of yoga involve physical activity. There are many forms of yoga, some of which involve meditation and others that involve movement. DIF:

Cognitive Level: Apply (Application)

REF:

p. 327

11. A nurse working at a holistic health center is asked by a person with severe rheumatoid arthritis if yoga would be a realistic therapy choice. Which of the following would be the best response by the nurse? a. “Let’s discuss other alternative therapies. Yoga involves positioning that you might not be able to do.” b. “Hatha yoga would be a great therapy option because it involves mediation rather than positioning.” c. “There are many types of yoga that would be realistic for you to try. Let’s talk about the types that would be suited to your needs.” d. “I will ask the instructor teaching karma yoga if there is room in her class for you. I think that you would enjoy this type of yoga.”


ANS: C There are many types of yoga that do not involve positioning, such as raja yoga, mantra yoga, and so forth. People should be informed about all of the choices and allowed to select the one most suited to their needs. Hatha yoga involves positioning, which might not be realistic for this individual. DIF:

Cognitive Level: Analyze (Analysis)

REF: p. 327

12. A nurse is describing how energy flow can affect health and well-being. Which of the following examples would the nurse discuss when sharing how energy flow can become chaotic? a. Believing that one has been abandoned by God b. Reestablishing contact with a friend who has recently moved back into the area c. Recovering from an acute illness d. Walking one mile a day ANS: A Spiritual distress can create a chaotic energy flow. Reestablishing friendships, having a positive outcome from an illness, and exercise can balance energy. DIF:

Cognitive Level: Apply (Application)

REF:

pp. 327-328

13. The nurse is asked to explain acupuncture to a person. Which of the following statements concerning acupuncture would the nurse include in the explanation? a. Acupuncture is accomplished by inserting very fine needles into various areas of subcutaneous tissue. b. Acupuncture can be used to either diagnose or treat a health deviation. c. The main focus of acupuncture is to relieve stress. d. Moxibustion is the term used to describe a disturbed energy flow, which results in disease. ANS: B Acupuncture is accomplished by inserting fine needles into the skin, not subcutaneous tissue. This treatment modality is used to either treat illness or diagnose illness. Moxibustion describes using burning herbs to increase point stimulation. DIF:

Cognitive Level: Apply (Application)

REF:

p. 328

14. A person asks the nurse to explain the difference between acupuncture and acupressure. Which statement would be the appropriate response by the nurse? a. Acupressure uses massage and oil aromatherapy to stimulate meridian points. b. Acupuncture has been shown to relieve nausea and vomiting, but acupressure is not effective in relieving these symptoms. c. Acupressure stimulates meridian points by using pressing and stretching movements. d. Acupressure is achieved by applying more pressure to the fine needles inserted as part of acupuncture. ANS: C Massage oil is not used during acupressure therapy. Both acupressure and acupuncture have been shown to relieve nausea and vomiting. Acupressure involves pressing, knuckling, rubbing, squeezing, and stretching, not by applying pressure to needles inserted during acupuncture. DIF:

Cognitive Level: Apply (Application)

REF:

pp. 328-329

15. The nurse has just discussed reflexology with an individual. Which of the following statements would the person make that demonstrates that learning has taken place? a. Reflexology is a therapy that uses upper back massage to control tension and stress.


b. c. d.

Reflexology uses massage of acupressure points to optimize positive energy flow. Reflexology uses pressure applied to specific hand and foot areas to relieve tension. Reflexology uses a squeezing or stretching motion to hands or feet to restore a positive energy flow.

ANS: C Reflexology is a hand pressure technique applied to the hands or feet. It does not involve massage, squeezing, or stretching like acupressure. DIF:

Cognitive Level: Apply (Application)

REF:

pp. 328-329

16. A nurse has completed an educational session about touch therapy with a group of adults. Which of the following statements made by an individual would demonstrate understanding of this topic? a. “Research has shown that touch therapy is effective because it relieves stress and tension.” b. “Touch therapy is effective because deep massage relaxes muscles.” c. “During touch therapy the therapist’s hands may not actually touch me.” d. “It is the massage skill of the therapist that produces the positive benefits of touch therapy.” ANS: C Why touch therapy is effective is not known. Touch therapy does not involve massage. DIF:

Cognitive Level: Apply (Application)

REF:

p. 329

17. A nurse is performing therapeutic touch with a person. Which of the following would be the first action that the nurse would take? a. Centering b. Movement of hands above the body c. Treatment technique d. Attunement ANS: A The first step in therapeutic touch therapy is for the therapist to become centered, so full attention can be paid to the person receiving the therapy. The second step is to assess for disturbances by moving the hands above the body. The last step involves the actual treatment techniques. DIF:

Cognitive Level: Apply (Application)

REF:

p. 329

18. A person wishes to participate in a therapy involving energy utilization and dietary design. Which of the following therapies would the nurse recommend to the person? a. Polarity therapy b. Reiki c. Qi gong d. Pranic healing ANS: A Reiki, Qi gong, and pranic healing are all forms of touch therapy. Polarity therapy utilizes touch therapy and dietary restructuring. DIF:

Cognitive Level: Apply (Application)

REF:

p. 329

19. The nurse is assisting an individual who is interested in music therapy. Which of the following statements made by the person indicates an understanding of this type of therapy? a. “I will select the music I listen to carefully. Energetic or rap music is not a good choice.” b. “I will need to find someone who has specialized in this type of therapy for best results.” c. “Chants, electronic sounds, or ‘designer music’ is generally the best choice.”


d.

“The music I choose may be different than what my spouse prefers.”

ANS: D Music selection is an individual choice. There is no one “best” type of music. Family members may also benefit from the positive effects of listening to music. DIF:

Cognitive Level: Apply (Application)

REF:

p. 330

20. A nurse would like to include holistic nursing care in her nursing practice. Which of the following actions must the nurse take first? a. Learn the specific therapy method. b. Engage in self-exploration. c. Explore the various types of complementary therapies available. d. Ask the person which type of therapy he or she prefers. ANS: B The first step to providing holistic care is an exploration of self. Each of the other steps follows that step. DIF: REF:

Cognitive Level: Apply (Application) p. 330

21. Which of the following describes a nurse who is “being with” the client? a. Answers a call bell b. Completes a dressing change c. Silently prays with a client when requested to do so d. Teaches a client about his or her medication regime ANS: C Performing one’s duty or completing a task allows a nurse to “be there” with the client, but entering the client’s world and praying with him or her when requested to do so exemplifies truly “being with” the client. DIF:

Cognitive Level: Apply (Application)

REF:

p. 330 | p. 330 (Table 14-3)

22. A nurse is providing education about safety precautions for people to take when using complementary/alternative medicine (CAM). Which of the following information would be included in this presentation? a. Most treatments are covered by insurance. b. The U.S. Food and Drug Administration regulates the quality of dietary supplements. c. It is important to research and understand the treatment before starting to use it. d. There are minimal interactions between the treatments prescribed by allopathic and holistic medicine providers. ANS: C It is important to take time to research and understand the treatment and ensure it is appropriate for the individual and the purpose. Treatments may not be covered by insurance. The U.S. Food and Drug Administration regulates dietary supplements, but does not always regulate their quality. There may be interactions between the treatment prescribed by allopathic and holistic practitioners so both providers should know about the care and treatment provided by the other. DIF:

Cognitive Level: Apply (Application)

REF:

p. 331

23. The nurse has formulated a nursing diagnosis for a person with carpal tunnel syndrome of “Energy Field Disturbance r/t Slowing or Blocking of Energy Flow Secondary to Repetitive Motion Injury.” Which of the following actions must the nurse take first when planning interventions for this person? a. Move his or her hands over the person. b. Obtain permission to use healing touch therapy. c. Explain healing touch therapy.


d.

Focus on the area of disturbed energy flow.

ANS: C Before giving informed consent, the person must understand the proposed therapy. DIF:

Cognitive Level: Apply (Application)

REF:

p. 332 (Care Plan)

24. A nurse is caring for a Hispanic individual. Which question should the nurse ask the person during the assessment process? a. “How many herbal medicines do you take?” b. “You cannot use any herbal products in the hospital. Did you bring any?” c. “Herbal products interact with medicine, so you cannot continue to use the herbal products. Which ones do you use?” d. “Tell me how you treat your asthma at home. What treatments work especially well for you?” ANS: D Many in the Hispanic culture use herbal products and alternative therapy. Keeping an open mind during the assessment process and asking open-ended questions will encourage the person to share his or her stories. DIF:

Cognitive Level: Analyze (Analysis)

REF:

p. 328 (Box 14-8)

MULTIPLE RESPONSE 1. Which of the following people are participating in alternative/complementary therapies? (select all that apply) a. A 15-year-old who listens to rap music to lessen anxiety related to a painful dressing change b. A 45-year-old who burns lavender-scented candles while meditating c. A 7-year-old who draws as a means of communicating feelings d. A 78-year-old who practices yoga to increase balance ANS: A, B, C, D Music, art therapy, aromatherapy, and movement therapy are all alternative/complementary therapies. DIF:

Cognitive Level: Apply (Application)

REF:

pp. 320-321 | p. 324

2. During an alternative therapy conference, a nurse attends a session related to massage therapy. Which of the following statements made by the presenter should the nurse question? (select all that apply) a. “Massage therapy is most useful for individuals who are between the ages of 13 and 60.” b. “Massage therapy involves the use of lotion or powder.” c. “Several massage techniques can be used, including kneading and pinching the skin.” d. “Massage therapy is best used by a licensed massage therapist.” ANS: A, D Nurses frequently use massage therapy by giving back rubs. All age groups from neonates to elders can benefit from massage therapy. DIF:

Cognitive Level: Apply (Application)

REF:

p. 326

3. A 66-year-old woman with osteoarthritis of the knees and hands tells the nurse she is interested in movement therapy. Which of the following would be the most appropriate for the nurse to suggest to this person? (select all that apply) a. Tai Chi b. Hatha yoga c. Qi gong d. Dance therapy


ANS: A, C Tai Chi combines slow, relaxed, gentle movement with controlled breathing. Qi-gong combines relaxed movements with a meditative aspect and controlled breathing. Hatha yoga involves poses, which might be difficult for this person to achieve. Dance therapy may also be difficult for this person to complete. DIF:

Cognitive Level: Apply (Application)

REF:

p. 327

4. A nurse consultant is helping redesign a pediatric unit which is undergoing renovation. Which actions by the nurse consultant are congruent with creating a positive energy force in the unit? (select all that apply) a. Selecting energizing colors for the playroom and soothing colors for the exam rooms b. Planning for a music source for all rooms c. Reducing the amount of natural light on the unit to promote relaxation d. Planning for an atrium garden in the center of the unit ANS: A, B, D Positive energy forces include light, color, and music. Limiting natural light has a depressive effect on some people. DIF:

Cognitive Level: Apply (Application)

REF:

p. 327

Chapter 15: Overview of Growth and Development Framework Edelman: Health Promotion Throughout the Life Span, 8th Edition MULTIPLE CHOICE 1. The nurse is explaining the concept of growth to a parent. Which of the following statements should be made by the nurse? a. “Growth means adding height and weight to your body.” b. “We continue to grow in various ways throughout life.” c. “How much we grow is based on gender.” d. “Growth stops when we are young adults.” ANS: B Growth, in some form, continues throughout life, not just during puberty. Not all growth is based on gender. Growth indicates more than a change in height and weight. DIF:

Cognitive Level: Apply (Application)

REF:

p. 336

2. Which of the following statements is correct related to the concept of growth a. Growth occurs mainly during early childhood. b. Older adults do not experience growth-related changes.. c. Growth refers only to the increase in the size of specific organs and systems. d. Growth occurs throughout the life cycle. ANS: D People experience developmental transitions throughout life, not just during times of rapid physical growth. Growth changes that take place in young, middle-aged, and older adults should be noted. People who think of growth only as it applies to infants, children and adolescents are missing important changes from conception throughout adulthood. DIF:

Cognitive Level: Understand (Comprehension) REF:

p. 336


3. The nurse at a well-baby clinic is assessing a 12-month-old child. At birth, the child weighed 7 lb. 3 oz. During this visit, the child weighs 21 lb. 10 oz. Which of the following conclusions would the nurse make about the child’s weight? a. The infant is gaining weight faster than anticipated. b. The infant is malnourished. c. The infant is at the expected weight for his or her age. d. The infant is having a growth spurt. ANS: C A 1-year-old infant is expected to have at least doubled his or her weight by 12 months of age. DIF:

Cognitive Level: Apply

(Application)

REF:

p. 343

4. The school nurse has been asked to order growth charts. Which of the following growth charts should be selected to assessing children in first through fifth grades? a. Generic body mass index chart b. Height and weight chart c. Revised CDC growth chart d. WHO international growth chart ANS: C The CDC recommends the revised 2000 CDC growth charts (including the BMI and the 3rd and 97th percentile) be used for children aged 2 to 20 years. DIF:

Cognitive Level: Apply (Application)

REF:

p. 343

5. A school nurse is obtaining the height and weight of a 9-year-old child. Which of the following is the best method to assess the child’s growth? a. Use serial measurements over time. b. Use this measurement. c. Use this measurement and compare it with the child’s peers. d. Use the WHO growth chart to assess this child’s measurements. ANS: A When assessing growth data for use in growth charts, it is important to remember that a single measurement taken at one point in time, although helpful in providing a baseline, does not allow for the best assessment of a child’s growth. Serial measurements, plotted on a growth chart over time, best reflect a child’s pattern of growth. DIF:

Cognitive Level: Apply (Application)

REF:

p. 344

6. A mother comes to the pediatric clinic and says to the nurse, “I am worried about my 11-month-old baby because he is not crawling yet and his brother crawled when he was 11 months old. Is something wrong?” Which of the following statements should the nurse make? a. “It is highly unusual for siblings not to reach developmental milestones at the same time. I’ll ask the doctor to assess him.” b. “All babies reach developmental milestones at different rates.” c. “This may be something to worry about. I’ll have the doctor examine him.” d. “Don’t worry; he’ll crawl soon enough.” ANS: B Although developmental milestones follow a predictable pattern, each child develops at his or her own rate. Telling the mother not to worry is not answering her question. DIF:

Cognitive Level: Apply (Application)

REF:

pp. 344-345


7. Which of the following statements describes Erikson’s theory of development? a. A healthy personality is defined by the lack of pathology. b. Stages of development advance based on psychosocial factors. c. The main premise of the theory is that individuals are interdependent beings and rely on each other in order to develop successfully. d. An individual must successfully accomplish the developmental stage before proceeding to the next stage. ANS: D Erikson’s theory of development is based on the need of each person to develop a sense of trust in self and others and a sense of personal worth. Each stage depends on the preceding stage, which must be accomplished successfully for the person to proceed. Although the environment and others may influence development, it is primarily an individual task. A person may regress to an earlier stage during times of stress. DIF:

Cognitive Level: Understand (Comprehension) REF:

pp. 345-346

8. According to Piaget’s theory of cognitive development, which of the following statements is correct? a. Cognitive function continues to become refined throughout life. b. A child’s greatest task is to make sense of the world around him or her. c. Culture plays a major role in cognitive development. d. Learning precedes development. ANS: B Piaget theorized that cognitive development was complete by about age 15. He did not factor the effects of culture into his theory and felt that learning followed cognitive development. DIF:

Cognitive Level: Understand (Comprehension) REF:

p. 346

9. Which of the following statements summarizes Vygotsky’s theory of development? a. All children develop cognitively at the same rate. b. Social and cultural influences are not major factors in cognitive development. c. Learning is an individual activity. d. Experience creates learning. ANS: D Vygotsky’s theory of development stresses that social and cultural influences can have a major impact on learning. He felt that cognitive development was dependent on exposure to many life experiences, was influenced by interaction with others, and did not proceed according to predetermined stages. DIF:

Cognitive Level: Understand (Comprehension) REF:

pp. 346-347

10. Kohlberg’s theory of moral development is based on: a. Erikson’s theory of psychosocial development b. Piaget’s theory of cognitive development c. Vygotsky’s theory of cognitive development d. Gilligan’s theory of moral development ANS: B Kohlberg’s theory of moral development is based on Piaget’s theory of cognitive development. DIF:

Cognitive Level: Remember (Knowledge)

REF:

pp. 347-348

11. Which of the following statements about Gilligan’s theory of moral development is accurate? a. This theory is based on a sense of absolute right and wrong. b. This theory is gender specific.


c. d.

This theory is dependent on one’s stage of cognitive development. This theory is based on the premise that women view situations in terms of rules.

ANS: B Gilligan’s theoretical work is based on women’s moral development. Kohlberg postulated that men view situations in terms of justice and rules, and that moral development depends on cognitive development. DIF:

Cognitive Level: Understand (Comprehension) REF:

p. 348

12. A nurse is caring for a person with the nursing diagnosis of chronic sorrow related to missed opportunities. Which of the following nursing interventions would be appropriate for this person? a. Sharing a personal story with the person to demonstrate empathy b. Assuring the person that he or she will be able to cope with the illness c. Encouraging the person to discuss his or her fears d. Contacting a support group representative for the person ANS: C It is not appropriate to discuss personal issues with an individual. Assuring the individual what he or she will be able to cope with is not appropriate because it does not encourage the person to talk about his or her fears. It is not appropriate to contact a support group without the person’s permission. The person may not be ready to discuss the issue with strangers. The correct response is part of grief work facilitation: helping another cope with painful feelings of actual or perceived responsibility. DIF:

Cognitive Level: Analyze (Analysis)

REF:

pp. 348-349 (Care Plan)

13. A nurse is teaching parents of a toddler about nutrition. Which of the following statements should be made by the nurse? a. “Raisins are a good finger food because they provide fiber.” b. “Grapes are a good snack choice because they help toddlers meet their daily fruit requirements.” c. “Hot dogs are not a safe food choice because they may cause choking.” d. “A small piece of chewing gum will help strengthen jaw muscle; just be sure he or she does not swallow it.” ANS: C Raisins, grapes, and chewing gum all pose a choking hazard and should not be given to toddlers. DIF: REF:

Cognitive Level: Apply (Application) p. 345 (Box 15-3)

14. A nurse who is conducting a parenting class is asked how to select a good day care center for an 18-month-old child. Which of the following responses should be made by the nurse? a. “Day care centers that schedule age-appropriate educational videos increase verbal ability.” b. “Human interaction increases a toddler’s verbal skills, so select a day care center that values adultto-toddler interaction.” c. “Learning to understand the meaning of words is increased when toddlers are exposed to educational DVDs, so select a day care center that has an extensive library of age-appropriate, educational DVDs.” d. “There really is no difference in the quality of day care centers based on whether or not videos or DVDs are used for educational purposes.” ANS: B Research has demonstrated that toddlers who are exposed to videos and DVDs have a more limited vocabulary than toddlers who interact with caregivers. DIF:

Cognitive Level: Apply (Application)

REF:

p. 347 (Box 15-4)


15. A high school sophomore engages in risky behavior to fit in with his peers. According to Erikson’s theory of human development, which of the following developmental conflicts is this student facing? a. Initiative versus guilt b. Industry versus inferiority c. Identity versus role confusion d. Intimacy versus isolation ANS: C According to Erikson, a preschool child has a developmental conflict between initiative and guilt, a school-age child has a developmental conflict between industry and inferiority, and a young adult has a developmental conflict between intimacy and isolation. An adolescent has a developmental conflict between identity and role confusion. DIF:

Cognitive Level: Apply (Application)

REF:

p. 346 (Table 15-3)

16. A child is focused on peer approval and avoidance of “not fitting in.” According to Kohlberg, this child is experiencing which of the following stages of moral development? a. Preconventional b. Conventional c. Postconventional d. Preoperational ANS: B The goal of the conventional stage is gaining approval and avoiding disapproval. The goal of the preconventional stage is avoiding punishment and gaining reward, while the goal of the postconventional stage is agreeing upon rights, establishing personal moral standards, and achieving justice. DIF:

Cognitive Level: Apply (Application)

REF:

p. 348 (Table 15-5)

17. The nurse is caring for a person who donated a kidney to a stranger in need. According to Gilligan’s stages of moral development, the person who donated the kidney is in which of the following stages? a. Preconventional b. Conventional c. Postconventional d. Midconventional ANS: B Characteristics of the conventional stage of Gilligan’s stages of moral development include sacrifice to fulfill the needs of others. DIF:

Cognitive Level: Apply (Application)

REF:

p. 348 (Table 15-6)

MULTIPLE RESPONSE 1. A nurse is attending a continuing education program about growth and development. Why is it necessary that nurses are well educated about this content? (select all that apply) a. Have a better understanding of genetics and genomics b. Can meet the requirements of Healthy People 2010 c. Can provide more effective health education d. Can deliver anticipatory guidance to families ANS: A, C, D


Genetics and genomics is an important part of our age of technology; understanding growth and development aids in the understanding of these topics as well. Health education is more effective when the nurse acknowledges and incorporates growth and developmental needs as well as the individual’s prior understanding of beliefs about health and health-related concepts. Health promotion is an important part of Healthy People 2010, but there are no requirements within this document that nurses need to be educated about growth and development. DIF:

Cognitive Level: Apply (Application)

REF:

p. 336 | p. 346 (Box 15-3)

2. A nurse is educating new parents about normal growth and development for their child. Which of the following items would the nurse include in this discussion? (select all that apply) a. Trends in childhood obesity b. Availability of a well-balanced diet c. Exposure to lead-based pain d. Height/weight of relatives ANS: B, C, D A well-balanced diet will help assure that the child reaches his or her growth potential. Lead poisoning, an environmental exposure, remains a significant threat to today’s children. Genetic factors greatly influence growth and development. DIF:

Cognitive Level: Apply (Application)

REF:

p. 336

3. The school nurse is conducting an in-service program for teachers that discusses the development of elementary school children. According to Piaget’s theory of cognitive development, which of the following information would the nurse include? (select all that apply) a. Use abstract thought to discuss a story. b. Make moral decisions concerning right and wrong actions. c. Listen to a peer’s point of view about a playground situation. d. Be influenced by his or her friends. ANS: C, D Children age 7 to 11 are in the concrete operations stage of development according to Piaget. They are influenced by friends and can listen to and think about a peer’s point of view. Moral decision making and use of abstract thought occur during the formal operations stage of development, usually between the ages of 11 and 15 years. DIF:

Cognitive Level: Analyze (Analysis)

REF:

p. 347 (Table 15-4)

Chapter 16: The Prenatal Period Edelman: Health Promotion Throughout the Life Span, 8th Edition MULTIPLE CHOICE 1. A pregnant woman reports to the nurse that the first day of her last menstrual period was January 22. Using Nägele’s rule, which of the following dates would be the most accurate estimated date of delivery? a. September 16 b. September 22 c. October 29 d. October 17 ANS: C


An accurate estimated date of delivery is determined by using Nägele’s rule. This is done by adding 7 days to the date of the first day of her last normal menstrual period and subtracting 3 months: 22 + 7 = 29. Subtracting 3 months from January is October. Thus, the estimated date would be October 29. DIF:

Cognitive Level: Apply (Application)

REF:

p. 352

2. Which of the following best describes an abnormality in structure or function that occurs during fetal development? a. Congenital defect b. Genetic defect c. Embryonic defect d. Chromosomal defect ANS: A During fetal development, an abnormality in structure or function is known as a congenital defect. DIF:

Cognitive Level:

Understand (Comprehension) REF:

p. 353

3. During which week would pregnancy begin to pose health concerns for the fetus? a. 36 weeks b. 38 weeks c. 40 weeks d. 42 weeks ANS: D When pregnancy continues beyond 42 weeks, or 2 weeks beyond the calculated due date, placental function decreases even more, posing concerns about the well-being of the fetus. DIF:

Cognitive Level: Remember (Knowledge)

REF:

p. 353

4. Which of the following changes is experienced by the urinary system during pregnancy? a. Increased urinary output secondary to total body water increase b. Decreased bladder capacity c. Decreased glomerular filtration rate (GFR) resulting from estrogen and progesterone surge d. Increased renal excretion of acidic drugs ANS: A The urinary system undergoes dramatic changes during gestation. The changes include a 50% increase in GFR, an increase in the diameter of the ureters, and increase in urinary output related to total body water increase and an increase in bladder capacity. DIF:

Cognitive Level: Understand (Comprehension) REF:

pp. 353-354

5. During a routine clinic visit, a pregnant woman expresses concern about reflux she is experiencing. Which statement should be made by the nurse when addressing the woman’s concern? a. “Frequent heartburn may be a sign of fetal distress and an ultrasound should be performed immediately.” b. “Frequent heartburn is caused by high levels of hormones during pregnancy.” c. “Frequent heartburn is a result of gastrointestinal system changes that occur during pregnancy.” d. “Frequent heartburn during pregnancy requires immediate consultation with a gastroenterologist.” ANS: C


The gastrointestinal system undergoes dramatic changes during pregnancy, which include frequent heartburn secondary to upward displacement of the stomach and a relaxed gastroesophageal sphincter. Therefore the appropriate response for this woman would be to provide her with information regarding normal changes during pregnancy. In this case the frequent heartburn is a result of gastrointestinal system changes that occur during pregnancy. DIF:

Cognitive Level: Apply (Application)

6. The client who is taking prescribed for drug-related birth defects. a. antiemetics b. antiepileptics c. iron d. non-narcotic analgesics

REF:

p. 354

would require preconception management to minimize the potential

ANS: B Oral antiepileptic medications require preconception management to minimize potential birth defects and minimize health problems. DIF:

Cognitive Level: Apply (Application)

REF:

p. 355

7. A nurse is caring for a pregnant woman who has a pre-pregnancy body mass index (BMI) of 27. Which of the following instructions should the nurse provide the woman regarding weight gain during pregnancy? a. “You should gain 11 to 20 pounds during your pregnancy.” b. “You should gain 15 to 25 pounds during your pregnancy.” c. “You should gain 25 to 35 pounds during your pregnancy.” d. “You should gain 28 to 40 pounds during your pregnancy.” ANS: B In 2009, the IOM released new weight gain guidelines that are based on revised BMI categories. Overweight women (BMI of 25 to 29.9) should gain 15 to 25 pounds. Healthy women at a normal weight for their height (BMI of 18.5 to 24.9) should gain 25 to 35 pounds. Underweight women (BMI less than 18.5) should gain 28 to 40 pounds. Obese women (BMI greater than 30) should limit their gain to 11 to 20 pounds. DIF:

Cognitive Level: Analyze (Analysis)

REF:

pp. 357-358

8. A nurse is caring for a woman who is in labor. The nurse anticipates that the labor will progress through which of the following sequences? a. Dilation stage, pressure stage, placental stage, recovery stage b. Dilation stage, pushing stage, pain stage, recovery stage c. Dilation stage, pushing stage, placental stage, recovery stage d. Dilation stage, pushing stage, placental stage, refractory stage ANS: C The sequence for the four stages of labor is dilation stage, pushing stage, placental stage, and recovery stage. DIF:

Cognitive Level: Apply (Application)

REF:

p. 358 9. A nurse is caring for a woman who is in labor and the fetus descends to the lower birth canal. The woman is in which of the following stages of labor? a. First b. Second c. Third d. Fourth


ANS: B During the second stage of labor, the fetus descends through the lower birth canal toward the woman’s perineum. DIF:

Cognitive Level: Apply (Application)

REF:

p. 358

10. A nurse is caring for a woman during the fourth stage of labor. Which of the following best describes an action the nurse would take during this stage? a. Providing constant reinforcement and education about the labor process and assisting the woman with pushing b. Explaining unusual interventions such as the use of a fetal heart monitor c. Assisting with labor discomfort by modeling breathing d. Teaching the new mother positioning of the infant to assist with breast-feeding ANS: D Active nursing support during the third and fourth stages of labor includes observing for excessive vaginal bleeding after the placenta is expelled, assisting the woman with breast-feeding, monitoring vital signs, implementing uterine massage as indicated, and providing emotional support. Thus, the best response is the nurse teaching the new mother the football hold to assist with breast-feeding. DIF:

Cognitive Level: Apply (Application)

REF:

p. 359

11. Which of the following statements about race and culture in the United States is correct? a. The United States ranks 22nd in the world for infant mortality rate. b. Ethnic minority populations have higher rates of low-birth-weight infants. c. Black infants are more likely to have cleft palates than Native American infants. d. Whites have more fraternal twin pregnancies than non-Whites. ANS: B Currently, the United States is 49th in the world ranking for infant mortality rate (IMR). This rate, which reflects the number of infants who die before the end of their first year of life is the leading indicator of a nation’s health, reflects the higher IMRs and low-birth-weight outcomes of Blacks, American Indians, and other ethnic minority populations in the United States. In the United States, non-Whites (mainly Blacks) have more fraternal twin pregnancies than Whites. More Native American, Latino, or Asian descent babies have cleft palates than do Black babies. DIF:

Cognitive Level: Understand (Comprehension) REF:

p. 360

12. During the second prenatal visit, the pregnant woman informs the nurse that she has stopped smoking. This finding is based on an assessment of which functional health pattern? a. Cognitive-perceptual pattern b. Self-perception–self-concept pattern c. Health-perception–health-management pattern d. Values-beliefs pattern ANS: C A woman’s acceptance of her pregnancy influences her health management practices. A woman who denies or has negative feelings about her pregnancy may fail to eat properly, get enough rest and exercise, and so on. A woman who has stopped smoking because she is pregnant indicates that she accepts her pregnancy and the health practices that must be instituted to promote a healthy pregnancy, delivery, and baby. Such an assessment is a part of the health perception–health management pattern. DIF:

Cognitive Level: Apply (Application)

REF:

pp. 362-363

13. A pregnant woman who is at the end of 20 weeks of gestation is at a prenatal visit. Which of the following changes in weight from pre-pregnancy would require the nurse to take additional action? a. 6 pound increase


b. c. d.

11 pound increase 14 pound increase 20 pound increase

ANS: A If at the end of 20 weeks of gestation the woman has not gained at least 10 pounds, she risks delivering an ill infant suffering from intrauterine growth restriction. DIF:

Cognitive Level: Apply (Application)

REF:

p. 363

14. During a prenatal visit, a nurse discovers that a Black woman has been occasionally craving and eating clay. Which of the following actions should be taken by the nurse? a. Tell the client that many pregnant women crave such nonfood substances, and it is not a problem. b. Ignore the comment because pica is acceptable in some rural Black cultures. c. Encourage the client to eat it with plenty of water because of the risk of constipation. d. Stress the importance of an appropriate diet and avoiding pica in a culturally sensitive way. ANS: D Pica may negatively influence the quality of a woman’s nutrition during pregnancy. The practice of pica is acceptable in some rural Black cultures. However, it can lead to lead poisoning, fecal impaction, parasitic infections, prematurity, perinatal mortality, low birth weight infants, and anemia in infants. When the nurse identifies instances of pica, she should suggest a culturally sensitive diet that will better meet the needs of the woman and her fetus. The nurse must remain nonjudgmental but stress the importance of an appropriate diet and the dangers of pica. DIF:

Cognitive Level: Apply (Application)

REF:

pp. 364-365

15. A nurse is assessing the nutritional-metabolic patterns of a pregnant woman. Which of the following findings would be of concern to the nurse? a. The woman drinks about 1 gallon of water a day. b. The woman works out at the gym daily. c. The woman does not like vegetables. d. The woman usually eats three meals a day in addition to two snacks a day. ANS: C Because of pressure from the enlarging uterus, slowed peristalsis, and supplemental iron and calcium intake, the woman is at risk for constipation that can then lead to hemorrhoids. Therefore, the woman should eat foods high in fiber, including vegetables and fruits. A woman who does not like or eat vegetables is at risk for constipation and hemorrhoids. DIF:

Cognitive Level: Analyze (Analysis)

REF:

p. 365

16. A woman who is 23 weeks pregnant is concerned because her baby is moving less than it was a few weeks ago. Which of the following statements would be the most appropriate response by the nurse? a. “Your infant is in distress. We should call an ambulance.” b. “You need to be more active; take a dance class once a week.” c. “At this point in your pregnancy, the baby moves less frequently because of lack of space in the uterus.” d. “You probably counted incorrectly.” ANS: C Education and guidance are important nursing functions. It is true that by the end of the second trimester fetal movement occurs less frequently because of lack of space in the uterus. The nurse should provide the woman with this information, so she knows what to anticipate.


DIF:

Cognitive Level: Apply (Application)

REF:

p. 365

17. Which of the following mothers would be most likely to bond appropriately with their infant? a. A mother who feels unloved will bond appropriately because now the baby will love her. b. A mother who feels great about herself will show love toward the infant and bond appropriately. c. A mother who feels empty by the birth of her child will bond appropriately once she holds the baby. d. A mother who feels ugly will bond appropriately with her child because her child will bring selfesteem. ANS: B After birth, the woman gradually sees the infant more and more as a separate individual, dependent on her care. The mother starts to bond with her baby based on her self-perception. If she feels good about herself, she will show love toward the infant. When she feels ugly or unlovable, she may make uncomplimentary remarks about the infant’s appearance. DIF:

Cognitive Level: Apply (Application)

REF:

p. 368

18. During the first prenatal visit, the pregnant woman informs the nurse that she cannot wait to start wearing maternity clothes. This finding is based on an assessment of which functional health pattern? a. Cognitive-perceptual pattern b. Self-perception–self-concept pattern c. Health-perception–health-management pattern d. Values-beliefs pattern ANS: B To develop a maternal identity, the woman must first accept the pregnant body image. The pregnant woman’s personality, maturity level, and psychological development influence her readiness to assume the role of mother. Being excited about wearing maternity clothes is an indication that she has accepted the pregnant body image. Such an assessment is a component of the self-perception–self-concept pattern. DIF:

Cognitive Level: Apply (Application)

REF:

p. 368

19. A nurse is caring for a client who is 30 weeks pregnant at a prenatal visit. Which of the following statements made by the client would be of concern to the nurse and warrant further explanation and close follow-up? a. “I have been feeling more tired lately.” b. “My husband complains every time I ask him to do something for me.” c. “Sometimes, the smell of food makes me nauseous.” d. “I need to get up two times a night to go to the bathroom.” ANS: B Fatigue, nausea, and increased urinary frequency are all normal discomforts associated with pregnancy. These discomforts warrant anticipatory guidance from the nurse. A husband who resents the attention that his pregnant wife is receiving and the additional demands she may make on his time may lead him to abuse his wife. Therefore the nurse must obtain additional information and provide close follow-up when a woman states that her husband complains every time she asks him to do something because she may be at risk for abuse. DIF:

Cognitive Level: Analyze (Analysis)

REF:

p. 369

20. A woman who is 30 weeks pregnant tells the nurse that she and her husband are having sexual difficulties. Which of the following actions should the nurse take? a. Tell the woman she should not be sexually active at this point in her pregnancy because it may harm the fetus. b. Ask her to elaborate on the difficulties. c. Tell her that they should modify their positions during intercourse and that will take care of the problem.


d.

Tell her men like the way pregnant women look.

ANS: B Some women worry about intercourse during pregnancy, fearing that it will cause miscarriage, infection, early delivery, or harm to the baby. Sexual dissatisfaction of the couple may result from restrictions in sexual positions, pain on penetration, increased vaginal discharge, breast tenderness, or the other physical discomforts of pregnancy such as fatigue or heartburn. The couple’s feeling about the woman’s changing body may alter their sexual relationship. The nurse’s first step in primary prevention intervention is to support the couple’s needs and relate, in a sensitive fashion, accurate information that facilitates couple’s intimacy during pregnancy. Asking her to elaborate on the difficulties allows the nurse to understand the root of the problem and develop an appropriate intervention. It also opens the lines of communication, so the nurse can discuss common difficulties experienced by couples and help her and her husband adjust to the challenges and discomforts of pregnancy. Giving the woman advice on position changes or body image may not be effective if it is not the cause of the difficulties. DIF:

Cognitive Level: Apply (Application)

REF:

pp. 371-372

21. During a prenatal visit, a pregnant woman informs the nurse that she and her husband have chosen godparents for the baby. This finding is based on an assessment of which functional health pattern? a. Cognitive-perceptual pattern b. Self-perception–self-concept pattern c. Health-perception–health-management pattern d. Values-beliefs pattern ANS: D For women with strong spiritual needs related to their cultural backgrounds, spiritual interventions will help them integrate various dimensions of their lives, develop the ability to parent successfully, and find meaning in the changes and goals of pregnancy. Identifying godparents will help the woman find meaning in the changes and goals of pregnancy. Such religious beliefs assessments are components of the values-beliefs pattern. DIF:

Cognitive Level: Apply (Application)

REF:

p. 372

22. A pregnant woman is having a TORCH screening done at today’s prenatal visit. She states she is anxious to get the results back because she recently had unprotected sex with someone she just met and is afraid she might have HIV. Which of the following statements would be the best response by the nurse? a. “You will know your HIV status in about 1 week when the results come back.” b. “You will require additional testing for HIV as it is not tested for with the TORCH screen.” c. “You will need to be tested for HIV after you deliver because pregnancy can produce false HIV results.” d. “You need to be exposed for at least 6 months before being tested for HIV in order for the results to be accurate.” ANS: B The TORCH screening helps detect toxoplasmosis, hepatitis B, rubella, cytomegalovirus, and herpes simplex. HIV is not detected with the TORCH screen. Therefore she should tell the woman that she will require additional testing for HIV. DIF:

Cognitive Level: Apply (Application)

REF:

p. 373

23. A nurse makes a home visit to monitor the blood pressure of a pregnant woman who is single. Which finding would be of most concern to the nurse? a. The bedroom is located on the second floor. b. She has a pet cat. c. There is an area rug in the living room. d. She has a 1-year-old child.


ANS: B Toxoplasmosis is caused by a protozoan that infects people through undercooked meat, handling of cat feces, and exposure to infected soil. Sixty percent of maternal infections acquired during the third trimester result in fetal infection, which can lead to rash, enlarged lymph nodes and liver, inflammation of the heart, pneumonia, jaundice, and severe central nervous system damage after birth or years later. Thus, pregnant women should avoid handling cats or cleaning cat litter boxes to avoid exposure to toxoplasmosis. DIF:

Cognitive Level: Analyze (Analysis)

REF:

pp. 373-374

24. A client who is 36 weeks of gestation is at her prenatal visit. The nurse tells the client that she will be screened for group B Streptococcus during today’s visit. Which of the following statements would be made by the nurse when providing client education about this screening test? a. “Screening for group B Streptococcus is necessary as this infection can cause complications with pregnancy and to the unborn infant.” b. “This screening will help us to determine if your unborn infant will be susceptible to respiratory distress when he or she is born.” c. “Screening for group B Streptococcus will help us identify if you have been exposed to this or any other infections during your pregnancy.” d. “This screening is necessary to determine if there is any Rh blood group incompatibility between you and your unborn child.” ANS: A Screening at 36 to 37 weeks of pregnancy for group B Streptococcus infection has been recommended because this infection causes preterm rupture of the amniotic membranes, premature labor, fetal respiratory distress syndrome, fetal septicemia, and meningitis. DIF:

Cognitive Level: Analyze (Analysis)

REF:

p. 375

25. A woman is Rh negative and the father of her baby is Rh negative. The woman states that her friend told her that she would need a “shot” to keep her baby alive. Which of the following statements would be the best response by the nurse? a. “Yes, you will need an injection of RhoGAM at 28 weeks gestation and within 72 hours after birth.” b. “Because the father of the baby is also Rh negative, your baby will not inherit Rh positive blood cells and you will not need the injection.” c. “Yes, you will need the injection because this is your first pregnancy. You will not need it with subsequent pregnancies.” d. “No, you will not need the injection because you are White. Rh incompatibility affects only Black women.” ANS: B Rh incompatibility sometimes affects fetal development. The problem usually occurs when the mother has Rhnegative blood cells and the fetus has Rh-positive blood cells, inherited from the father who also has Rh-positive blood cells. All women should be assessed for blood type, Rh factor, and antibody development at their first prenatal visits and again at 24 to 28 weeks, unless the father is Rh-negative. Rh incompatibility may be prevented by administering RhoGAM at 28 weeks and within 72 hours after birth. In this case, because the father is Rh negative, RhoGAM is not necessary. DIF:

Cognitive Level: Apply (Application)

REF:

p. 376

26. A woman who just found out she is pregnant starts crying. She tells the nurse that over the last 3 days she was not feeling herself, so she took some Tylenol. She states if she knew she was pregnant, she never would have taken the medication. Which of the following actions should be taken by the nurse? a. Tell the client that there is a high probability that her baby will have a birth defect b. Tell the client not to worry because research indicates that in recommended doses Tylenol is safe. c. Tell the client not to worry because she did not know she was pregnant, but in the future, she should use herbal products instead of over-the-counter medication.


d.

Tell the client that there is a high probability of a stillbirth.

ANS: B Research on aspirin and Tylenol indicates that both medications are safe in recommended doses. Little is known about the effects of herbal products and their interaction with other medications. As a result, the nurse may need to encourage a client to reconsider the use of herbs when evidence exists that the herbs may harm the fetus or the mother. Providing the mother with appropriate information will help to decrease the mother’s anxiety level. DIF:

Cognitive Level: Apply (Application)

REF:

p. 377

27. A nurse is discussing the harmful effects that chemical agents can have on an unborn child with a woman who is pregnant. Which of the following statements made by the woman indicates a need for further teaching? a. “It is safe to eat up to 12 ounces of cooked fish weekly.” b. “I should avoid using caffeine during pregnancy.” c. “Consumption of one drink per day will not cause any harm to my unborn baby.” d. “Use of nicotine during pregnancy may cause my child to be born prematurely.” ANS: C Numerous studies have shown that no safe level of alcohol use exists during pregnancy; therefore alcohol should be avoided during this time and when attempting conception. DIF:

Cognitive Level: Analyze (Analysis)

REF: p. 378

28. During a prenatal visit, pregnant parents ask the nurse what they can do to prepare their 4-year-old child for the birth of the new baby. Which of the following actions would be most appropriate for the nurse to suggest? a. Discourage any negative comments the child makes about the baby or the pregnancy. b. Do not make any plans regarding the baby in front of the 4-year-old child. c. Provide him with very detailed information about the pregnancy and the birth. d. Ask the 4-year-old boy to help decorate the new baby’s room. ANS: D Children in the family also experience role changes during and after the pregnancy. Having a child participate in decisions about the baby helps to prepare the child for the arrival of the neonate. Helping the parents decorate the new baby’s room allows the child to participate in the arrival of the new baby. DIF:

Cognitive Level: Analyze (Analysis)

REF:

p. 371 (Box 16-8)

29. A Mexican American woman comes to the office for a visit. She is found to be 30 weeks pregnant. Which of the following conclusions can the nurse draw from this finding? a. The woman does not value prenatal care. b. Client education may require a different approach because of dissimilar cultural beliefs. c. This culture does not believe in traditional medicine. d. Signs of pregnancy were not recognized by the woman. ANS: B Cultural groups have unique ideas and beliefs related to pregnancy, childbirth, and childbearing that must be understood by the nurse to render individualized care to each pregnant woman. Mexican Americans may consider prenatal care not needed because pregnancy is a normal life event. In addition, education influences their prenatal care access. Therefore finding out that she is 30 weeks pregnant may be an indication that this woman may require a different approach to education because of dissimilar cultural beliefs. DIF:

Cognitive Level: Analyze (Analysis)

REF:

p. 356 (Table 16-1)

30. At 1 minute of age, an infant has a heart rate of 95, a strong cry, some flexion of extremities, a cry reflex, and is completely pink. Which of the following is a correct assessment of the infant’s Apgar score?


a. b. c. d.

4 6 8 10

ANS: C There are five signs that are scored using the Apgar scoring system. A heart rate under 100 receives a score of 1, a strong cry receives a score of 2, some flexion of extremities receives a score of 1, a cry for the reflex irritability sign receives a score of 2, and an infant whose color is completely pink receives a score of 2: 1 + 2 + 1 + 2 + 2 = 8. DIF:

Cognitive Level: Apply (Application)

REF:

p. 360 (Table 16-3)

31. A woman usually eats an 1800-calorie diet. She is now in the second trimester of her pregnancy. Which of the following best describes the caloric nutritional needs for this woman during the second and third trimester? a. 1800 calories b. 2100 calories c. 2400 calories d. 2700 calories ANS: B During pregnancy, a woman’s caloric needs increase by 300 calories. Someone who usually eats an 1800-calorie diet should increase her caloric intake to 2100 calories (1800 + 300 = 2100). DIF:

Cognitive Level: Apply (Application)

REF:

p. 364 (Table 16-4)

MULTIPLE RESPONSE 1. Which properties must the sperm possess for conception to occur? (select all that apply) a. High motility b. Uniform size c. Ability to secrete enzymes that dissolve the membrane surrounding the egg d. Life span of at least 3 hours ANS: A, B, C The sperm must be of uniform size, be normally formed, possess high motility, and have an ability to secrete enzymes that dissolve the membrane surrounding the egg. DIF:

Cognitive Level: Remember (Knowledge)

REF:

p. 352

2. A nurse is assessing a woman for positive signs of pregnancy. Which of the following assessment findings would the nurse discover? (select all that apply) a. Positive test for HCG in the maternal urine b. Detection of fetal heart tones c. Enlargement of the uterus d. Palpation of fetal body parts ANS: B, D Positive signs of pregnancy include: detection of fetal heart tones by auscultation, ultrasonography, or a Doppler; palpation of fetal body parts using Leopold maneuvers; fetal movements visible and detected by examiner; and radiological or ultrasonographic demonstration of fetal parts. Enlargement of the uterus and positive test for HCG in the maternal urine are both probable signs of pregnancy. DIF:

Cognitive Level: Apply (Application)

REF:

p. 354 (Box 16-1)


Chapter 17: Infant Edelman: Health Promotion Throughout the Life Span, 8th Edition MULTIPLE CHOICE 1. An infant is lying in a crib watching and listening to a colorful mobile. What is the purpose of having a mobile? a. Provides stimulation necessary for continued growth and development for the infant b. Provides too much stimulation for the infant, making it difficult for the infant to focus c. Serves as inadequate stimulation for the infant because it is an inanimate object d. Serves as a distractor for the infant, making it easier to fall asleep ANS: A The mobile is providing the stimulation necessary for continued growth and development because both auditory and visual stimuli are necessary to promote growth and development. Additionally, soft sounds and voices are soothing to infants. DIF:

Cognitive Level: Analyze (Analysis)

REF:

p. 391

2. The nurse performs a home visit for a new mother and infant. The nurse observes that the infant lacks a regular feeding schedule, has diapers that are not changed promptly, and cries for long periods of time before being consoled. Which of the following problems is the infant most at risk for encountering in the future? a. Developing enuresis as a toddler b. Having difficulty forming relationships c. Becoming a picky eater d. Taking longer to toilet train ANS: B The infant may have difficulty forming relationships in the future because trust influences future relationships. By attending to the infant’s needs, parents are helping the infant develop trust in them. Not having his needs attended to hinders that developmental task. DIF:

Cognitive Level: Analyze (Analysis)

REF:

p. 392

3. By what age is an infant’s retina fully developed? a. Birth b. 2 months c. 4 months d. 6 months ANS: C Rod cells in the retina of the eyes, which are responsible for light perception, are functional at birth; however, the retina (the organ of visual perception) is not fully developed until approximately 4 months of age. DIF:

Cognitive Level: Remember (Knowledge)

REF:

p. 392

4. A nurse is completing a well-child assessment of a 6-month-old infant. Which of the following guidelines should be used when assessing the infant’s height and weight? a. CDC growth charts b. CDC body mass index (BMI) charts c. WHO growth standards d. Denver Developmental Screening height and weight standards ANS: C


The CDC recommends that health care providers use the WHO growth standards to monitor growth for infants and children ages 0 to 2 years of age in the United States and use the CDC growth charts for children age 2 years and older in the United States. DIF:

Cognitive Level: Apply (Application)

REF:

p. 394

5. A delivery room nurse senses disappointment when a mother is told she just delivered a baby girl. Which of the following would be the most therapeutic response by the nurse? a. “Yes, you just delivered a healthy, beautiful baby girl.” b. “This is your first baby. You can always try again for a boy.” c. “Don’t worry, many parents often feel the same way you do.” d. “You are tired. I think you should get some rest.” ANS: A The nurse should respond by saying, “Yes, you just delivered a healthy, beautiful baby girl.” This response provides information that will help increase the mother’s acceptance of having a girl rather than a boy. Acceptance and bonding are important in preventing child abuse. The fact that the mother appeared disappointed should be acknowledged with a positive comment. DIF:

Cognitive Level: Analyze (Analysis)

REF:

p. 394

6. A nurse is working with a family of an infant to improve their health-perception and health management pattern. Which of the following interventions would be most appropriate for the nurse to implement? a. Assess the home for safety hazards. b. Teach parents appropriate coping mechanisms. c. Demonstrate effective parenting skills. d. Encourage appropriate stimulation of the infant. ANS: C Health is largely a subjective judgment. With this understanding, the nurse uses every opportunity to convey confidence in the parents’ health perception-health management pattern and their ability to act to enhance the infant’s health. When parents learn and adopt behaviors to improve their own health, they are more likely to ensure that the health needs of their infant are met. Parental modeling increases the changes that good health practices will be retained throughout the child’s life. DIF:

Cognitive Level: Apply (Application)

REF:

pp. 395-396

7. The mother of an 8-month-old asks what the minimum recommended amount of fat intake is for her 10-kg infant. Which of the following would be the minimum recommended fat intake? a. 18 grams a day b. 28 grams a day c. 38 grams a day d. 48 grams a day ANS: C The minimum recommended fat intake for this 8-month-old, 10-kg infant is 38 grams per day. The minimum recommended fat intake for an infant this age is 3.8 g/kcal. Thus, 3.8  10 = 38. DIF:

Cognitive Level: Apply (Application)

REF:

p. 396

8. The mother of a 3-month-old is concerned because her infant usually falls asleep halfway through his bottle. Because of this, the mother is afraid he will not grow well and get sick. Which of the following would be the most therapeutic response by the nurse? a. Ask the mother about the infant’s feeding schedule. b. Assure the mother that he is growing fine. c. Tell the mother that half a bottle is probably enough for him.


d.

Tell the mother she should start to add solids to his diet.

ANS: A The most therapeutic response would be to ask the mother about the infant’s feeding schedule. The mother’s concern needs to be explored further so that the nurse can understand if the child is receiving adequate nutrition. This response will open the lines of communication and allow the mother to discuss her fears as well as the feeding schedule. Parents look to nurses for anticipatory guidance. The nurse should listen before giving advice. Knowing the feeding schedule will help to determine the best intervention. DIF:

Cognitive Level: Analyze (Analysis)

REF:

p. 396 | p. 399

9. A nurse is completing a home visit of a new mother who is breast-feeding her infant. Which of the following instructions should the nurse provide during this visit? a. Consume 500 kcal/day above pre-pregnancy energy intake to avoid excessive weight loss. b. Drink 1 quart of fluids daily to produce sufficient quantity of breast milk. c. Use formula to provide the infant supplementation between feedings. d. Use a pacifier to help the infant to fall asleep so he does not fall asleep when breast-feeding. ANS: B Community nurses who are caring for breast-feeding mothers should stress the following tips to increase the duration of this activity: drink 1 quart of fluids daily to produce sufficient quantity of breast milk; consume 300 to 400 kcal/day above pre-pregnancy energy intake to avoid excessive weight loss; learn the appropriate interventions for engorged breasts, sore nipples, plugged ducts, infection, and leaking; learn about the use of breast pumps and milk storage; join breast-feeding support groups for continued help within the community; learn about the effects of drugs, environmental pollutants, alcohol, and nicotine on breast milk. The use of formula and pacifiers should be discouraged with new infants who are breast-feeding. DIF:

Cognitive Level: Apply (Application)

REF:

pp. 397-398

10. A mother asks the nurse when she should start feeding her infant solid foods. Which of the following is the most appropriate response by the nurse? a. “It is recommended that solid foods are introduced no earlier than 3 months of age.” b. “It is recommended that solid foods are introduced no earlier than 6 months of age.” c. “You should wait until your child is 9 months old to start solid foods.” d. “You should wait until your child is 1 year old to start solid foods.” ANS: B The AAP (2011) recommends that waiting until the child is 6 months of age to introduce solid food decreases the tendency to develop food allergies and reduces the risk of childhood obesity. DIF:

Cognitive Level: Apply (Application)

REF:

p. 398

11. A new mother has decided to breast-feed her infant after having bottle-fed her other two children. Which of the following information about having a breast-fed infant should the nurse discuss with the mother? a. Breast-fed infants have darker bowel movements than bottle-fed babies. b. Breast-fed infants have smellier bowel movements than bottle-fed babies. c. Breast-fed infants have more frequent bowel movements than bottle-fed babies. d. Breast-fed infants have harder bowel movements than bottle-fed babies. ANS: C A breast-fed infant’s stools have an orange-yellow color and a soft, even consistency, with a slightly sour but clean smell, dissimilar to stools passed later in life. A bottle-fed infant’s stools are harder, smellier, and resemble those of an infant eating solid food. The breast-fed infant has many daily stools during the first and second months of life. The bottle-fed infant has two to four stools per day during the first month.


DIF:

Cognitive Level: Apply (Application)

REF:

p. 399

12. During a home visit, the nurse finds a 9-month-old in a playpen with a couple of toys. Which of the following instructions would be most appropriate for the nurse to provide to the parent to encourage growth and development of the child? a. Encourage the addition of a few more toys to the playpen. b. Suggest keeping the infant in the playpen as much as possible to promote safe play. c. Encourage providing the infant with supervised time outside of the playpen. d. Suggest removing all toys from the playpen. ANS: C The nurse should encourage the mother to provide the infant with sufficient supervised time outside of the playpen to allow the child to crawl and explore. These activities help promote growth and development. Additionally, by 9 months of age, most infants are crawling. DIF: Cognitive Level: Apply (Application) REF: pp. 400-401 | p. 390 (Box 17-2) 13. A new young mother asks the nurse what she should do to play with her 3-month-old infant. Which of the following suggestions should the nurse provide? a. Take the infant for a walk outside. b. Place several toys around the infant when lying on the floor. c. Rock the infant in a rocking chair. d. Search the Internet for toys highly recommended for infants. ANS: C Parental stimulation of the infant is an important developmental technique. The infant needs stimulation to learn about the world. This activity does not require expensive objects, but rather involves experiences in sight, sound, and touch that are free and can be provided by any parent. One example of a stimulating experience for infants is being rocked in a rocking chair. Infant who are 3 months old will hold toys, but not actively reach for them so surrounding the infant with toys would not be suggested. DIF:

Cognitive Level: Apply (Application)

REF:

p. 401

14. The mother of a 4-month-old infant is concerned about the possibility of SIDS because her neighbor’s daughter passed away last year as a result of SIDS. The mother reports she is so nervous that she has taken up smoking again. The mother informs you that the 4-month-old sleeps supine and takes a pacifier to help her fall asleep. Which of the following statements would be the best response from the nurse? a. “You should place the infant in the bed with you until 6 months of age.” b. “You should never give the infant a pacifier while she is sleeping.” c. “You should try to stop smoking.” d. “You should place the infant on her stomach.” ANS: C The 2011 report from the American Academy of Pediatrics Task Force on SIDS recommended the following: healthy infants should be placed to sleep in the supine position; infants should be placed on a firm sleep surface keeping soft objects, loose bedding, or any objects that could increase the risk of entrapment, suffocation, or strangulation out of the crib; avoid the infant getting too hot by dressing him/her in too many clothes for the environment; keep the infant away from smokers and places where people smoke; sleep with the infant in the same room where you sleep, but not the same bed, for 6 months; and offer a pacifier at nap time and bedtime. DIF:

Cognitive Level: Apply (Application)

REF:

p. 402 | p. 402 (Box 17-10)

15. A nurse is developing an educational program about SIDS for a new mothers’ support group in the community. Which of the following information would be included in this presentation? a. Parents are encouraged to place their infants in the supine sleeping position.


b. c. d.

SIDS is the leading cause of death among infants age 6 months to 1 year old. SIDS is associated with infants who have had difficulty sleeping at night. Parents are encouraged to limit the amount of “tummy time” of their infant.

ANS: A Through the Back to Sleep program developed by the AAP, there has been a lower number of infant deaths from SIDS. This has encouraged the supine sleeping position. One of the consequences of the supine sleeping position is plagiocephaly. The primary intervention to decrease the risk of plagiocephaly is “tummy time.” Parents should be instructed to allow supervised tummy time while the infant is awake and cautioned about the amount of time their infant spends in a car seat. SIDS is the leading cause of death among infants 28 to 365 days of age. By definition, the cause of SIDS is not known. DIF:

Cognitive Level: Apply (Application)

REF:

p. 402

16. A nurse is assessing the language development of a 6-month-old infant. Which of the following findings would the nurse anticipate? a. The infant forms two-syllable sounds. b. The infant coos and makes vowel sounds. c. The infant babbles. d. The infant says “ma-ma” and “bye-bye.” ANS: C By 6 months, babbling sounds are heard, and by 9 to 10 months, the infant forms two-syllable sounds. By 12 months, words such as “ma-ma” and “bye-bye” are emerging. Cooing and vowel sounds are heard at approximately 2 to 3 months. DIF:

Cognitive Level: Apply (Application)

REF:

p. 404

17. Which of the following infants is most at risk to experience child abuse? a. Father has experienced paternal engrossment b. Responds to parental touch c. Parents have low-self esteem d. Has been adopted ANS: C Because the parents’ self-esteem is associated closely with their infant’s interactions and accomplishments, when parents’ self-esteem is low, disappointment, anger, and a disturbance in the relationship with their infant can occur. When a disturbed parent-infant relationship continues, the infant is at risk for abuse and behavior problems. The process of paternal engrossment has been used to describe the behavior patterns of fathers when they interact with their infants. DIF:

Cognitive Level: Apply (Application)

REF:

p. 406

18. A nurse is assessing a 4-month-old infant during a well-child visit. Which of the following findings will require the nurse to collect additional information? a. The infant’s shirt is wet from drooling. b. The infant has gained one pound since her 2-month well-child visit. c. The infant holds his or her head steady when in a sitting position. d. The infant grasps objects with two hands. ANS: B One of the passive manifestations of abuse includes poor nutrition, failure to thrive, and severe malnutrition. Weight gain of 1 pound in 2 months indicates a problem with the infant’s growth and requires further data collection by the nurse. The other assessment findings are normal growth and development for a 4-month-old infant.


DIF:

Cognitive Level: Analyze (Analysis)

REF: p. 406

19. A nurse visits a 3-month-old infant of a single mother for a weight check. She finds two older children running around the home, the infant is crying, the mother yelling at the two children and a half-lit cigarette is in an ashtray. The mother tells the nurse the visit needs to be cut short today because she has to pick up her other children from the bus stop. Which of the following would be the most beneficial intervention this nurse can provide during this visit? a. Leave and tell the mother to call her to reschedule the appointment. b. Weigh the infant and contact the Department of Children and Families. c. Weigh the infant and offer community resources to the mother. d. Offer to go pick up the other children at the bus stop. ANS: C The most beneficial intervention the nurse can provide is to keep the visit short, weigh the infant, and offer the mother the help of community services. Providing anticipatory guidance is important in maintaining health and preventing abuse. Abusing parents are often socially isolated and have few people to whom to turn. Women are more frequent abusers than men. Furthermore, economic conditions play a role in the care of children. DIF:

Cognitive Level: Analyze (Analysis)

REF: p. 407

20. Where do most accidents occur? a. In the home b. In the car c. At day care d. On the playground ANS: A Accidents occur in many situations: in the home, outside, on the playground, and in automobiles. However, most accidents occur in the home. DIF:

Cognitive Level: Remember (Knowledge)

REF:

p. 410

21. A nurse is encouraging parents to immunize their infants. Which of the following strategies would be the best way for the nurse to facilitate this? a. Send a reminder for immunizations to the parents. b. Talk to the parents about immunizations during office visits. c. Track clients who have not been immunized. d. Memorize the CDC immunization schedule. ANS: B The best way for a nurse to encourage immunization is to talk to parents about immunizations during office visits. Immunizations are important in the prevention of disease. Actively broadening one’s knowledge base regarding immunizations and developing a close relationship with the parents that promotes open communication are important factors for motivating parents. DIF:

Cognitive Level: Apply (Application)

REF:

22. Which of the following is the most common poison ingested by infants? a. Houseplants b. Lead c. Cleaning agents d. Aspirin ANS: D

p. 413


The most common ingested poison is aspirin, followed by houseplants and cleaning agents. DIF:

Cognitive Level: Remember (Knowledge)

REF:

pp. 413-414

23. A new father is installing a car seat for an infant in the car. Which of the following information should be given to the father by the nurse? a. The infant should be in a rear-facing car seat in the front seat. b. The infant should be in a rear-facing car seat in the back seat. c. The infant should be in a front-facing car seat in the front seat. d. The infant should be in a front-facing car seat in the back seat. ANS: B The AAP recommends rear-facing car safety seats for most infants up to 2 years of age. The back seat is recommended because of the danger posed by air bags. DIF:

Cognitive Level: Apply (Application)

REF:

p. 417

24. A 15-month-old infant has been brought into the office by his Spanish-speaking mother for symptoms of an upper respiratory infection. Which of the following should be the initial action taken by the nurse? a. Ask the mother if she would like an interpreter. b. Ask the mother what home remedies have been used. c. Ask the mother how long the child has had these symptoms. d. Ask the mother about any allergies the child may have. ANS: A The nurse should start by asking the mother if she would like an interpreter. Once the need for an interpreter has been established, the nurse can continue with the other questions. DIF:

Cognitive Level: Analyze (Analysis)

REF:

pp. 419-420

25. A nurse would like to be involved in promoting change in health care policies. Which of the following actions would be most appropriate for the nurse to accomplish this goal? a. Be an active member of a national nursing organization. b. Vote for political leaders in favor of health care reform. c. Enroll in a health policy class. d. Learn about community resources. ANS: A The best way for a nurse to promote change in health care policies is to be an active member of a national nursing organization. As an active member, she can join other professionals in lobbying for change. DIF:

Cognitive Level: Apply (Application)

REF:

p. 421

26. A mother asks a nurse what the best toy would be for her 2-month-old. Based on the growth and development of the infant, which of the following recommendations should be made by the nurse? a. Colorful mobile with music b. Rattle c. Stuffed teddy bear d. Play telephone ANS: A The most appropriate toy based on the child’s age and developmental level would be a colorful mobile with music. At this age, the infant can listen to sounds and follow objects. The mobile provides visual and auditory stimulation to continue to promote growth and development.


DIF:

Cognitive Level: Analyze (Analysis)

REF:

p. 390 (Box 17-2)

27. A 5-month-old boy was born at 6 pounds, 7 ounces. He is being seen in the office for a well-child visit. The nurse would expect him to weigh approximately: a. 10 pounds, 14 ounces b. 11 pounds, 7 ounces c. 12 pounds, 14 ounces d. 13 pounds, 7 ounces ANS: C The child should weigh approximately 12 pounds, 14 ounces, because by 5 months of age, the child should weigh twice the birth weight. DIF:

Cognitive Level: Apply (Application)

REF:

p. 390 (Box 17-2)

28. A 9-month-old has mastered the pincer grasp. Which of the following tasks would the nurse anticipate that the infant can do by himself? a. Play with a rattle. b. Eat cheerios off a hard surface. c. Hold a spoon to feed himself. d. Drink from a cup. ANS: B The pincer grasp is a fine motor skill that is necessary for picking items up off a surface. By this age, handmouth coordination has usually been met and therefore feeding the self with finger foods becomes possible. DIF:

Cognitive Level: Apply (Application)

REF:

p. 390 (Box 17-2)

29. A nurse makes a home visit to a 15-month-old following a hospitalization for a fall. Which finding would be concerning to the nurse? a. Radiator heating system in the home b. The child drinking from a cup c. The child experiencing a temper tantrum d. A decorative bowl filled with colorful marbles on the coffee table ANS: D A decorative bowl filled with colorful marbles on the coffee table is concerning because by 15 months of age the infant is mobile and can eat finger foods. By 7 months of age, the infant fixates on small objects; therefore a safe environment must be created for the infant. Small objects are the most common causes of foreign body aspiration. A bowl of colorful marbles poses a risk for foreign body obstruction. DIF:

Cognitive Level: Analyze (Analysis)

REF:

p. 390 (Box 17-2)

30. A nurse assesses that the task of development of affection for and from others has been met or is in the process of being met by an infant. Which of the following observations has the nurse made? a. The infant smiles at mirror images. b. The infant makes cooing noises. c. The infant begins to smile socially. d. The infant likes to be picked up. ANS: D An infant who likes to be picked up is displaying affection for others. This task is usually accomplished by 6 months of age. DIF:

Cognitive Level: Apply (Application)


REF:

p. 390 (Box 17-2) | p. 392 (Box 17-3)

31. An infant is going through the toys in his toy box. He looks at each one before dropping it to the floor and picking the next toy out of the box. Using a growth and development perspective, which of the following conclusions can the nurse draw by assessing this behavior? a. The infant is testing his limits. b. The infant is exploring his environment. c. The infant is expressing his emotions. d. The infant is developing fine motor skills. ANS: B The infant is exploring his environment. By looking at each toy, he is developing the task of learning to understand and control his world through exploration. DIF:

Cognitive Level: Analyze (Analysis)

REF:

p. 392 (Box 17-3)

32. The mother of a 16-month-old infant asks a nurse her opinion about holiday decorating. Which of the following recommendations should the nurse provide to the mother? a. Avoid hanging a wreath with holly and berries on the front door. b. Avoid hanging decorative pictures of a bearded Santa Claus on the window. c. Avoid placing poinsettia plants around the home. d. Avoid hanging stockings on the fireplace mantle. ANS: C The mother should avoid placing poinsettia plants around the home because, if swallowed, they could be fatal. By 16 months the infant can walk, and infants are more susceptible than adults to the effects of poisonous plants. Plants should be kept out of reach of infants. Berries on the wreath can also be fatal, but at 16 months, the child would not be able to reach the wreath. DIF: REF:

Cognitive Level: Analyze (Analysis) p. 390 (Box 17-2) | p. 414 (Table 17-8)

33. During a home visit to the home of a 3-month-old infant, which environmental finding requires immediate anticipatory guidance? a. A pool in the backyard without a gate b. The lack of smoke detectors c. The lack of childproof latches on drawers and cupboards d. Two azalea plants in the living room ANS: B The lack of a smoke detector places the infant at immediate risk if a fire were to occur. The majority of fire deaths occur in the home, and most people die from smoke inhalation, not burns. At 3 months of age, the infant is not yet able to crawl; thus, the other three findings, although risks, are not immediate. DIF: REF:

Cognitive Level: Analyze (Analysis) pp. 409-410 | p. 411 (Box 17-15)

34. A 2-month-old infant is in for a well-baby visit. Which of the following immunizations should the nurse administer to the infant? a. DTaP-1 b. MMR c. Varicella d. Influenza ANS: A


The Centers for Disease Control and Prevention (2007) recommends the following vaccines at 2 months of age: DTaP-1, Hib-1, PCV, Polio, RV. DIF:

Cognitive Level: Apply (Application)

REF:

p. 413 (Table 17-7)

MULTIPLE RESPONSE 1. A nurse is using the Denver Developmental Screening Test (DDST) II to assess a 10-month-old infant. Which of the following purposes does this screening serve? (select all that apply) a. Screens apparently healthy infants for developmental problems b. Allows the infant’s growth patterns to be compared with other infants c. Validates intuitive concerns about an infant’s development using an objective test d. Monitors high-risk children for developmental problems ANS: A, C, D The Denver Developmental Screening Test (DDST) is one of several standardized tools that screens for developmental problems in children from birth to 6 years of age. The purposes identified for administering the Denver II: screening apparently healthy infants for developmental problems; validating intuitive concerns about an infant’s development with an objective test, and monitoring high-risk children for developmental problems. DIF:

Cognitive Level: Apply (Application)

REF:

p. 393

2. A nurse is completing an initial genetic counseling interview with a couple. Which of the following data would be collected during the interview? (select all that apply) a. Paternal age b. Maternal age c. Family history d. Religious affiliation ANS: B, C An important aspect of genetic counseling is identifying families at increased risk and referring them as necessary. Aspects that would be reviewed in the initial interview include: maternal age, ethnic background, family history, reproductive history, and maternal disease. DIF:

Cognitive Level: Apply (Application)

REF:

p. 395

Chapter 18: Toddler Edelman: Health Promotion Throughout the Life Span, 8th Edition MULTIPLE CHOICE 1. At a well-child visit, a 2-year-old toddler measures 2 feet 8 inches tall. The nurse estimates the toddler’s approximate final adult height to be: a. 4 feet 8 inches. b. 5 feet. c. 5 feet 2 inches. d. 5 feet 4 inches. ANS: D The toddler’s height at 2 years is approximately 50% of final adult height. Thus, at 2 feet 8 inches, the child’s approximate final adult height would be 5 feet 4 inches tall (2 feet 8 inches  2). DIF:

Cognitive Level: Apply (Application)

REF:

p. 429


2. A -year-old toddler is in for an office visit. He was born at 6 pounds, 10 ounces. At today’s visit, the nurse expects his weight to be: a. 13 pounds, 4 ounces. b. 19 pounds, 14 ounces. c. 26 pounds, 8 ounces. d. 33 pounds, 2 ounces. ANS: C Birth weight usually quadruples by

years of age: 6 pounds, 10 ounces  4 = 26 pounds 8

ounces. DIF: REF:

Cognitive Level: Apply (Application) p. 429

3. At a well-child visit, the parents of a toddler ask the nurse how they should teach their toddler the importance of dental hygiene. Which of the following actions should the nurse recommend to the parents? a. Remind the toddler to brush his teeth every day. b. Have the dentist tell the toddler about the importance of dental hygiene. c. Schedule a time to brush their own teeth with their toddler. d. Have the toddler watch an educational cartoon video on the teeth brushing. ANS: C Toddlers identify with parents, caregivers, and other important role models, internalizing a wider range of lifestyle attributes. Parents’ and caregivers’ health perceptions and health behaviors should model the perceptions and behaviors desired for health promotion. Thus, scheduling a time to brush their teeth with their toddler is the best way for parents to teach their toddler the importance of dental hygiene. DIF:

Cognitive Level: Apply (Application)

REF:

p. 431

4. A mother is concerned because her 22-month-old has been skipping meals lately. Which of the following actions should the nurse recommend to the mother? a. Put the child in time out whenever he skips a meal. b. Offer him foods he likes such as cookies and chips. c. Limit the child’s consumption of juices and cookies. d. Talk to the child about how important it is to eat three meals a day. ANS: C Toddlers often use mealtime to assert their individuality. They sometimes do this by refusing to eat. Additionally they are often not hungry because they have been given empty calories by their parents, because the parents want to make sure the toddler eats something. However empty calories should be avoided. Parents should take control and offer the toddler healthy, age-appropriate foods. Punishment and focusing on food should be avoided. DIF:

Cognitive Level: Apply (Application)

REF:

p. 432 | p. 434 (Box 18-4)

5. Which of the following toddlers would require further investigation from the nurse? a. Sitting quietly on a couch b. Experiencing a temper tantrum c. Playing with a toy next to another child also playing with a toy d. Exploring his environment and playing with new toys ANS: A Toddlers are always busy. They spend most of their day playing and exploring their environment. Thus, a toddler sitting quietly is not an anticipated behavior and should be evaluated further. DIF:

Cognitive Level: Analyze (Analysis)

REF: p. 434


6. Approximately how much sleep do toddlers require each day? a. 10 hours b. 11 hours c. 12 hours d. 13 hours ANS: C Toddlers’ need for sleep decreases to 12 hours a day during this developmental period. DIF:

Cognitive Level: Remember (Knowledge)

REF:

p. 435

7. The inability of a toddler to put him- or herself in another’s shoes is known as: a. autonomy. b. egocentrism. c. self-perception. d. integrity. ANS: B The inability of a toddler to put him- or herself in another’s shoes is known as egocentrism. DIF:

Cognitive Level:

Remember (Knowledge)

REF:

p. 436

8. A nurse is assessing the cognitive-perceptual pattern of a toddler. Which of the following findings would be cause for concern for the nurse? a. Uses the word blanky to mean both that he wants to go to bed and that he is cold b. Refuses to eat something he ate and enjoyed the day before c. Has visual acuity that is not 20/20 d. Has a history of recurrent ear infections ANS: D Vision, hearing, speech, and taste/smell are all assessed as part of the cognitive-perceptual pattern. A toddler with recurrent ear infections is at risk for hearing loss, which can also lead to speech delay; thus this finding would be cause for concern. Visual acuity during the toddler years is approximately 20/40. DIF:

Cognitive Level: Apply (Application)

REF:

pp. 436-437

9. A deviation of the line of vision from the midline resulting from extraocular muscle weakness or imbalance is known as: a. amblyopia. b. farsightedness. c. nearsightedness. d. strabismus. ANS: D A deviation of the line of vision from the midline resulting from extraocular muscle weakness or imbalance is known as strabismus. DIF:

Cognitive Level: Remember (Knowledge)

REF: p. 438

10. A toddler has been diagnosed with amblyopia. Which of the following treatment measures would most likely be prescribed for the child? a. Using corrective lenses b. Patching of the eye c. Applying artificial tears to the eye d. Allowing the passage of time


ANS: B Management of amblyopia depends on the cause and includes surgery or paralytic and autonomic, centrally acting pharmacologic agents. In addition, management includes strengthening of the eye with reduced vision by applying atropine eye drops or patching the stronger eye. DIF:

Cognitive Level: Apply (Application)

REF:

p. 438

11. Which of the following behaviors exemplifies a toddler working through his developmental task? a. Clinging to his mother b. Refusing to take a bath c. Playing quietly with one toy d. Having no interest in a new toy ANS: B The development task of toddlers is to acquire a sense of autonomy while overcoming a sense of doubt and shame. Toddlers need to explore the world physically and interpersonally to develop a true sense of autonomy. The child explores relationships with others by searching for the limits of the child’s power. A child refusing to take a bath is trying to exert his autonomy. DIF:

Cognitive Level: Apply (Application)

REF:

pp. 438-439

12. A nurse is providing anticipatory guidance to parents about the self-perception–self-concept pattern of a toddler. Which of the following statements would the nurse most likely make? a. “Temper tantrums are expected, so always give in to them.” b. “Put your toddler in time out if he has a temper tantrum.” c. “Childproof your home.” d. “Always provide your toddler with challenging toys.” ANS: C The developmental task of toddler is to acquire a sense of autonomy. During this stage toddlers do a lot of exploring, and this includes climbing, crawling, and tasting. As a result, they are at increased risk for injury and should be provided with a childproof environment. DIF:

Cognitive Level: Apply (Application)

REF:

pp. 438-439

13. The mother of a 2-year-old toddler recently lost her job. In addition, her husband asked for a separation. Which of the following actions should the nurse take based on this information? a. Give the mother information about local support groups. b. Provide the mother the name of a prominent local attorney. c. Do not offer any advice unless the mother asks. d. Help the mother find a job. ANS: A Child abuse is more likely to occur with a major change or turmoil in the family. Toddlerhood is a trying time for even patient parents. Nurses need to be alert to family changes and stress. Additionally a major nursing role is to provide anticipatory guidance. Therefore the nurse should give the mother information about local support groups. DIF:

Cognitive Level: Apply (Application)

REF:

p. 440

14. A 2-year-old Asian American boy is being seen for an office visit. While examining the child, the nurse notices round bruises on his back. The mother does not volunteer any information during the exam. Which of the following actions should the nurse take after examining the child? a. Report the suspected child abuse to the Department of Children and Families.


b. c. d.

Assume the bruises are the result of a fall. Ask the mother about the bruises. Schedule a follow-up appointment in 2 weeks to see if the bruises are still present.

ANS: C Many injuries are difficult to differentiate from accidental injuries or culturally appropriate healing practices. Abusive parents are often hesitant to provide information, may be evasive, and may not exhibit feelings of guilt. However, these signs may also be present in nonabusive families. Therefore signs consistent with abusive parents should serve as a cue for further assessment. Additionally the nurse who is aware of cultural differences knows that coining, which can result in bruising, is a common cultural practice. Therefore in this case the nurse should ask the mother about the bruises. DIF:

Cognitive Level: Apply (Application)

REF:

p. 440 | p. 440 (Box 18-8)

15. An individual’s style of emotional and behavioral response across situations, especially those involving change or stress, is known as: a. ritual. b. temperament. c. coping. d. autonomy. ANS: B An individual’s style of emotional and behavioral response across situations, especially those involving change or stress, is known as temperament. DIF:

Cognitive Level: Remember (Knowledge)

REF:

p. 441

16. A parent expresses her frustration over the constant fighting between her toddler and preschooler. Which of the following would be an appropriate response from the nurse? a. Tell the mother not to worry about it and continue with the office visit. b. Tell the mother this is normal behavior for the toddler, and she should be firm with the child without taking sides. c. Say to the mother, “Don’t you remember fighting with your sibling? Don’t worry; everything will work out.” d. Say to the mother, “Don’t worry; this behavior will all end soon.” ANS: B Toddlers often imitate those older than them and this can create sibling rivalry. A major role of the nurse is to provide anticipatory guidance. The nurse should include normal family development as part of the anticipatory guidance because discipline can often be influenced by lack of knowledge and skill. Parental anger that goes unchecked can lead to abuse. DIF:

Cognitive Level: Apply (Application)

REF:

p. 441

17. A toddler needs to have blood drawn at his next appointment. Which of the following actions would the nurse suggest that the mother do with the child before the next visit? a. Tell the child about the blood work on the day that the appointment is scheduled. b. Tell the child about the blood work right before leaving the house for the appointment. c. Tell the child when they are in the office right before the phlebotomist enters the room. d. Do not tell the child about the blood work. ANS: B


Parents can help to prepare toddlers for stressful experiences before they happen. However, toddlers’ sense of time and ability to remember are limited. Enough time should be allowed for the toddler to rehearse the coping behavior with the parent but not so far off that the child will forget. Thus, the best time to tell the toddler is right before leaving for the appointment. DIF:

Cognitive Level: Apply (Application)

REF:

p. 441

18. What would be the best way for a parent to explain to a toddler that he needs to receive the influenza vaccine at today’s appointment? a. “You need the shot to keep you healthy. You will feel a little pinch when you get the shot.” b. “You need the shot to prevent the flu. They will use a small needle to administer the shot. It will not take long.” c. “You need a shot, but don’t worry, you won’t feel a thing.” d. “No, you will not get any needles today.” ANS: A Parents can help to prepare toddlers for stressful experiences before they happen. Preparation should be honest, simple, and focused on what the toddler will experience. Therefore, the best way to tell the toddler is to say, “You need the shot to keep you healthy. You will feel a little pinch when you get the shot.” DIF:

Cognitive Level: Analyze (Analysis)

REF:

p. 441

19. A nurse is assessing the home environment of a toddler. Which of the following poses the greatest and most immediate risk to the child? a. A fenced-in pool in the backyard b. An entertainment center with the remote control on the top shelf c. A closed window in the child’s bedroom d. A toy box overflowing with toys ANS: B Toddlers are at high risk for accidental injury because they lack judgment, and have limited physical coordination and a heightened level of curiosity. Toddlers love to explore and climb objects. Injuries to toddlers occur most often when they fall from furniture, stairs, or windows. Trying to reach the remote control on the top shelf of the entertainment center puts the child at immediate and greatest risk because of the potential for the entertainment center to fall on the child while he is trying to climb it. DIF:

Cognitive Level: Analyze (Analysis)

REF:

p. 442

20. A nurse is providing primary prevention education to reduce the risk of lead poisoning to a mother of a toddler. Which of the following recommendations would the nurse make? a. Encourage the toddler to drink orange juice and eat yogurt. b. Encourage the toddler to eat plenty of cakes and cookies to help absorb the lead. c. Limit foods high in iron such as spinach to help reduce lead levels. d. Encourage dark-chocolate–containing products that contain antioxidants to help reduce lead levels. ANS: A Primary prevention for lead poisoning involves teaching parents about risk factors and the importance of a diet that encourages decreasing fat intake, because lead is retained in fat. Vitamin C, calcium, and iron products should be encouraged. Thus, orange juice, which contains vitamin C, and yogurt, which contains calcium, should be encouraged. Chocolate products should not be encouraged. DIF:

Cognitive Level: Apply (Application)

REF:

p. 444

21. An 18-month-old toddler is being seen for a well-child visit. Which of the following strategies would the nurse use to enlist the toddler’s cooperation? a. Ask the toddler if he wants to be examined today.


b. c. d.

Ask his mother leave the room while the nurse examines him. Have him hold the stethoscope diaphragm on his chest while the nurse listens to his chest. Have his mother hold him down while the nurse examines him.

ANS: C Toddlers need to explore their environment to master it. This includes health care visits. They need to observe and listen to their parents interact with the health care provider, be allowed to manipulate the equipment, and be given simple instructions and choices so they can maintain some control. Therefore having the child hold the stethoscope on his chest while being examined will help make the visit more productive. DIF:

Cognitive Level: Apply (Application)

REF:

p. 446

22. A mother expresses her concern and frustration over the fact that her toddler has not been successful at toilet training yet. Which of the following would be the best response by the nurse? a. “Don’t worry; it will happen eventually.” b. “How long does the child stay dry during the day?” c. “Train him on a regular toilet instead of a potty chair.” d. “Here is a training schedule to follow.” ANS: B Toilet training is a major parental concern during toddlerhood. The nurse anticipates this and initiates discussion with the parent to determine their understanding of the child’s signs of development readiness. Before beginning toilet training, parents should check their toddler’s prerequisite skills. These include being able to walk, stop, and recover, stay dry for at least 2 hours during the day, and communicate sensation before elimination, as well as about the discomfort of wet or messy pants and the need for assistance. These prerequisites rarely develop before 18 months of age. Determining whether the child is ready will help to guide the discussion with the mother as well as help to determine the appropriate recommendations. This response also helps the mother understand and accept the reason for the potential delay in toilet training. DIF:

Cognitive Level: Analyze (Analysis)

REF:

p. 434 (Box 18-5)

23. A mother tells the nurse that ever since she started toileting training, the toddler has been touching his genitals. Which of the following statements is the most appropriate response by the nurse? a. “This is a sign that he is not ready to toilet train yet.” b. “This behavior is developmentally normal.” c. “Tell him to stop touching himself.” d. “Don’t worry about it.” ANS: B The toilet training process may precipitate curiosity about the genital organs. The nurse should provide anticipatory guidance to help the parents approach this curiosity and exploration, as well as masturbation as a normal developmental process. Telling the mother not to worry does not address her concern. Thus to address the mother’s concern and provide anticipatory guidance, the nurse should tell the mother that the behavior is developmentally normal. DIF:

Cognitive Level: Apply (Application)

REF:

p. 434 (Box 18-5)

24. Which is a common temperament pattern described by Chess and Thomas? a. The fussy child b. The happy child c. The unhappy child d. The slow-to-warm-up child ANS: D


Chess and Thomas originally described three common temperament patterns they believe are innate. The three patterns are the easy child, the difficult child, and the slow-to-warm-up child. DIF:

Cognitive Level: Remember (Knowledge)

REF:

p. 441 (Box 18-9)

25. How many words are in a toddler’s receptive language at 24 months of age? a. 500 words b. 1200 words c. 2400 words d. 3600 words ANS: B By 24 months of age, a toddler’s receptive language is up to 1200 words. DIF:

Cognitive Level: Remember (Knowledge)

REF:

p. 437 (Table 18-1)

26. A mother asks her toddler if she is tired. The toddler responds by saying, “Sammy bed.” Based on her expressive language and speech pattern, the nurses determines that this child is likely how old? a. 18 months b. 24 months c. 30 months d. 36 months ANS: B By 24 months of age, the toddler talks in two- to three-word sentences and averages two words per response. At this age, the toddler also refers to him- or herself by name. Adjectives are only beginning at this age. Thus, a response of “Sammy bed” is likely that of a 24-month-old. DIF:

Cognitive Level: Analyze (Analysis)

REF:

p. 437 (Table 18-1)

MULTIPLE RESPONSE 1. Which scenario places a toddler at risk for injury? (select all that apply) a. Toddler playing with his preschool sibling’s toy b. Mop and a bucket of clean water in the kitchen c. Cup of hot coffee on the kitchen counter d. Twenty-five pound toddler sitting in a forward-facing car seat in the back seat of a car ANS: A, B, C Toys are a common source of injury, especially toys intended for older children. Hot water, boiling water, coffee, tea, and food are the most common sources of injury and should not be kept within reach of the child. Additionally, toddlers can drown in water just deep enough to cover their noses and mouths, and this includes pails of water. Once children reach 20 lb, they should be switched to a forward-facing child safety seat, and preferably be placed in the rear seat because of the risk of injury from air bags in the front seat. DIF:

Cognitive Level: Apply (Application)

REF:

pp. 442-443

2. A nurse is assessing a toddler during a well-child visit. Which of the following findings warrant further investigation? (select all that apply) a. Bare spots on the scalp, and broken hair b. Overly concerned parents c. Burns with sharply demarcated edges d. Bruises anywhere on the body ANS: A, C


Bare spots and broken hair as well as burns with sharply demarcated edges are warning signs of child abuse. Abusive parents often do not demonstrate any guilt and are hesitant to provide information. Typically, abusive parents have difficulty leaving a child and attempt to identify with their feelings. In addition, bruises caused by abuse are usually located on the back side of the body, from the neck to the knees. DIF:

Cognitive Level: Apply (Application)

REF:

p. 440 (Box 18-7)

Chapter 18: Toddler Edelman: Health Promotion Throughout the Life Span, 8th Edition MULTIPLE CHOICE 1. At a well-child visit, a 2-year-old toddler measures 2 feet 8 inches tall. The nurse estimates the toddler’s approximate final adult height to be: a. 4 feet 8 inches. b. 5 feet. c. 5 feet 2 inches. d. 5 feet 4 inches. ANS: D The toddler’s height at 2 years is approximately 50% of final adult height. Thus, at 2 feet 8 inches, the child’s approximate final adult height would be 5 feet 4 inches tall (2 feet 8 inches  2). DIF:

Cognitive Level: Apply (Application)

REF:

p. 429

2. A -year-old toddler is in for an office visit. He was born at 6 pounds, 10 ounces. At today’s visit, the nurse expects his weight to be: a. 13 pounds, 4 ounces. b. 19 pounds, 14 ounces. c. 26 pounds, 8 ounces. d. 33 pounds, 2 ounces. ANS: C Birth weight usually quadruples by

years of age: 6 pounds, 10 ounces  4 = 26 pounds 8

ounces. DIF: REF:

Cognitive Level: Apply (Application) p. 429

3. At a well-child visit, the parents of a toddler ask the nurse how they should teach their toddler the importance of dental hygiene. Which of the following actions should the nurse recommend to the parents? a. Remind the toddler to brush his teeth every day. b. Have the dentist tell the toddler about the importance of dental hygiene. c. Schedule a time to brush their own teeth with their toddler. d. Have the toddler watch an educational cartoon video on the teeth brushing. ANS: C Toddlers identify with parents, caregivers, and other important role models, internalizing a wider range of lifestyle attributes. Parents’ and caregivers’ health perceptions and health behaviors should model the perceptions and behaviors desired for health promotion. Thus, scheduling a time to brush their teeth with their toddler is the best way for parents to teach their toddler the importance of dental hygiene. DIF:

Cognitive Level: Apply (Application)

REF:

p. 431


4. A mother is concerned because her 22-month-old has been skipping meals lately. Which of the following actions should the nurse recommend to the mother? a. Put the child in time out whenever he skips a meal. b. Offer him foods he likes such as cookies and chips. c. Limit the child’s consumption of juices and cookies. d. Talk to the child about how important it is to eat three meals a day. ANS: C Toddlers often use mealtime to assert their individuality. They sometimes do this by refusing to eat. Additionally they are often not hungry because they have been given empty calories by their parents, because the parents want to make sure the toddler eats something. However empty calories should be avoided. Parents should take control and offer the toddler healthy, age-appropriate foods. Punishment and focusing on food should be avoided. DIF:

Cognitive Level: Apply (Application)

REF:

p. 432 | p. 434 (Box 18-4)

5. Which of the following toddlers would require further investigation from the nurse? a. Sitting quietly on a couch b. Experiencing a temper tantrum c. Playing with a toy next to another child also playing with a toy d. Exploring his environment and playing with new toys ANS: A Toddlers are always busy. They spend most of their day playing and exploring their environment. Thus, a toddler sitting quietly is not an anticipated behavior and should be evaluated further. DIF:

Cognitive Level: Analyze (Analysis)

REF: p. 434

6. Approximately how much sleep do toddlers require each day? a. 10 hours b. 11 hours c. 12 hours d. 13 hours ANS: C Toddlers’ need for sleep decreases to 12 hours a day during this developmental period. DIF:

Cognitive Level: Remember (Knowledge)

REF:

p. 435

7. The inability of a toddler to put him- or herself in another’s shoes is known as: a. autonomy. b. egocentrism. c. self-perception. d. integrity. ANS: B The inability of a toddler to put him- or herself in another’s shoes is known as egocentrism. DIF:

Cognitive Level:

Remember (Knowledge)

REF:

p. 436

8. A nurse is assessing the cognitive-perceptual pattern of a toddler. Which of the following findings would be cause for concern for the nurse? a. Uses the word blanky to mean both that he wants to go to bed and that he is cold b. Refuses to eat something he ate and enjoyed the day before c. Has visual acuity that is not 20/20 d. Has a history of recurrent ear infections


ANS: D Vision, hearing, speech, and taste/smell are all assessed as part of the cognitive-perceptual pattern. A toddler with recurrent ear infections is at risk for hearing loss, which can also lead to speech delay; thus this finding would be cause for concern. Visual acuity during the toddler years is approximately 20/40. DIF:

Cognitive Level: Apply (Application)

REF:

pp. 436-437

9. A deviation of the line of vision from the midline resulting from extraocular muscle weakness or imbalance is known as: a. amblyopia. b. farsightedness. c. nearsightedness. d. strabismus. ANS: D A deviation of the line of vision from the midline resulting from extraocular muscle weakness or imbalance is known as strabismus. DIF:

Cognitive Level: Remember (Knowledge)

REF:

p. 438

10. A toddler has been diagnosed with amblyopia. Which of the following treatment measures would most likely be prescribed for the child? a. Using corrective lenses b. Patching of the eye c. Applying artificial tears to the eye d. Allowing the passage of time ANS: B Management of amblyopia depends on the cause and includes surgery or paralytic and autonomic, centrally acting pharmacologic agents. In addition, management includes strengthening of the eye with reduced vision by applying atropine eye drops or patching the stronger eye. DIF:

Cognitive Level: Apply (Application)

REF:

p. 438

11. Which of the following behaviors exemplifies a toddler working through his developmental task? a. Clinging to his mother b. Refusing to take a bath c. Playing quietly with one toy d. Having no interest in a new toy ANS: B The development task of toddlers is to acquire a sense of autonomy while overcoming a sense of doubt and shame. Toddlers need to explore the world physically and interpersonally to develop a true sense of autonomy. The child explores relationships with others by searching for the limits of the child’s power. A child refusing to take a bath is trying to exert his autonomy. DIF:

Cognitive Level: Apply (Application)

REF:

pp. 438-439

12. A nurse is providing anticipatory guidance to parents about the self-perception–self-concept pattern of a toddler. Which of the following statements would the nurse most likely make? a. “Temper tantrums are expected, so always give in to them.” b. “Put your toddler in time out if he has a temper tantrum.” c. “Childproof your home.” d. “Always provide your toddler with challenging toys.”


ANS: C The developmental task of toddler is to acquire a sense of autonomy. During this stage toddlers do a lot of exploring, and this includes climbing, crawling, and tasting. As a result, they are at increased risk for injury and should be provided with a childproof environment. DIF:

Cognitive Level: Apply (Application)

REF:

pp. 438-439

13. The mother of a 2-year-old toddler recently lost her job. In addition, her husband asked for a separation. Which of the following actions should the nurse take based on this information? a. Give the mother information about local support groups. b. Provide the mother the name of a prominent local attorney. c. Do not offer any advice unless the mother asks. d. Help the mother find a job. ANS: A Child abuse is more likely to occur with a major change or turmoil in the family. Toddlerhood is a trying time for even patient parents. Nurses need to be alert to family changes and stress. Additionally a major nursing role is to provide anticipatory guidance. Therefore the nurse should give the mother information about local support groups. DIF:

Cognitive Level: Apply (Application)

REF:

p. 440

14. A 2-year-old Asian American boy is being seen for an office visit. While examining the child, the nurse notices round bruises on his back. The mother does not volunteer any information during the exam. Which of the following actions should the nurse take after examining the child? a. Report the suspected child abuse to the Department of Children and Families. b. Assume the bruises are the result of a fall. c. Ask the mother about the bruises. d. Schedule a follow-up appointment in 2 weeks to see if the bruises are still present. ANS: C Many injuries are difficult to differentiate from accidental injuries or culturally appropriate healing practices. Abusive parents are often hesitant to provide information, may be evasive, and may not exhibit feelings of guilt. However, these signs may also be present in nonabusive families. Therefore signs consistent with abusive parents should serve as a cue for further assessment. Additionally the nurse who is aware of cultural differences knows that coining, which can result in bruising, is a common cultural practice. Therefore in this case the nurse should ask the mother about the bruises. DIF:

Cognitive Level: Apply (Application)

REF:

p. 440 | p. 440 (Box 18-8)

15. An individual’s style of emotional and behavioral response across situations, especially those involving change or stress, is known as: a. ritual. b. temperament. c. coping. d. autonomy. ANS: B An individual’s style of emotional and behavioral response across situations, especially those involving change or stress, is known as temperament. DIF:

Cognitive Level: Remember (Knowledge)

REF:

p. 441


16. A parent expresses her frustration over the constant fighting between her toddler and preschooler. Which of the following would be an appropriate response from the nurse? a. Tell the mother not to worry about it and continue with the office visit. b. Tell the mother this is normal behavior for the toddler, and she should be firm with the child without taking sides. c. Say to the mother, “Don’t you remember fighting with your sibling? Don’t worry; everything will work out.” d. Say to the mother, “Don’t worry; this behavior will all end soon.” ANS: B Toddlers often imitate those older than them and this can create sibling rivalry. A major role of the nurse is to provide anticipatory guidance. The nurse should include normal family development as part of the anticipatory guidance because discipline can often be influenced by lack of knowledge and skill. Parental anger that goes unchecked can lead to abuse. DIF:

Cognitive Level: Apply (Application)

REF:

p. 441

17. A toddler needs to have blood drawn at his next appointment. Which of the following actions would the nurse suggest that the mother do with the child before the next visit? a. Tell the child about the blood work on the day that the appointment is scheduled. b. Tell the child about the blood work right before leaving the house for the appointment. c. Tell the child when they are in the office right before the phlebotomist enters the room. d. Do not tell the child about the blood work. ANS: B Parents can help to prepare toddlers for stressful experiences before they happen. However, toddlers’ sense of time and ability to remember are limited. Enough time should be allowed for the toddler to rehearse the coping behavior with the parent but not so far off that the child will forget. Thus, the best time to tell the toddler is right before leaving for the appointment. DIF:

Cognitive Level: Apply (Application)

REF:

p. 441

18. What would be the best way for a parent to explain to a toddler that he needs to receive the influenza vaccine at today’s appointment? a. “You need the shot to keep you healthy. You will feel a little pinch when you get the shot.” b. “You need the shot to prevent the flu. They will use a small needle to administer the shot. It will not take long.” c. “You need a shot, but don’t worry, you won’t feel a thing.” d. “No, you will not get any needles today.” ANS: A Parents can help to prepare toddlers for stressful experiences before they happen. Preparation should be honest, simple, and focused on what the toddler will experience. Therefore, the best way to tell the toddler is to say, “You need the shot to keep you healthy. You will feel a little pinch when you get the shot.” DIF:

Cognitive Level: Analyze (Analysis)

REF:

p. 441

19. A nurse is assessing the home environment of a toddler. Which of the following poses the greatest and most immediate risk to the child? a. A fenced-in pool in the backyard b. An entertainment center with the remote control on the top shelf c. A closed window in the child’s bedroom d. A toy box overflowing with toys ANS: B


Toddlers are at high risk for accidental injury because they lack judgment, and have limited physical coordination and a heightened level of curiosity. Toddlers love to explore and climb objects. Injuries to toddlers occur most often when they fall from furniture, stairs, or windows. Trying to reach the remote control on the top shelf of the entertainment center puts the child at immediate and greatest risk because of the potential for the entertainment center to fall on the child while he is trying to climb it. DIF:

Cognitive Level: Analyze (Analysis)

REF:

p. 442

20. A nurse is providing primary prevention education to reduce the risk of lead poisoning to a mother of a toddler. Which of the following recommendations would the nurse make? a. Encourage the toddler to drink orange juice and eat yogurt. b. Encourage the toddler to eat plenty of cakes and cookies to help absorb the lead. c. Limit foods high in iron such as spinach to help reduce lead levels. d. Encourage dark-chocolate–containing products that contain antioxidants to help reduce lead levels. ANS: A Primary prevention for lead poisoning involves teaching parents about risk factors and the importance of a diet that encourages decreasing fat intake, because lead is retained in fat. Vitamin C, calcium, and iron products should be encouraged. Thus, orange juice, which contains vitamin C, and yogurt, which contains calcium, should be encouraged. Chocolate products should not be encouraged. DIF:

Cognitive Level: Apply (Application)

REF:

p. 444

21. An 18-month-old toddler is being seen for a well-child visit. Which of the following strategies would the nurse use to enlist the toddler’s cooperation? a. Ask the toddler if he wants to be examined today. b. Ask his mother leave the room while the nurse examines him. c. Have him hold the stethoscope diaphragm on his chest while the nurse listens to his chest. d. Have his mother hold him down while the nurse examines him. ANS: C Toddlers need to explore their environment to master it. This includes health care visits. They need to observe and listen to their parents interact with the health care provider, be allowed to manipulate the equipment, and be given simple instructions and choices so they can maintain some control. Therefore having the child hold the stethoscope on his chest while being examined will help make the visit more productive. DIF:

Cognitive Level: Apply (Application)

REF:

p. 446

22. A mother expresses her concern and frustration over the fact that her toddler has not been successful at toilet training yet. Which of the following would be the best response by the nurse? a. “Don’t worry; it will happen eventually.” b. “How long does the child stay dry during the day?” c. “Train him on a regular toilet instead of a potty chair.” d. “Here is a training schedule to follow.” ANS: B Toilet training is a major parental concern during toddlerhood. The nurse anticipates this and initiates discussion with the parent to determine their understanding of the child’s signs of development readiness. Before beginning toilet training, parents should check their toddler’s prerequisite skills. These include being able to walk, stop, and recover, stay dry for at least 2 hours during the day, and communicate sensation before elimination, as well as about the discomfort of wet or messy pants and the need for assistance. These prerequisites rarely develop before 18 months of age. Determining whether the child is ready will help to guide the discussion with the mother as well as help to determine the appropriate recommendations. This response also helps the mother understand and accept the reason for the potential delay in toilet training. DIF:

Cognitive Level: Analyze (Analysis)

REF:

p. 434 (Box 18-5)


23. A mother tells the nurse that ever since she started toileting training, the toddler has been touching his genitals. Which of the following statements is the most appropriate response by the nurse? a. “This is a sign that he is not ready to toilet train yet.” b. “This behavior is developmentally normal.” c. “Tell him to stop touching himself.” d. “Don’t worry about it.” ANS: B The toilet training process may precipitate curiosity about the genital organs. The nurse should provide anticipatory guidance to help the parents approach this curiosity and exploration, as well as masturbation as a normal developmental process. Telling the mother not to worry does not address her concern. Thus to address the mother’s concern and provide anticipatory guidance, the nurse should tell the mother that the behavior is developmentally normal. DIF:

Cognitive Level: Apply (Application)

REF:

p. 434 (Box 18-5)

24. Which is a common temperament pattern described by Chess and Thomas? a. The fussy child b. The happy child c. The unhappy child d. The slow-to-warm-up child ANS: D Chess and Thomas originally described three common temperament patterns they believe are innate. The three patterns are the easy child, the difficult child, and the slow-to-warm-up child. DIF:

Cognitive Level: Remember (Knowledge)

REF:

p. 441 (Box 18-9)

25. How many words are in a toddler’s receptive language at 24 months of age? a. 500 words b. 1200 words c. 2400 words d. 3600 words ANS: B By 24 months of age, a toddler’s receptive language is up to 1200 words. DIF:

Cognitive Level: Remember (Knowledge)

REF:

p. 437 (Table 18-1)

26. A mother asks her toddler if she is tired. The toddler responds by saying, “Sammy bed.” Based on her expressive language and speech pattern, the nurses determines that this child is likely how old? a. 18 months b. 24 months c. 30 months d. 36 months ANS: B By 24 months of age, the toddler talks in two- to three-word sentences and averages two words per response. At this age, the toddler also refers to him- or herself by name. Adjectives are only beginning at this age. Thus, a response of “Sammy bed” is likely that of a 24-month-old. DIF:

Cognitive Level: Analyze (Analysis)

MULTIPLE RESPONSE

REF:

p. 437 (Table 18-1)


1. Which scenario places a toddler at risk for injury? (select all that apply) a. Toddler playing with his preschool sibling’s toy b. Mop and a bucket of clean water in the kitchen c. Cup of hot coffee on the kitchen counter d. Twenty-five pound toddler sitting in a forward-facing car seat in the back seat of a car ANS: A, B, C Toys are a common source of injury, especially toys intended for older children. Hot water, boiling water, coffee, tea, and food are the most common sources of injury and should not be kept within reach of the child. Additionally, toddlers can drown in water just deep enough to cover their noses and mouths, and this includes pails of water. Once children reach 20 lb, they should be switched to a forward-facing child safety seat, and preferably be placed in the rear seat because of the risk of injury from air bags in the front seat. DIF:

Cognitive Level: Apply (Application)

REF:

pp. 442-443

2. A nurse is assessing a toddler during a well-child visit. Which of the following findings warrant further investigation? (select all that apply) a. Bare spots on the scalp, and broken hair b. Overly concerned parents c. Burns with sharply demarcated edges d. Bruises anywhere on the body ANS: A, C Bare spots and broken hair as well as burns with sharply demarcated edges are warning signs of child abuse. Abusive parents often do not demonstrate any guilt and are hesitant to provide information. Typically, abusive parents have difficulty leaving a child and attempt to identify with their feelings. In addition, bruises caused by abuse are usually located on the back side of the body, from the neck to the knees. DIF:

Cognitive Level: Apply (Application)

REF:

p. 440 (Box 18-7)

Chapter 19: Preschool Child Edelman: Health Promotion Throughout the Life Span, 8th Edition MULTIPLE CHOICE 1. A 5-year-old is prescribed a medication that is renally excreted. Which of the following considerations should be made when the nurse examines the dosage of the medication? a. The dosage should be lower than the recommended dose. b. The dosage should be at the recommended dose. c. The dosage should be higher than the recommended dose. d. The medication should not be administered. ANS: B The dose should be at the recommended dose. Kidneys reach full maturity by the end of infancy and early toddler years. As a result, medications that are renally excreted can be given at the recommended doses. DIF:

Cognitive Level: Analyze (Analysis)

REF:

pp. 450-451

2. A nurse is discussing genetic disorders with a local community group. Which of the following statements would the nurse make during the presentation? a. Downs syndrome is most likely to appear during the preschool years. b. Hemophilia is most likely to appear during the preschool years. c. Cystic fibrosis is most likely to appear during the preschool years.


d.

Multiple sclerosis is most likely to appear during the preschool years.

ANS: C The genetic problems most likely to appear during the preschool years are cystic fibrosis, Duchenne muscular dystrophy, fragile X syndrome, Williams’s syndrome, and autism. DIF:

Cognitive Level: Apply (Application)

REF:

p. 453

3. A mother comments to the school nurse that her child has poor hygiene habits and asks the nurse for suggestions as to how to teach her child to improve on his hygiene. Which of the following recommendations would the nurse give the mother? a. Have him learn it at school. b. Model the behavior at home. c. Lecture him about hygiene. d. Punish him when he forgets to brush his teeth. ANS: B The best way for a parent to teach a child good hygiene is to model the behavior at home. Reinforcement of health-promotion activities helps to instill behaviors. Additionally, preschoolers like to imitate adults. DIF:

Cognitive Level: Apply (Application)

REF:

p. 453

4. A 5-year-old is consuming a daily caloric intake of 1400 calories. Approximately what portion of the diet should be comprised of carbohydrates? a. 300 calories b. 700 calories c. 900 calories d. 1200 calories ANS: C Preschoolers should consume approximately half of their diet in carbohydrates. Children ages 3 to 5 years old should receive 1200 to 1600 calories per day depending on their activity level and gender. DIF:

Cognitive Level: Apply (Application)

REF:

p. 454

5. A nurse is determining caloric needs for a preschooler who weighs 18 kilograms. Which of the following represents the daily caloric needs for this child? a. 1360 calories b. 1540 calories c. 1620 calories d. 1800 calories ANS: C The approximate number of calories required by an 18-kg preschooler is 1620. Preschoolers need approximately 90 kcal/kg of body weight per day for health maintenance, activity, and growth. DIF:

Cognitive Level: Apply (Application)

REF:

6. Which of the following foods will a 5-year-old child need help eating? a. Pudding b. Macaroni and cheese c. Chicken d. Jelly sandwich

p. 454


ANS: C The preschooler will need help with the chicken. In the later preschool years, most children need help cutting meats. DIF:

Cognitive Level: Apply (Application)

REF:

pp. 454-455

7. A 2-year-old child has hives. Of the following items ingested, which is most likely to have caused the allergic reaction? a. Apple juice b. Oatmeal raisin cookies c. Jelly sandwich d. Banana ANS: B Oatmeal raisin cookies are most likely to have caused the allergic reaction. Nuts are a food that is likely to cause allergic reactions, and oatmeal raisin cookies often contain nut products. DIF:

Cognitive Level: Apply (Application)

REF:

p. 455

8. A 4-year-old boy had an episode of enuresis. Which of the following actions should be taken by the parents? a. Have him change his own pajamas before going to bed. b. Take away one video game. c. Tell his 6-year-old sister about the incident. d. Make him wear his diaper to bed. ANS: A The parents should have the child change his own pajamas before going to bed. Preschoolers should not be ridiculed or punished. They should be responsible for changing their own clothes and encouraged to avoid problems in the future. DIF:

Cognitive Level: Apply (Application)

REF:

p. 455

9. A nurse is working with a local daycare center to develop appropriate outdoor play activities for 5-year-old children at the center. Which of the following activities would the nurse most likely recommend? a. Having the children bring their bikes to the center so they can go for bike rides in the neighborhood b. Playing hopscotch on the sidewalk in front of the daycare center c. Taking the children swimming at the neighborhood pool d. Playing on the outdoor preschool equipment while being supervised by daycare personnel ANS: D Appropriate outdoor play for a 5-year-old child includes playing on age-appropriate playground equipment while being supervised by daycare personnel. At the age of 5, the child is still not proficient at controlling a bike in the streets or being safe with many children at a neighborhood pool. Additionally, children of this age have less fear of strangers. DIF:

Cognitive Level: Apply (Application)

REF:

pp. 455-456 | p. 468

10. A 4-year-old girl asks her mother for a second cup of yogurt before starting to eat the first cup she was given. What will the child most likely do with the extra helping? a. Eat it herself. b. Ask her brother if he wants it. c. Feed it to her dog. d. Give it to her imaginary friend. ANS: D


The extra helping is likely for the child’s imaginary friend. Preschoolers may have one or more imaginary companions. They are usually another child. Preschoolers eat, play, and sleep with their imaginary companions. DIF:

Cognitive Level: Analyze (Analysis)

REF:

p. 456

11. Which sleep ritual is age appropriate for a preschooler? a. Asking for every light in the home to be turned on at bedtime b. Saying good night to all 20 stuffed animals in the room before going to sleep c. Going right to sleep d. Asking the parent to read all books on the bookshelf before going to bed ANS: B An age-appropriate sleep ritual for a preschooler is to say good night to all 20 stuffed animals in the room before going to sleep. Preschoolers have longer, more rigid bedtime rituals than toddlers. They prolong bedtime routines, which for them generally last about 30 minutes. Reasonable rituals should be honored. DIF:

Cognitive Level: Apply (Application)

REF:

p. 457

12. During a well-child visit, a parent tells the nurse that their preschooler occasionally wakes up during the middle of the night. Based on growth and development, which of the following would be the most common reason for the preschooler to wake up? a. Nightmares b. Night terrors c. Enuresis d. Hunger ANS: A Nightmares are a common reason for nighttime awakenings. Nightmares fully waken the child and give them a feeling of fearfulness and helplessness. DIF:

Cognitive Level: Apply (Application)

REF:

p. 457

13. Which of the following is a hallmark of Piaget’s preconceptual substage? a. Using language to function symbolically b. Using concrete thought processes c. Using mental abstracting d. Using transductive reasoning ANS: A The hallmark of Piaget’s preconceptual substage is using language to function symbolically. DIF:

Cognitive Level: Remember (Knowledge)

REF:

p. 458

14. A preschooler and her school-age brother both received the same swimming tube as a gift. The brother accidentally took his sister’s swimming tube. The sister protested that she needed her swimming tube to go swimming. Which of the following is being demonstrated by this behavior by the sister? a. Abstract thinking b. Concrete thinking c. Jealousy d. A temper tantrum ANS: B The behavior is indicative of concrete thinking. Egocentrism exemplifies the concept of concrete thinking. Children can only concentrate on their own perspective. The preschooler is not able to consider more than one factor at a time when solving problems.


DIF:

Cognitive Level: Apply (Application)

REF:

p. 458

15. A school nurse is planning to assess the visual acuity of the preschool students at the school. Which of the following tests would be used? a. Snellen Screening test b. Denver Eye Screening test c. Ishihara’s test d. Cover test ANS: A The Snellen Screening test is a reliable estimate of actual visual acuity. The Snellen Screening test is performed with the Snellen E chart. DIF:

Cognitive Level: Apply (Application)

REF:

p. 459

16. A nurse is reviewing the chart of a preschool child who has been diagnosed with Asperger’s syndrome. Which of the following findings is the nurse most likely to discover? a. Eating finger foods at 10 months old b. Absent crawling at 10 months old c. Separation anxiety at 10 months old d. Saying “ma-ma” at 10 months old ANS: B Manifestations of autism spectrum disorders vary considerably. However, these children usually have had some childhood stressors such as chronic ear infections, sleeping/eating problems, absent/brief crawling phase, language delays, atypical responses to sensory stimulation, lack of separation anxiety, and failure to attain motor skill developmental milestones as warning signs that warrant further assessment. DIF:

Cognitive Level: Analyze (Analysis)

REF:

p. 463

17. A 4-year-old child tells the school nurse that he invented a new song. Which of the following would be the best response by the nurse? a. Congratulate the child on a job well done. b. Tell the child that he is too young to invent anything. c. Nicely tell the child that it does not sound like a song. d. Ask the child to come back later when there is more time to listen to the song. ANS: A The nurse’s best response is to congratulate the child on a job well done. When children perceive their value, they experience good feelings about themselves. They develop initiative through active imagination. According to Erikson, this is the most important developmental task in the development of self-concept. Reinforcement of behaviors and praising are methods that help develop self-concept. DIF:

Cognitive Level: Apply (Application)

REF:

p. 463

18. During the 5-year-old well-child visit, the father tells the nurse that he is unsure if the child is socially ready to attend school. The nurse asks the father to describe some of the social interactions of the child. Which of the following behaviors would indicate that the preschooler is socially ready to attend school? a. He gives his brother the ball when he is tired of playing with it. b. He never engages in any activity until his mother tells him to. c. He plays well with a group of neighborhood children. d. He moves from one activity to another. ANS: C


Playing well with a group of neighborhood children best indicates the preschooler is ready to attend school. Social interaction prepares preschoolers for school. The child acquires readiness to interact in group situations, follow directions, and take turns by interacting with others. DIF:

Cognitive Level: Apply (Application)

REF:

p. 464

19. A nurse is using primary prevention strategies to prevent child abuse in the community. Which of the following interventions would the nurse implement? a. Educate the public about violence and the potential for abuse. b. Remove a child who has been abused from the home. c. Report a case of child abuse. d. Refer a parent who abused their child to counseling. ANS: A Educating the public about violence and the potential for abuse is a form of primary prevention. Nurses provide primary prevention for violence by promoting awareness and providing anticipatory guidance. Primary prevention is used before violence actually occurs. DIF:

Cognitive Level: Analyze (Analysis)

REF:

p. 465

20. During a well-child visit, a father tells the nurse that the preschool-aged child has been enjoying their “road trips” that they take together when the child gets to sit in the front seat of the car with him. Which of the following actions should the nurse take next? a. Report the father to the Department of Human Services. b. Encourage the father to have the child sit in the back seat instead. c. Instruct the father on the dangers of having small children in the front seat. d. Provide the parent with educational materials about car seat safety. ANS: C Federal investigations concluded that children under age 13 should ride in the back seat of a motor vehicle, particularly because of potential injury or death from a passenger seat air bag that could inflate in a severe car accident. Although parents may know that infants should be placed in the back seat, they do not always know that children in the front seat of a motor vehicle face danger, even if they are using a restraint system. DIF:

Cognitive Level: Apply (Application)

REF:

p. 467

21. A nurse is discussing with parents how to prevent burns in a preschooler. Which of the following recommendations is the nurse most likely to give the parents? a. Do not read to the child while sitting on the sofa in front of the fireplace. b. Do not cook on a gas grill until the child is a teenager. c. Do not leave cigarettes and matches on the kitchen counter. d. Do not cook with the child in the kitchen. ANS: C Advising a parent to not leave cigarettes and matches on the kitchen counter is the most useful recommendation for preventing burns in a preschooler. Preschoolers like to imitate and act older than they are. Having cigarettes and matches on the kitchen counter makes them available for use by the preschooler and can lead to burns. Preschoolers should also be taught the dangers of matches and fire. DIF:

Cognitive Level: Analyze (Analysis)

REF: p. 469

22. A mother thinks her 2-year-old child drank Drano. What should the nurse tell the mother to do next? a. Call the pediatrician on call. b. Give ipecac syrup. c. Call the Poison Control Center. d. Wait to see if the child will vomit.


ANS: C The first thing the mother should do is call the Poison Control Center. Parents should have the number for the poison control center handy. Ipecac syrup should also be in the home; however, The Poison Control Center should be contacted before administration of ipecac syrup in order to determine if this would be appropriate following ingestion of this substance. DIF:

Cognitive Level: Analyze (Analysis)

REF:

pp. 470-471

23. Which of the following is the most common intraocular tumor in young children? a. Neuroblastoma b. Wilms tumor c. Retinoblastoma d. Leukemia ANS: C Retinoblastoma is the most common intraocular tumor in young children. DIF:

Cognitive Level: Remember (Knowledge)

REF:

p. 471

24. The nurse suspects a 5-year-old child has asthma. Which information from the child’s past medical history is a risk factor? a. The child was breast-fed. b. The child had respiratory infections as an infant. c. The child was born via a C-section. d. The child had pressure equalizer tubes as an infant. ANS: B Having respiratory infections (such as RSV) as an infant is a risk factor for asthma. Other potential causes of asthma include genetic predisposition, allergens such as animal dander or dust mites, and other nonspecific precipitants such as exercise, weather, or stress. DIF:

Cognitive Level: Apply (Application)

REF:

p. 472

25. The nurse makes a home visit to a child who has recently been diagnosed with asthma. Which environmental finding has the potential to trigger an asthma exacerbation? a. Radiator heating system b. Air conditioner c. Hardwood flooring d. Leaky roof ANS: D A leaky roof has the potential to trigger an asthma exacerbation because water or moisture can lead to mold growth, and mold is an allergen that can trigger asthma. DIF:

Cognitive Level: Apply (Application)

REF:

p. 472

26. The nurse is conducting a physical exam on a 3-year-old child. Which of the following would be the best way for the nurse to approach the child during the exam? a. Let the child listen to the nurse’s heart with the stethoscope. b. Tell him to sit quietly while the nurse examines him. c. Ask his parents to leave the room to minimize distractions. d. Ask another nurse to hold him while the nurse examines him.


ANS: A The best way for the nurse to approach a 3-year-old during a physical exam is to let him listen to the nurse’s heart with the stethoscope. Preschoolers like to imitate adults and like to play with medical devices. DIF:

Cognitive Level: Apply (Application)

REF:

p. 474

27. A nurse is using the Healthy People 2020 health promotion and disease prevention objectives for preschoolers when planning for health policy development in the community. Which of the following health policies would the nurse support? a. Elimination of the school requirement for the MMR vaccine b. Regulation of the use of electronic devices used by preschoolers c. Regulation of the paint allowed for use with toys d. Elimination of WIC programs ANS: C One way to address a Healthy People 2020 health promotion and disease prevention objective for preschoolers is to regulate the paint that is allowed for use in toys. Paints can contain lead, and lead poisoning can lead to disabilities. A Healthy People 2020 objective is to eliminate elevated blood-lead levels in children. DIF:

Cognitive Level: Analyze (Analysis)

REF:

p. 450 (Box 19-1)

28. The school nurse is observing the development of gross motor skills of young children at the school and watches a child who is learning how to throw a ball overhand. How old is this child? a. 3 years old b. 4 years old c. 5 years old d. 6 years old ANS: B At the age of 4, most children have mastered throwing a ball overhand. DIF:

Cognitive Level: Apply (Application)

REF:

p. 451 (Table 19-1)

29. How old is a preschooler with a receptive vocabulary of up to 5600 words? a. 42 months old b. 48 months old c. 54 months old d. 60 months old ANS: B By 48 months of age, most preschoolers have receptive languages skills of up to 5600 words. DIF:

Cognitive Level: Remember (Knowledge)

REF:

p. 461 (Table 19-2)

MULTIPLE RESPONSE 1. A mother tells the nurse her 3-year-old child has not been himself lately. The nurse examines the child. Which of the following physical findings would be cause for concern? (select all that apply) a. Bruises b. Pale skin c. Enlarged lymph nodes d. Hives ANS: A, B, C Bruises, pale skin, and enlarged lymph nodes are all warning signs that may indicate childhood cancer.


DIF:

Cognitive Level: Apply (Application)

REF:

p. 472 (Box 19-9)

2. A nurse administers the Draw-a-Person and Draw-a-Family tests with a preschool student. Which of the following purposes will these tests serve? (select all that apply) a. Assess verbal development. b. Assess intelligence development. c. Assess social development. d. Assess emotional development. ANS: B, D Draw-a-Person and Draw-a-Family tests approximate intelligence and emotional development; however, scoring figure drawings requires standardized test conditions and analysis of findings by a psychometric specialist. Nurses, however, use these methods in conjunction with other information to help establish rapport and provide a starting point to guide assessment. Drawings may indicate general developmental level, fine motor control, and evidence of concept formation. DIF:

Cognitive Level: Apply (Application)

REF:

p. 462

3. Which of the following behaviors by a preschooler indicates use of an effective coping mechanism? (select all that apply) a. The preschooler walks away from the situation. b. The preschooler blames a friend for causing the problem. c. The preschooler states what he is upset about. d. The preschooler has a temper tantrum. ANS: B, C Preschoolers use many of the coping mechanisms developed during their toddler years, but they generally show greater ability to verbalize frustration, fewer temper tantrums, and more patience in experimentation to resolve difficulty than the typical toddler. Preschoolers refine their problem-solving skills. Through fantasy play, they investigate solutions or responses to stressful events and find inner control for challenging situations. Preschoolers tend to project blame. DIF:

Cognitive Level: Apply (Application)

REF:

p. 466

Chapter 20: School-Age Child Edelman: Health Promotion Throughout the Life Span, 8th Edition MULTIPLE CHOICE 1. The school nurse conducting an in-service program for teachers is asked about general growth and development of school-age children. Which of the following statements would be the best response by the nurse? a. “By age 11, blood pressure readings are the same as for an adult.” b. “Because of the rapid growth pattern in this age group, snacks are important to ensure sufficient caloric intake.” c. “Children in this age group have some difficulty with coordination and balance, so playground supervision is important.” d. “Overall growth in school-age children is slower than during infancy and adolescence.” ANS: D Children grow (physically) much more slowly during this time period as compared with growth during infancy and adolescence. Blood pressure readings for children in this age group are lower than for adults. Growth slows until adolescence. Children in this age group generally have good coordination and balance. DIF:

Cognitive Level: Apply (Application)

REF:

p. 480


2. According to the American Heart Association, at what age should blood pressure screening begin? a. 3 b. 6 c. 9 d. 12 ANS: A Blood pressure screening should begin at 3 years of age and should be measured annually. DIF:

Cognitive Level: Remember (Knowledge)

REF:

p. 481

3. The school nurse has seen several students in the health office. For which of the following students should the nurse suggest a follow-up exam? a. A 14-year-old girl who has not experienced menarche b. A 7-year-old boy who has grown 2 inches in a year c. A 10-year-old girl who has gained 11 pounds in a year d. An 8-year-old boy who has lost four deciduous teeth in the past year ANS: C The average age of menarche is between 11 and 15 years of age. School-age children grow approximately 2 inches a year, gain between approximately 4.4 to 6.6 pounds a year, and lose four deciduous teeth a year between ages 6 and 13. A weight gain of 11 pounds in 1 year requires a follow-up because of the rising rate of obesity in children. DIF:

Cognitive Level: Apply (Application)

REF:

p. 481

4. The president of the Parent-Teacher Association asks the school nurse about the appropriateness of sponsoring a health fair that focuses on oral health for school-age children. Which of the following statements indicates the nurse’s correct understanding of this issue? a. “Dental problems are not as prevalent a problem as asthma, so you might want to consider a health fair about respiratory problems instead.” b. “Over 40% of children age 2 to 11 have dental caries, so a health fair that focuses on oral care is very appropriate.” c. “Children are not usually receptive to learning about dental care, so the topic is not the best choice for a health fair.” d. “This is not an appropriate focus for a health fair because the incidence of dental caries has dropped dramatically due to the use of sealants.” ANS: B Dental caries are present in up to 42% of children 2 to 11 years of age. Dental caries and other oral health issues remain a problem in the school-age population. Small gains have been made in number of children receiving sealants to prevent caries. Dental caries is the most common chronic disease of children 5 to 17 years of age, 4 times more prevalent than asthma. DIF:

Cognitive Level: Apply (Application)

REF:

pp. 481-482

5. Which of the following statements is correct? a. Tonsils in a school-age child are smaller than in an adult. b. The nervous system generally matures between ages 11 to 13. c. Fractures heal more slowly in a 10-year-old child than in an adult. d. Overweight children have muscle pain more often than average-weight children. ANS: D


Overweight children are more likely to suffer bone fractures, have joint pain, and more muscle pain than their normal weight counterparts. Tonsils, which are lymphoid tissue, are larger in a child than in an adult. The nervous system matures by ages 7 to 8. Fractures heal more quickly in children than in adults. DIF:

Cognitive Level: Understand (Comprehension) REF:

p. 482

6. Which of the following elements is most likely deficient in a school-age child’s diet? a. Protein b. Sodium c. Iron Vitamin B12 d. ANS: C The diet of school-age children is frequently deficient in calcium, iron, and vitamin C, while it is high in fat and sodium. DIF:

Cognitive Level: Understand (Comprehension) REF:

p. 484

7. Which of the following initiatives related to childhood nutrition outlined in Healthy People 2010 was most successful? a. Reduction of fat consumption b. Increase in consumption of fruit c. Adequate food supplies at home d. Reducing obesity rates ANS: C The target rate of 94% related to household adequate food supplies was surpassed with up to 98% of households found to be “food secure.” Children continue to eat too many foods high in fat, obesity rates are increasing, and consumption of fruit and vegetables is low. DIF:

Cognitive Level: Understand (Comprehension) REF:

p. 484

8. A school nurse is planning a series on nutrition. Which of the following factors should the nurse take into account? a. Television commercials frequently advertise food-related products. b. Children watch approximately two hours of television a day. c. Each day, approximately 60% of children eat fast food. d. A child is three times more likely to eat in a restaurant than at home. ANS: A Children watch over 3 hours of television a day. About one third of children eat fast food each day, and parents spend approximately 40% of their food budget for restaurant meals, including fast food restaurants. DIF:

Cognitive Level: Apply (Application)

REF:

p. 484

9. A nurse is planning activities to help children learn about good nutritional practices. Which of the following strategies should the nurse implement? a. Coordinate a “Try This” food day offering a variety of nutritious snacks. b. Find a book about nutrition to read the students. c. Plan a game using the concept of MyPlate. d. Search on-line for a video about nutrition to the students. ANS: A Children frequently prepare their own snacks and are more likely to learn from activities that foster active participation rather than passivity.


DIF:

Cognitive Level: Apply (Application)

REF:

p. 484

10. A child who has a BMI of 34 visits the school nurse. Which of the following factors does the nurse recognize that the child may be experiencing? a. Increased bullying by peers b. Increased self-esteem c. Increased resiliency d. Increased motivation to lose weight ANS: A Children may not be concerned about being overweight/obese and may therefore not be motivated to adopt healthier eating/exercise patterns. They may demonstrate impulsive behavior and have low self-esteem. Obese children are often ridiculed by peers. DIF:

Cognitive Level: Apply (Application)

REF:

p. 485

11. During a well-child visit, parents report to the nurse that they are concerned that their 8-year-old son has started wetting the bed at night. Which of the following statements should the nurse make to the parents? a. “I’ll be sure to mention this to the doctor because we rarely see a child who wets the bed at this age.” b. “There is not much you can do to stop this problem until he gets older.” c. “This is a common disease in children his age. Let’s talk about some options for helping him stay dry.” d. “I’m going to collect a urine sample from him and then we will talk about bed wetting.” ANS: D Enuresis may be caused by a UTI. It is not a disease and not uncommon in school-age children. Various therapies exist to help the child maintain nighttime bladder control. DIF:

Cognitive Level: Apply (Application)

REF:

p. 486

12. Which of the following children is at risk for developing encopresis? a. A 6-year-old child who takes Colace on a daily basis to soften stool b. A 3-year-old child who is not fully toilet-trained c. A 5-year-old child who has been in four foster care homes since age 3 d. A 4-year old child who sleeps soundly at night ANS: C Encopresis is diagnosed after age 4. It occurs most frequently during the day, if the child has chronic constipation, which would not occur if the child takes Colace on a daily basis, and if the child has experienced early life stress. DIF:

Cognitive Level: Apply (Application)

REF:

p. 487

13. A nurse is discussing sleep patterns in school-age children. Which of the following should be stressed by the nurse? a. Sharing a bed with siblings/parents has a long-lasting psychological impact. b. School-age children need between 6 and 9 hours of sleep a night. c. Daytime naps are important in children until they reach the age of 9 or 10. d. School-age children generally agree with and adhere to a regular bedtime. ANS: D Research has not demonstrated that sharing a bed with siblings/parents has a long-lasting psychological impact on the child. Children sleep between 8 and 12 hours per night and do not need daytime naps.


DIF:

Cognitive Level: Apply (Application)

REF:

p. 488

14. Parents of an 8-year-old boy tell the nurse that they are concerned about their son because he occasionally sleepwalks. Which of the following is the best response from the nurse? a. “Sleepwalking is much more common in girls than boys.” b. “I am glad you mentioned this. Sleepwalking is sometimes a sign of a neurological deficit.” c. “Does he also have nightmares? Do the two problems often occur together?” d. “This is not abnormal for a child of this age. He will most likely outgrow this tendency as his nervous system matures.” ANS: D Sleepwalking occurs more frequently in boys than girls, is most likely caused by an immature CNS, and is not associated with nightmares. DIF:

Cognitive Level: Apply (Application)

REF:

pp. 488-489

15. During a school gift fair, a parent asks the school nurse what gift would be most appropriate for her 6-year-old child. Which of the following should the nurse encourage the parent to purchase? a. A box of wooden blocks of various sizes for sorting b. A jar of sea shells in various shapes, colors, and textures c. A world map d. A beginning encyclopedia of animals ANS: B Classifying and ordering sea shells according to color, size, shape, and texture will help the child learn about classifying and numbering. These two concepts are necessary in order to learn to read and so on. A world map and encyclopedia are not age-appropriate, nor are wooden blocks. DIF:

Cognitive Level: Apply (Application)

REF:

p. 489

16. The school nurse has written a grant to obtain funding for a vision screening program and glasses for children in his pre-K to 6th grade elementary school. Which of the following statements should be included with the rationale for the need for funding? a. Many preschool children are hyperopic and need glasses to prevent headaches. b. Fewer than 10% of children entering school have had eye exams. c. Vision screening can diagnose vision/eye problems before they become severe. d. Approximately one in four children has a vision problem. ANS: D Preschool children who are farsighted do not need corrective glasses. Their vision will improve as they mature. Approximately 33% of children entering school have had eye exams. Vision screening does not diagnose vision problems. DIF:

Cognitive Level: Apply (Application)

REF:

p. 490

17. Which of the following statements concerning hearing in school-age children is correct? a. The rate of otitis media infections met Healthy People 2010 targets. b. Hearing and visual problems occur with the same frequency. c. All 50 states require routine hearing evaluations. d. The use of tympanograms has improved the accuracy of hearing screenings. ANS: A Hearing problems occur with less frequency than do vision problems. Many states, but not all, require routine hearing evaluations. Tympanograms measure how well the tympanic membrane vibrates.


DIF:

Cognitive Level: Understand (Comprehension) REF:

p. 490

18. A third grade teacher asks the school nurse what she can reasonably expect from children in her classroom related to language ability. Which of the following statements would be the best response from the nurse? a. “Children in the second grade should have about a 4000-word vocabulary.” b. “Research has shown that children will learn to read more quickly if you use the whole word approach.” c. “You can expect that the children will write ‘b’ and ‘d’ without reversing them, and all of the letters in a word will be about the same size.” d. “You can evaluate the cognitive ability of the children in your classroom by noting how legible their writing is.” ANS: C The child will not have a 4000-word vocabulary until about the age of 12. Both the whole word approach and the phonics approach will help children learn to read. The legibility of handwriting has no correlation to intelligence. DIF:

Cognitive Level: Apply (Application)

REF:

pp. 490-491

19. Each of the following children visits the school nurse on a regular basis. Which of these children would the nurse identify as being at risk for educational difficulty and suggest follow-up care? a. 9-year-old whose parents recently divorced b. 10-year-old who cannot sit still longer than 15 minutes c. 12-year-old with a 4000-word vocabulary d. 11-year-old who scored 102 on an IQ test ANS: B A 10-year-old who displays inattentive behavior should have follow-up care. Hyperactivity may indicate ADHD, which would require follow-up care and potential educational difficulty. An environmental deficit such as a divorce may cause school difficulty for a child, but this does not indicate a learning disability. The average vocabulary for a 12-year-old is 4000 words. A score of 102 on an IQ test falls within the average range of 90 to 110. DIF:

Cognitive Level: Apply (Application)

REF:

p. 492 | p. 492 (Box 20-4)

20. A parent asks the nurse working in a pediatric clinic how she can help her 6-year-old child feel good about himself. Which of the following statements should be made by the nurse? a. “Doing most things for your child will give them a sense of security, and they will have a positive self-image.” b. “Having your child help around the house with tasks such as setting the table will increase selfesteem.” c. “Make sure to highly praise even the smallest effort your child makes at completing a task.” d. “There is little you can do to increase your child’s self-esteem. Their peers have much more of an influence than parents at this age.” ANS: B When children succeed at age-specific tasks, they increase their sense of self-worth. Doing everything for a child prevents him or her from becoming proficient at a task. Too much praise makes positive feedback lose its value and gives the child a false sense of mastery. Although peers are important, at age 6, parents influence their children more than do peers. DIF:

Cognitive Level: Apply (Application)

REF:

p. 493

21. Which of the following statements made by a parent is most helpful when discussing rules of the household? a. “You did not feed the dog like you promised, so you cannot have ice cream for dessert.” b. “I was upset when you did not feed the dog like you promised. We all need to help take care of him


c. d.

because he depends on us. Let’s talk about what happened.” “Why did you forget to feed the dog? Would you like me to forget to feed you?” “What you did was bad. We all need to care for the dog.”

ANS: B When correcting a child, explaining how one feels, the consequence of the action and then listening to the child discuss the situation is more beneficial than punishment and labeling the child as “bad.” DIF:

Cognitive Level: Analyze (Analysis)

REF:

p. 494

22. A nurse has recently accepted a position as a school nurse at a middle school and plans to offer sex education to the students and parents. Which of the following should be in the initial action taken by the nurse? a. Offer gender-segregated sex education classes. b. Develop information to teach about abstinence and condom use. c. Determine how this education has been provided in the past. d. Survey parents to determine their desire for this education to be provided. ANS: C The nurse employed in the school is in an ideal position to teach group sex education programs using literature and games. Children at this age appear to respond most favorably with gender-segregated classes, based on their general discomfort with sexual topics and unique needs and questions. Some schools appropriately incorporate these classes into school curricula as part of a health-promotion curriculum. Other schools have special programs focused only on sex education based on parental desires or school board policies. Therefore, the first action by the nurse should be to determine what has been done in the past. DIF:

Cognitive Level: Apply (Application)

REF:

p. 495

23. The school nurse has been asked to conduct a community awareness program about accidents that impact the school-age child. Which of the following facts should be included in the program? a. Poisonings and falls are the leading causes of death. b. Drowning and motor vehicle accidents are the leading causes of fatality. c. The majority of accidents happen in the home. d. Burn accidents are more common in the summer months. ANS: B Most fatal accidents during the school-age period derive from motor vehicle accidents when the child (host) is a passenger or pedestrian (walking or riding a bike). Other fatal accidents occur from fires and burns, bicycles, drowning, and firearm accidents. Accidents are the leading cause of death in children over 12 months of age. The majority of accidents occur outdoors. Burn accidents are more common in the winter. DIF:

Cognitive Level: Apply (Application)

REF:

p. 498

24. A school nurse is assessing the child’s social environment. Which of the following strategies would the nurse implement? a. Interview family members. b. Conduct a windshield assessment. c. Review the child’s past medical history. d. Evaluate the child’s activity and rest patterns. ANS: A The child’s social environment includes the family, school, and playmates. Thus, the best way to collect information about this is to interview the child’s family members. DIF:

Cognitive Level: Apply (Application)

REF:

pp. 498-499


25. A school nurse is caring for a child with an upper respiratory tract infection. Which of the following nursing interventions should the nurse implement? a. Send the child with a mild sore throat back to class. b. Advise the parent of a child with a group A strep throat infection that they cannot return to school for 5 days. c. Educate the parent that treatment of group A strep infections usually requires extra fluid intake. d. Ask the parent of a child with a mild fever and mild sore throat to take the child to his or her health care provider. ANS: D Untreated strep throat caused by group A bacteria can result in serious complications. Signs and symptoms may include a mild sore throat. After 24 hours on antibiotics, the child can return to school. Thus, the parents of a child with a mild fever and mild sore through should be evaluated by his or her health care provider. DIF:

Cognitive Level: Apply (Application)

REF:

p. 501

26. Which of the following statements about smoking is true? a. Black children begin smoking at an earlier age than do White children. b. Almost 80% of schools prohibit tobacco use in all locations. c. Each day over 6000 children try smoking. d. Tobacco use prevention programs should be initiated in eighth grade. ANS: C Every day, more than 6000 school students try to smoke a cigarette. Most students who smoke initiate their habit around 11 years of age. White children begin smoking at an earlier age than do Black children. Approximately 64% of schools are smoke free. Tobacco use prevention programs should begin in elementary school. DIF:

Cognitive Level: Understand (Comprehension) REF:

pp. 502-503

27. A child’s parents ask the school nurse if there is any help available to help their child receive health insurance. They state they earn too much money to qualify for Medicaid but not enough to purchase health insurance. Which of the following recommendations should the nurse provide? a. “Currently, there is no health care program available for your child.” b. “The school district has a health insurance policy that you can access.” c. “The state has an insurance program to assist you.” d. “You will need to find coverage through another member of your family.” ANS: C The State Children’s Health Insurance Program was created to assist parents in this economic category. This program supports comprehensive care to children 0 to 18 years who do not meet Medicaid criteria but live in families too poor to afford private insurance. DIF:

Cognitive Level: Apply (Application)

REF:

p. 506

28. During orientation, the new school nurse asks his preceptor about the role of the school nurse. Which of the following statements is the best response by the nurse? a. “School nurses need to focus their attention on children who are economically disadvantaged, because other children have all the resources necessary to assist them maintain a healthy lifestyle.” b. “There is little school nurses can do to improve the health of children. That really is the job of parents and caregivers. Our role is to conduct routine screening and deal with emergencies.” c. “We are primarily focused on physical and psychological health, rather than environmental concerns.” d. “We are an important part of the team that helps to maintain and improve children’s health in a variety of ways from screenings to advocacy.”


ANS: D School nurses care for children in all economic groups and recognize the unique challenges faced by each population. Nurses are a primary force related to improving children’s health and well-being taking into account all facets of a child’s world from the environment to health deviations. DIF:

Cognitive Level: Apply (Application)

REF:

p. 507

29. During a parent-nurse conference, the parent tells the nurse that her child sleeps approximately 7 hours per night. Which of the following potential health problems would be of concern to the nurse? a. Hypertension b. Hyperactivity c. Obesity d. Type 2 diabetes ANS: C Based on the study by Lumeng et al, children who do not get adequate sleep tend to be less active, which promotes overweight/obesity. Type 2 diabetes and HTN may result from obesity. DIF:

Cognitive Level: Analyze (Analysis)

REF:

p. 486 (Box 20-2)

MULTIPLE RESPONSE 1. Which of the following children should the school nurse monitor closely for hypertension (HTN)? (select all that apply) a. 7-year-old White male with no known family history of HTN b. 9-year-old White female whose maternal grandfather has HTN c. 6-year-old Black child with no family history of HTN d. 10-year-old Mexican American child with known family history of HTN ANS: B, C, D A White child with no family history of HTN can receive routine monitoring. The other children are in the highrisk category for HTN because of race and family history. DIF:

Cognitive Level: Apply (Application)

REF:

p. 481

2. A nurse is planning a flu prevention fair for 9- to 11-year-old children. Which of the following activities should be included? (select all that apply) a. Microscopes set up to show bacteria on common objects b. Games such as “find the germs” in a cartoon picture c. Research poster showing germ transmission d. Hand-washing booth with colored soap ANS: A, B, D Age-appropriate interactive activities such as games and demonstrations are effective teaching strategies. Formal research poster presentations are not appropriate for this age group. DIF:

Cognitive Level: Apply (Application)

REF:

p. 483

3. The school nurse has been asked to plan play and recess activities for children in grades 2 through 6. Which of the following activities should be developed by the nurse? (select all that apply) a. Highly competitive games that focus on individual accomplishment and winning b. Family activity days that promote nature walks and biking c. Co-ed organized sports d. Library outings


ANS: B, D Emphasis should move away from highly competitive activities that promote more of a feeling of superiority than team work to reach a goal. Children in this age group generally prefer to play with children of the same gender. DIF:

Cognitive Level: Apply (Application)

REF:

pp. 487-488

4. Which of the following children may have difficulty with Erikson’s “industry versus inferiority” stage of development? (select all that apply) a. Poor motor skills and is rarely chosen for team play during recess b. Difficulty with math homework c. Difficulty making friends d. Cannot speak his or her native language fluently ANS: A, C, D The “industry versus inferiority” stage of development as described by Erikson is mastered when the child has a sense of accomplishment, rather than an inability to perform at the expected level. Children with poor motor skills who are not selected by their peers for a team, those who find it difficult to make new friends, and those who have difficulty assimilating into their dominant culture all are likely to have a sense of inferiority. A child who has difficulty with math homework may not be noticed by his/her peer group. DIF:

Cognitive Level: Apply (Application)

REF:

p. 493

5. The school nurse is evaluating children for signs and symptoms of depression. Which of the following children may be most at risk for depression? (select all that apply) a. 11-year-old girl who has been involved in fighting during the past month b. 10-year-old boy who no longer wants to play on the school’s soccer team c. 12-year-old boy who has started lifting weights d. 9-year-old girl who visits the nurse on a daily basis for vague complaints ANS: A, B, D Symptoms of depression in school-age children include anorexia, sleeplessness, lethargy, changed affect, aggressive behavior, frequent crying, or withdrawal from previously enjoyed activities. DIF:

Cognitive Level: Analyze (Analysis)

REF:

p. 497

6. Which of the following is responsible for ensuring that children are up-to-date on their immunizations? (select all that apply) a. School nurse b. Parent or caregiver c. Teacher d. School principal ANS: A, B The parent or caregiver bears the major responsibility for ensuring that their child has had all of the required immunizations; however, it is also the school nurse’s responsibility to ensure that the immunizations are current. DIF:

Cognitive Level: Understand (Comprehension) REF:

p. 502

7. A nurse is conducting a school-wide in-service program about child abuse and is asked to describe common indicators of abuse. Which of the following signs would the nurse include? (select all that apply) a. Caregivers and parents who are reluctant to take the child for treatment for an injury b. Statement by the day care provider that the child was injured by falling off a chair c. Explanation of the injury from the child and caregiver or parent that are similar d. Lack of emotional response from child to a playground injury


ANS: A, B, D Signs of abuse may include physical evidence, conflicting stories about the “accident” or injury from parents or others, an injury or complaint inconsistent with the child’s history or developmental level, inappropriate response of the caregiver or child, the child’s report of abuse, previous reports of abuse in the family, or repeated visits to emergency facilities with injuries. DIF:

Cognitive Level: Apply (Application)

REF:

p. 495 (Box 20-5)

Chapter 21: Adolescent Edelman: Health Promotion Throughout the Life Span, 8th Edition MULTIPLE CHOICE 1. When do females usually begin puberty? a. 1 year before males b. 1 year after males c. 2 years before males d. 2 years after males ANS: C Females usually begin puberty 2 years before males and experience their growth spurts earlier. DIF:

Cognitive Level: Remember (Knowledge)

REF:

p. 515

2. A teenager has been using acne medications for the last 14 days. Her acne is no better, and in fact, it is a little worse. What information should the nurse offer this girl? a. “Wash your face at least four times a day, making sure to scrub well.” b. “The medications can make acne appear worse at first; try to give it a few more weeks.” c. “Avoid all chocolate products.” d. “Because it is summertime, it would be good for your skin if you lie out in the sun for a few hours each day.” ANS: B Acne is common in adolescents. Evidence indicates that dietary restrictions for acne are unnecessary. Although washing with soap and water is the best way to remove dirt, vigorous scrubbing should be discouraged. Furthermore, although sunlight can have a beneficial effect on acne, prolonged exposure should be avoided. Topical acne products make acne appear worse initially, and improvement occurs slowly over several months. Nurses should provide this information to teenagers planning to use acne products, so they are not discouraged early in therapy. DIF:

Cognitive Level: Apply (Application)

REF:

p. 517

3. The mother of a 13-year-old girl is concerned because her daughter has not started menstruating yet. The girl developed breast buds and started her height spurt at 12 years of age. Which of the following statements would be the most appropriate response from the nurse? a. “Usually, girls start menstruating when they develop breast buds; an endocrine evaluation might be warranted.” b. “Everyone is different; it will happen when it happens.” c. “Based on when the breast buds first appeared, she should starting menstruating around the age of 14.” d. “She will not start menstruating until her breasts are of adult size and contour.” ANS: C


The first sign of puberty in females is the appearance of breast buds, followed by a growth spurt. The onset of menstruation occurs approximately 2 years after the appearance of breast buds and near the end of the growth spurt. Thus, based on the age of breast bud appearance, the girl should start menstruating around age 14. DIF:

Cognitive Level: Apply (Application)

REF:

p. 518

4. Which female disorder is characterized by only one X chromosome present instead of two? a. Gynecomastia b. Menstruation c. Turner syndrome d. Klinefelter syndrome ANS: C A female disorder in which only one X chromosome is present instead of two is known as Turner syndrome. DIF:

Cognitive Level: Remember (Knowledge)

REF:

p. 518

5. Which of the following disorders is characterized by males having an extra chromosome, typically being tall and initially thin, and not developing secondary sex characteristics? a. Gynecomastia b. Menstruation c. Turner syndrome d. Klinefelter syndrome ANS: D A disorder in which males have an extra chromosome and typically are tall and initially thin, and do not develop secondary sex characteristics is known as Klinefelter syndrome. DIF:

Cognitive Level: Remember (Knowledge)

REF:

pp. 518-519

6. Parents recently discovered that their teenage daughter has been consuming alcohol on a regular basis. They ask the nurse for advice on how to best help their daughter. Which of the following recommendations should the nurse give to the parents? a. Talk to their daughter at length regarding the dangers associated with drinking. b. Tell their daughter she will not be allowed to use her cell phone if this continues. c. Enroll their daughter in a peer support group focused on teenage drinking. d. Provide their daughter with literature regarding the dangers of drinking. ANS: C Teens do not always consider health risks associated with their risky behaviors. Additionally, peer influence is primary and parental input is often rejected. In this case, the teenager may risk losing her cell phone and privileges by continuing with her risky behavior, but because peer influence is primary, the parents should enroll her in a peer support group focused on teenage drinking. DIF:

Cognitive Level: Apply (Application)

REF:

7. Which of the following behaviors is most reflective of a teenager? a. Riding an ATV without a helmet b. Not skiing for fear of falling and getting hurt c. Obeying a new parental rule without a challenge d. Walking away from a group of friends who are smoking cigarettes ANS: A

p. 519


Teenagers are known for their risk-taking behaviors. They do not always consider the health risks associated with their behaviors. In addition, they often do things because their friends do it. Therefore, riding an ATV without a helmet and risking a head injury is most reflective of a teenager’s behavior. DIF:

Cognitive Level: Analyze (Analysis)

REF:

p. 519

8. What percentage of adolescent females suffer from an eating disorder in the United States? a. 5% b. 10% c. 15% d. 20% ANS: B Ten percent of adolescent females suffer from an eating disorder in the United States. DIF:

Cognitive Level: Remember (Knowledge)

REF:

pp. 519-520

9. During a physical exam, a teenager asks the nurse if she can have a stronger medication for her constipation because the laxatives she has been taking are not helping. During the physical exam, the nurse observes mild to moderate erosion of the tooth enamel. Which of the following disorders is the client most likely experiencing? a. Bulimia nervosa b. Binge eating disorder c. Idiopathic constipation d. Irritable bowel syndrome ANS: A Bulimia nervosa is characterized by binge eating and then purging by self-induced vomiting and/or laxatives. Purging can also lead to erosion of the tooth enamel. Bulimia nervosa occurs more commonly in females. A teenage girl asking for stronger laxatives with tooth enamel erosion should cause any health care provider to consider the possibility of bulimia nervosa. DIF:

Cognitive Level: Apply (Application)

REF:

p. 520

10. A teenager has a family history of Type 2 diabetes mellitus. He asks the nurse what he can do to try to prevent developing it himself. Which of the following instructions should the nurse give this client? a. Tell him to avoid any and all sweets. b. Tell him to be active and eat a balanced diet. c. Tell him he should talk to his physician about genetic testing. d. Tell him that very little can be done to avoid developing this disorder. ANS: B Obesity is making Type 2 diabetes mellitus common among teens and young adults. Being overweight and inactive increases the risk. Therefore, encouraging activity and a balanced diet is the best information the nurse can give to the client. Telling him to avoid all sweets would be an impossible request, and genetic testing does not detect the development of this disorder. DIF:

Cognitive Level: Apply (Application)

REF:

11. Which of the following teenage behaviors is cause for concern? a. Has difficulty waking up in the morning and naps during the day b. Goes to bed late and naps during the day c. Takes frequent naps and states it is because he is bored d. Participates in after school activities and has difficulty waking up ANS: C

p. 521


Adolescents need at least 8 hours of sleep per night. Often, they stay up late and wake up for school before their sleep cycles have finished; therefore, napping is common. However, those with a decreased energy level who use words such as “bored” or “sad” should be further evaluated for suspected depression. Therefore, a teenager who takes frequent naps and states it is because he is bored is cause for concern because this behavior might indicate depression. DIF:

Cognitive Level: Analyze (Analysis)

REF:

p. 522 | p. 525

12. A mother is concerned because her daughter has been taking a nap every day after school. Which of the following statements would be the best response by the nurse? a. “Don’t worry; all teenagers do that.” b. “Tell me about your daughter’s schedule.” c. “Let’s bring her in for some blood work.” d. “Have you addressed the topic of drugs with you daughter?’ ANS: B Many teenagers take naps, although this does not address the mother’s concern. Although the nap can be a symptom of a health problem or drug use, additional information is necessary to make that determination. Asking the mother to tell you about her daughter’s schedule provides information to determine whether the behavior is normal or whether it requires further investigation. DIF:

Cognitive Level: Analyze (Analysis)

REF:

p. 522

13. Which of the following teenagers is at risk for not meeting his developmental task? a. Arrested for petty theft b. Decided he wants to major in accounting c. Has recently broke up with his girlfriend d. Is a member of the varsity basketball team ANS: A The central task of adolescence according to Erikson is the establishment of identity, with the primary risk being role confusion. As the adolescent searches for direction, he makes choices in social, recreational, volunteer, academic, familial, and occupational activities. When the threat of confusion is exceedingly great, delinquent behavior and alterations in mental health can occur. Thus, a teenager arrested for petty theft is displaying delinquent behavior that may place him at risk for not meeting his developmental task of identity. DIF:

Cognitive Level: Analyze (Analysis)

REF:

p. 522

14. Which of the following actions should parents take to help their teenager meet his developmental task? a. Set strict rules to prevent the teenager from getting into any trouble. b. Maintain an ongoing dialogue about important issues and upcoming challenges. c. Maintain a friendship with the teenager, so he views parents as peers. d. Be very lenient with the teenager so that he can figure out who he is as a person. ANS: B Imposing strict limits and eliminating all rules are not helpful to teenagers. Families in which parents maintain a willingness to listen, demonstrate an ongoing affection for and acceptance of their adolescent, yet still maintain some consistent limits, experience more constructive, positive outcomes during adolescence. Thus, maintaining an ongoing dialogue about important issues and upcoming challenges demonstrates the parents’ willingness to listen and their concern. DIF:

Cognitive Level: Apply (Application)

REF:

pp. 523-524

15. Which of the following teenagers is at increased risk for acquiring a sexually transmitted disease and should be provided with anticipatory guidance? a. Asks the nurse for information about condoms


b. c. d.

Feels her friends and family do not care about her Has multiple friends and feels good about herself Talks to her boyfriend on the phone for hours

ANS: B Adolescents often talk on the phone for hours, providing a safe way for them to interact with members of the opposite sex. However, adolescence is a time when teenagers fantasize about relationships and sex. Teenagers decide to become sexually active for a number of reasons, including affection. This can put them at increased risk for sexually transmitted diseases; therefore, anticipatory guidance regarding protection from sexually transmitted diseases needs to be provided before the adolescent is in a situation that can place him or her at risk. DIF:

Cognitive Level: Analyze (Analysis)

REF:

p. 524

16. A mother is concerned because her 13-year-old daughter comes home happy one day and sad the next. The girl also argues with her younger sister more than she did in the past, and does not like participating in family activities anymore. Which of the following is the best initial response from the nurse? a. “The changes during adolescence can be stressful as they struggle to come to terms with who they are. What other types of behaviors has she been displaying?” b. “Changes in affect can be a warning sign for suicide. Has she had any increase in risk-taking behaviors or physical violence?” c. “Is there any history of psychological disorders in your family?” d. “Withdrawal can be a sign of substance abuse. Has she given you any indication she might be using illicit drugs?” ANS: A Although dramatic swings in affect can be a warning sign for suicide, and withdrawal can be a sign of substance abuse, additional information is required before suggesting a diagnosis. Often, mood swings and spending more time with peers and less time with the family is normal adolescent behavior during this period of rapid change in physical, psychosocial, spiritual, moral, and cognitive growth. Parents and teenagers should be aware of and prepared for these changes. DIF:

Cognitive Level: Analyze (Analysis)

REF:

pp. 525-526

17. Which of the following is the third leading cause of death in adolescents between 10 and 24 years of age? a. Accidents b. Suicide c. Cancer d. Sport injuries ANS: B The third leading cause of death in adolescents between 10 and 24 years of age is suicide. DIF:

Cognitive

Level: Remember (Knowledge)

REF:

p. 526

18. After offering a teenager the HPV vaccine, the teenager responds by saying, “I can’t have the vaccine because I have already been sexually active.” Which of the following statements is the best response by the nurse? a. “You can still benefit from the vaccine, but there is a chance it will not be as effective.” b. “The best way to protect yourself now is to make sure all your future sexual partners have received the vaccine.” c. “You are correct, the vaccine is only indicated for females who have never been sexually active.” d. “It depends on the number of partners you have had. If you have had less than five partners, you are still eligible for the vaccine.” ANS: A


The HPV vaccine is recommended for 11- to 12-year-old girls and for 13- to 26-year olds who have not yet received or completed the vaccine series. Ideally, females should receive the vaccine before they are sexually active because girls who have not been infected with any of the four HPV types covered by the vaccine will receive the full benefits of the vaccine. Females who are sexually active may also benefit but to a lesser degree. It is not known if the vaccine is effective in boys or men. Thus, the best response is, “You can still benefit from the vaccine, but there is a chance it will not be as effective.” DIF:

Cognitive Level: Apply (Application)

REF:

p. 530

19. A nurse is initiating a discussion about alcohol consumption during a physical exam with a teenager. Which of the following questions would be most appropriate for the nurse to ask? a. “So, what do you and your friends do for fun and excitement?” b. “Do you have access to alcohol?” c. “Do any of your friends drink?” d. “Do you want to talk about alcohol or drinking?” ANS: C Anticipatory guidance during a physical exam is important. This includes asking about substance use. Questions should not be vague because they may be misinterpreted. Direct questions are useful when discussing sensitive topics. However, asking first about friends and the adolescent’s feelings about them is a good lead-in approach to a discussion about substance use. DIF:

Cognitive Level: Analyze (Analysis)

REF:

p. 532

20. The school nurse is working on a campaign in the school to prevent adolescent pregnancies. Which of the following interventions should the nurse implement? a. Talk to teenagers about sex, including abstinence, contraception, and sexually transmitted diseases. b. Tell the teenagers they should abstain from sex until they are old enough to cope with the potential consequences such as an unwanted pregnancy. c. Encourage parents of all teenage girls to make sure their daughters start oral contraceptives early. d. Offer all teenagers free condoms when they come in for physical exams. ANS: A The emergence of secondary sexual characteristics increases adolescents’ awareness of themselves as sexual human beings leading to experimentation. Primary prevention is necessary to help prevent adolescent pregnancies. Parents, teachers, and health care providers will be more successful in assisting teens to manage their health needs if they treat them as joint partners. Talking to teenagers about sex, contraceptive options, and sexually transmitted diseases gives them the information and anticipatory guidance they need to help them make a decision. DIF:

Cognitive Level: Analyze (Analysis)

REF:

p. 524 | p. 520 (Box 21-3)

21. A teenager asks the school nurse for advice before getting a tattoo. Which of the following responses would be most appropriate for the nurse? a. “You should not get a tattoo.” b. “It is okay to get a tattoo if you go to a licensed artist.” c. “Tattoos are popular; however, they can lead to infection and/or bleeding.” d. “A body piercing would be safer and less permanent.” ANS: C Tattoos and piercings carry similar risks, which include localized infection, bleeding, and dermatitis. In addition, tattoos carry a heightened concern for blood-borne disease such as hepatitis and HIV. When providing anticipatory guidance, nurses should treat teenagers as partners and give them the information they need to make an informed decision. Therefore, informing the teenager about the risk of infection is the best response. DIF:

Cognitive Level: Apply (Application)

REF:

p. 523 (Box 21-5)


22. A teenager is expecting her menstrual period on May 14. On which of the following dates would the nurse recommend that she perform a self-breast exam? a. May 11 b. May 14 c. May 17 d. May 23 ANS: D The best day to perform a self-breast exam is 2 to 3 days after one’s period when breasts are least likely to be tender or swollen. A typical period lasts about 6 days. Thus, a teenager who starts her period on May 14 should perform a breast self-exam on May 23 (14 + 6 = 20 + 3 = 23). DIF:

Cognitive Level: Apply (Application)

REF:

p. 525 (Box 21-6)

23. Which of the following teenagers is exhibiting a behavior that cause for concern and requires further evaluation? a. Is always very happy b. Is agitated and has trouble sleeping c. Has a lot of homework and stays up late to finish it d. Values his friend’s opinion more than his parent’s opinion ANS: B Nurses should be aware that the problem of substance abuse exists. They should also be aware of what the signs and symptoms are. Agitation and insomnia are signs of substance abuse and should therefore be evaluated further. DIF:

Cognitive Level: Analyze (Analysis)

REF:

p. 529 (Box 21-10)

24. Which stage of male genital development is characterized by initial enlargement of the penis, mainly in length along with further growth of the testes and scrotum? a. 1 b. 2 c. 3 d. 4 ANS: C In stage 3, male genital development is characterized by initial enlargement of the penis, mainly in length along with further growth of the testes and scrotum. DIF:

Cognitive Level: Remember (Knowledge)

REF:

p. 516 (Table 21-1)

25. Which stage of female breast development is characterized by enlargement of the areolar diameter along with a small area of elevation around the papillae? a. 1 b. 2 c. 3 d. 4 ANS: B In stage 2, female breast development is characterized by enlargement of the areolar diameter along with a small area of elevation around the papillae. DIF:

Cognitive Level: Remember (Knowledge)

REF:

p. 516 (Table 21-1)

26. A 14-year-old female is in for a physical exam. Which finding requires further investigation?


a. b. c. d.

A height spurt since her last visit Facial acne The lack of breast buds Noticeable sweating and body odor with activity

ANS: C The onset of puberty usually begins around 11 to 13 years of age. Stage 2 is the usual time of peak height velocity for girls. Additionally, both sweat and sebaceous glands become more active during adolescence leading to sweat, body odor, and acne. Breast buds also usually appear during stage 2 of puberty. Females who have not begun pubertal development by age 14 should have an endocrine evaluation. Thus, the finding of lack of breast buds on this 14-year-old girl requires further investigation. DIF: REF:

Cognitive Level: Analyze (Analysis) p. 516 (Table 21-1) | p. 518 (Figure 21-3)

MULTIPLE RESPONSE 1. Which of the following are symptoms or warning signs of anorexia nervosa? (select all that apply) a. Lack of menstruation in females b. Preoccupation with food c. Eating only small amounts of certain foods d. Dramatic weight fluctuations ANS: A, B, C, D Symptoms or warning signs of anorexia nervosa include the relentless pursuit of thinness, self-starving with significant weight loss, lack of menstruation in females, decreased sexual interests in males, compulsive physical activity, preoccupation with food, portioning food carefully, eating only small amounts of only certain foods, and a distorted body image. Dramatic weight fluctuations are seen with bulimia nervosa. DIF:

Cognitive Level: Understand (Comprehension) REF:

p. 520

2. A nurse is discussing being a safe driver as part of a driver’s education course. Which of the following recommendations would the nurse include in this presentation? (select all that apply) a. Carpooling with friends b. Avoiding talking on cell phones c. Driving during daytime hours d. Wearing safety belt ANS: B, C, D The risk of motor vehicle crashes is four times higher among 16- to 19-year-old adolescents than among any other age group. Research suggests that distractions, such as talking or texting on cell phones, eating, or playing with the radio, increase teen drivers’ risk of being involved in a crash. Nurses should talk to teens about these distractions, which can also include riding with a car full of other teens. Teens have a much higher nighttime crash fatality rate as well. DIF:

Cognitive Level: Apply (Application)

REF:

p. 527

Chapter 22: Young Adult Edelman: Health Promotion Throughout the Life Span, 8th Edition MULTIPLE CHOICE 1. A nurse is planning a community health education program for young adults. Which of the following considerations should be made by the nurse?


a. b. c. d.

The age span encompassing young adulthood is between 20 to 30 years of age. The number one cause of death for young adults is injury. The number of young adults in the United States is increasing. The maternal mortality rate is at its lowest point since 1980.

ANS: B Young adulthood spans the years between ages 18 to 35. The number of young adults in the United States is declining, and the maternal mortality rate is at its highest level since 1980. DIF:

Cognitive Level: Apply (Application)

REF:

p. 536

2. A nurse is completing a health counseling session with a 25-year-old woman. Which of the following statements should be made by the nurse during this session? a. “A yearly Pap test is recommended for detection of ovarian cancer.” b. “Research supports that breast self-exams reduce breast cancer mortality.” c. “The incidence of cervical cancer is very low in your age group.” d. “Daily physical activity and weight control is one way to stay healthy.” ANS: D Screening is strongly recommended for cervical cancer in women who have been sexually active (Papanicolaou [Pap] smears). The incidence of carcinoma in situ is high in young adults. The US Preventative Services Task Force recommends against teaching breast self-examination and concludes that the current evidence is insufficient to assess the additional benefits and harm of clinical breast examination beyond screening mammography for women 40 and older. After age 25, the preventive emphasis is on modifying coronary disease risk factors, which would be accomplished by discussing activity and weight control. DIF:

Cognitive Level: Apply (Application)

REF:

pp. 538-539

3. A 26-year-old client has a total cholesterol of 206 mg/dL and an LDL of 110. Which of the following conclusions can be drawn by the nurse? a. The client has achieved the target levels for cholesterol in young adults. b. The client has a very low risk of developing heart disease. c. The client requires counseling about cardiac disease risk factors. d. The client has early-onset cardiac disease. ANS: C Cardiovascular assessment of the young adult includes determining the presences of hyperlipemia, hypertension, diabetes, chest pain, or heart disease. This client has an elevated cholesterol levels and requires additional education about risk factors that may lead to cardiac disease. The Healthy People 2020 target is to reduce the mean total blood cholesterol levels among adults to 177.9 mg/dL. A diagnosis of heart disease cannot be made on the basis of these lab values. DIF:

Cognitive Level: Analyze (Analysis)

REF:

pp. 538-539

4. The public health nurse is conducting a screening of young adults for metabolic syndrome. When asked about the syndrome, which of the following responses should be made by the nurse? a. “Anyone who has low blood sugar, high cholesterol, and high blood pressure has this syndrome and is at risk for cardiac disease.” b. “This syndrome helps predict heart disease. Once it is diagnosed, the correct medication can be prescribed and heart disease avoided.” c. “This syndrome is a warning sign that the person could develop heart disease. When someone has metabolic syndrome, dietary and activity level changes are recommended.” d. “Anyone who has high cholesterol, high blood pressure, and high blood sugar has coronary artery disease. Once we make the diagnosis, we can begin to treat the disease.”


ANS: C Metabolic syndrome is diagnosed when a person has high cholesterol, high blood pressure, and high blood sugar. A person with this syndrome is at risk for coronary artery disease. Lifestyle changes are tried before medication is used. DIF:

Cognitive Level: Analyze (Analysis)

REF:

p. 541

5. The university health services nurse is preparing a disease prevention program for college students. Which of the following information should the nurse include in the program? a. Living in the dorm increases one’s risk of developing meningococcal disease. b. The mortality rate from meningococcal disease is very low. c. There are no antiviral medications that can treat meningococcal outbreaks. d. A vaccine for meningococcal disease has not been developed. ANS: A Although most outbreaks of meningococcal disease are sporadic, young adults living in dormitories or crowded conditions may be more susceptible than young adults not living in close settings. Meningococcal disease has a high mortality rate. It is caused by a bacterium, not a virus, and antibiotics to treat the disease are readily available. A preventive vaccine is available. DIF:

Cognitive Level: Apply (Application)

REF:

p. 543

6. A 30-year-old woman with a BMI of 36 is counseled by the nurse regarding interventions to assist with weight reduction. In which order would the nurse discuss the implementation of these interventions? 1. Gastric stapling 2. Medication to reduce appetite 3. Nutrition and exercise education 4. Assessment of lipid profile and blood pressure a. 4, 3, 2, 1 b. 4, 2, 3, 1 c. 3, 4, 2, 1 d. 3, 2, 1, 4 ANS: A Assessment data must be complete before intervention. Diet and exercise are used first, and then pharmaceutical agents to reduce appetite. Bariatric surgery is used when all other options have failed. DIF:

Cognitive Level: Analyze (Analysis)

REF:

p. 544

7. A nurse is discussing optimal activity for young adults. Which of the following information should be stressed? a. Forty-five minutes of moderate exercise per day are required to maximize health benefits. b. An increase in heart rate from 70 to 200 beats/min during exercise is optimal. c. Moderately brisk walking is an appropriate aerobic exercise. d. Sixty minutes of exercise three times a week is recommended. ANS: C The Healthy People 2020 goal is to increase the proportion of adults who engage in moderate physical activity for 150 minutes a week (30 minutes five times per week) to 47.9% from 43.5%. Aerobic conditioning achieves cardiovascular fitness through five periods of moderately intense exercise weekly for about 30 minutes or more at a heart rate of approximately 220 minus the age of the person multiplied by 65% to 85%. Young adults are encouraged to engage in fitness activities that increase the heart rate to approximately 150 beats or more per minute. Moderately brisk walking that increases the heart rate is an appropriate aerobic exercise. DIF:

Cognitive Level: Apply (Application)

REF:

p. 545


8. The nurse working in the college health center is asked about a safe way to obtain a tan. Which of the following would be the best response by the nurse? a. “There is no safe way. Be sure to stay out of the sun as much as possible.” b. “Tanning beds are much safer than exposure to actual sunlight.” c. “Be sure to use sunblock agents and do not sunbathe between 10 AM and 2 PM.” d. “Once you apply sunblock, you are protected all day, so be sure to use it before any outdoor activity.” ANS: C Young adults should avoid sunbathing during the 2-hour period before and after noon because two-thirds of the day’s ultraviolet light comes through the earth’s atmosphere during this time. There is no need to stay completely out of the sun. Tanning beds are not a safe way to tan. Sunblock should be reapplied if certain outdoor activities are part of the day, such as swimming. DIF:

Cognitive Level: Apply (Application)

REF:

p. 545

9. A nurse is working in the community to prevent bicycle and motorcycle fatalities. Which of the following measures would be most effective for the nurse to promote? a. Use of helmets b. Increasing the minimum age for motorcycle licensure c. Construction of bike paths along busy roads d. Right-of-way laws for bicyclists ANS: A Head injury is the major cause of death related to bicycle or motorcycle accidents. Bicycle helmets are believed to be the single most effective preventive measure available to decrease the incidence of brain and head injury. DIF:

Cognitive Level: Apply (Application)

REF:

p. 545

10. Which of the following reflects Piaget’s theory as applied to young adults? a. Voting for a candidate based on popularity b. Voting for a candidate based on media advertisements c. Voting for a candidate based on support of the candidate by peers d. Voting for a candidate based on analysis of views about various issues ANS: D Young adults are in Piaget’s formal operational thinking stage during which they analyze issues objectively and insightfully. DIF:

Cognitive Level: Analyze (Analysis)

REF:

p. 546

11. Which of the following adults has transitioned to Erikson’s intimacy versus isolation and loneliness stage of development? a. A 21-year-old man who has a part-time job, spends most of his leisure time with his buddies, and has numerous short-term intimate relationships b. A 25-year-old woman who is very concerned with how she is perceived by her coworkers and friends c. A 30-year-old man who just graduated with a PhD and is looking for his first full-time job d. A 26-year-old woman who has a long-term relationship with a female companion ANS: D Erikson’s intimacy versus isolation and loneliness stage of development involves reciprocal expressions of affection and trust. Promiscuous behavior is not characteristic of this stage of development. Reliance on a positive perception by friends and coworkers for self-esteem is not characteristic of this stage of development.


DIF:

Cognitive Level: Analyze (Analysis)

REF:

p. 546

12. Which of the following young adults exemplifies Kohlberg’s postconventional level of moral reasoning? a. 28-year-old person who calls in sick so as to help a friend move to a new location b. 22-year-old man who uses marijuana on the weekends with friends c. 30-year-old woman who volunteers at a soup kitchen one day a month d. 35-year-old who works long hours so as to be considered for a promotion ANS: C Someone in Kohlberg’s postconventional level of moral reasoning is able to do what is beneficial or “right” regardless of peer pressure to take another course of action. DIF:

Cognitive Level: Analyze (Analysis)

REF:

p. 546

13. Which of the following statements should be considered by a nurse working in occupational health who organizes a stress reduction workshop? a. Women generally experience less workplace stress than do men. b. One’s first job is exciting and leads to numerous opportunities, and therefore causes little stress. c. Young adults adapt easily to changing job demands such as telecommuting and long work hours. d. Work is of primary importance to young adults and helps form their self-image. ANS: D Employment is more than a source of income to young adults; it provides self-esteem and social interaction. Women experience as much, and sometimes more, stress than do men because of child care issues. The demands of work are stressful and sometimes lead to unhealthy behavior. DIF:

Cognitive Level: Apply (Application)

REF:

p. 547

14. The parents of a young adult female have recently divorced. Which of the following is the young adult most likely to experience? a. Relief b. Depression c. Unemployment d. Inability to sustain own relationships ANS: B Although dissatisfaction and unhappiness are frequent precursors to separation and divorce, the decision to dissolve a marriage is not easy. Divorced young adults frequently suffer severe emotional strain and depression. Some young adults are unable to adjust to role and status changes and to threats of self-concept. Young adults whose parents divorce have an increased rate of divorce, but that does not mean that they are unable to sustain their own relationships. DIF:

Cognitive Level: Apply (Application)

REF:

p. 548

15. A 19-year-old college student is seen by the health services nurse. The young woman is distraught that she had unprotected sex 4 days ago and is afraid she may have become pregnant. Which of the following statements should be made by the nurse? a. “Let’s talk about your options because you must be 21 years of age to purchase emergency contraception.” b. “Don’t worry; you can purchase Plan B, an emergency contraception drug, without a prescription and it will terminate the pregnancy.” c. “I know you are upset, but we are in one of the 40 states that allow drugstores to carry Plan B, an emergency contraception drug. Let’s talk about how to use the medication.” d. “You have several options. Let’s discuss what would be the best thing for you to do next.”


ANS: D Emergency contraception can reduce the number of unintended pregnancies. There is a generic two-dose form Next Choice, and a one-dose form, Plan B One-Step. Both require a prescription for girls under 17. Emergency contraception must be used within three days of unprotected sex. It works by either altering tubal transport of either sperm or ova, inhibiting implantation. It will not terminate an existing pregnancy and does not provide protection against sexually transmitted diseases. DIF:

Cognitive Level: Apply (Application)

REF:

pp. 550-551

16. The nurse working in the college health center is planning a program about sexually transmitted diseases (STDs). Which of the following information should the nurse include in the program? a. STDs are the third most common infection in young adults. b. STDs can be transmitted by any intimate contact, not just sexual intercourse. c. STDs usually occur as a single infection. d. STDs are the most common reason for a visit to the health center. ANS: B STDs can be transmitted from oral and anal sex, not just vaginal intercourse, and many young adults do not understand this. STDs are the most common infection in persons age 15 to 24. A person may have multiple STDs. STDs are generally unreported and not treated. DIF:

Cognitive Level: Apply (Application)

REF:

pp. 551-552

17. A 32-year-old woman visits the occupational health nurse because of malaise. Which of the following assessment data indicates that the client may be experiencing achievement-oriented stress? a. Sleeping 10 hours per day b. Unintentional loss of 10 pounds in the past 3 weeks c. Expressed fear of company layoffs d. Calling in sick once every 2 weeks for the past month ANS: B Achievement-oriented stress is characterized by lack of sleep, skipping meals, and workaholic habits. It differs from situational stress, which would be caused by changes in job function for example. DIF:

Cognitive Level: Apply (Application)

REF:

p. 553

18. A nurse volunteered to work at a natural disaster site. Which of the following questions should the nurse ask to determine if the 26-year-old individual being screened is depressed or has suicide ideation? a. “Tell me about your pets.” b. “What do you do for work? How has your work been affected by this disaster?” c. “Have you thought of harming yourself?” d. “Have you seen your friends since the disaster?” ANS: C Direct questioning about the intent to harm oneself is appropriate for the nurse to ask during screening/counseling sessions. DIF:

Cognitive Level: Apply (Application)

REF:

pp. 553-554

19. A nurse is planning health care for young adults. Which of the following factors about the interventions should be recognized by the nurse as having the greatest impact on whether they are effective? a. Culturally sensitive b. Match the person’s health care beliefs c. Cost effective d. Gender neutral


ANS: A Unless the interventions are culturally sensitive, they are not likely to be successful. Although the other factors are important, the priority consideration is cultural sensitivity. DIF:

Cognitive Level: Analyze (Analysis)

REF:

pp. 554-555

20. Which of the following circumstances accounts for one of the leading cause of death in young adults? a. Tobacco use b. Binge drinking and driving c. Illegal drug use d. HIV infection ANS: B Alcohol-related accidents among individuals ages 15 to 24 continue to be a leading cause of preventable morbidity, disability, and death. DIF:

Cognitive Level: Understand (Comprehension) REF:

p. 556

21. A nurse is counseling a 23-year-old woman about oral, transdermal, injectable, vaginal ring, and implant contraceptive methods. Which of the following risks of using these methods should be stressed by the nurse? a. Toxic shock syndrome b. Nausea c. Blood clots d. Ectopic pregnancy ANS: C Blood clots are a potential risk with using all of these contraceptive methods. Ectopic pregnancy is a possible complication of IUDs. Nausea may occur with the use of Plan B, and toxic shock syndrome may occur with use of a diaphragm. DIF:

Cognitive Level: Apply (Application)

REF:

p. 551 (Table 22-2)

22. Which of the following interventions are appropriate relative to the nursing diagnosis: Health-Seeking Behaviors Related to Preconceptual Assessment and Preparation for Childbearing? a. Encouraging the consumption of a moderate-fat diet to prepare for pregnancy b. Referral to genetic counseling based on family history c. Counseling to avoid “hard” liquor d. Substituting a nutritional supplement for a meal so as to assure adequate vitamin intake ANS: B Assessment of biophysical risk factor is an important intervention, which includes review of genetic disorders, nutrition problems, and current medical problems. Before and during pregnancy, the diet should be wellbalanced, low in fat and sodium, and high in calcium and iron. Nutritional supplements are not required unless the woman is malnourished. Alcohol in all forms including beer and wine should be avoided. DIF:

Cognitive Level: Apply (Application)

REF:

p. 559 (Care Plan)

MULTIPLE RESPONSE 1. An occupational health nurse is conducting a blood pressure screening. Which of the following clients is at greatest risk of mortality from cardiac disease? (select all that apply) a. Black man with a blood pressure of 130/80 b. Mexican American man with a blood pressure of 110/60 c. White woman with a bold pressure of 120/80 d. Black woman with a blood pressure of 128/78


ANS: A, D According to the JNC VII, the risk for cardiovascular events rises when blood pressure is greater than 115/75. Blacks have a higher death rate from heart disease than do Whites. Mexican Americans have the lowest rate of hypertension. DIF:

Cognitive Level: Analyze (Analysis)

REF:

pp. 538-539

2. A public health nurse is interested in writing a grant to improve vaccination rates of young adults. His focus should be on trying to obtain funding to support which of the following vaccination initiatives? (select all that apply) a. Rubella prevention b. Lyme disease prevention c. Hepatitis B prevention d. Human papilloma virus (HPV) prevention ANS: C, D Rubella in young adults is generally a minor disease; the population at risk is women of childbearing age. There is not a vaccine available for Lyme disease. Hepatitis B and HPV can both be prevented through immunization and young adults are at high risk for contracting these diseases. DIF:

Cognitive Level: Apply (Application)

REF:

p. 543

3. A nurse is conducting a community health education program for Hispanic women ages 18 to 35. Which of the following items should the nurse recommend that they increase in their diet on a daily basis? (select all that apply) a. Calcium b. Folic acid c. Iron d. Sodium ANS: A, B, C Women of childbearing age frequently consume a diet deficient in calcium, iron, and folic acid. DIF:

Cognitive Level: Apply (Application)

REF:

p. 544

4. A community health nurse has received a grant to present a program about violence prevention. Which of the following individuals has a higher than average risk of homicide? (select all that apply) a. 33-year-old woman who served 5 years in prison b. 20-year-old homeless man c. 28-year-old man with a history of bipolar disorder d. 24-year-old woman who has a female partner ANS: A, B, C Homicide is closely associated with alcohol and drug abuse and frequently is related to other violent acts, such as robbery. Other risk factors for homicide include a history of loss of employment, detention or prison experience, access to firearms, abuse in the home, mental illness, social isolation, and homelessness. DIF:

Cognitive Level: Apply (Application)

REF:

p. 549

5. A nurse is working in a domestic violence shelter. Which of the following individuals would the nurse expect to encounter? (select all that apply) a. 34-year-old White woman who is vice president of a communications company b. 26-year-old Black woman who has a live-in boyfriend c. 29-year-old Hispanic man who has been married for 2 years


d.

21-year-old unemployed White woman who has been with her boyfriend for 3 years

ANS: A, B, C, D Abusive behavior victims come from all racial, ethnic, and socioeconomic levels. DIF:

Cognitive Level: Apply

(Application)

REF:

p. 549

6. A nurse is planning community health programming for young adults. Based on the objectives for Healthy People 2020, which of the following topics would be priority areas for programming? (select all that apply) a. Prenatal care b. Physical activity c. HIV d. Suicide ANS: A, B Healthy People 2020 objectives targeted for the young adult include increasing the proportion of adults who engage in regular aerobic activity of moderate intensity and increasing the proportion of women who receive early and adequate prenatal care. HIV and suicide rates are not mentioned in the target areas for young adults. DIF:

Cognitive Level: Apply (Application)

REF:

p. 537 (Box 22-2)

Chapter 23: Middle-Age Adult Edelman: Health Promotion Throughout the Life Span, 8th Edition MULTIPLE CHOICE 1. A 35-year-old person asks the nurse working in a dermatology clinic what causes skin to wrinkle. Which of the following statements would be most appropriate way for the nurse to reply? a. “There is really nothing you can do to prevent a lot of wrinkles. It is part of aging.” b. “As we age, we lose fat beneath the skin so wrinkles appear. There is nothing we can do to prevent it.” c. “Wrinkles appear for a number of reasons, including loss of fat beneath your skin and too much sun exposure.” d. “Wrinkles are caused by genetic factors.” ANS: C Loss of subcutaneous fat and collagen damage related to sun exposure cause wrinkles. DIF:

Cognitive Level: Apply

(Application)

REF:

p. 563

2. A 45-year-old client who has experienced a ten pound weight gain during the past year asks the nurse if she thinks iron supplements would help increase her energy. Which of the following would be the most appropriate action for the nurse to take? a. Ask the client to describe her daily activity and diet b. Leave a note requesting an iron supplement on the client’s chart for the doctor c. Provide the client with information related to aerobic exercise classes d. Ask the client if she is still menstruating ANS: A A lack of routine exercise and activity plus a diet high in calories contributes to weight gain and a lack of energy. The priority action by the nurse is assessment, which will reveal the reason for a lack of energy. DIF:

Cognitive Level: Apply (Application)

REF:

p. 563


3. Which of the following is a normal physiological change that occurs during middle age? a. Increase in gastric emptying resulting in acid reflux b. Reduction in height caused by kyphosis c. Reduction of glomerular filtration by 50% d. Increase in the amount of estrogen produced in both genders ANS: C Gastric emptying is decreased, kyphosis is not an expected part of aging, and estrogen levels decrease in women and increase in men as male testosterone levels decrease. DIF:

Cognitive Level: Understand (Comprehension) REF:

p. 564

4. A nurse is conducting a community education program. Which of the following should be stressed as the leading cause of death in middle-age adults? a. Automobile accidents b. Malignant disease c. Homicide d. Suicide ANS: B The leading causes of death during middle adulthood are heart disease, cancer, and accidents. Accidents are the major cause of death for children and young adults. Homicide and suicide are major causes of death for young adults. DIF:

Cognitive Level: Apply (Application)

REF:

p. 564 | p. 564 (Box 23-1)

5. A nurse formed a political action committee to advocate for health insurance for the uninsured. On which of the following populations would the nurse focus the group’s efforts? a. Asian Americans b. Blacks c. Hispanic Americans d. Working-poor White Americans ANS: C Hispanic Americans are the largest minority group in the United States and have the highest uninsured rate. DIF:

Cognitive Level: Apply (Application)

REF:

p. 567 6. A nurse has been asked to develop a visit plan for a mobile mammogram van. Which of the following considerations would be made by the nurse when deciding which neighborhoods to visit? a. Hispanic women have the highest rate of breast cancer of all minority groups in the United States. b. The death rate from breast cancer is higher in Hispanic women than for women in other ethnic groups. c. The death rate from breast cancer in Hispanic women has risen in the past 10 years. d. Breast cancer in Hispanic women is usually diagnosed at an early stage. ANS: B Even though Hispanic women have a breast cancer rate approximately 40% lower than non-Hispanic White women, their death rate is higher and the cancer is diagnosed at a later stage. DIF:

Cognitive Level: Apply (Application)

REF:

p. 567

7. A nurse has received a grant to provide a community-based education series for middle-age adults. Which of the following topics would allow the nurse to have the greatest impact on the health of members of this age group?


a. b. c. d.

Diet and exercise Seat belt use Depression screening Cancer prevention

ANS: A Obesity is a major risk factor for heart disease, metabolic syndrome, and Type 2 diabetes mellitus. An educational series devoted to discussing diet and exercise will have the greatest impact on health. DIF:

Cognitive Level: Analyze (Analysis)

REF:

pp. 568-569

8. The nurse collects the following assessment data from a woman: Hispanic ethnicity, BMI 29 kg/m 2, age 41. Which of the following actions should be taken by the nurse? a. Encourage the client to continue her dietary and exercise patterns. b. Discuss her current dietary and exercise lifestyle habits. c. Counsel the client that she will face chronic health problems because of her weight. d. Encourage the client to adopt a low-calorie diet to help control her weight. ANS: B Obesity is defined as a BMI of 30 kg/m2. The client is at risk for obesity. Further data should be gathered related to her dietary and exercise patterns so as to plan appropriate interventions. Adoption of a low-calorie diet may not be an appropriate intervention depending on what additional data collection reveals. Also, dietary changes should be accompanied by regular aerobic exercise. DIF:

Cognitive Level: Analyze (Analysis)

REF:

p. 568

9. A nurse working in a rural location is interested in starting a weight management group for clients in the area. Which of the following interventions is most likely to ensure success? a. Using a pre-set list of topics for discussion b. Relying on printed material to relay information to the group c. Asking for a list of topics that interest the group d. Beginning each session with a 2-mile walk ANS: C When participants plan the topics of discussion, interest is maintained. A pre-set list of topics may not meet the learning needs of this particular group. A 2-mile walk may not be realistic for participants. DIF:

Cognitive Level: Apply (Application)

REF:

p. 568

10. A 52-year-old postmenopausal woman tells the nurse that she is afraid she will develop osteoporosis like her mother did. Which of the following statements is the most appropriate response by the nurse? a. “The American diet is much better now than when your mother was your age, so you have enough calcium in your diet to keep your bones strong.” b. “You have a strong genetic risk factor. There is not much we can do to stop the process.” c. “You need about 1000 mg of calcium a day to keep your bones strong. Let’s talk about your diet.” d. “Tell me about your diet, how much walking you do each day, and what medications you take.” ANS: D The postmenopausal woman needs 1500 mg of calcium a day if she is not taking estrogen. Dietary calcium and vitamin D intake, and walking, can help prevent osteoporosis. DIF:

Cognitive Level: Apply (Application)

REF:

11. Which of the following statements about alcohol intake is correct? a. Women have a higher death rate related to alcoholism than men.

p. 569


b. c. d.

A woman who drinks two glasses of wine per day with her evening meal is considered a moderate drinker. More women than men are considered problem drinkers. Women who experience fewer cognitive effects from drinking are less likely to drink heavily than do women who get a “buzz” from drinking.

ANS: A The death rate for female alcoholics is 50% to 100% higher than for male alcoholics. More men than women are considered problem drinkers. Moderate drinking for women is defined as one alcohol-containing drink a day. Women who are able to “hold their liquor” are more likely to drink excessively than those who cannot “hold their liquor.” DIF:

Cognitive Level: Understand (Comprehension) REF:

pp. 569-570

12. The occupational health nurse is planning a health fair for employees. Which of the following information about dental health should be included? a. Drinking bottled water helps prevent tooth decay. b. Gingivitis results in bone destruction and can be eliminated by flossing daily. c. Preventive dental care requires a visit to the dentist every 4 months. d. In addition to screening for tooth decay, dentists screen for mouth cancer. ANS: D Dental health is essential to overall health. Dental professionals may be the first to detect a symptom or irregularity that points to a potentially dangerous condition, such as cancer of the mouth and esophageal cancer. Bottled water does not contain fluoride. Periodontitis is oral bone destruction. An annual dental check-up is recommended for the average person, not a check-up every 4 months. DIF:

Cognitive Level: Apply (Application)

REF:

p. 570

13. Which of the following people has exceeded the recommended guidelines for activity? a. A 36-year-old man who engages in bike riding and has a pulse rate of 137 during activity b. A 45-year-old woman who swims and has a pulse rate of 145 during activity c. A 50-year-old man who jogs and has a pulse rate of 120 during activity d. A 60-year-old woman who walks briskly and has a pulse rate of 125 during activity ANS: D During activity, the person’s pulse rate should not exceed 220 minus the age times 0.75. Thus the woman who is 60 years old would subtract (220 – 60 = 180); then multiply 180 by 0.75 to give 120 for the optimal heart rate for this client. A pulse rate of 125 exceeds the recommended optimal rate of 120. DIF:

Cognitive Level: Apply (Application)

REF:

p. 572

14. A 63-year-old woman complains to the nurse that she has insomnia and sleeps only 6 hours a night. Which of the following responses should be made by the nurse? a. “Do you want a sleeping pill?” b. “Make sure that you do not nap during the day even if you are tired.” c. “Do you feel excessively tired during the day?” d. “The nurse practitioner may want to run some simple tests to find out why you are having trouble sleeping.” ANS: C It is important to assess the quality of sleep as well as the effects that the limited sleep has on this individual’s daily activities. Medication should not be the first intervention used to help someone attain restful sleep. People should nap when tired, if they are able to do so, rather than forcing themselves to stay awake.


DIF:

Cognitive Level: Apply (Application)

REF:

p. 572

15. A nurse conducted a 5-week education series about health maintenance. Which of the following statements by a participant indicates the analysis stage of cognition as defined by Bloom’s taxonomy? a. “If I eat too much food and do not exercise, I will gain weight.” b. “Fat is more calorie-dense than is protein. If I eat calorie-dense foods and do not do moderate exercise at least 5 days a week, I will gain weight.” c. “I will walk briskly at least a mile a day, 6 days a week, to maintain a healthy weight.” d. “I can eat more if I exercise more.” ANS: B During the analysis stage of cognition, all aspects of learning come together in thought and the individual is cognizant of the relationships and interactions of all the parts. “If I eat too much food and do not exercise, I will gain weight” indicates knowledge (recall of facts). “I will walk briskly at least a mile a day, 6 days a week, to maintain a healthy weight” indicates application. “I can eat more if I exercise more” indicates comprehension. DIF:

Cognitive Level: Apply (Application)

REF:

p. 573

16. A 62-year-old person states to the nurse, “I have trouble with my peripheral vision. Sometimes, I do not notice objects unless they are in front of me.” Which of the following disorders is the client likely experiencing? a. Presbyopia b. Glaucoma c. Cataracts d. Diabetic retinopathy ANS: B Glaucoma occurs as a result of increased intraocular pressure, which can damage the optic nerve. Damage to the optic nerve is irreversible, but visual loss can be prevented if damage is identified early and treatment is initiated. Peripheral vision is affected in glaucoma. Presbyopia is farsightedness. Cataracts cause cloudy vision. DIF:

Cognitive Level: Apply (Application)

REF:

p. 573

17. Which of the following middle-age adults has successfully transitioned to Erikson’s generativity-versusstagnation stage of development? a. A 60-year-old woman who retired last year and volunteers at a homeless shelter 2 days a week b. A 63-year-old man who regrets his career choice and feels he cannot change careers c. A 45-year-old woman who resents having to care for her aging mother-in-law d. A 50-year-old man who has not saved for retirement but spends his discretionary income on vacations he takes by himself ANS: A Generativity according to Erikson’s theory involves a sense of productivity, creativity, and the desire to help others, whereas stagnation involves a sense of isolation and focus on oneself. DIF:

Cognitive Level: Apply (Application)

REF:

p. 574

18. A 52-year-old woman going through menopause tells the nurse working in the women’s health center that she is interested in slowing down the aging process and asks if she should take estrogen. Which of the following statements would be the best response by the nurse? a. “The latest research has shown that estrogen is safe to take and will help keep you healthy. I’ll tell the nurse practitioner that you are interested in estrogen therapy.” b. “Estrogen is safe to take for about 10 years. I’ll get you some information on estrogen therapy that you can take home and read.” c. “The safety of long-term estrogen therapy is inconclusive. It is primarily used short-term to relieve hot flashes.” d. “You can get the same effects from natural products in a health food store, and each product is


backed up by research showing that it is effective.” ANS: C Research about the safety of estrogen therapy is mixed, therefore it is only recommended for short-term use (1 to 3 years) and only to relieve hot flashes. Natural products do not need research to back up their claims of safety or efficacy. DIF:

Cognitive Level: Apply (Application)

REF:

pp. 574-575

19. The occupational health nurse is asked to develop a plan to reduce work-related injuries. Which of the following recommendations would the nurse stress as the most effective way to reduce work-related injuries? a. Make the work environment smoke-free. b. Reduce environmental noise levels. c. Conduct regular walk-through assessments at the worksite. d. Increase paid personal days from one to three. ANS: A Accidents are twice as high among smokers than nonsmokers. Possible explanations include the loss of attention, the use of one hand for smoking, and irritation of the eyes. Because smoking has such an impact on accident rates, this would have the largest effect on reduction of work-related injuries. DIF:

Cognitive Level: Apply (Application)

REF:

p. 576

20. Which of the following effects of divorce on middle-age adults and children has been documented by research? a. Children adapt to divorce more rapidly than do adults. b. Emotional healing after a divorce takes approximately 24 months in adults. c. Children of divorced parents may have life-long difficulty forming relationships. d. Children of divorced parents undergo similar emotional turmoil as their parents. ANS: C Although research is continuing, longitudinal studies by Wallerstein and her colleagues beginning in the early 1970s indicate that children bear the emotional scars of a divorce for a lifetime. The emotional response to divorce is different for children and parents, with most parents adjusting to the divorce within 3 years after the event. DIF:

Cognitive Level: Understand (Comprehension) REF:

p. 578

21. A nurse is discussing sexuality with middle-age men and women. Which of the following information should be stressed by the nurse? a. Contraception usage should be decreased as a woman nears menopause. b. Difficulty with sexual performance among men happens near age 60. c. HIV infection after age 50 is rare. d. The majority of new HIV/AIDS cases occur among middle-age adults. ANS: D Adults in middle age represent 71% of all new cases of HIV/AIDS. Unintended pregnancy occurs most often in middle-age women. Male sexual performance issues begin between the ages of 40 to 50. HIV infection is not rare among middle-age adults. DIF:

Cognitive Level: Apply (Application)

REF:

p. 579

22. An occupational health nurse is planning injury-prevention educational sessions. Which of the following considerations should be made by the nurse? a. Education should be focused on workers with the least experience. b. All workers need continued education related to safety issues.


c. d.

OSHA rules apply to all companies with more than 25 employees. Middle-age workers have the lowest rate of injury of any age group.

ANS: B The role of the occupational health nurse is focused on keeping the worksite safe for employees. Experienced workers may not know how to prevent injury. OSHA applies to all companies. There are no data to support that middle-age workers have the lowest rate of injury. Physical and cognitive changes, which are part of middle age, may predispose workers to injury (i.e., decline in vision/hearing). DIF:

Cognitive Level: Apply (Application)

REF:

p. 584

23. Taking into account all of the agents that affect middle-age adults, the nurse plans a health education workshop focused on developing an individualized lifestyle change program for each participant. Which of the following actions should the nurse stress that individuals take first? a. Limiting unprotected exposure to sun b. Starting a smoking cessation program c. Exercising at least three times a week d. Scheduling a complete physical and dental exam ANS: D Although limited unprotected exposure to the sun, starting a smoking cessation program, and regular exercise are important to overall health, assessment data must first be collected to create an effective lifestyle change program. Health care needs and issues vary according to race and ethnicity, socioeconomic status, marital status, and so on. Approaches to health care/education must be individualized. DIF:

Cognitive Level: Analyze (Analysis)

REF:

pp. 585-586

24. Which of the following medications is generally recommended to help preserve cardiovascular health in middleage women? a. Aspirin b. Folic acid c. Estrogen d. Estrogen receptor modulator ANS: A Only aspirin is recommended to help maintain cardiovascular health by lowering the risk of clot formation. DIF:

Cognitive Level: Understand

(Comprehension) REF:

p. 577 (Box 23-7)

MULTIPLE RESPONSE 1. Which of the following individuals is at risk for developing osteoporosis? (select all that apply) a. A 55-year-old man who is on a low dose of oral prednisone every day because of rheumatoid arthritis b. A 40-year-old woman who works as a secretary c. A 60-year-old woman who walks three miles a day d. A 50-year-old man who works as a mason ANS: A, B Osteoporosis occurs most frequently in postmenopausal women who have fair complexions and are small, sedentary individuals, and people on long-term steroid use. It increases with age. DIF:

Cognitive Level: Apply (Application)

REF:

p. 563


2. Blacks are more likely than non-Hispanic White Americans to be diagnosed with cancer at a later stage and have a less optimal outcome after the diagnosis is made. Which of the following reasons help explain this difference? (select all that apply) a. Lack of access to, or utilization of, cancer screening b. Lack of health insurance c. Genetic variations d. Lack of resiliency against disease ANS: A, B, C Although poverty and lack of access may account for lack of screening and early diagnosis, genetic variations in tumor growth and response to medication along with other major health problems also affect outcomes. DIF:

Cognitive Level: Understand (Comprehension) REF:

p. 566

3. A community health nurse is caring for a variety of different people in the community. Which of the following individuals would the nurse recognize as being at risk to experience increased stress related to family dynamics? (select all that apply) a. A 54-year-old couple whose young adult daughter and grandson move back into their home b. A 44-year-old single mother raising three adolescent children without child support c. A 60-year-old couple whose children have established successful careers in another state d. A 36-year-old couple whose child plays competitive tennis ANS: A, B, D Stressors of middle age include adult children moving back home, heading a single parent household, and parents whose child plays competitive sports with an emphasis on winning. The couple who has children living out of state who have established successful careers are at the least risk to have stress related to family dynamics. DIF:

Cognitive Level: Apply (Application)

REF:

pp. 575-576

4. The occupational health nurse working for a large corporation is a member of a task force to improve the work environment. Which of the following suggestions by the nurse would help to improve work conditions? (select all that apply) a. On-site walking paths b. On-site food vendors c. On-site counseling services d. On-site meetings of support groups such as Alcoholics Anonymous ANS: A, C, D On-site facilities that improve health (walking paths) and psychological well-being (support groups, counseling) will help to create a positive work environment. Depending on the food vendor who is on-site, the healthy food options may not be beneficial to the employees. DIF:

Cognitive Level: Apply (Application)

REF:

p. 578

Chapter 24: Older Adult Edelman: Health Promotion Throughout the Life Span, 8th Edition MULTIPLE CHOICE 1. A nurse is planning to write a grant for a health promotion project in the community. Which of considerations should the nurse make when determining the appropriate population for this project? a. The fastest growing population is children under the age of 12. b. The fastest growing population is young Black American adults. c. The fastest growing population is White middle-age adults.


d.

The fastest growing population is non-White older adults age 65 and older.

ANS: D Adults age 85 and older are the fastest growing population in the United States, and the percentage of White Americans greater than age of 65 is decreasing compared with other ethnic groups. DIF:

Cognitive Level: Apply (Application)

REF:

p. 592

2. Which of the following statements made by a client best identifies someone who would benefit from health promotion interventions? a. “I have a new grandchild and want to be part of her life.” b. “My mother lived until she was 90, so a long life is in my genes.” c. “Some chronic illness is just a part of aging.” d. “I hope I’ll live a lot longer, but one never knows.” ANS: A Motivation to adopt a healthy lifestyle is a primary predictor of successful interventions. Feeling that illness is just part of aging or leaving health up to chance will not help the person make necessary lifestyle changes. DIF:

Cognitive Level: Analyze (Analysis)

REF:

p. 595

3. Which of the following statements concerning nutrition and older adults is true? a. Older adults who live in their own homes are more likely to be malnourished than older adults living in a nursing home. b. Older adults lose their desire for high-fat, salty foods. c. Meals served in long-term care institutions are usually more well-balanced than foods eaten by older adults living at home. d. Many older adults are not aware of food assistance programs. ANS: D Barriers that may interfere with the ability of independent seniors to obtain adequate nutritional food include limited transportation, income, and social support resources. Many older persons are unaware that they are eligible to participate in SNAP. The percent of older adults who live in their own homes and are malnourished is lower than that of older adults who live in a nursing home. Older adults do not lose their desire for high-fat, highcholesterol, high-sodium foods. Fresh fruits and vegetables are not always available for older adults living in nursing homes. DIF:

Cognitive Level: Understand (Comprehension) REF:

pp. 596-597

4. Which of the following statements concerning urinary incontinence is correct? a. Urge incontinence occurs when one has the urge to void but is unable. b. Stress incontinence is associated with emotional turmoil. c. Functional incontinence occurs is associated with environmental barriers. d. Incontinence with high post-void residual occurs when someone laughs or sneezes. ANS: C Functional incontinence is associated with environmental barriers, physical limitations, or cognitive impairment in which the client is unable to reach the toilet. Stress incontinence occurs when someone coughs or sneezes. Urge incontinence occurs when someone cannot wait to void once the urge to void is felt. Urge, mixed, or stress incontinence with high post-void residual is caused by physiological changes that affect voiding, such as an enlarged prostate gland. DIF:

Cognitive Level: Understand (Comprehension) REF:

p. 597

5. Which of the following nursing diagnoses is a priority related to the problem of urinary incontinence?


a. b. c. d.

Risk for social isolation Risk for impaired skin integrity Risk for inadequate fluid intake Risk for impaired coping

ANS: B Although social isolation and decreased fluid intake may occur because of urinary incontinence, impaired skin integrity is a major concern because of its impact on physical and psychological health. DIF:

Cognitive Level: Analyze (Analysis)

REF:

p. 597

6. A 75-year-old man tells the nurse that he has difficulty staying asleep during the night. Which of the following responses should be made by the nurse? a. “Older adults need as much sleep as middle-age and younger adults. Let’s talk about your sleep patterns.” b. “I’ll ask the nurse practitioner to prescribe a sleeping pill for you.” c. “Taking naps during the day can help relieve excessive fatigue and will help you sleep better.” d. “Let’s talk about your sleep schedule and things that may prevent you from getting a restful night’s sleep.” ANS: D Nurses should assist older adults in achieving a good night’s sleep through assessment that might reveal possible causes of sleep disturbances. Older adults need less sleep than younger adults. Daytime naps may prevent sleeping adequately at night. Medications to assist in sleeping are not the initial therapy choice because of side effects. DIF:

Cognitive Level: Apply (Application)

REF:

pp. 599-560

7. A nurse is conducting a community education program about cognition in older adults. Which of the following information should be included? a. Some form of mild dementia is a normal part of aging. b. All forms of dementia have the same symptoms, but progress at different rates. c. Assessment for dementia should be part of routine physical exams. d. Elders who score below 27 points on the Mini-Mental State Exam (MMSE) are not likely to have a cognition problem. ANS: C Cognitive alterations are key symptoms that indicate changes in physiological function among older persons. Thus, assessment of cognition is an important part of routine assessments. Dementia is not a normal part of aging. Dementia symptoms vary according to the specific type of dementia. A score of 23 or lower on the MMSE indicates a problem with cognition. DIF:

Cognitive Level: Apply (Application)

REF:

pp. 600-601

8. A nurse has recently accepted the position of unit nurse manager on a long-term care unit. Which of the following directives should the nurse give to the staff nurses? a. Encourage residents to participate in unit activities such as Scrabble™ and bingo. b. Demonstrate the caring aspect of nursing by dressing residents before breakfast. c. Fill in “missing words” during conversation with the residents to avoid embarrassment. d. Instruct the aides to get residents ready for breakfast as quickly as possible to make sure they are ready to eat. ANS: A


To encourage cognitive health nurses should inspire older individuals to take classes, read, engage in stimulating conversation and entertainment, keep their minds active, and continue learning throughout their lives. They are encouraged to continue with self-care activities rather than relinquish them to caregivers. Residents should be encouraged to perform self-care activities themselves. When help is required, it should be provided in an unhurried manner. Filling in missing words makes the resident dependent on the staff and reduces motivation to communicate, and the words supplied may not be the intended words. DIF:

Cognitive Level: Apply (Application)

REF:

p. 602

9. A nurse is assisting with development of menu items at a long-term care facility. Which of the following menus would the nurse most likely recommend? a. One egg over easy, one slice of whole wheat toast with sugar-free jam, 4 oz orange juice b. 5-oz cheeseburger cooked medium well with lettuce and tomato, 10 baked potato chips, and 1 cup of decaf coffee with low-fat milk and artificial sweetener c. Chef salad with 2 oz each ham and turkey, lettuce, tomato, bean sprouts, onion, green pepper with low-fat dressing, and one small roll with low-fat spread d. Turkey bacon, lettuce, and tomato sandwich on whole wheat bread with two teaspoons of low-fat mayonnaise, 10 vegetable chips, and ice tea with artificial sweetener ANS: D The United States Department of Agriculture recommends that infants and young children, pregnant women, older adults, and those who are immune compromised should not consume raw (unpasteurized) milk or any products from unpasteurized milk, raw or partially cooked eggs or foods containing raw eggs, raw or undercooked meat and poultry, raw or undercooked fish or shellfish, unpasteurized juices, or raw sprouts. DIF:

Cognitive Level: Analyze (Analysis)

REF:

p. 603

REF:

p. 603

10. Which of the following individuals would be at greatest risk of injury? a. 80-year-old who does not have air conditioning or a fan b. 70-year-old who has new dentures c. 68-year-old who has difficulty tasting salt in food d. 84-year-old who needs hearing aids ANS: A Sweating decreases in older adults, predisposing them to heat stroke. DIF:

Cognitive Level: Analyze (Analysis)

11. According to Erikson’s theory of development, which of the following older adults has successfully navigated the stage of ego integrity versus despair? a. 72-year-old man who is reluctant to retire because “work is everything to me” b. 78-year-old woman who has scheduled her third face lift c. 80-year-old man who has informed his children that he has made his funeral arrangements d. 67-year-old woman who is depressed because she has not been promoted at work for the past 10 years ANS: C Successfully navigating the stage of ego integrity versus despair requires that the individual accept normal bodily changes associated with aging, find meaning in life apart from work, accepts the inevitability of death, and is at peace with his or her life. DIF:

Cognitive Level: Analyze (Analysis)

REF:

p. 604

12. A couple in their sixties has assumed responsibility for raising their two grandchildren ages 9 and 12 because the children’s parents died in a car accident. Which of the following are priority nursing interventions that the nurse can implement to help the grandparents cope with the responsibility of raising their grandchildren?


a. b. c. d.

Connecting them with the “Grandparents Raising Grandchildren” support group Encouraging them by reinforcing how they successfully raised their own children Volunteering to provide respite time for them as needed Assuring them that their stress level will lessen as they gain confidence

ANS: A Stress experienced by grandparents who must raise grandchildren can be lessened by counseling and participation in support groups. DIF:

Cognitive Level: Apply (Application)

REF:

pp. 604-605

13. A nurse is conducting a staff development program about human sexuality for nurses who work in a long-term care facility. Which of the following statements would most likely be made by the nurse? a. “The desire for sexual intimacy declines rapidly with age.” b. “Older adults are very well-informed about sexuality.” c. “Older adults need to practice safe sex.” d. “The danger of contracting STDs during sexual intimacy is extremely low in older adults.” ANS: C The desire for sexual intimacy remains a primary force throughout life. Older adults need teaching related to safe sexual practices, STD transmission, and so on. HIV infection is a concern in all age groups. DIF:

Cognitive Level: Apply (Application)

REF:

p. 606

14. The clinic nurse has seen the following four people today. Which of the following people is demonstrating a healthy coping pattern? a. Discusses very little except the loss of her husband 3 years ago b. Talks about the “good times” she and her husband had before his illness and death 1 year ago c. States that life is not worth living without her husband, who died 2 years ago d. Blames herself for her husband’s death because she did not “make him go” to the doctor ANS: B Research has demonstrated that rumination, self-blame, and catastrophizing have negative effects on coping, whereas positive reappraisal has a positive effect. DIF:

Cognitive Level: Apply (Application)

REF:

p. 606

15. A nurse is assigned to provide care for a 96-year-old bedridden man who experiences severe pain from a chronic neurological disease. The man asks the nurse to please help him leave this world so that his suffering ends. Which of the following actions would be most appropriate for the nurse to take? a. Assist the doctor with the man’s request b. Refuse to care for the man c. Contact the pain care specialist d. Provide the man with names of doctors who participate in assisted suicide ANS: C Nurses caring for chronically ill older adults have the added responsibility of determining who is at risk for wanting physician-assisted suicide and helping them to be as comfortable as possible and free of pain through the use of pharmacological and nonpharmacological interventions. Refusing to care for the man could be construed as abandonment and is not necessary. Consulting a pain care specialist is the correct action. DIF:

Cognitive Level: Apply (Application)

REF:

p. 607

16. A nurse is conducting a home visit for an older adult in the summer. Which of the following items should the nurse should locate in the home?


a. b. c. d.

Fan Porch chair Synthetic blend clothing Thermometer

ANS: A Older adults are susceptible to heat stroke. The availability of a fan will help cool the ambient air. Light-weight cotton clothing “breathes” and facilitates cooling, whereas synthetic material retains heat. DIF:

Cognitive Level: Analyze (Analysis)

REF:

p. 609

17. The community health nurse has obtained a grant to offer a “Safe Driving” series for older adults. Which of the following information should the nurse stress during the series? a. Traffic fatalities involving older adults occur mainly at dusk. b. The cause of two-vehicle accidents is frequently an older driver striking another car. c. The average blood alcohol level in older adults involved in accidents is higher than the blood alcohol level of younger drivers involved in accidents. d. Relearning the art of defensive driving can help reduce the likelihood that an older driver will be involved in a motor vehicle accident. ANS: D Traffic fatalities involving older adults occur mainly during the daytime. The cause of two vehicle accidents is frequently a younger driver striking the car driven by an older adult. The average blood alcohol level in older adults involved in accidents is lower than the blood alcohol level of younger drivers involved in accidents. DIF:

Cognitive Level: Apply (Application)

REF:

p. 609

18. As part of a community-wide flu vaccination program for older adults, the nurse is assisting at a vaccination clinic. Which of the following questions should the nurse ask before administering the vaccine? a. “Do you have any food allergies?” b. “Have you had the pneumococcal vaccine?” c. “Have you had the flu in the past 5 years?” d. “Do you have Medicare?” ANS: A An allergy to eggs is a contraindication to receiving flu vaccine grown in chick embryo cells. DIF:

Cognitive Level:

Analyze (Analysis)

REF:

p. 610

19. A man tells a nurse that he is concerned about his risk of developing cancer. Which of the following behaviors would place the man at higher risk of developing cancer? a. Smoking a half of a pack of cigarettes a day b. Exercising three times a week c. Consuming green, leafy vegetables several times a week d. Visiting a chiropractor every week ANS: A Habits that place older adults at a high risk for developing cancer: not following nutritional guidelines, high stress levels, not engaging in a regular exercise program, and smoking cigarettes and using other tobacco products. DIF:

Cognitive Level: Apply (Application)

REF:

p. 611

20. Which of the following statements is the best definition of polypharmacy? a. The use of more than five medications prescribed by different physicians


b. c. d.

The use of multiple medications for the same or different health problems The use of multiple pharmacies and self-management of medications The use of multiple medications and supplements simultaneously

ANS: B Polypharmacy is the use of multiple medications for the same or different health problems. It is a major concern for elderly people, with the rate of polypharmacy in this population being between 9% and 39%. DIF:

Cognitive Level: Understand (Comprehension) REF:

p. 611

21. A person is preparing to return home following a 2-week hospital stay for congestive heart failure and pneumonia. Which of the following referrals would be most beneficial for the person? a. Transportation assistance b. Home-delivered meals c. Adult day care d. Home health nursing ANS: D Home care nurses provide health care information and services to individuals and families. The resources available to community health nurses frequently are rich and enable the nurses to draw on a variety of sources to assist in promoting the health of community-dwelling older adults. Although all of the referrals listed may be of benefit to the person, the home health nurse will provide the most comprehensive services and be able to connect the person to necessary community resources. DIF:

Cognitive Level: Analyze (Analysis)

REF:

p. 614

22. The son of a 70-year-old man dying of cancer asks the nurse to tell him about the hospice program, so he can help his father decide if the program would be of benefit to him. Which of the following responses should be given by the nurse? a. “At hospice, your father will be heavily medicated, so he will not have pain.” b. “All of your father’s care will be provided by nurses at hospice.” c. “Hospice care can take place either at home or in a hospice facility.” d. “Visiting hours are about the same at hospice as they are here in the hospital.” ANS: C Once the person is placed in hospice care, treatment to relieve pain and other symptoms is continued by the physician after they have decided to end all curative treatment. Hospice care can occur in the home, a hospice inpatient facility, nursing home, and acute care hospital. The focus of hospice care is to help the person remain alert but free of pain. Families are encouraged to participate in care giving. There are no restrictions to visiting hours. DIF:

Cognitive Level: Apply (Application)

REF:

p. 614

23. A community health nurse is working with a diverse population of older adults within the community. Based on the data from the National Health Interview Survey, which of the following assumptions should be made by the nurse when working with this diverse population? a. White Non-Hispanic older adults tend to require more assistance with their activities of daily living. b. Hispanic older adults tend to rate their health as poorer than other ethnic groups. c. Older adults who tend to rate their health as fair or poor are also financially poor. d. Older adults tend to require more assistance with activities of daily living than with meeting their routine needs. ANS: C


The National Health Interview Survey conducted by the Centers for Disease Control and Prevention in 2010 provides interesting information about the perceptions of a variety of ethnic groups regarding their health status, their perceived need for assistance with routine needs, and the need for assistance with activities of daily living. In all groups, those who described their health as fair or poor were also more likely to be financially poor. DIF:

Cognitive Level: Apply (Application)

REF:

p. 594 (Box 24-2)

24. A woman who is being discharged from the hospital is oriented to person, place, and time, and her memory is intact. She has occasional forgetfulness without a consistent pattern of memory loss. Which of the following housing options would be most appropriate for this woman? a. Independent living b. Retirement community c. Assisted living d. Nursing facility ANS: B Retirement community living is suited for a person who is oriented to person, place, and time and has a memory that is intact, but has occasional forgetfulness without a consistent pattern of memory loss. Independent living may be appropriate for the person if she had a consistent pattern of memory loss. Assisted living or nursing facility living may be appropriate if the woman has difficulty with orientation to person, place, or time or has difficulty with confusion that results in anxiety, social withdrawal, or depression. DIF:

Cognitive Level: Analyze (Analysis)

REF:

p. 612 (Table 24-4)

MULTIPLE RESPONSE 1. Which of the following interventions will have the greatest impact on reducing constipation in older adults? (select all that apply) a. Offering pureed foods that are easy to digest b. Encouraging fluids between meals c. Administering laxatives on a daily basis d. Assisting with ambulation ANS: B, D Nurses can help reduce the incidence of constipation by encouraging older adults to exercise and increase their fluid and dietary intake. Dietary modifications, such as increasing fiber and fluid intake, can stimulate the colon and resolve constipation. Pureed foods do not contain adequate fiber. Increased fluid intake and exercise should be implemented before relying on medication to relieve constipation. DIF:

Cognitive Level: Apply (Application)

REF:

p. 597

2. A nurse has accepted a position as director of health services in a retirement community. Which of the following statements about retirees is important for the nurse to consider as she plans programs? (select all that apply) a. Depressed and inactive. b. Left their job to spend more time with family. c. Left their job because of ill health. d. Happy and well-adjusted to retirement. ANS: B, D Research has demonstrated that the majority of retirees are in good health and happy with their decision to leave work so as to spend more time with their families. DIF:

Cognitive Level: Apply (Application)

REF:

pp. 604-605


3. The nurse working in a retirement community for older adults is conducting a psychosocial assessment of all residents. Which of the following questions should the nurse ask during the assessment process? (select all that apply) a. “Have you thought about harming yourself?” b. “Are you still playing bridge every Thursday?” c. “How does your congestive heart failure affect your life?” d. “Has your financial situation changed since last year?” ANS: A, B, C Suicide ideation, loss of interest in activities, and chronic illness are signs of, or may lead to, depression. Economic difficulty is not a predictor of suicide in older adults. DIF:

Cognitive Level: Analyze (Analysis)

REF:

p. 607

4. Which of the following nursing interventions implies respect for a person’s spirituality? (select all that apply) a. Changing the position of the bed in a Muslim client’s room so that it faces east b. Contacting the chaplain to assess the client’s spiritual needs c. Saying a silent prayer with a client at the client’s request d. Praying out loud in the room of a dying client ANS: A, C Because of the highly personal quality of spirituality, an unobtrusive and sensitive presence by the nurse is needed to allow the person in any setting to achieve spiritual health. The nurse can provide an environment that is supportive to the practice of the person’s spirituality. Praying out loud does not demonstrate respect for a person’s spirituality but imposes the nurse’s belief system on the person. Nurses should be able to assess the person’s spirituality initially and determine if it is necessary to further consult the chaplain. DIF:

Cognitive Level: Apply (Application)

REF:

p. 607

5. A nurse is conducting a community health education program about osteoporosis with a group of older adults. Which of the following individuals in attendance is at greatest risk for developing osteoporosis? (select all that apply) a. 65-year-old Asian woman who smokes one pack of cigarettes per week b. 80-year-old White man who has three glasses of wine per day c. 74-year-old White woman on daily oral prednisone for emphysema d. 68-year-old Black man who has a sedentary lifestyle ANS: A, B, C Osteoporosis is a disease of bone loss common to women age 70 and older and men age 80 and older. The disease occurs six times more frequently in women than it does in men. Risk factors include a small, thin frame; White or Asian ancestry; family history; excessive thyroid medication or high doses of cortisone-like drugs for asthma, arthritis, or cancer; a diet low in dairy products and other sources of calcium; physical inactivity; smoking cigarettes; and drinking alcohol. DIF:

Cognitive Level: Apply (Application)

REF:

pp. 607-608

6. An 80-year-old person is admitted to the hospital. During the health assessment process, which of the following conditions should the person be screened for by the nurse? (select all that apply) a. Polypharmacy b. Kidney failure c. Illegal drug use d. Alcoholism ANS: A, B, C, D


Alcoholism and illegal drug use are often overlooked by health care professionals because of a person’s age. Many older adults take multiple over-the-counter and prescription medications and are at risk for drug interactions. DIF:

Cognitive Level: Apply (Application)

REF:

pp. 611-612

Chapter 25: Health Promotion in the Twenty-First Century Edelman: Health Promotion Throughout the Life Span, 8th Edition MULTIPLE CHOICE 1. A nurse is encouraging members of the community to advocate for the prevention of premature deaths in children under the age of 5 at the international level. Which of the following interventions would most likely be discussed by the nurse? a. Using mosquito netting in sleeping areas b. Participating in international vaccination programs c. Implementing accident prevention programs d. Providing nutritional supplements ANS: D Protein-energy malnutrition can be severely harmful to the mental and physical development of individuals, especially young children under the age of 5. Worldwide, one out of two deaths among children younger than 5 years old stems from protein-energy malnutrition. DIF:

Cognitive Level: Apply (Application)

REF:

pp. 622-623

2. A nurse is working at a clinic in a third world country and suspects a child has severe acute malnutrition (SAM). Which of the following assessment findings would be expected? a. Distended abdomen b. Bulging eyes c. Large tears when crying d. Lethargy ANS: A Severe wasting is characterized by sunken eyes, visible ribs, and protruding shoulder blades. Children with severe wasting usually have a distended abdomen and general overall appearance in some way similar to an older adult. In general, these children are irritable, anxious, and cry easily; yet they will often have absence of tears while crying due to lachrymal gland atrophy. DIF:

Cognitive Level: Apply (Application)

REF:

pp. 622-623

3. A nurse is determining if children with severe acute malnutrition (SAM) should receive facility-based or community-based care. Which of the following children should receive care in the community? a. 3-year-old male who has a history of dehydration b. 9-month-old male whose family has adequate support and resources to obtain the recommended treatment c. 4-year-old female who has sensitivity to milk products d. 6-month-old female who has a good appetite and no underlying medical conditions ANS: D When a child has a good appetite with no medical conditions (hypoglycemia, hypothermia, dehydration, electrolyte imbalance, and/or infections), Ready-to-Use Therapeutic Foods (RUTF) under community-based care are appropriate. RUTF are special milk-based foods which are soft, crushable, and tasty nutrient- and energy-rich foods that can be consumed by children six months or older.


DIF:

Cognitive Level: Apply (Application)

REF:

p. 623

4. A nurse working on a post-partum unit of a large urban hospital has received a grant to begin a “Baby-Friendly Hospital Initiative” developed by WHO. Which of the following statements would the nurse most likely make when explaining the major premises of the initiative to the unit manager? a. “This program is meant to encourage sibling visits in the early postpartum period, so the adjustment to a new baby is a smooth process.” b. “A major goal of this program is to ensure that all infant/pediatric units have implemented basic safety initiatives.” c. “Improving nutrition by promoting breast-feeding of newborns through four months of age is what this program is all about.” d. “The purpose of this program is to make sure that all health care providers who care for newborns and children up to the age of 18 really understand basic principles of growth and development.” ANS: C The purpose of the initiative is to improve newborn nutrition by promoting breast-feeding of newborns through age 4 months. DIF:

Cognitive Level: Apply (Application)

REF:

pp. 623-624

5. Which of the following projects would most likely be administered by International Micronutrient Malnutrition Prevention and Control (IMMPaCT) Program? a. Researching best methods to preserve the freshness of foods b. Surveying various populations about consumption patterns c. Comparing the effectiveness of vitamin pills versus fresh fruit and vegetables d. Analyzing the relationship between BMI and cardiovascular disease ANS: B IMMPaCt provides its skills and resources toward working to eradicate vitamin and mineral deficiencies around the globe. The IMMPaCT program’s activities include conducting surveys; providing micronutrients to infants, young children, and women of childbearing age; and monitoring and evaluating intervention systems. DIF:

Cognitive Level: Analyze (Analysis)

REF:

p. 624

6. The school nurse has been asked to present a workshop on CA-MRSA for high school teachers. Which of the following information would the nurse most likely include? a. CA-MRSA is a common viral infection that responds well to antiviral meds. b. CA-MRSA is a serious bacterial infection that does not spread easily. c. CA-MRSA is a fungal infection common in school settings. d. CA-MRSA is a potentially fatal skin infection that is difficult to treat. ANS: D CA-MRSA is a highly contagious bacterial infection that is difficult to treat and can result in pneumonia and death. DIF:

Cognitive Level: Apply (Application)

REF:

p. 624

7. The school nurse has been asked to present a workshop on how to prevent CA-MRSA spread in the school. Which of the following interventions should be emphasized? a. Completing prophylactic antibiotic therapy for those in close contact with an infected person b. Using correct hand-washing technique c. Requiring those infected stay home from school until the infection has resolved d. Disinfecting all classrooms on a daily basis


ANS: B Hand washing is an effective way to prevent spread of the infection. In order to contract the infection, students must come into contact with a draining wound, etc. DIF:

Cognitive Level: Apply (Application)

REF:

p. 624

8. A nurse is providing care to HIV positive clients living in sub-Saharan communities in Africa. When discussing prevention of the transmission of HIV, which of the following statements should be made by the nurse? a. “Alternative practices to traditional sexual cleansing should be considered after the death of a spouse.” b. “Traditional sexual cleansing should be performed with a cleanser who is known to not have HIV.” c. “Virgin cleansing is a safer option than traditional sexual cleansing.” d. “Traditional sexual cleansing can lead to further transmission of HIV.” ANS: A Although traditional sexual cleansing can lead to further transmission of HIV is a true statement, it is not the most culturally sensitive response by the nurse. The nurse should respect the cultural practices of the clients living in the sub-Saharan community and discuss that alternative practices may be a better option to traditional sexual cleansing. DIF:

Cognitive Level: Apply (Application)

REF:

p. 625

9. A nurse is working with UNAIDS to fight against HIV/AIDS at the global level. Which of the following activities would the nurse most likely be involved in? a. Determining prevalence rates of HIV/AIDS in African countries b. Administering antiretroviral medications to pregnant women who are HIV positive c. Providing treatment access for all HIV-positive individuals who need medication d. Reporting confirmed cases of HIV to the World Health Organization ANS: C UNAIDS has outlined three strategic directions: revolutionizing HIV prevention; catalyzing the next generation for treatment, care, and support; and advancing human rights and gender equality in responses to HIV. Providing treatment access for all HIV-positive individuals who need medication meets the strategic direction of catalyzing the next generation for treatment, care, and support. DIF:

Cognitive Level: Analyze (Analysis)

REF: p. 626

10. A nurse is using the four-step public health approach to address violence in the community. Which of the following actions would the nurse take first? a. Analyze a variety of data sources to determine the cause of the problem. b. Examine risk factors that have contributed to the violence. c. Develop an evidence-based program to address the problem. d. Train community members to discuss the problem within their neighborhoods. ANS: B The WHO has proposed a four-step public health approach which includes: defining the problem; identifying risks and protective factors; devising and testing means of dealing with violence; and applying successful means on a large scale. Defining the problem is the first step in this process. Examination of different data sources are needed to understand the magnitude of the problem. DIF:

Cognitive Level: Analyze (Analysis)

REF: pp. 626-627

11. A nurse using the four-step public health approach to respond to violence. Which of the following actions would occur as the nurse identifies risk and protective factors? a. Planning a community-based educational program to respond to violence b. Reviewing police reports and population-based surveys


c. d.

Interviewing local community members about their experiences with violence Examining characteristics in the community where violence has not occurred

ANS: D A “risk factor” is defined as “a characteristic that increases the likelihood of a person becoming a victim or perpetrator of violence,” and a “protective factor” as “a characteristic that decreases the likelihood of a person becoming a victim or perpetrator of violence.” Knowing risk and protective factors can help responsible organizations and personnel to estimate violence magnitudes and devise appropriate prevention measures. DIF:

Cognitive Level: Apply (Application)

REF:

p. 627

12. The school nurse is planning a workshop for high school students about interpersonal violence (IPV). Which of the following information would the nurse discuss as risk factors for IPV? a. Cultural norms b. Family support c. Drug and alcohol abuse d. Depression ANS: C Risk factors for IPV may include a victim’s low self-esteem, low self-control, and personality/conduct disorders. Other risk factors are reported to be lack of social support, dysfunctional family structure, family history of violence, and drug and alcohol abuse. DIF:

Cognitive Level: Apply (Application)

REF:

p. 627

13. The nurse is working at a crisis hotline center. Which of the following callers would be most at risk for suicide? a. A 16-year-old girl who broke up with her boyfriend b. A 26-year-old male who has been laid off from his job c. A 50-year-old female who has a history of alcohol abuse d. A 40-year-old male whose brother died suddenly ANS: C Older adults have a greater risk of committing suicide than do younger people. Major factors contributing to suicide in Europe and North America include mental disorders especially depression and alcohol use disorders. DIF:

Cognitive Level: Analyze (Analysis)

REF:

p. 628

14. Which of the following age groups is experiencing the greatest increase in suicide rates? a. 15 to 24 b. 25 to 39 c. 40 to 59 d. 60 to 70 ANS: A Suicide rates among people aged 15 to 24 have been increasing to such an extent that they are the second leading cause of death in this age group in some developed and developing countries. DIF:

Cognitive Level: Remember (Knowledge)

15. Which of the following information about anthrax is correct? a. Anthrax is a viral infection. b. Anthrax can cause skin lesions or pneumonia. c. Anthrax is spread by person-to-person contact. d. Anthrax is treated by IV penicillin for 30 days.

REF:

p. 628


ANS: B Anthrax is a bacillus that is not spread person-to-person. An anthrax infection is treated by using ciprofloxacin, levofloxacin, doxycycline, or penicillin for 60 days. DIF:

Cognitive Level: Remember (Knowledge)

REF:

p. 629

16. Which of the following statements about smallpox is accurate? a. Smallpox results in a rash that looks similar to the chickenpox rash. b. Side effects from a smallpox vaccine include acute renal failure. c. The last case of smallpox occurred in 1997. d. The rash associated with smallpox is most prominent on the face and extremities. ANS: D A rash from smallpox is most prominent on the face and extremities with the same stage of legion development. Additionally, smallpox symptoms usually resemble influenza symptoms. The smallpox rash is distinctive from the varicella rash. Side effects from the vaccination mostly involve a low fever, soreness in the injection area, and enlarged glands under the armpit. The last case of smallpox was in 1977. DIF:

Cognitive Level: Understand (Comprehension) REF:

p. 630

17. A nurse is working in the emergency department (ED). Which of the following situations would cause the nurse to suspect bioterrorism? a. When two people out of twenty who ate at a restaurant come to the ED with nausea and vomiting b. When a worker in a lab comes to the ED complaining of a severe headache c. When thirteen people, aged 24 to 33, come to the ED with ascending flaccid muscle paralysis d. When five people come to the ED with an upper respiratory infection ANS: C The other examples are common problems for which people seek health care. Flaccid paralysis is a symptom of botulism. DIF:

Cognitive Level: Analyze (Analysis)

REF:

p. 630

18. Which of the following statements best explains priorities related to preventative health care? a. Global initiatives have been developed to respond to health care needs around the world. b. Health care initiatives are focused mainly on people in developed countries because of the access to medication. c. Health care initiatives are focused mainly on people in developing countries because of their great need. d. Public health agencies are leaders in health care development. ANS: A Health care initiatives have a global focus and involve developing as well as developed countries. Nurses play a major role in this initiative. DIF:

Cognitive Level: Understand (Comprehension) REF:

p. 632

19. A nurse states that many community members are concerned about the increasing number of natural disasters that have been occurring in the United States. Which of the following would the community members be referring to? a. Fires and suicides b. Wildfires and hurricanes c. Traffic accidents and floods d. Tornadoes and food poisonings


ANS: B Natural disasters phenomena that occur through natural forces involving land, air, or water, and they often have large-scale negative impacts on humans who live in the affected areas. Examples of natural disasters include tsunamis, earthquakes, floods, landslides, mudslides, tornadoes, hurricanes, cyclones, typhoons, wildfires, volcano eruptions, extreme heat, winter weather, and others. DIF:

Cognitive Level: Apply (Application)

REF:

p. 630

20. A hurricane has just caused extensive damage within a local community. Which of the following are community members at risk to experience? a. Unequal access to health care b. Exposure to community-associated methicillin-resistant Staphylococcus aureus c. Increased incidence of suicide d. Outbreaks of acute respiratory infection ANS: D Victims of natural disasters tend to have limited access to essential infrastructures for survival related to food, water, shelter, and sanitation. Infection outbreaks commonly diagnosed after natural disasters include diarrhea, acute respiratory infections, viral hepatitis, and snail and trematode infections. DIF:

Cognitive Level: Apply (Application)

REF:

p. 631

21. A primary care clinic is being established in a rural county that recently experienced a hurricane severely damaging its infrastructure for medical care. Which of the following considerations would be the priority when planning for this clinic? a. The ability to provide mental health services to community members b. The ability to deliver primary care services to community members c. The ability to refer community members to other necessary services d. The ability to supply community members with daily necessities ANS: A The priority in planning should be the ability to provide mental health services to the community members. In 2007, the Inter-Agency Standing Committee developed its Guidelines on Mental Health and Psychosocial Support in Emergency Setting. It emphasizes the importance of primary care clinics being able to provide mental health services to victims of the disaster. DIF:

Cognitive Level: Analyze (Analysis)

REF:

p. 631

22. A nurse is advocating for improved disease prevention and health promotion within the local community. Which of the following interventions would the nurse most likely implement? a. Educate community members about the violence rates that exist within the community. b. Administer flu shots for elderly at a local senior citizens center. c. Participate in a disaster planning committee within the community. d. Talk with a state legislator about improved access to health care for the uninsured and underinsured. ANS: D In order to be advocates for newly emerging priorities for disease prevention and health promotion, nurses in the twenty-first century need to participate in policy development for health promotion as the health care of individuals in acute settings shifts from hospitals to home and community settings, influence public expectations about health promotion, and promote equitable access to preventive health care. Talking with a state legislator about improving access to health care for the uninsured and underinsured is the only example that demonstrates the nurse in the role of an advocate working in one of these priority areas. DIF:

Cognitive Level: Analyze (Analysis)

REF:

pp. 634-635


MULTIPLE RESPONSE 1. Which of the following are classifications within the World Health Organization’s (WHO) definition of malnutrition? (select all that apply) a. Obesity b. Micronutrients deficiencies c. Protein-energy malnutrition d. Water insufficiency ANS: A, B, C Inadequate or excessive intake of protein and or vitamins and obesity are components of the WHO’s definition of malnutrition. DIF:

Cognitive Level: Remember (Knowledge)

REF:

p. 622

2. Which of the following individuals would be susceptible to CA-MRSA? (select all that apply) a. 16-year-old student who plays football b. 48-year-old person who has taken antibiotics for sinus infections c. 36-year-old man serving 12 months in jail for robbery d. 22-year-old woman who shares a towel after a swimming meet ANS: A, B, C, D CA-MRSA is spread by close skin-to-skin contact such as during contact sports, by sharing towels and clothing, and by those living in crowded conditions. People who take or who have been on antibiotic therapy are also susceptible to CA-MRSA. DIF:

Cognitive Level: Analyze (Analysis)

REF:

p. 624

3. A nurse employed by the World Health Organization is working with HIV positive clients in Africa and is teaching them about ways to reduce transmission of this disease. Which of the following barriers should be considered when planning this education? (select all that apply) a. Culture-specific practices b. Unavailability of antibiotic therapy c. Lack of understanding about the transmission process d. Overcrowded living areas ANS: A, C Culture-specific sexual practices such as “sexual cleansing” and a lack of understanding about how HIV is spread contribute to its transmission. HIV is a viral infection, not a bacterial infection. Simply living with someone who has HIV will not result in transmission unless the person comes into contact with blood or bodily fluid from the infected person. DIF:

Cognitive Level: Apply (Application)

REF:

pp. 625-626

4. A school nurse is planning an educational presentation for high school students about violence. Which of the following statements would be included in the presentation? (select all that apply) a. “Violent behavior happens when someone bullies another person.” b. “Withholding food from a child as a punishment is violent behavior.” c. “Collective violence occurs during an argument between two people.” d. “Painting a racial slur on a building is violent behavior.” ANS: A, B, D


Violent behavior as defined by WHO is the intentional use of physical force or power, threatened or actual against oneself, another person, or against a group or community, that either results in or has a high likelihood of resulting in injury, death, psychological harm, maldevelopment, or deprivation. Collective violence is defined as the instrumental use of violence by a particular group of people for specific political, economic, or social objectives. DIF:

Cognitive Level: Apply (Application)

REF:

p. 626

5. Which of the following individuals assessed by the nurse is experiencing interpersonal violence? (select all that apply) a. A 16-year-old female who hides in her apartment due to civil unrest in the country b. A 32-year-old male whose co-worker sends him insulting e-mails at work c. A 19-year old male who has attempted suicide d. A 78-year-old female living in a nursing home who is force-fed by aides ANS: B, D Interpersonal violence is violence committed by an individual or a small group of people in a wide range of acts and behaviors (emotional, physical, sexual, and psychological). The violence can happen to people of any age and at any place. Self-directed violence occurs with self-abuse and suicide. Collective violence is defined as the instrumental use of violence by a particular group of people for specific political, economic, or social objectives. DIF:

Cognitive Level: Analyze (Analysis)

REF:

p. 627

6. A nurse is participating in disaster preparedness planning for the local community. Which of the following would be an appropriate responsibility for the nurse? (select all that apply) a. Completing a risk assessment in the community b. Implementing strategies to decrease posttraumatic stress disorder c. Coordinating meetings among multiple agencies and disciplines d. Evaluating the economic impact of the disaster ANS: A, C The International Council for Nurses spells out its position on the role of the nurse concerning disaster preparedness to include risk assessment as well as management strategies bridging multiple disciplines and system levels. To coordinate health sector preparedness, regulatory meetings and frameworks across cultures will help nurses plan for and streamline health care responses to such mass events internationally. DIF:

Cognitive Level: Analyze (Analysis)

REF:

pp. 628-629


Turn static files into dynamic content formats.

Create a flipbook
Issuu converts static files into: digital portfolios, online yearbooks, online catalogs, digital photo albums and more. Sign up and create your flipbook.